344

FORWARD - WordPress.com · Return of premium p 6-8 6.0 Annuities p 7-1 Annuity principles and concepts p 7-1 Accumulation period versus annuity period p 7-1 Owner, annuitant and beneficiary

  • Upload
    others

  • View
    2

  • Download
    0

Embed Size (px)

Citation preview

Page 1: FORWARD - WordPress.com · Return of premium p 6-8 6.0 Annuities p 7-1 Annuity principles and concepts p 7-1 Accumulation period versus annuity period p 7-1 Owner, annuitant and beneficiary
Page 2: FORWARD - WordPress.com · Return of premium p 6-8 6.0 Annuities p 7-1 Annuity principles and concepts p 7-1 Accumulation period versus annuity period p 7-1 Owner, annuitant and beneficiary

FORWARD

The topical material presented throughout this text is designed to help the license candidate receive a passingscore on his or her state licensing examination. All state licensing examinations are extremely challenging. It is imperative that each candidate utilize a study text which provides an in-depth view of the material uponwhich the state examination is based. Numerous texts in publication today are excellent reference pieces. However, many of these texts are written generically for multi-state use and are not specifically designedto assist candidates to pass their state examination. Such texts include an exorbitant amount of extraneousmaterial. While relevant for possible use in life insurance today, nowhere is a candidate able to find in sucha text, what topical information will or will not appear on his or her state examination. In other words,volumes of information appear in these generic texts, much of which does not need to be reviewed by thecandidate in order to prepare for the state exam. Therefore, important self-study time may be centered oninformation that will not aid the candidate in passing the state examination.

The information throughout this study text will provide the licensing candidate with the topical areas thathe or she MUST comprehend in order to pass the state exam. Little extraneous information will beprovided. Numerous illustrations and examples will be provided to aid in this objective and to further clarifythe material. Sample quiz questions will also appear to test candidate retention of the material previouslyreviewed. A final examination is also provided to ensure retention of the necessary topical material. Again,the primary purpose of this text is to help each candidate to:

PASS THE STATE EXAMON THE FIRST ATTEMPT

* * * * *

CIG©

L /H Licensing

Introduction—1

Page 3: FORWARD - WordPress.com · Return of premium p 6-8 6.0 Annuities p 7-1 Annuity principles and concepts p 7-1 Accumulation period versus annuity period p 7-1 Owner, annuitant and beneficiary

NOTE

Information in this text followed by an asterisk ( * ) has been referenced from one of the following sources:

! All About Medicare, National Underwriter, 2010

! Social Security Manual, National Underwriter, 2010

! Life Insurance, Prentice - Hall, 1997, 2002, 2004, 2006

! Group Insurance, Burton Beam, The American College, 2004

! Tax Facts, National Underwriter, 2010

! Essentials of Life, Accident and Health Insurance, National Insurance Licensing Associates,1992, 1994, 1996, 1998, 2000, 2002, 2004, 2006, 2008

! Law and the Life Insurance Contract, Irwin and Company, 1997

Copyright 2010, National Insurance Licensing Associates (NILA). All rights reserved. No portion of the specific material

protected by this copyright may be reproduced or utilized in any fashion without the express written consent of NILA.

CIG©

L /H Licensing

Introduction—2

Page 4: FORWARD - WordPress.com · Return of premium p 6-8 6.0 Annuities p 7-1 Annuity principles and concepts p 7-1 Accumulation period versus annuity period p 7-1 Owner, annuitant and beneficiary

LIFE, ACCIDENT & HEALTHEXAMINATION CONTENT OUTLINE

The following outline of topical material identifies the subject matter from which state examination questions aredeveloped for the Thomson Prometric state examinations.

The Thomson Prometric state examinations are extremely challenging. The license candidate must not only familiarizehim/herself with the basic concepts and principles of life and health insurance but MUST also comprehend the materialas well. It is important that the license candidate read through the entire text.

Life Insurance

1.0 Insurance Regulation - see Chapter 20

2.0 General Insurance2.1 Concepts

Needs analysis p 1-3Risk management key terms p 1-1

Risk p 1-1Exposure p 1-1Hazard p 1-1Peril p 1-1Loss p 1-1

Methods of handling risk p 1-1 and 2Avoidance p 1-1 and 2Retention p 1-1 and 2Sharing p 1-1 ans 2Reduction p 1-1 and 2Transfer p 1-1 and 2

Elements of insurable risk p 1-2Adverse selection p 1-3Law of large numbers p 1-2Reinsurance p 1-8Insurers p 1-6 and 7

Types of insurers p 1-7Stock companies p 1-7Mutual companies p 1-7Fraternal benefit societies p 1-7Lloyd’s associations p 1-7Risk retention groups p 1-7

Private versus government insurers p 1-8Admitted versus nonadmitted insurers p 1-7Domestic, foreign and alien insurers p 1-6 and 7Financial status (independent rating services) p 1-9Marketing (distribution) systems p 1-9

Producers p 1-10Law of agency p 1-10

Insurer as principal p 1-10Producer of insurer p 1-10

Authority and powers of producers p 1-10Express p 1-10Implied p 1-10Apparent p 1-10

Responsibilities to the applicant/insured p 1-10Contracts p 2-1Elements of a legal contract p 2-1

Offer and acceptance p 2-1

Consideration p 2-1Competent parties p 2-1Legal purpose p 2-2

Characteristics of insurance contracts p 2-2Contract of adhesion p 2-2Aleatory contract p 2-2Personal contract p 2-3Unilateral contract p 2-2Conditional contract p 2-3

Legal concepts and interpretations affecting contracts p 2-2Ambiguities in a contract of adhesion p 2-2Reasonable expectations p 2-2Indemnity p 1-1 and 12-13Utmost good faith p 2-3Representations / misrepresentations p 2-3Warranties p 2-3Concealment p 2-4Fraud p 2-4Waiver and estoppel p 2-4

3.0 Life Insurance BasicsInsurable interest p 1-2Personal uses of life insurance p 1-3

Survivor protection p 1-3Estate creation p 1-3Cash accumulation p 1-3Security p 1-3Liquidity p 1-3Estate conservation p 1-3Viatical settlements p 6-8

Determining amount of personal life insurance p 1-3Human life value approach p 1-3Needs approach p 1-3

Types of information gathered p 1-3Determining lump-sum needs p 1-4Planning for income needs p 1-4

Business uses of life insurance p 10-1Buy-sell funding p 10-2Key person p 10-1Executive bonuses p 10-3Business continuation p 10-2

Differences in life insurance policies p 4-1 & 3, p 9-7Group versus individual p 9-9Permanent versus term p 4-1 and 4-3Participating versus nonparticipating p 6-2Fixed vs variable life insurance and annuities p 4-9; 7-3, 7-6Regulation of variable products p 4-9

CIG©

Life & Health Licensing

Introduction – 3

Page 5: FORWARD - WordPress.com · Return of premium p 6-8 6.0 Annuities p 7-1 Annuity principles and concepts p 7-1 Accumulation period versus annuity period p 7-1 Owner, annuitant and beneficiary

Factors in premium determination p 1-4Mortality p 1-4Interest p 1-4Expense p 1-4

Premium conceptsNet single premium p 1-5Gross annual premium 1-5

Premium payment mode p 1-5Producer responsibilities p 1-10 and 3-1

Field underwriting p 3-1Notice of information practices p 3-1Application procedures p 3-2

Delivery p 3-5Policy review p 3-5Effective date of coverage p 3-2Premium collection p 3-2Statement of good health p 3-2

Individual underwriting by the insurer p 3-1 through 5Information sources and regulation p 3-1

Application p 3-1Producer report p 3-4Attending physician statement p 3-4Investigative consumer (inspection) report p 3-4Medical Information Bureau (MB) p 3-4Medical examination and lab tests p 3-4Selection criteria and unfair discrimination p 3-4 and 5Classification of risks p 3-4 and 5

Preferred p 3-4 and 5Standard p 3-4 and 5Substandard p 3-5Declined risks p 3-5

4.0 Life Insurance Policies p 4-1Term Life insurance p 4-1

Level term p 4-1Annual renewable term p 4-2Convertible term p 4-2Level premium term p 4-1

Increasing and decreasing term p 6-8 and 4-1Whole life insurance p 4-3

Continuous premium (straight life) p 4-3Limited payment p 4-4Single premium p 4-4Modified life p 4-4 and 5Graded premium p 4-4 and 5Interest sensitive p 4-8Equity indexed p 4-6

Flexible premium policies p 4-7Adjustable life p 4-7Universal life p 4-8

Specialized policies p 4-10Joint life (first-to-die) p 4-11Survivorship life (second-to-die) p 4-11Juvenile life p 4-11

Group life insurance p 9-7Characteristics of group plans p 9-7Types of plan sponsors p 9-8 Group underwriting requirements p 9-7Conversion to individual policy p 9-8

Credit life insurance (individual versus group) p 4-12

5.0 Life Ins. Policy Provisions, Options and Riders p 5-1Required provisions p 5-1

Ownership p 5-2Assignment p 5-2Entire contract p 5-2Modifications p 5-3Right to examine (free look) p 5-3Payment of premiums p 5-1Grace period p 5-1Reinstatement p 5-1Incontestability p 5-1Misstatement of age p 5-3Exclusions p 5-4Statements of the applicant p 5-7Interest on proceeds p 5-8

Beneficiaries p 5-4 through 6Designation options p 5-4 and 5

Individuals p 5-5Classes (per stirpes / per capita) p 5-6Estate p 5-6Minors p 5-6Trusts p 5-6

Succession p 5-5Facility of payment clause p 5-7Revocable versus irrevocable p 5-6Common disaster clause p 5-6Spendthrift clause p 5-6

Settlement options p 6-4Cash payment p 6-4Interest only p 6-5Fixed-period installments p 6-4Fixed-amount installments p 6-4Life income p 6-5

Single life p 6-5Joint and survivor p 6-5

Nonforfeiture options p 6-1Cash surrender value p 6-1Extended term p 6-2Reduced paid-up insurance p 6-2

Policy loan and withdrawal options p 5-8Cash loans p 5-8Automatic premium loans p 5-9Withdrawals or partial surrenders p 5-8

Dividend options p 6-2Cash payment p 6-3Reduction of premium payments p 6-3Accumulation at interest p 6-3One-year term options p 6-3Paid-up additions p 6-3

Disability riders p 6-6Waiver of premium p 6-5Waiver of cost of insurance p 6-5Disability income benefit p 6-6Payor benefit life/disability (juvenile insurance) p 6-8

Accelerated (living) benefit provisions/riders p 6-6Conditions for payment p 6-6Effect on death benefit p 6-6

Riders covering additional insureds p 6-6Spouse/other-insured term rider p 6-6Children’s term rider p 6-6Family term rider p 6-6

CIG©

Life & Health Licensing

Introduction – 4

Page 6: FORWARD - WordPress.com · Return of premium p 6-8 6.0 Annuities p 7-1 Annuity principles and concepts p 7-1 Accumulation period versus annuity period p 7-1 Owner, annuitant and beneficiary

Riders affecting the death benefit amount p 6-7Accidental death p 6-7Guaranteed insurability p 6-7Cost of living p 6-7Return of premium p 6-8

6.0 Annuities p 7-1Annuity principles and concepts p 7-1

Accumulation period versus annuity period p 7-1Owner, annuitant and beneficiary p 7-1Insurance aspects of annuities p 7-1

Immediate versus deferred annuitiesSingle premium immediate annuities p 7-2Deferred annuities p 7-3

Premium payment options p 7-2Nonforfeiture p 7-6Surrender and withdrawal charges p 7-6Death benefits p 7-1 and 7-2

Annuity (benefit) payment options p 7-4Life contingency options p 7-4

Pure life vs life with guaranteed minimum p 7-4Single life versus multiple life p 7-5

Annuities certain (types) p 7-5Annuity products p 7-1

Fixed annuities p 7-3General account assets p 7-3Interest rate guarantees (minimum vs current) p 7-3Level benefit payment amount p 7-2

Equity indexed annuities p 7-7Market value adjusted annuities p 7-7

Uses of annuities p 7-6Lump-sum settlements p 7-6Qualified retirement plans p 7-6

Group versus individual annuities p 7-6Personal uses p 7-7

Individual retirement annuities (IRAs) p 7-6Tax-deferred growth p 7-7Retirement income p 7-7Education funds p 7-7

7.0 Federal Tax Considerations for Life Insurance andAnnuities p 8-1

Taxation of personal life insurance p 8-1Amounts available to policy owner p 8-1

Cash value increases p 8-1Dividends p 8-1Policy loans p 8-1Surrenders p 8-2

Amounts received by beneficiary p 8-1General rule and exceptions p 8-1Settlement options p 8-2

Values included in insured’s estate p 8-2Modified endowment contracts (MECs) p 8-3

Definition of modified endowment p 8-3Seven-pay test p 8-3Distributions p 8-3

Taxation of non-qualified annuities p 8-1 and 4Individually owned p 8-4

Accumulation phase (taxation issues related towithdrawals) p 8-4Annuity phase and the exclusion ratio p 8-4

Distributions at death p 8-4Corporate (Non-living entity) owned p 8-4

Taxation of individual retirement annuities (IRAs) p 8-5Traditional IRAs p 9-2

Contributions and deductible amounts p 9-2Premature distributions including tax issues p 9-2Annuity phase benefit payments p 8-4Values included in the annuitant’s estate p 8-3 and 4Amounts received by beneficiary p 8-2

Roth IRAs p 9-3Contributions and limits p 9-3Distributions p 9-3

Rollovers and transfers (IRAs and qualified plans) p 9-2Section 1035 exchanges p 8-5

8.0 Qualified plans p 9-1General requirements p 9-1

Federal tax considerations p 9-1Tax advantages for employers and employees p 9-1Taxation of distributions (age-related) p 9-1

Plan types, characteristics and purchasersSimplified employee pensions (SEPs) p 9-4Self-employed plans (HR 10 or Keogh plans) p 9-4Profit-sharing and 401(k) plans p 9-5SIMPLE plans p 9-6403(b) tax-sheltered annuities (TSAs) p 9-5529 plans p 9-6

Accident and Health Insurance

1.0 Insurance Regulations Same as life2.0 General insurance Same as life3.0 Health Insurance BasicsDefinitions of perils p 12-1

Accidental injury p 12-1; 14-2Sickness p 12-1; 14-2

Principal types of losses and benefits p 14-1; 15-1Loss of income from disability p 14-1Hospital and medical expense p 15-1Long-term care expense p 16-7Dental expense p 15-6

Classes of health insurance coverage p 12-10Individual, franchise, and group p 12-10Private versus government p 12-4; 12-10Limited versus comprehensive p 12-10

Limited policies p 15-7 and 8Limited benefits p 15-8Required notice to insured p 15-8Types of limited policies p 15-8

Accident-only p 15-8Specified (dread) disease p 15-8Hospital indemnity (income) p 15-8Dental insurance p 15-6Credit disability p 15-8Prescription drugs p 15-8Vision care p 15-9Medical supplements p 16-5

Common exclusions from coverage p 14-5; 15-6Producer responsibilities in individual heath insurance p 13-1

Marketing requirements p 13-1Advertising p 13-1

CIG©

Life & Health Licensing

Introduction – 5

Page 7: FORWARD - WordPress.com · Return of premium p 6-8 6.0 Annuities p 7-1 Annuity principles and concepts p 7-1 Accumulation period versus annuity period p 7-1 Owner, annuitant and beneficiary

Sales presentations p 13-1Outline of coverage p 13-1

Field underwriting p 13-1Name and purpose p 13-1Disclosure of information about individuals p 13-1Application procedures p 13-2Requirements at delivery of policy p 13-4

Common situations for errors/omissions p 12-11Individual underwriting by the insurer p 13-1

Underwriting criteria p 13-1Sources of underwriting information p 13-1

Application p 13-2Agent report p 13-2Attending physician statement p 13-2Investigative consumer report p 13-2Medical Information Bureau (MB) p 13-2Medical examinations and lab tests including HIVp 13-2 and 3

Prohibited use of generic information p 12-10, 15-16Unfair discrimination p 13-3Classification of risks p 13-4

Preferred p 13-4Standard p 13-4Substandard p 13-4Declined risks p 13-4

Consideration in replacing health insurance p 12-10Pre-existing conditions p 12-10Benefits, limitations and exclusions p 12-10Underwriting requirements p 12-11Producer liability for errors and omissions p 12-11

4.0 Individual Health Insurance Policy General Provisions p 17-1Required provisions p 17-1

Entire contract; changes p 17-2Time limit on certain defenses p 17-2Grace period p 17-1Reinstatement p 17-1Claim procedures p 17-2Physical examinations and autopsy p 17-2Legal actions p 17-2Change of beneficiary p 17-2

Optional provisions p 17-3Change of occupation p 17-3Misstatement of age p 17-3Other insurance in this insurer p 17-3Insurance with other insurers p 17-3

Expense-incurred basis p 17-3Other benefits p 17-3

Unpaid premium p 17-4Cancellation p 17-4Conformity with state statutes p 17-4Illegal occupation p 17-4Intoxicants and narcotics p 17-4

Other general provisions p 17-4Right to examine (free-look) p 17-4Insuring clause p 17-4Consideration clause p 17-5Renewability clause p 17-5

Noncancelable p 17-5Guaranteed renewable p 17-5

Conditionally renewable p 17-6Renewable at option of insurer p 17-6Nonrenewable (cancelable, term) p 17-5

Military suspense provision p 14-5

5.0 Disability Income and Related InsuranceQualifying for disability benefits p 14-1

Inability to perform duties p 14-1Own occupation p 14-1Any occupation p 14-1

Pure loss of income (income replacement contracts)p 14-1Presumptive disability p 14-1Requirement to be under physician care p 14-1

Individual disability income insurance p 14-3Basic total disability plan p 14-3

Income benefits (monthly indemnity) p 14-3Elimination and benefit periods p 14-3 and 4Waiver of premium feature p 14-3

Coordination with social insurance and Workers’Compensation benefits p 14-9

Additional monthly benefit (AMB) p 14-6Social insurance supplement (SIS) p 14-6Occupational vs nonoccupational coverage p 14-5

At-work benefits p 14-1Partial disability benefit p 14-1Residual disability benefit p 14-1

Other provisions affecting income benefits p 14-6Cost of living adjustment (COLA) rider p 14-6Future increase option (FIO) rider p 14-6Relation of earnings to insurance p 17-3

Other cash benefits p 14-6Accidental death and dismemberment p 14-6Rehabilitation benefit p 14-7Medical reimbursement benefit (nondisabling injury)p 14-7

Refund provisions p 14-7Return of premium p 14-7Cash surrender p 14-7

Exclusions p 14-5Unique aspects of individual disability underwriting p 14-3

Occupational considerations p 14-4Benefit limits p 14-3Policy issuance alternatives p 14-4

Group disability income insurance p 14-8Group versus individual plans p 14-8Short-term disability p 14-8Long-term disability p 14-8

Business disability insurance p 14-7Key person disability income p 14-8Disability buy-sell policy p 14-8Business overhead expense policy p 14-7

Social Security disability p 16-9Qualifications for benefits p 16-10Definition of disability p 16-10Waiting period p 16-10Disability income benefits p 16-10

6.0 Medical PlansMedical plan concepts p 15-1

Fee-for-service basis versus prepaid basis p 15-2

CIG©

Life & Health Licensing

Introduction – 6

Page 8: FORWARD - WordPress.com · Return of premium p 6-8 6.0 Annuities p 7-1 Annuity principles and concepts p 7-1 Accumulation period versus annuity period p 7-1 Owner, annuitant and beneficiary

Specified coverages versus comprehensive carep 15-2Benefit schedule versus usual/reasonable/customarycharges p 15-2Any provider versus limited choice of providers p 15-2Insureds versus subscriber/participants p 15-2

Types of plans p 15-1 and 2Basic hospital, basic medical, basic surgical p 15-2

Characteristics p 15-2Common limitations p 15-2Exclusions from coverage p 15-3Provisions affecting cost to insureds p 15-4

Major medical insurance (indemnity plans) p 15-3Characteristics p 15-3Common limitations p 15-3Exclusions from coverage p 15-3Provisions affecting cost to insureds p 15-4

Health Maintenance Organizations (HMOs) p 12-13General characteristics p 12-13Preventive care services p 12-13Primary care physician (PCP) p 12-13Referral (specialty) physician p 12-14Emergency care p 12-13Hospital services p 12-13Other basic services p 12-13

Preferred provider organizations (PPOs) and point-of-service (POS) plans p 12-15

General characteristics p 12-15In-network and out-of-network provider access p 12-15Primary Care Physician referral p 12-15Indemnity plan features p 12-15

High deductible health plan p 15-21Cost containment in health care delivery p 15-9

Cost saving services p 15-9Preventive care p 15-9Hospital outpatient benefits p 15-9Alternatives to hospital services p 15-9

Utilization review p 15-9Prospective review p 15-9Concurrent review p 15-10

HIPAA (Health Insurance Portability and Accountability Act) p 15-15 and 16

Medical savings accounts (MSAs) p 15-20Definition p 15-20Eligibility p 15-20Contribution limits p 15-20

Health Savings Accts/Flexible Savings Accts p.15-21 and 22

7.0 Long-Term Care (LTC) Insurance p 16-7LTC, Medicare and Medicaid compared p 16-7Eligibility for benefits p 16-7Levels of care p 16-7

Skilled care p 16-7Intermediate care p 16-7Custodial care p 16-7Home health care p 16-7Adult day care p 16-8Respite care p 16-7Assisted living p 16-7

Benefit periods p 16-8

Benefit amounts p 16-8Optional benefits p 16-8

Guarantee of insurability p 16-8Return of premium p 16-8

Qualified LTC plans p 16-8Exclusions p 16-9Underwriting considerations p 16-9Additional information regarding LTC insurance appearsin the law and regulations chapter.

8.0 Group Health and Blanket InsuranceCharacteristics of group insurance p 15-10

Group contract p 15-10Certificate of coverage p 15-10Experience rating versus community rating p 15-10

Types of eligible groups p 15-10Employment related groups p 15-10

Individual employer groups p 15-10Multiple Employer Trusts METs) or Welfare Plans(MEWAs) p 15-10Taft-Hartley Trusts p 15-10

Association (alumni, professional, other) p 15-10Customer groups (depositors, creditor-debtor, other)p 15-10Blanket customer groups (teams, passengers, andothers) p 15-10

Regulation of employer group insurance plans p 15-14Employee Retirement Income Security Act (ERISA)p 15-14

Applicability p 15-14Fiduciary responsibilities p 15-14Reporting and disclosure p 15-14

Age Discrimination in Employment Act (ADEA) p 15-14Applicability to employers and workers p 15-14Permitted reductions in insured benefits p 15-14Permitted increases in employee contributionsp 15-14Requirements for medical expense coveragep 15-14

Civil Rights Act/Pregnancy Discrimination Act p 15-15Applicability p 15-14Guidelines p 15-14

Relationship with Medicare p 15-15Medicare secondary rules p 15-15Medicare carve-outs and supplements p 15-15

Nondiscrimination rules (highly-compensated) p 15-14Types of funding and administration p 15-17 and 15-18

Conventional fully-insured plans p 15-17Modified fully-insured plans p 15-18

Premium-delay arrangements p 15-18Reserve-reduction arrangements p 15-18Retrospective-rating arrangements p 15-18

Partially self-funded plans p 15-18Stop-loss coverage p 15-18Administrative services only (ASOs) arrangementsp 15-18501(c)(9) trust p 15-19

Fully self-funded (self-administered) plans p 15-19Characteristics p 15-19Conditions suitable for self-funding p 15-19Benefits suitable for self-funding p 15-19

CIG©

Life & Health Licensing

Introduction – 7

Page 9: FORWARD - WordPress.com · Return of premium p 6-8 6.0 Annuities p 7-1 Annuity principles and concepts p 7-1 Accumulation period versus annuity period p 7-1 Owner, annuitant and beneficiary

Marketing considerations p 13-1, 15-11Advertising p 13-1Regulatory jurisdiction / place of delivery p 15-11

Employer group health insurance p 15-11Insurer underwriting criteria p 15-11

Characteristics of group p 15-11Plan design factors p 15-11Persistency factors p 15-11Administrative capability p 15-11

Eligibility for coverage p 15-11Annual open enrollment p 15-12Employee eligibility p 15-11Dependent eligibility p 15-11, 15-26

Coordination of benefits provisions p 15-12Change of insurance companies or loss of coveragep 15-12

Coinsurance and deductible carryover p 15-12No-loss no-gain p 15-12Events that terminate coverage p 15-12Extension of benefits p 15-12Continuation of coverage under COBRA p 15-12Conversion privilege p 15-12

Small employer medical plans p 15-13Definition of small employer p 15-13Benefit plans offered p 15-13Availability of coverage p 15-13Renewability p 15-13

9.0 Dental Insurance p 15-6Types of dental treatment p 15-6

Diagnostic and preventive p 15-6Restorative p 15-6Oral surgery p 15-6Endodontics p 15-7Periodontics p 15-7Prosthodontics p 15-7Orthodontics p 15-7

Indemnity plans p 15-7Choice of providers p 15-7Scheduled versus nonscheduled plans p 15-7Benefit categories p 15-7

Diagnostics/preventive services p 15-7Basic services p 15-7Major services p 15-7

Deductibles and coinsurance p 15-7Combination plans p 15-7Exclusions p 15-7Limitations p 15-7Predetermination of benefits p 15-7

Employer group dental expenses p 15-7Integrated deductibles vs stand alone plans p 15-7Minimizing adverse selection p 15-7

10.0 Government Health Insurance Plans p 14-9Workers’ Compensation p 14-9

Eligibility p 14-9Benefits p 14-9

Social Security Disability p 16-9Qualifications for disability benefits p 16-10Definition of disability p 16-10Waiting period p 16-10

Disability income benefits p 16-10Medicaid p 16-4

Eligibility p 16-4Benefits p 16-4

Medicare p 16-1Nature, financing, and administration p 16-1Part A - Hospital insurance p 16-1

Individual eligibility requirements p 16-1Enrollment p 16-1Coverages and cost-sharing amounts p 16-1 and16-2

Part B - Medical insurance p 16-3Individual eligibility requirements p 16-3Enrollment p 16-3Coverages and cost-sharing amounts p 16-3Exclusions p 16-3Claims terminology and other key terms p 16-3

Part C - Medicare Advantage p 16-3Part D - Prescription Drug Ins. P. 16-4

11.0 Private Insurance for Senior Citizens and SpecialNeeds Individuals

Medicare supplements p 16-5Purpose p 16-5Open enrollment p 16-5Standardized Medicare supplement plans p 16-5

Core benefits p 16-5Additional benefits p 16-5

Other Medicare options for individuals p 16-2Disabled individuals p 16-2Individuals with kidney failure p 16-2Employer group health plans p 15-11

Employees age 65 and older p 15-11, 14See additional Medicare Supplement information in thelaw and regulations chapter.

12.0 Federal Tax Considerations for Health Insurancep 14-10

Personally owned health insurance p 14-10Disability income insurance p 14-10Medical expense insurance p 14-10Long-term care insurance p 14-10

Employer group health insurance p 14-10Disability income (STD, LTD) p 14-10Benefits subject to FICA 14-10Medical and dental expense p 14-10Long-term care insurance p 14-10Accidental death and dismemberment p 14-10

Medical expense coverage for sole proprietors and partnersand limited liability corps p 14-10Business disability insurance p 14-7 and 14-10

Key person disability income p 14-8 and 14-10Buy-sell policy p 14-8 and 10

Medical savings accounts (MSAs) p 15-19 and 20Health savings accounts (HSAs) p. 15-21Flex spending accounts p. 15-22

CIG©

Life & Health Licensing

Introduction – 8

Page 10: FORWARD - WordPress.com · Return of premium p 6-8 6.0 Annuities p 7-1 Annuity principles and concepts p 7-1 Accumulation period versus annuity period p 7-1 Owner, annuitant and beneficiary

SECTION I — LIFE INSURANCE BASICS

Chapter 1. Introduction to Life Insurance 1-1Chapter 2. Law of Contracts and Other Contract Features 2-1

Chapter 3. Field and Home Office Underwriting and Policy Delivery 3-1

SECTION II — LIFE INSURANCE POLICYPROVISIONS, OPTIONS AND RIDERS

Chapter 4. Life Insurance Policies 4-1Chapter 5. Life Insurance Policy Provisions 5-1

Chapter 6. Life Insurance Policy Options and Benefit Riders 6-1

SECTION III — ADDITIONAL LIFE INSURANCEPRODUCTS AND CONCEPTS

Chapter 7. Annuities 7-1Chapter 8. Tax Treatment of Life Insurance and Annuities 8-1

Chapter 9. Qualified Retirement Plans and Group Life Insurance 9-1Chapter 10. Business Uses of Life Insurance 10-1

Chapter 11. Life Insurance Sample Exam 11-1

NILA ©

Life & Health Licensing

Page 11: FORWARD - WordPress.com · Return of premium p 6-8 6.0 Annuities p 7-1 Annuity principles and concepts p 7-1 Accumulation period versus annuity period p 7-1 Owner, annuitant and beneficiary

SECTION IV — ACCIDENT & HEALTHINSURANCE BASICS

Chapter 12. Accident & Health Insurance Basic Concepts and Principles 12-1Chapter 13. Marketing, Underwriting and Policy Delivery 13-1

SECTION V — ACCIDENT & HEALTH PRODUCTS

Chapter 14. Disability Income Insurance 14-1Chapter 15. Individual and Group Medical Expense Policies 15-1

SECTION VI — SOCIAL SECURITY AND OTHERACCIDENT & HEALTH CONCEPTS

Chapter 16. Insurance for Senior Citizens and Special Needs Individuals 16-1Chapter 17. Accident & Health Policy Provisions 17-1

Chapter 18. Accident & Health Sample Examination 18-1Chapter 19. Glossary of Life and Health Terms 19-1

SECTION VII — STATE INSURANCE LAW

Chapter 20. State Insurance Law 20-1

NILA ©

Life & Health Licensing

Page 12: FORWARD - WordPress.com · Return of premium p 6-8 6.0 Annuities p 7-1 Annuity principles and concepts p 7-1 Accumulation period versus annuity period p 7-1 Owner, annuitant and beneficiary

SECTION I

LifeInsurance

Basics

Page 13: FORWARD - WordPress.com · Return of premium p 6-8 6.0 Annuities p 7-1 Annuity principles and concepts p 7-1 Accumulation period versus annuity period p 7-1 Owner, annuitant and beneficiary

INTRODUCTIONTO LIFE

INSURANCE

Page 14: FORWARD - WordPress.com · Return of premium p 6-8 6.0 Annuities p 7-1 Annuity principles and concepts p 7-1 Accumulation period versus annuity period p 7-1 Owner, annuitant and beneficiary

INTRODUCTION — Life insurance has been recognized for more than a century as an essential elementin an individual's or family's financial planning program. A financial planning program should include thegeneral and specific financial goals of the individual or family and a plan to achieve those objectives. Oneof the primary tools used to assist in meeting these objectives is a sound life insurance program. This chapterwill review the basic definition of life insurance and the legal aspects of insurance contracts.

INTRODUCTION TO LIFE INSURANCE

CONCEPT OF INSURANCE

DEFINITION OF INSURANCE — The concept of insurance, in generic terms, may be defined as the transferof risk from one party to another through a legal contract; or the transfer of risk through the pooling or accumulationof funds. This means that one person (i.e. the applicant / policy owner) is transferring the chance of a possiblefinancial loss to another party (i.e. the insurer) while the insurer assumes the risk since it receives a premium. Thetransfer is accomplished through an insurance contract between two parties. The two parties, a policy owner andinsurer, ultimately arrive at an agreement for insurance protection. Insurance helps to reduce the financial uncertaintyof the policy owner with regard to possible future losses. Insurance spreads the risk of loss from one person to a largenumber of persons through the pooling of premiums. In other words, insurance reduces financial risk and spreadsthe risk of loss from one individual to many. When the transfer of risk is accomplished by purchasing an insurancepolicy, the policy owner obtains a large quantity of coverage in return for a small fee (i.e. premium). Insurancepolicies are contracts of indemnity. A contract of indemnity specifies that insurance should restore an individualto the same or a similar financial position in which they existed prior to the loss.

The concept of risk may be identified as the uncertainty or chance of financial loss. A pure risk is the only typethat may be insured since it involves a chance of loss only, whereas a speculative risk (i.e., placing a wager on theSuper Bowl) provides the opportunity for loss or gain. A speculative risk may not be insured since it possesses thechance for loss or financial gain. In other words, speculative risk involves an opportunity for gain that could resultin loss. Loss is defined as the decrease or disappearance of economic value. Again, by purchasing insurance, aninsured's financial uncertainty with regard to a possible loss is reduced.

RISK MANAGEMENT / METHODS OF HANDLING RISK — This is a process used by an insurer oftreating and managing loss exposures. An exposure is defined as a hazardous condition brought about by the natureof an insured’s activities. Risk management may be accomplished by: (1) detecting the potential loss exposure; (2)selecting a method or tool to reduce risk; (3) executing a course of action, and (4) periodically reviewing the measurestaken. Risk may be reduced or managed by purchasing an insurance contract. There are many methods of riskmanagement available to handle risk such as avoidance, retention, sharing, reduction and transfer (please referto page 1-13). The primary peril or hazard covered by a life insurance policy is premature death. A peril is thecause of a loss while a hazard is a condition present that increases the chance of a loss occurring . There are threetypes of hazards including physical hazards (i.e., a slippery floor, icy steps, faulty wiring, leaving matches withinthe reach of children, etc.), moral hazards (i.e., dishonest actions by an policy owner / insured), and morale hazards(i.e. driving under the influence, apathetic attitudes).

NILA ©L/H LicensingChapter 1—1

Page 15: FORWARD - WordPress.com · Return of premium p 6-8 6.0 Annuities p 7-1 Annuity principles and concepts p 7-1 Accumulation period versus annuity period p 7-1 Owner, annuitant and beneficiary

ELEMENTS OF INSURABLE RISK — It is not possible to insure every type of risk. In order for a pure riskto be insurable it must involve a chance of loss that is accidental, measurable and definable. In addition, the lawof large numbers must also apply. This is a mathematical law of probability that states that the larger the numberof occurrences (i.e., the number of lives covered), the more predictable losses will be. As the number of exposuresincrease, the more the actual results will approach the results expected for a specific event. In other words, the largerthe number of homogeneous units (i.e., similar risks), loss predictability will increase. Other elements of insurablerisk include the fact that: the loss must be large enough to cause a financial hardship (i.e., death); the loss must notbe catastrophic (i.e., loss due to war); the cost for coverage (i.e., the premium) must not be unreasonable; and theloss must be accidental. Again, methods available to manage risk include but are not limited to: (1) avoidance; (2)retention (i.e., a deductible); (3) reduction; (4) sharing; or (5) transfer. The most common way to manage risk iswhen an individual transfers risk to an insurer by purchasing an insurance policy. The concept of insurance mayalso be referred to as an economic or social device used for the transfer of risk from one party to another through theaccumulation or pooling of funds (i.e., premiums).

LIFE INSURANCE

DEFINITION OF LIFE INSURANCE — Life insurance involves the transfer of the risk of premature deathfrom one party (i.e., policy owner/insured) to another party (i.e., insurer). When a life insurance contract is payableupon the death of the insured, it instantly creates funds for a named beneficiary. In other words, a life insurancecontract instantly or immediately creates an estate. The word "estate" means value, money, funds, capital or assets. Therefore, life insurance involves a method of estate creation.

O PARTIES INVOLVED IN THE LIFE INSURANCE CONTRACT — There are two partiesinvolved in any insurance contract. The policyowner is responsible for the payment of premiums andpossesses all ownership rights of the contract. These rights will be reviewed later. The insurer issues thepolicy once the application is approved. The action taken by the insurer when the policy is issued anddelivered indicates that the offer made by the applicant/future policy owner has been accepted by the insurer.When a policy owner is different from the insured party, third party ownership is in existence. In otherwords, when a policy owner is a different person than the individual who is insured, third party ownershipexists. For example, a wife owning a policy covering her husband or a business owning life insurance ona key employee are common illustrations of third party ownership.

O COSTS ASSOCIATED WITH DEATH — Several financial or post mortem burdens may exist whena person dies. Funeral and burial expenses are most obvious. However, there are numerous additionallump sum needs or costs associated with death such as possible estate tax liability; possible state inheritance(death) tax liability; current debt obligations at the time of death that need to be paid so they will not bepassed on to survivors or heirs; outstanding medical expenses incurred prior to death; attorney or probatefees or other costs associated with estate administration; and other lump sum needs to provide for emergencyreserve funds, education funds, retirement funds and other types of bequests made through the deceased'slast will and testament.

O INSURABLE INTEREST — An individual may not purchase life insurance covering the life of anotherperson unless he or she possesses an insurable interest in that person. This involves a financial or economicinterest in the person to be covered. This doctrine or rule was designed to prevent one person from insuringanother and then murdering the insured in order to collect policy proceeds. Insurable interest must existin life insurance at the time of application whereas in property insurance it is required to exist at the timeof loss. In some instances, the insurable interest may not exist at the time of application but must exist, atleast, when the policy is issued. In addition, with a few exceptions (i.e., buying insurance on a minor), aperson may not purchase life insurance on another without his or her consent. Anyone purchasing life

NILA ©L/H LicensingChapter 1—2

Page 16: FORWARD - WordPress.com · Return of premium p 6-8 6.0 Annuities p 7-1 Annuity principles and concepts p 7-1 Accumulation period versus annuity period p 7-1 Owner, annuitant and beneficiary

insurance on his or her own life possesses an unrestricted or unlimited insurable interest in himself orherself. Insurable interest also exists automatically in marital relationships, between parents and children,in a business situation between a business and a key employee, or in a debtor- creditor relationship. Anindividual would possess an insurable interest in a nephew or niece if the latter lived in the individual'shousehold and he or she was their guardian. Again, the basic requirement which must exist when lifeinsurance is purchased on another's life is insurable interest. In addition, once in effect, a policy owner maycontinue to pay premiums on a policy covering another even if insurable interest no longer exists (i.e.,divorce, dissolution of a partnership, etc.). This means that insurable interest need not exist at the time ofloss (i.e., death).

O ADVERSE SELECTION — The concept of adverse selection is paramount to a life insurer. This is aprocess that involves the danger an insurer is exposed to when approving more bad risks than good ones sincethere is a tendency for poorer risks to seek insurance. It may also occur when an applicant for insurancehas a greater bias to an exposure than a random sampling of the population. When an applicant who is inpoor health applies for life insurance and the insurer's underwriters allow the application to somehow beapproved, the insurer will be adversely affected. In other words, if the underwriting department of an insurerdoes not select risks according to insurer guidelines, this action (or lack thereof) may adversely affect thefinancial condition of the insurer. Therefore, sound and competent underwriting may reduce the chancefor adverse selection. All insurers attempt to reduce or avoid adverse selection by thoroughly underwritingall individual risks submitted. Even some policy provisions found in a policy attempt to reduce adverseselection after underwriting such as the suicide clause, the incontestable clause or in health insurance, thepre-existing condition limitation/provision. War and aviation exclusions also assist in avoiding adverseselection as well. In group insurance situations, the larger the number of participants, the less the chance foradverse selection.

O PERSONAL USES OF LIFE INSURANCE — Life insurance provides for the immediate creationof funds payable to a beneficiary when an insured person dies. We all purchase life insurance for a varietyof reasons including but not limited to: (1)for final expenses; (2) estate protection and conservation; (3)funds for survivor protection and security; (4) education expenses; (5) to provide funds to pay off a debt;(6) to supplement retirement income; (7) charitable contributions; (8) in business situations (i.e., to funda buy-sell agreement or key employee insurance); and (9) to accumulate cash and for liquidity purposes. These personal uses of life insurance are similar to and related to the costs associated with death that werepreviously reviewed. Therefore, life insurance may be used for survivor protection, estate creation, cashaccumulation, liquidity and estate conservation especially since it is utilized many times to pay off estatetaxes that are incurred.

DETERMINING LIFE INSURANCE NEEDS

Traditionally there are two primary approaches that can be used to determine how much life insurance an individualor family needs. The human life value approach is a capitalized value of an individual’s net future earnings. In otherwords, it looks at the potential lost earnings of a person as a measure of how much insurance to purchase. When aperson dies prematurely his or her future earning capacity has obviously come to an end. Therefore, to determinehow much life insurance is needed to protect this individual's dependents, we may multiply projected earned incomeper year by the number of years until retirement. Generally, the present value of the individual's projected earningsless expenses (i.e., income taxes and cost of living) are multiplied by the years to retirement. This provides anapproximate coverage amount that is needed. Therefore, determining the value of an individual's earning potentialover a period of time is known as the human life value approach.

NILA ©L/H LicensingChapter 1—3

Page 17: FORWARD - WordPress.com · Return of premium p 6-8 6.0 Annuities p 7-1 Annuity principles and concepts p 7-1 Accumulation period versus annuity period p 7-1 Owner, annuitant and beneficiary

The needs approach is used in cases where the amount of life insurance needed is based upon the individual's (orfamily's) financial goals and objectives. Therefore, education fund goals, emergency funds, bequests, charitablegifting or retirement income goals of a spouse will influence the amount of coverage needed. This indicates thatthe ages of family members, the wages of family members and the health history of all parties concerned should bereviewed. In addition, purchasing life insurance as a charitable gift also has its tax advantages. For instance, if theowner of a policy transfers all or a part of an existing whole life policy to a charitable organization, he or she willreceive an income tax deduction based on the cash value of the policy at the time of the transfer. In addition, if a newpolicy is purchased and the charity is named owner and beneficiary, future premium payments made by the purchaserare tax deductible (i.e. tax deductible gift). The needs approach will focus on determining lump sum needs and willutilize all the costs associated with death (i.e. post mortem costs) plus financial objectives in order to arrive at aperson's or family's total capital needs. Once this is done, the liquid assets of the person should be calculated whichincludes savings, pension or profit sharing benefits, life insurance proceeds and any other income the person isentitled to such as Social Security retirement income, interest from bonds or other investments, dividends frommutual funds or stock, or rental income. It is especially important to consider Social Security since no retirementincome will be provided to survivors during the so-called "blackout period." This is the period of time from theinsured's death until the surviving spouse is permitted to receive retirement income benefits. However, benefits areprovided for other dependents (i.e., children) during the blackout period until the youngest child reaches ageeighteen (18). By subtracting liquid assets from total capital needs, the individual will arrive at the approximateamount of life insurance "needed."

Planning for income needs of survivors is extremely important. The planning process involves: (1) informationgathering including personal information (i.e., ages, health history) and financial information such as wages,personal assets, investments and earnings, pension plans and savings; (2) identifying and prioritizing the client’sobjectives; (3) analyzing the client’s current financial condition; (4) developing and implementing a plan; and (5)periodically reviewing the plan. Life insurance proceeds many times will be used to replace the salary or the lostservices of the deceased. The producer must also aid the proposed insured and family in determining the properamount of life insurance when considering what amount of capital should be retained or be available at death andif these available funds will be sufficient enough to protect against a forced liquidation of property. Either of theseapproaches may be utilized successfully although the human life value approach does not consider those who receivefinancial benefit from the individual's continued life. To learn more about the human life value and needs approacheswith regard to life insurance refer to "Estimating Human Life Values," a text by Philip Eden (TechnipressInternational). There are other methods available that may be used to determine the amount of life insurance neededincluding: a multiple of earnings such as five times a person’s annual salary; the interest only method where it isdetermined how much insurance is needed to maintain after-tax family consumption levels if the principle is held bythe insurer for future payments; the single needs method which identifies the amount of insurance needed based upona specific need (i.e., loan or debt, education fund, death taxes, etc.); the “seat of the pants” method where an amountis arbitrarily selected; and the capital needs analysis. This last method involves determining the immediate cashneeds of an individual or family such as final expenses, medical expenses associated with death, probate costs, costof living expenses, debt elimination, an emergency fund, education funds and Federal and State death taxes whichmust be paid within six months of the death; and continuing income needs such as readjustment income, dependenceperiod income, life income for a survivor and retirement income.

DETERMINING LIFE INSURANCE PREMIUMS — There are three primary factors or elements utilizedto aid in determining what an insurer will charge for its life insurance product. The three factors that influence thegross premiums charged for life insurance are: (1) mortality; (2) interest; and (3) expenses. The mortality factoris determined from a mortality table which provides an indication of the "probability of death" of an individual ata particular age. In other words, the mortality table provides us with the death rate. This mortality factor is generallydetermined from a Commissioners Standard Ordinary Mortality table (1980, 2001 or 2004 table). This is thegeneric table used by most insurers today, although some insurance companies utilize mortality factors derived fromtheir own experience (i.e., death claims paid). The interest factor is a reflection of an insurer's return on its

NILA ©L/H LicensingChapter 1—4

Page 18: FORWARD - WordPress.com · Return of premium p 6-8 6.0 Annuities p 7-1 Annuity principles and concepts p 7-1 Accumulation period versus annuity period p 7-1 Owner, annuitant and beneficiary

investments. Insurers invest the premiums they collect in a myriad of investment vehicles. The wiser theirinvestment decisions, the better return they will realize. The expense factor is derived from the funds the insurer"pays out". These expenses include but are not limited to: (1) death benefits paid; (2) commissions or compensationto producers and other employees (i.e., officers, managers or clerical staff); and (3) other administrative costs (i.e.,rent).

The gross annual premium charged by an insurer is the one comprised of or influenced by the aforementionedfactors of mortality, interest and expenses. A net single premium is one that makes provision for mortality (deathbenefit) losses only, while being influenced by the interest rate assumed, gender, benefit to be provided and themortality rate. When an amount for expenses is added to the net premium, the gross premium has been determined. A final premium will be arrived at when an insurer considers other factors such as the applicant's health history, hisor her age and even the premium payment mode used to pay premiums. The mode utilized by the policy ownerallows the insurer to assess an extra charge if premiums are paid quarterly, semi-annually or monthly (i.e., anythingother than on an annual basis).

QUIZ

1. All of the following influence the premiums charged for life insurance policies, EXCEPT?

A. Mortality C. Investment returnB. Agent commissions D. Insurance commissioner

2. Which of the following spreads the risk of loss from one person to a large number of people through the pooling ofcollected premiums.

A. Transference C. Pure riskB. Insurance D. Risk management

3. Insurance aids in reducing uncertainty with regard to possible financial losses. Which of the following best identifiesthe uncertainty concerning loss or chance of loss?

A. Hazard C. Law of large numbersB. Risk D. Retention

4. Which of the following best defines the concept known as life insurance?

A. The liquidation of funds C. The immediate creation of fundsB. The investment of funds D. The financial interest in another's life

5. An insurance contract is an agreement between two parties. Which of the following are the parties involved in a lifeinsurance contract?

A. Policy owner and beneficiary C. Insured and beneficiaryB. Insurer and policy owner D. Insurer and beneficiary

6. Several financial burdens exist upon the death of an individual. Each of the following is a cost associated with death,EXCEPT:

A. Burial expenses C. Indemnification expensesB. Estate taxes D. Debt obligations

NILA ©L/H LicensingChapter 1—5

Page 19: FORWARD - WordPress.com · Return of premium p 6-8 6.0 Annuities p 7-1 Annuity principles and concepts p 7-1 Accumulation period versus annuity period p 7-1 Owner, annuitant and beneficiary

7. In order for one person to purchase life insurance on the life of another a valid insurable interest must be present. When

must insurable interest exist in life insurance?

A. At the time of loss C. When death occurs

B. At the time of application D. At the time of underwriting

8. Which of the following help to reduce or avoid adverse selection?

A. Speculative risk C. Human life values

B. Investment income D. Sound underwriting

9. Which of the following approaches used to determine the appropriate amount of life insurance involves an individual's

future earning capacity?

A. Needs approach C. Projection approach

B. Human life value approach D. Financial objectives approach

10. A net single premium primarily looks to which of the following ?

A. Mortality, interest and expenses C. Mortality and expenses

B. Interest and expenses D. Death benefit

ANSWERS 1. D 5. B 9. B2. B 6. C 10. D3. B 7. B 4. C 8. D

TYPES OF INSURERS

ACCORDING TO DOMICILE — A domestic insurer is one which has its principal or home office in a statewhere it is authorized. In other words, a domestic insurer is chartered in or formed under the laws of the State whereit is authorized. For instance, Nationwide Insurance Company of Columbus, Ohio operates as a domestic insurer inthe State of Ohio. A foreign insurer is one that is authorized in one State but its charter or principal office is inanother State. Nationwide Insurance Company is also licensed or authorized in the State of Illinois. Therefore, itoperates throughout the State of Illinois as a foreign insurer. An alien insurer is one which is authorized in any Statewithin the U.S. but its principal office is located outside this country. Nippon Life of Tokyo, Japan or Sun FinancialServices of Toronto, Canada are examples of alien insurers.

ACCORDING TO OWNERSHIP — A mutual company is one which is “owned” by its policyholders and isincorporated without capital stock. A mutual company issues participating life insurance policies and may allowall policyholders to share (i.e., participate) in its surplus in any particular year by receiving a dividend. A stockcompany is one owned by its stockholders. Any profits realized by the stock company are distributed to theshareholders or stockholders. Stock companies generally do not pay dividends to policy holders. Therefore, amutual company and a stock company differ according to ownership. The two major insurer classifications in theprivate sector are stock and mutual insurers.

ACCORDING TO AUTHORIZATION — An insurer which is allowed or authorized to conduct insurancebusiness in a particular State is referred to as an authorized insurer. Also known as admitted insurers, an authorized

NILA ©L/H LicensingChapter 1—6

Page 20: FORWARD - WordPress.com · Return of premium p 6-8 6.0 Annuities p 7-1 Annuity principles and concepts p 7-1 Accumulation period versus annuity period p 7-1 Owner, annuitant and beneficiary

company is issued a certificate of authority. An unauthorized insurer, also referred to as a nonadmitted company,is not permitted nor allowed to conduct insurance operations in a particular State.

OTHER INSURER CLASSIFICATIONS — There are several additional classifications of insurers includingbut not limited to: (1) Fraternal associations are benevolent societies or social organizations which provideinsurance only for their members. These organizations operate on a lodge system and due to their charitable naturethey are exempt from federal income and state taxes. Organizations such as the Independent Order of Foresters orthe Knights of Columbus are examples of a fraternal; (2) A reciprocal is an unincorporated organization in whichall members insure one another. Reciprocals are managed by an attorney-in-fact (i.e., manager) who attempts tosecure additional subscribers for the organization. Other duties of the attorney-in-fact include collecting premiumsfrom members, administration and adjusting losses; (3) Self-insuring is a method used where an individual orbusiness assumes the total risk of possible losses. An individual may choose to self-insure, or in the case of WorkersCompensation, permission must be granted by a State authority and proof of the ability to self-insure must also beprovided (i.e., financial statements). Self insurance groups are not for profit unincorporated associations generallyconsisting of five or more private or public employers who are engaged in the same trade or professional associationwhich has not been in existence for less than five years, and who enter into agreements to pool their liabilities forWorkers Compensation and other specified forms of liability insurance; (4) A Lloyds organization is a group ofindividuals who share risk or underwriting exposures. The most recognized Lloyds organization is Lloyds ofLondon. This group is comprised of individual underwriters of a particular risk. If a loss occurs, they share in theexpenses (i.e., pay-out). These associations also function in this country. "American Lloyds" organizations functionin a similar fashion to their British counterpart; (5) risk retention groups provide product liability coverage tobusinesses that make up the group; and (6) surplus lines insurance is available to those who need protection but itis not available through private or commercial carriers. "Surplus lines" refers to the nontraditional insurance market.A person will seek coverage through a surplus lines broker in order to secure coverage for substandard or unusualrisks (i.e., hole in one insurance or nonappearance coverage). In order to qualify for surplus lines coverage, an efforthas to be made to secure coverage in the authorized market. An individual may not attempt to secure coverage justbecause it may be less expensive. Surplus lines brokers must also maintain adequate documentation of all transactionsand a list of the standard insurers who have rejected coverage. A surplus lines broker is also responsible for collectinga premium tax from the insured in return for placing this business and must pass this fee along to the appropriate Stateregulatory authorities.

REINSURANCE — This is a device which helps an insurer avoid catastrophic losses. Reinsurance is actuallyinsurance from insurers. In a reinsurance transaction, the insurer seeking reinsurance is known as the ceding insurer. The insurer assuming the risk is known as the reinsurer. The portion of the risk that the ceding insurer retains iscalled the net retention (or net line). Transferring the risk to the reinsurer is called "ceding" while the risk itself isreferred to as cession. For example, assume that Maine Mutual writes property insurance. Maine Mutual is a smallinsurer and cannot afford to cover any particular risk for more than $1 million. If an applicant seeks coverage of $5million, Maine Mutual will retain coverage up to its line limit (i.e., $1 million), and will "cede" the excess $4 millionof protection to Richmond Reinsurance Company. Therefore, Maine Mutual can continue to provide protection forits client but it does not assume the entire risk. Reinsurance can be defined as the transfer of risk from one insurancecompany to another. There are two basic types of reinsurance including:

O FACULTATIVE — Under a facultative agreement, the risks are considered individually by both parties (i.e., ceding company and reinsurer). In other words, the ceding insurer may submit the risk to the reinsurerand the latter may either accept or reject it on a case by case basis.

O AUTOMATIC — Under an automatic or treaty agreement, the reinsurer agrees, in advance, toautomatically accept or assume a portion of a risk written by the ceding insurer. Conversely, the cedinginsurer has agreed to "cede" a portion of the risk automatically to the reinsurer.

NILA ©L/H LicensingChapter 1—7

Page 21: FORWARD - WordPress.com · Return of premium p 6-8 6.0 Annuities p 7-1 Annuity principles and concepts p 7-1 Accumulation period versus annuity period p 7-1 Owner, annuitant and beneficiary

O OTHER FORMS — A quota share agreement involves a ceding insurer agreeing to pay a reinsurer aspecific percentage (i.e., 40%) of the premium collected if the reinsurer agrees to pay the same percentage(i.e., 40%) of any loss that occurs. An excess loss agreement involves the reinsurer agreeing to pay onlywhen a loss exceeds a specific amount.

PRIVATE VERSUS GOVERNMENT INSURERS — Private or commercial insurance companies are ownedby private citizens or groups. They may be proprietary or cooperative. An example of a proprietary insurer wouldbe a profit motivated stock company. Private insurers offer individual, group, industrial or blanket insurance policies. Government insurers are owned and operated by a Federal or State entity. Government insurers may either: (1) writeinsurance to cover perils that are not insurable by commercial insurers (i.e., war, flood, nuclear reaction) or (2) writeinsurance on risks that are insurable and thus compete with the commercial marketplace (i.e., Social Insurance suchas Workers' Compensation). Government insurers generally write insurance to cover catastrophic perils (i.e., flood)or to protect a segment of society (i.e., the elderly) against catastrophic medical costs (i.e., Medicare).

The Federal Government provides life insurance coverage for those in the armed services and other federalemployees. Serviceman's Group Life Insurance (SGLI) is provided up to $400,000 for full time members of thearmed services. The coverage provided is group term life insurance and all active members are covered unless theychoose otherwise. Federal Employees Group Life Insurance (FEGLI) provides group term life insurance for all otherfederal employees or civil service workers.

FEDERAL REGULATIONS PERTINENT TO INSURANCE — Federal Regulation 18 USC 1033 and 1034stipulates that anyone who is engaged in the business of insurance whose activities affect interstate commerce andknowingly, with the intent to deceive, makes any false material statement or report, may be subject to a monetarypenalty and imprisonment. Interstate commerce means any business conducted within the District of Columbia, anyState within the U.S. or any of its territories or possessions. Any violation of this federal law will subject anindividual to a monetary fine of up to $50,000, or imprisonment for up to ten (10) years, or both. In addition, if thematerial misrepresentation jeopardized the safety and soundness of an insurer and was a significant cause of theinsurer being placed in conservation, rehabilitation, insolvency or liquidation, the agent making the falsestatements may be subject to imprisonment of not more than fifteen (15) years. In other words, if the insurer’ssolvency is threatened due to the material misrepresentations of a licensee, a prison sentence of up to 15 years maybe assessed the guilty individual.

The term “interstate commerce” means: (1) commerce within the District of Columbia, or any territory or possessionof the United States; (2) all commerce between any point in the State, territory, possession, or the District ofColumbia and any point outside thereof; (3) all commerce between points within the same State through any placeoutside such State; or (4) all other commerce over which the United States has jurisdiction. The term “State”includes any U. S. State, the District of Columbia, the Commonwealth of Puerto Rico, the Northern Mariana Islands,the Virgin Islands, American Samoa, and the Trust Territory of the Pacific Islands.

In addition, USC 18 1033 also describes the Violent Crime Control Act. It provides criminal and civil enforcementprovisions for insurance fraud committed by persons in the insurance industry. The main aspect of this Act involvesthe fact that it is illegal for an individual convicted of a crime involving dishonesty, breach of trust or fiduciaryresponsibilities or a violation of the Act to work or continue to work in the insurance business without receivingwritten consent from an insurance regulatory official (i.e., Commissioner or Superintendent of Insurance). ThisAct contains no “grandfather provision” for persons already employed by, licensed in or transacting the business ofinsurance. Persons who fail to comply with this Act face federal fines and imprisonment. INSURER FINANCIAL STATUS AND RATING SERVICES — The primary objective of all Statesthroughout the country is to regulate insurance companies in order to protect the insurance buying public. State

NILA ©L/H LicensingChapter 1—8

Page 22: FORWARD - WordPress.com · Return of premium p 6-8 6.0 Annuities p 7-1 Annuity principles and concepts p 7-1 Accumulation period versus annuity period p 7-1 Owner, annuitant and beneficiary

insurance departments regularly examine (i.e., audit) insurers to ascertain their financial stability. In addition toinsurance regulatory authorities, there are numerous independent rating services which evaluate the financial stabilityof insurers including but not limited to:

! A.M. Best Review (A++ to F)! Moody's Investment (Exceptional to Lowest)! Standard and Poor's (AAA to D)! Weiss Research (Excellent to Very Weak)

These rating services, in the past, may not have included as much data on an insurer's "risky investments" as it shouldhave. Most of these rating services analyze criteria such as expense ratios of insurers (i.e., death claims paid), thereserve adequacy of insurers and the insurer's product mix. Criteria such as an insurer's personnel practices are nottaken into consideration when determining its financial strength. Today, the organization that appears to be the mostdiscerning insurer rating service is Weiss Research.

INSURER MARKETING AND DISTRIBUTION SYSTEMS — There are four principal marketing systemsused to distribute insurance products to the public. They include (1) the independent agency system which placesbusiness with an assortment of insurers. These independent agents own and control this business. In other words,independent agents own the renewals or expirations of the business; (2) direct writers are insurers who employcaptive agents that sell the products of the one individual direct writing insurer they represent. The insurer owns therenewals or expirations. A captive agent is bound only to place insurance business with the insurer who sponsorshim or her; (3) the exclusive agency system is a distribution system made up of independent contractors who agreeto market products of one exclusive insurer; (4) direct mail or direct response systems which market their productby phone, mail or through the media (i.e., newspaper, T.V., radio). The initial three systems are the principal onesinvolved in the marketing of insurance products.

Insurers manage their distribution systems through the use of the branch office system or the general agency system. These management systems generally provide technical support and evaluate the production of its sales force. Thebranch office system is made up of insurer employees including a branch manager, sales staff and clerical employees. The general agency (GA) system may be of the managing general agent (MGA) type or the personal producing type(PPGA). The MGA generally appoints (i.e., hires) and supervises producers while also performing some servicework such as underwriting and claims. A PPGA does not appoint producers and is generally a personal producer.

PRODUCERS AND PRODUCER AUTHORITY

This moniker refers to the wide range of individuals who solicit insurance products to the public. The various typesof producers include: (1) Agents who represent the insurer which sponsors them; (2) Brokers who representthemselves and the insured (i.e., the client or customer); (3) Solicitors who represent and solicit insurance on behalfof an agent; and (4) Service Representatives who are employees of an insurer. Service representatives who do notengage in sales activities which pay commissions need not be licensed. If commissions are paid, the recipient mustpossess an insurance license. In some locales any person who solicits or countersigns policies or collect premiumsfrom policy owners must be licensed.

CONCEPT (LAW) OF AGENCY — This is the legal relationship between two parties when one of the partiesacts on behalf of another. This concept does not just apply to the insurance field. However, it may be applied to theinsurance business to illustrate the relationship between an insurer and a producer. For example, this legalrelationship exists when a producer (i.e., an agent) represents an insurer. The insurer is referred to as the principal. The producer may be referred to as the agent or authorized representative. When the producer / agent acts within

NILA ©L/H LicensingChapter 1—9

Page 23: FORWARD - WordPress.com · Return of premium p 6-8 6.0 Annuities p 7-1 Annuity principles and concepts p 7-1 Accumulation period versus annuity period p 7-1 Owner, annuitant and beneficiary

the scope of his or her authority, the insurer or principal is bound or responsible for such action. There are severaltypes of agent authority in existence:

1. ACTUAL OR EXPRESS AUTHORITY — This is the type of “written” authority an agent possesseswhich is defined or contained in the agent contract or agreement. When this contract is in force, the insureris responsible for the acts of its agents / producers. This agreement specifies the actual powers and functionsof the agent. This type of authority is generally in written form. Producers or agents are allowed to solicitappointments, select or reject risks in the field, complete applications, collect premiums, submit applicationsand premiums to the insurer and other various duties. Express authority is extended to a producer in anagency contract or agreement.

2. IMPLIED AUTHORITY — This type of “unwritten” authority is not specifically identified in the agencyagreement. However, it is an extension of an agent's regular duties (i.e., authority exercised by theproducer). Any business activities that are usual and customary are implied. For instance, it is implied thatan agent (i.e., producer) who solicits insurance is permitted to sign the application, or it is implied that theproducer may ask an applicant questions concerning the health history of the proposed insured.

3. APPARENT AUTHORITY — This is the type of authority the insurance buying public perceives theagent possesses based on his or her actions. In other words, this authority is not given to the producer butthe public believes that the agent possesses this authority.

PRODUCER RESPONSIBILITIES — A producer assumes a fiduciary responsibility with regard to an insurerand his or her clients. Any producer is required to act in a fiduciary capacity when collecting premiums or dealingwith the public. This means that all producers possess a fiduciary responsibility when engaging in insurancetransactions. The producer possesses the duty to act honestly with applicants for insurance coverage and to representhis or her insurer faithfully. For instance, a producer must account for all funds collected from a consumer.Therefore, the producer possesses financial and professional responsibilities. Producers must also strive to increasetheir knowledge with regard to financial services topics. This is why most States now have in place continuingeducation laws to ensure that the producer continually increase his or her knowledge for the benefit of the consumer. A producer who has made an unintentional error or honest mistake has committed a tort known as an error andomission. This is why an insurer purchases Errors and Omissions (E+O) insurance to cover the malpractice ornegligence of producers . Independent producers or agents purchase their own E+O coverage.

Producers must represent the insurer and their clients in a fiduciary capacity. For example, any premiums collectedby an agent must be submitted to the insurer. They cannot be used for personal use or commingled with a personalaccount. Producers are also required to follow and adhere to the insurer's underwriting rules and guidelines. Unlesspremium notices are billed directly to an insured by an insurer or agency, the agent generally collects insurancepremiums.

PREMIUM COLLECTION AND RESERVES — Agents generally collect premiums from the insured at thetime of application. All future premiums are billed to the insured by the insurer and are remitted by the insured tothe company. Insureds who cannot afford premiums may sometimes use a premium financing organization whichwill function similar to an installment loan. The earned premium is the amount an insurer is entitled to since itprovided coverage for a specific period of time. An unearned premium is an amount that is returned to an insuredby the insurer since it was paid and no coverage was provided. The earned and unearned premium make up the totalpremium. The reserves of an insurer are funds set aside to pay future claims.

NILA ©L/H LicensingChapter 1—10

Page 24: FORWARD - WordPress.com · Return of premium p 6-8 6.0 Annuities p 7-1 Annuity principles and concepts p 7-1 Accumulation period versus annuity period p 7-1 Owner, annuitant and beneficiary

QUIZ

11. An insurer that is chartered in the State of New Jersey and is authorized in Connecticut is referred to in the latter State

as which of the following?

A. Domestic insurer C. Alien insurer

B. Foreign insurer D. Nonadmitted insurer

12. An authorized company whose principal office is located in this State operates in this State as:

A. A domestic company C. An alien company

B. A foreign company D. A nonadmitted company

13. An insurer owned by its policyholders best describes?

A. A stock company C. An alien company

B. A mutual company D. A reciprocal company

14. A reciprocal is managed by which of the following?

A. A branch manager C. A fraternal

B. A company officer D. An attorney-in-fact

15. Fraternals offer various types of insurance programs for which of the following?

A. For those who cannot obtain standard coverage C. For their members only

B. For those over age 65 D. For labor unions and their members

16. Insurance rating services analyze insurers based upon all of the following criteria, EXCEPT?

A. Personnel practices C. Product mix

B. Expense ratios D. Insurer reserve adequacy

17. An agent generally represents which of the following?

A. The insured C. A service representation

B. The insurer D. The Department of Insurance

18. A legal relationship between two parties when one of the parties acts on behalf of another is known as:

A. Authority C. Fiduciary capacity

B. Agency D. Declaratory judgement

19. Which of the following types of authority is an extension of an agents regular duties?

A. Express C. Apparent

B. Implied D. Warranted

20. The total premium is a combination of earned premium and:

A. Net premium C. Annual premium

B. Gross premium D. Unearned premium

NILA ©L/H LicensingChapter 1—11

Page 25: FORWARD - WordPress.com · Return of premium p 6-8 6.0 Annuities p 7-1 Annuity principles and concepts p 7-1 Accumulation period versus annuity period p 7-1 Owner, annuitant and beneficiary

ANSWERS 11. B 16. A12. A 17. B13. B 18. B14. D 19. B15. C 20. D

CHAPTER 1 KEY CONCEPTS

NILA ©L/H LicensingChapter 1—12

Page 26: FORWARD - WordPress.com · Return of premium p 6-8 6.0 Annuities p 7-1 Annuity principles and concepts p 7-1 Accumulation period versus annuity period p 7-1 Owner, annuitant and beneficiary

Adverse SelectionAgencyAgent AuthorityAlien InsurerDomestic InsurerForeign InsurerHuman Life ValueInsurable InterestInsurance

Law of Large NumbersLife InsuranceMutual InsurerNeeds ApproachPremiums ProducersRiskRisk ManagementStock Insurer

NILA ©L/H LicensingChapter 1—13

Page 27: FORWARD - WordPress.com · Return of premium p 6-8 6.0 Annuities p 7-1 Annuity principles and concepts p 7-1 Accumulation period versus annuity period p 7-1 Owner, annuitant and beneficiary

METHODS OF MANAGING RISK

The concept of risk involves the uncertainty of financial loss. There are two types of risk, one of which is insurable:(1) pure risk and (2) speculative risk. Pure risk involves the chance of loss only and may be insurable. Speculativerisk is that type which involves the chance for gain or loss. There are several methods available in which to manageinsurable risk including:

1. AVOIDANCE — To avoid the risk of loss, a person need not engage in a particular activity, ownspecific property or manufacture a product. Avoidance makes loss from a specified exposureimpossible. It eliminates the probability of loss by deciding not to acquire an asset or undertake anactivity.

2. RETENTION — Loss exposures can be retained consciously or unconsciously. Eliminatingcollision coverage on an old auto or increasing one’s health insurance deductible are examples of(conscious) retention. Someone who is underinsured and does not realize it is an example ofunconscious retention. A benefit of using either form of retention is that a person or organization doesnot need to pay for retained losses before they occur. If the person or organization saves money byusing retention, the savings can be wiped out if a loss or larger loss than expected occurs.

3. TRANSFER — This is accomplished when a person or business purchases an insurance policy. Purchasing insurance coverage is the most common method of risk transfer (i.e., from an insured toan insurer). Risk may also be transferred through a non-insurance technique such as a hold-harmlessagreement or through incorporation.

4. SHARING — In this method, risk is distributed among an assortment of persons. Each party agreesto assume a specific portion of the loss. It involves, in some cases, a non-binding agreement betweenthe parties. For example, store owners located nearby a football stadium suffer break-ins followinggames. Since their insurance policies may not cover these losses, they form an agreement that if anyof them suffer a loss related to the large crowds at the game, they will all share in any loss not coveredby their property or crime insurance. The non-binding aspect of the agreement involves the fact thatthe store owners may be relieved of liability if the one who suffers a loss inadvertently left the storeunlocked (therefore contributing to the loss). Another example of risk sharing is a partnership or areinsurance transaction.

5. REDUCTION — Loss reduction measures reduce the severity of losses that do occur. Constructinga structure with brick or masonry rather than frame (i.e., wood) will reduce a fire loss. Installing firealarms, burglar alarms, smoke detectors or a sprinkler system may all reduce a possible loss. A flushot reduces a possible health insurance loss or when an individual decides to set aside funds to coverlosses that are not covered by insurance, he or she has engaged in risk reduction.

6. PREVENTION — This is any measure that reduces the probability or frequency of a loss but doesnot eliminate completely all possibility of that loss. For example, removingunnecessary flammables from a premises reduces the possibility of a lossoccurring.

NILA ©L/H LicensingChapter 1—14

Page 28: FORWARD - WordPress.com · Return of premium p 6-8 6.0 Annuities p 7-1 Annuity principles and concepts p 7-1 Accumulation period versus annuity period p 7-1 Owner, annuitant and beneficiary

LAW OF CONTRACTSand OTHER

CONTRACT FEATURES

Page 29: FORWARD - WordPress.com · Return of premium p 6-8 6.0 Annuities p 7-1 Annuity principles and concepts p 7-1 Accumulation period versus annuity period p 7-1 Owner, annuitant and beneficiary

INTRODUCTION

An insurance policy is a legal contract between two parties. These parties generally include theinsurer and the policy owner. The policy owner is also the insured in life and health insurance. A lifeand health policy is a personal contract owned by the policy owner/insured party. Each party to thecontract promises to perform in a specified manner.

LAW OF CONTRACTS ANDOTHER CONTRACT FEATURES

CONTRACTS

CONTRACT LAW — According to the law of contracts, elements that must be present in order for any contractto be legal and enforceable in a court of law include: (1) offer and acceptance, (2) consideration, (3) competentparties, and (4) legal purpose.

Agreement — A valid offer and unconditional acceptance must be present in order for the contract to be enforceable. The offer and acceptance is sometimes referred to as the agreement. An offer is a proposal by one party that, ifaccepted by another, will create an agreement. In an insurance contract the "offer" is generally made by the applicantfor insurance. The insurer either accepts or declines (rejects) the offer based on its underwriting criteria. If theinsurer accepts the offer, both parties have arrived at a meeting of the minds which, in effect, is the agreement. Therefore, the acceptance can be demonstrated by the insurer issuing or the producer delivering the policy. Theremust be a genuine assent between the parties which means that neither party must be under duress or any undueinfluence. If an applicant submits an application without an initial premium, he or she is making an invitation. Theoffer is not complete unless the premium is included. If the insurer makes a counter-offer, the original offer madeby the applicant has been rejected by the insurer. The original offer is off the table so to speak. No contract will existunless the applicant accepts the insurer’s counter-offer.

Consideration — In order for the agreement to be binding, both parties must provide each other with something ofvalue. Therefore, there must be an exchange of values between the parties to make the agreement binding. Theapplicant's/insured's consideration is the premium paid and the representations (i.e., statements) he or she makeson the application. The insurer's consideration is the promise to pay legitimate claims for coverage provided duringthe policy period. Sometimes a consideration is referred to as a bargained exchange. Therefore, the considerationis the binding force in the insurance policy.

Legal Capacity — The parties (i.e., applicant and insurer) arriving at an agreement must possess the capacity toenter into that contract. In other words, the parties involved in the agreement must be competent to enter into thatagreement. If a person is competent, he or she possesses such capacity. This requirement may also be referred to ascompetent parties. Most people (and insurers) are considered competent to enter into a contract with someexceptions including but not limited to:

NILA ©

L/H Licensing

Chapter 2—1

Page 30: FORWARD - WordPress.com · Return of premium p 6-8 6.0 Annuities p 7-1 Annuity principles and concepts p 7-1 Accumulation period versus annuity period p 7-1 Owner, annuitant and beneficiary

! Minors, except those entering into agreements for necessities (i.e., food). Laws will vary by state as far asdetermination of "age of majority."

! Insane or mentally incompetent individuals.! Individuals under the influence of alcohol or drugs at the time of application.! Persons forced or coerced into a contract.! Enemy aliens.! Convicts (based on state law).

Legal Purpose — A contract must be in existence with the public interest in mind. Contractual arrangements maynot be contrary to public policy and must be created in the best interest of the public. Therefore, an organized crime"hit" contract is not valid nor is it in the best interest of the public.

DISTINCT CHARACTERISTICS OF AN INSURANCE CONTRACT — There are many legal conceptsand interpretations which affect all types of contracts including insurance contracts. Insurance policies also possessadditional features and characteristics which distinguish them in the "world of contract law" including but not limitedto:

Unilateral — An insurance policy may be described as a unilateral contract since its language is developed by oneparty – the insurer. Therefore, it is sometimes considered to be a one-sided contract. An offer for a unilateralcontract generally requires the performance of an act in order for the contract to be binding. For example, Johncompletes a life insurance application and submits it to ABC Mutual. The insurer informs John that it has acceptedthe offer by the issuance of the policy. The act performed by the insurer is the "promise" to provide coverage andpay valid claims. A unilateral contract may also be described as a promise in exchange for an act already performedor an act in exchange for a promise. This also means that it is a contract where only one party must perform (i.e.the insurer makes the promise).

Adhesion — An insurance policy is a contract of adhesion. This means that it is drafted by the insurer and acceptedby an applicant. In other words, the insurer "draws up" all the contract language and the applicant (if he or shedesires coverage) must act according to the language of the agreement. Since the insurer drafts the language of thecontract, any language that is unclear or ambiguous that later becomes apparent may be decided by a court in favorof the person who did not draft the language (i.e., the insured). A contract of adhesion is drawn up by the insurer andthe applicant has the option of purchasing it or not purchasing it. The fact that an insurance contract is a contract ofadhesion and unilateral makes it "one-sided". The possibility of ambiguities or confusing language arising in acontract of adhesion involve legal interpretations affecting contracts. For example, a common legal interpretationof this situation entails the doctrine of reasonable expectations. According to this doctrine, a court of law willgenerally state that an insurance contract may be interpreted by a "reasonable" consumer to mean what the produceror insurer has indicated it means or what he or she has interpreted or expected it to mean.

Aleatory — An insurance contract is an aleatory contract since one party may recover more in value than he or shehas parted with based upon a possible future event (i.e., death). The potential value (i.e., claim payment) receivedby the insured is generally greater than the value (i.e., premium) received by the insurer. An insurance policy isconsidered to be aleatory in nature since the insurer promises to pay funds if a covered loss results. Performance isbased upon an uncertain future event involving unequal bargaining value. In other words, an aleatory contract isone characterized by an unequal payment or consideration. This aleatory feature of an insurance contract involvesa policy owner obtaining a lot of coverage in return for a small fee (i.e. premium).

Conditional — An insurance policy may also be described as a conditional contract since its function is conditionalupon the performance of the parties arriving at an agreement. The insurer promise or performance (i.e., payingclaims) is conditional upon the payment of premium to keep coverage in force. In addition, the policy owner andinsurer both must satisfy certain aspects before the contract can be executed.

NILA ©

L/H Licensing

Chapter 2—2

Page 31: FORWARD - WordPress.com · Return of premium p 6-8 6.0 Annuities p 7-1 Annuity principles and concepts p 7-1 Accumulation period versus annuity period p 7-1 Owner, annuitant and beneficiary

Personal — A property and casualty contract is between an insurer and a named insured (i.e., a person). Even thoughit provides protection for a life, a home or a car, the agreement is between the company and a person.

Utmost Good Faith — This means that both parties must deal with one another in accordance with standards ofhonesty and sincerity. These parties rely on the statements of the other. Insurance policies are contracts of utmostgood faith. The representations made by an applicant that are made part of the contract must be true. Materialmisrepresentations made by the applicant / insured or intentional concealment may void the contract since one ofthe parties has not acted "in good faith". In addition, both parties must not engage in concealment or fraud or thecontract may be void. Both parties rely on the statements and promises (i.e., insurer) of the other.

OTHER CONTRACTUAL DOCTRINES AND PRINCIPLES — An insurance contract may also bedescribed as a contract of utmost good faith. Trust by both parties is important when finalizing the agreement. Theapplicant for insurance must provide to the insurer, full and fair disclosure of the exposure being insured. In otherwords, the risk or exposure that the insurance company is assuming must be the exact risk or exposure that theinsured is transferring. Most insurance contracts are policies of indemnity which means that the policy owner /insured is made financially whole after a loss is paid. Therefore, the insurer “indemnifies” the policy owner /insured. Therefore, the insured should make truthful statements on the application and refrain from concealing anyimportant information from the insurer. Some additional legal principles regarding the utmost good faith of anapplicant / insured for insurance coverage include but are not limited to:

Representations — These are oral or written statements made by an applicant for insurance that are recorded onthe application. They are generally answers to specific questions on the application regarding the insured and theexposure (i.e., health history etc.) to be insured. A representation is a statement made by an applicant that is trueto the best of his or her knowledge. All statements recorded on an insurance application are (hopefully) truerepresentations and not warranties.

Misrepresentations — This is a false statement made by an applicant. If an applicant incorrectly states hisresidence address on an application, this would be considered a misrepresentation — simply, a false fact. Misrepresentations generally do not affect coverage unless the applicant has made false statements regarding his orher health status. This would be an indication of a material misrepresentation.

A material misrepresentation involves a material fact that, had it been known by the insurer at the time ofapplication, would have caused the insurer to reject the application. For instance, an applicant for life insurance isasked on the application if he has suffered any illness during the past five years. He answers no when, in fact, he hasbeen seriously ill on three occasions in the past two years. This involves a material misrepresentation. If the policyis issued and the insurer later learns of the material misrepresentation (i.e., the lie), the policy is void. Insurers usethis principle to varying degrees and depending upon their underwriting guidelines.

Warranty — According to Life Insurance, by Black and Skipper, a warranty is "a representation made by the appli-cant/insured that is guaranteed to be absolutely true." Such a statement is made part of the contract and thenbecomes a warranty — a promise made by the insured on which coverage is based. *

For example, due to several robberies at a small branch office of a bank, an insurer informs the bank that it will notprovide crime coverage unless the bank installs a new alarm system and hires a guard who will always be on dutyduring banking hours. In order to receive coverage, the bank warranties (i.e., guarantees in writing) that it willcomply. This warranty is physically made part of the policy. If the bank is robbed when the guard left the premises,the insurer could deny coverage because the warranty was breached.

Concealment — An applicant for insurance is also obligated to voluntarily disclose any other material factconcerning the exposure to be insured, even if the insurer and its agent do not ask questions regarding it. The failureto disclose material facts is considered to be concealment. In other words, an insured may not withhold any material

NILA ©

L/H Licensing

Chapter 2—3

Page 32: FORWARD - WordPress.com · Return of premium p 6-8 6.0 Annuities p 7-1 Annuity principles and concepts p 7-1 Accumulation period versus annuity period p 7-1 Owner, annuitant and beneficiary

facts that he should realize the insurer would want to know. Concealment is fraudulent in nature and is grounds forvoiding the policy. For example, if an applicant withholds information about his health status that he should haveprovided, he has probably engaged in concealment.

Fraud — This involves deliberate knowledge of or intentional deceit with the objective of making false statementsin order to be compensated by an insurance contract (i.e., filing a false claim).

Waiver and Estoppel — These are two legal principles that may prevent an insurer from denying an insuranceclaim if specific conditions are present or satisfied. A waiver is the voluntary surrendering of a known right. It mayalso be defined as "the deliberate, voluntary or intentional abandonment of a known right by the insurer". It usuallyinvolves conduct of an insurer or its sales representative which intentionally relinquishes a defense against a claim. For instance, assume that a life insurer issues a policy which states that if the insured enters the military the policyis void. The insured joins the army and is killed during a battle. A company officer informs the insured's beneficiarythat since the insured died in defense of his country the company will waive its defense of military service death. Later the insurer denies the claim. However, they will have to pay the claim since the company officer's communic-ation (written or verbal) constitutes a waiver and prevents the insurer from denying the claim. Another example ofan insurer engaging in a waiver is when it mistakenly accepts an incomplete application and issues a policy. If theinsurer later attempts to rescind the policy or deny a claim because the application was incomplete, they will beprevented from doing so since they have engaged in a waiver. Their mistake prevents them from denying the claimor attempting to take back or rescind the policy. A waiver can also occur if an insurer fails to enforce a provisionin the policy. If an insurer finds after it issues a policy that an individual lied about his health and the insurer doesnot inform him within a reasonable time that the contract is going to be void or rescinded, it has engaged in a “waiverby silence.” Estoppel is a legal principle involving a broken promise. It prohibits an insurer from denying a claimdue to specific actions by the insurer or its representatives or due to inaction by the insurer. One party makes a falserepresentation (i.e., an agent who states that the insured is fully covered). Another party relies on that statement(i.e., the insured). Harm then results to the insured (i.e., the insurer attempts to deny the claim). If these threeelements are present the insurer will be "estopped" or prevented from denying the claim.

QUIZ

1. Each of the following must be present in order for an insurance contract to be legal and enforceable, EXCEPT:

A. Offer C. Competent underwriting

B. Acceptance D. Legal purpose

2. Delivery of an insurance contract is an illustration of:

A. The offer C. A consideration

B. An acceptance D. An aleatory action

3. An insurance contract is created by an insurer. Any ambiguities that appear in the future will be decided by a court in

favor of the insured since the policy is:

A. A contract of adhesion C. A unilateral contract

B. An aleatory contract D. A personal contract

4. The statements made by an applicant in an insurance application are considered to be:

A. Warranties C. Misrepresentations

B. Representations D. Countersignatures

NILA ©

L/H Licensing

Chapter 2—4

Page 33: FORWARD - WordPress.com · Return of premium p 6-8 6.0 Annuities p 7-1 Annuity principles and concepts p 7-1 Accumulation period versus annuity period p 7-1 Owner, annuitant and beneficiary

5. A statement made by an applicant for insurance that is guaranteed to be absolutely true and is made part of the contract

is called a:

A. Representation C. Consideration

B. Conditional promise D. Warranty

6. Which of the following is considered to be a voluntary relinquishment of a known right?

A. Warranty C. Waiver

B. Estoppel D. Parole evidence

7. Deliberate or intentional deceit by making false statements is known as:

A. Larceny C. Fraud

B. Rebating D. Misrepresentation

8. Each of the following is a characteristic of an insurance contract, EXCEPT?

A. Aleatory C. Personal

B. Adhesion D. Bilateral

9. A statement made by an applicant for insurance that is made part of the contract and upon which coverage is conditioned

is known as:

A. Warranty C. Material misrepresentation

B. Waiver D. Consideration

10. Several elements must be present in order for an insurance contract to be enforceable. One required element is that all

parties be competent. Each of the following is competent to enter into a contract, EXCEPT:

A. Minor contracting for necessities C. An officer of an insurer

B. An insured suffering a serious illness D. An individual under duress

11. An applicant for insurance submits, through a producer, a completed application and the initial premium. Once it is

received by the insurance company underwriter, the risk is assessed and the insurer makes a counter-offer. Which of the

following has occurred?

A. The offer has been accepted by the insurer

B. The original offer has been rejected by the insurer

C. The negotiations between the prospective parties have concluded

D. The contract is void

12. Which of the following is the binding force in an insurance contract?

A. The agreement C. The consideration

B. The offer and acceptance D. Legal capacity

13. Which of the following best describes when an insurer fails to enforce a provision in an insurance policy?

A. Concealment C. Adhesion

B. Waiver D. Fraud

NILA ©

L/H Licensing

Chapter 2—5

Page 34: FORWARD - WordPress.com · Return of premium p 6-8 6.0 Annuities p 7-1 Annuity principles and concepts p 7-1 Accumulation period versus annuity period p 7-1 Owner, annuitant and beneficiary

ANSWERS

1. C2. B3. A4. B5. D6. C7. C

8. D 9. A10. D11. B12. C13. B

CHAPTER 2 KEY CONCEPTS

AdhesionAleatoryCompetent PartiesConsiderationContract LawEstoppelLegal Purpose

Offer & AcceptanceParole EvidencePersonalRepresentationUnilateralWaiverWarranty

NILA ©

L/H Licensing

Chapter 2—6

Page 35: FORWARD - WordPress.com · Return of premium p 6-8 6.0 Annuities p 7-1 Annuity principles and concepts p 7-1 Accumulation period versus annuity period p 7-1 Owner, annuitant and beneficiary

FIELD AND HOME OFFICEUNDERWRITING AND

POLICYDELIVERY

Page 36: FORWARD - WordPress.com · Return of premium p 6-8 6.0 Annuities p 7-1 Annuity principles and concepts p 7-1 Accumulation period versus annuity period p 7-1 Owner, annuitant and beneficiary

INTRODUCTION

This chapter will review the basic principles with regard to underwriting and agent responsibilities. The application information including its required signatures will be reviewed as well as an applicant'sresponsibility in providing truthful responses to an insurance producer. The types of premiumreceipts, insurable interest, risk classification and policy delivery information will also be presented.

FIELD AND HOME OFFICEUNDERWRITING AND POLICY ISSUANCE

APPLICATION

THE LIFE INSURANCE APPLICATION — The primary duties of an underwriter are to assess risks (i.e.,applications), approving or declining applications, determining premiums and protecting the insurer against adverseselection. The principal source or tool utilized by the underwriting department of an insurer to determine whetheror not an applicant for life insurance is eligible for coverage is the application. The application provides anassortment of information to the insurer. It is generally divided into two sections. Section I includes generalinformation about an applicant such as name, address, birth date, salary and occupation. Section II of an applicationinvolves information with regard to the proposed insureds health history. An underwriter is provided with theapplication and will review Section I and II information to determine if the proposed insured is insurable. All of thefact finding information that appears on an application will assist the underwriting department in determiningindividual premiums to be charged. Mortality tables inform us that the older an individual is, the greater theprobability of death. This is why age affects insurance costs. Mortality information also informs us that women

statistically live longer than men. Therefore, the risk an insurer assumes when providing coverage for a femaleis lower. This is why the cost is lower.

The field underwriting performed by the producing agent will also assist the home office underwriters. Theagent should complete the application providing as much detail as possible. Once the application isforwarded to the insurer, the underwriter may require additional information from the applicant's doctor toassess the risk. However, the applicant must sign a disclosure statement permitting such action. In someinstances insurers utilize a non-medical application which requires no additional information other than theapplication. This type of application is sometimes used when the proposed insured desires a low amountof coverage and he or she is a younger age (i.e., 20s, or 30s).

FIELD UNDERWRITING

The initial underwriting done by an insurer is performed by a producer. This is known as field underwriting and isextremely important. During this process the producer determines which risks are desirable and submits those to theunderwriting department for approval. The producer also possesses some additional duties with regard to theapplication process. A field underwriter or producer may solicit appointments, complete applications, collect

NILA ©

L/H Licensing

Chapter 3—1

Page 37: FORWARD - WordPress.com · Return of premium p 6-8 6.0 Annuities p 7-1 Annuity principles and concepts p 7-1 Accumulation period versus annuity period p 7-1 Owner, annuitant and beneficiary

premiums and submit applications to the home office underwriter. The producer does not issue the policy. He orshe is also responsible for providing any required notice of information practices to an applicant such as a noticeregarding replacement, a life insurance buyer’s guide, an outline of coverage or a policy summary .

APPLICATION PROCEDURES — When a producer completes an application, he or she must include at leasttwo required signatures: that of the applicant and the producer. If third party ownership is present, three signaturesare required: the applicant, the proposed insured and the producer. All insurers require that the producer sign theapplication as well or it will not be underwritten. The application must be filled out completely. Severalconsequences of an incomplete application will be present if the producer fails to submit one that is completed.Several delays will follow including a delay in underwriting, policy issuance, policy delivery and payment ofcommissions to the producer. The applicant may also decide to do business elsewhere as a result of such delays.

If the producer alters or changes the application information in any way without informing the applicant or insurer,he or she may be engaging in fraud. Therefore, any changes made to an application by a producer must be initialedby the applicant (and the producer) before the application is submitted to underwriting. In addition, all the repliesmade by the applicant to specific questions on the application regarding health history are representations. Arepresentation is a true statement made by the applicant in response to Section I or II questions on the application. If the applicant lies about his or her health status, material misrepresentation is present which may void the policyor cause it to be canceled by the insurer. Whenever a policy is canceled by the insurer, a premium is generallyrefunded to the policy owner / applicant. If an applicant makes any statements that are "guaranteed" to be true, heor she has made a warranty. Therefore, statements made by an applicant that are recorded on the application areconsidered to be representations and not warranties.

PREMIUMS PAID WITH THE APPLICATION — The producer should collect a premium from theapplicant at the time of application (or as early as possible). The premium is generally forwarded with the applicationto the underwriting department. However, in some cases, an application may be submitted without a premium. Suchan application is referred to as a trial application. Since a premium did not accompany the application, a statementof continued good health must be completed by the proposed insured before the policy, if issued, can be delivered. If an application is submitted without a premium and the agent or producer later learns that the proposed insured hascontracted a serious illness, the producer should inform the insurer before it continues with or concludes itsunderwriting. This would obviously influence whether or not the policy would be issued. It is the responsibility ofthe agent or producer to inform the insurer of any information which may impact policy issuance. Whenever apremium is collected, the producer must leave a premium receipt with the applicant. An applicant is provided witha premium receipt at the time the application is completed and an initial premium is paid. This receipt is proof thatan initial premium was paid with the application. The type of receipt provided may determine when coverage willbe effective.

PREMIUM RECEIPTS

TYPES OF PREMIUM RECEIPTS — There are two types of receipts in existence that may be provided toan applicant when the producer collects a premium including a conditional receipt and a binding receipt. Conditionalreceipts generally provide coverage "as of the date of the receipt as long as a specific condition is satisfied." Thismeans that coverage can be provided if the proposed insured demonstrates insurability. This may also be accomplished simply by submitting the application to the underwriting department and it is approved (or declined)on its own merits. No other information may be required. Another "condition" that may have to be satisfied is thatinsurability may have to be proven in other ways. For example, the insurer may wish to receive more informationbefore it approves the application or agrees that the proposed insured is, in fact, insurable. Information may beobtained from other areas as well such as the medical information bureau. a consumer report, an attending physicianstatement, a medical exam or other tests (i.e., blood test). These receipts identify the amount of premium collected

NILA ©

L/H Licensing

Chapter 3—2

Page 38: FORWARD - WordPress.com · Return of premium p 6-8 6.0 Annuities p 7-1 Annuity principles and concepts p 7-1 Accumulation period versus annuity period p 7-1 Owner, annuitant and beneficiary

and may inform us when coverage is effective. The conditional receipt is predominantly utilized today whereas thebinding receipt is used in a limited fashion. The date appearing on a conditional receipt always reflects a date earlierthan the issue date of the policy. The type of receipt used will also indicate the effective date of coverage.

Insurability Language in a Conditional Receipt — This provides that the insurer has made an offerconditional upon the proposed insured's insurability, and the applicant accepts the conditional offer by paying thepremium. Therefore, coverage becomes effective as of the date of the conditional receipt (or if later, as of the dateof the medical exam if the proposed insured is found to be insurable). For example, if Ed signs an application forcoverage on August 2nd and takes a required medical exam on August 4th, protection actually begins on the 4thbecause the medical exam was required in order for coverage to be provided. Delivery is not necessary in order forcoverage to be provided. If, in another example, the proposed insured is not required to pass a physical exam andhe or she dies before the application is underwritten (i.e., a clean application), the insurer must still proceed with theunderwriting phase and determine insurability according to its normal underwriting standards. If the application isapproved, even after the insured dies, the claim will be paid.

Approval Language in a Conditional Receipt — This type provides that coverage is effective only after theapplication has been approved by the insurer. Therefore, if the producer collects the premium and application onJune 15th and the application is approved by underwriting on June 29th, coverage is effective as of the latter date.

Binding Receipt — Also referred to as an unconditional receipt, issuance of a binding receipt means that coveragebecomes effective as soon as the premium is collected by the producer (i.e., immediate coverage). The receipt willverify this since it allows for instant coverage. Life insurance protection will continue during the underwriting periodand remain in effect until the application is declined and premium is refunded. If the applicant dies before this occurs,the insurer must pay the claim.

OTHER PRODUCER RESPONSIBILITIES — It is the producer's responsibility to review the receipt with theapplicant. In addition to identifying the amount of the premium, the receipt also provides some additionalinformation that must be communicated to the insurer. It is the producer's responsibility to discuss with the applicantduring the field underwriting process the applicable notice of information practices such as the Medical InformationBureau (MIB) and the Fair Credit Reporting Act. A notice to the applicant appears in the receipt and explains theseinformation practices. Producers must also secure an HIV consent form from the applicant in applicable situationsand communicate that blood tests may be a required underwriting practice. In other words, even though the insurerrequires a blood test as part of its normal underwriting activity, it must still secure a signed consent form whichindicates to the applicant that any blood taken will be screened for the HIV virus and that he or she is providingpermission for such testing to be completed.

MEDICAL INFORMATION BUREAU

The MIB is a clearing house of health information supplied by insurers from information on applications. The MIBprovides assistance to an insurer in the underwriting of life insurance. Underwriting is complemented by the MIBsince it helps to detect any adverse health conditions that the applicant possesses. The MIB contains informationabout an applicant's previous health history. An agent must inform an applicant, at the time of application, that theinsurer has the right to request an MIB report with regard to the applicant's health history. The MIB does not includeinformation from the proposed insured's physician or hospital. It only includes information from the current applica-tion that has been provided by the applicant to the producer. The MIB allows an insurer to check applicationinformation against a record of the proposed insured's health history and also permits an insurer to avoid high riskapplicants. It also allows for an exchange of medical information between insurance companies only. There arenumerous MIB organizations in existence that collect and store data for insurers. MIBs are supported by insurersand collect information on their behalf. Most are non-profit entities.

NILA ©

L/H Licensing

Chapter 3—3

Page 39: FORWARD - WordPress.com · Return of premium p 6-8 6.0 Annuities p 7-1 Annuity principles and concepts p 7-1 Accumulation period versus annuity period p 7-1 Owner, annuitant and beneficiary

CONSUMER REPORTS

FAIR CREDIT REPORTING ACT OF 1970 — This federal law enacted in 1970 provides consumers withspecific rights if they are denied employment, credit or insurance coverage and also establishes procedures forconsumer reporting agencies. If an individual is denied any of these aforementioned items, he or she has the rightto be provided with the source of the adverse information. For example, if an insurer denies coverage based oninformation derived from a consumer or other type of inspection report, the applicant has the right, according to thislaw, to request a personal interview or a copy of the report in order to correct any false information. A consumerreport may be performed by an insurer and may include a credit check or the request of information from anapplicant's current or previous landlord, employer or neighbors. Interviewing any individuals in person in thissituation is known as an "investigative" consumer report. Any information derived that reflects adversely upon anapplicant may be challenged if claimed to be inaccurate or untrue. A "Notice to the Applicant" explaining theaforementioned information is generally contained in whatever premium receipt the insurer utilizes. Again, the rightsof the consumer are outlined in this notice with regard to declinations.

CONSUMER REPORTS — A consumer report may be performed by an insurer to further qualify an applicantfor life insurance. Since an insurer may be providing a large death benefit to the applicant, it wants to make sure thatit has all the information necessary in order to arrive at a decision for coverage. According to the Fair CreditReporting Act, whenever a consumer report of any kind is requested on an applicant's life, he or she must be notifiedwithin a short period of time (i.e., usually three days) that such a report has been requested. The producer possessesseveral responsibilities to an applicant following an adverse underwriting decision (i.e., application is declined). Theproducer should inform the applicant of his or her rights with regard to consumer reports, if applicable; provide thereasons for the adverse decision; and provide the names of the institutional sources which provide informationregarding the applicant.

OTHER UNDERWRITING SOURCES

UNDERWRITING — The underwriting department utilizes numerous sources of information in order to accept orreject an application for coverage. Again, insurable interest must be present at the time of application. Adetermination of insurable interest is performed by the field underwriter (i.e., agent). The field underwriter maycomplete applications, collect premiums and provide a premium receipt to the applicant. Although the primary sourceof information available to an underwriter is the application, he/she will also base a final decision on an assortmentof other information, including the producer or agent's report, an attending physician statement (APS), MIB,consumer or inspection reports, medical or physical exam results (i.e. medical report), or laboratory tests (i.e., bloodtests for HIV). When a medical exam or medical report is required by the insurer, it will pay for the exam and usea physician or medical professional of its choice. The producer or agent's report is a small segment of theapplication where the producer includes any other information that may not have been included in other portions ofthe application. The insurer wishes to receive as much information as possible in order to make a sound decisionwhether or not to accept or reject applications for life insurance.

SELECTION CRITERIA — Life insurers have established basic requirements or "average risk profiles" to obtaina desired type of life insurance business. Ideally, insurers desire risks that are standard in order to realize a profit. Most applicants for life insurance are in this category. Those in "perfect" health may be considered a "preferred"risk and receive an even lower premium rate than the standard risk. A third classification of risks for life insuranceis substandard. These applicants are generally declined because of poor health. Some types of substandard risksmay be issued a policy with a health condition rider or amendment of some sort which requires an extra or higherpremium rate since the risk to the insurer is greater. Other substandard risks may be declined. These may be referredto as declined risks. Methods available to "rate up" a life policy include flat extra rate (i.e., flat additional premium),

NILA ©

L/H Licensing

Chapter 3—4

Page 40: FORWARD - WordPress.com · Return of premium p 6-8 6.0 Annuities p 7-1 Annuity principles and concepts p 7-1 Accumulation period versus annuity period p 7-1 Owner, annuitant and beneficiary

step-up or "rate-up" in age, tabular rating, or a percentage of the standard rate. * In addition, due to the inclusion ofa war clause which excludes death while in the military, insurers are legally discriminating, in effect, againstmembers of the armed forces. This clause or provision will be reviewed in chapter 5.

POLICY DELIVERY

Once the application is submitted and underwritten and coverage is approved, the policy is issued and must bedelivered to the policy owner. When a producer delivers the policy, this means that the insurer has "accepted" theapplicant's offer. It also means that the producer should review with the insured the reasons why it was purchased. Once delivered, the 10-day free-look time frame begins to elapse. In addition, once the policy is issued, the suicideclause and the incontestable period time frames commence.

1. PRODUCER RESPONSIBILITIES AT DELIVERY — The producer should review with the policyowner at delivery the benefits of the insurance contract just purchased. With regard to life insurance policies,the producer should point out and review the following including but not limited to:

! The person insured.! The coverage amount.! The effective date of coverage.! The policy provisions.! The names of the primary and contingent beneficiaries.! The annual premium and the frequency with which it is paid.! The ownership rights that the policy owner now possesses.! The causes of death that are not covered (i.e., exclusions such as war or aviation).! Any benefit or exclusion riders added to the policy plus the applicable premium.

If a statement of continued good health is required, the premium must be collected and this statementcompleted before the policy is delivered by the producer. If a premium has not been paid with the applicationbut the policy has been issued, the agent may deliver the policy as soon as the statement of good health andthe premium has been collected. If, however, the producer arrives to deliver the policy but finds that theproposed insured is now in the hospital, he or she should not deliver the policy. The home officeunderwriters will want to know more about the proposed insured’s condition before following through withdelivery.

QUIZ

1. An MIB contains information about:

A. Consumer reports C. Risk classifications

B. An applicant's previous health history D. Adverse selection

2. All of the following are factors which affect a life insurance risk, EXCEPT:

A. Age C. Unearned income

B. Occupation D. Family health history

NILA ©

L/H Licensing

Chapter 3—5

Page 41: FORWARD - WordPress.com · Return of premium p 6-8 6.0 Annuities p 7-1 Annuity principles and concepts p 7-1 Accumulation period versus annuity period p 7-1 Owner, annuitant and beneficiary

3. Which of the following is true regarding the doctrine of insurable interest?

A. Insurable interest must exist at the time of death

B. Insurable interest must exist during policy delivery

C. Insurable interest must exist at the time of application

D. Insurable interest must exist at the time of loss

4. All of the following are types of premium receipts, EXCEPT:

A. Binding receipt C. Countersignature receipt

B. Approval conditional receipt D. Insurability conditional receipt

5. An applicant who conceals material information from an insurer may be subject to which of the following?

A. Policy cancellation C. Fine and imprisonment

B. Warranty revocation D. Policy cancellation with returned premium

6. An underwriter receives the greatest amount of insurability information from which of the following sources?

A. The insurance application C. An attending physician's statement

B. The medical information report D. A consumer report

7. Each of the following methods may be utilized to rate up a life insurance policy, EXCEPT:

A. Flat rate increase C. Percentage of the standard rate

B. Tabular rating D. Preferred risk method

8. Which of the following receipts provides coverage for an insured immediately upon the payment of the premium?

A. Binding receipt C. Conditional receipt

B. Approval receipt D. Insurability receipt

9. Which of the following risk classifications is characterized by a lower annual premium?

A. Preferred risk C. Standard risk

B. Substandard risk D. Personal risk

10. John applies for life insurance and pays an initial premium. He is provided with an insurability type of conditional

receipt. The next day John dies in an auto accident. The insurer is notified by John's wife the following day. What

action will the insurer take?

A. Refuse to underwrite and retain the premium

B. Underwrite the policy and if John is found insurable, the insurer will pay the claim

C. Deny any death claim and refund any premium paid

D. Issue a policy if John is found insurable but deny any claim since he died before the completion of the free look

period

11. Which of the following is a primary purpose of a Medical Information Bureau (MIB) report?

A. Allows the insurer to receive medical history from an applicant's personal physician

B. Allows the insurer to deny a claim for a pre-existing condition

C. Allows the insurer to reduce death claim amounts

D. Allows the insurer to avoid high risk applicants

NILA ©

L/H Licensing

Chapter 3—6

Page 42: FORWARD - WordPress.com · Return of premium p 6-8 6.0 Annuities p 7-1 Annuity principles and concepts p 7-1 Accumulation period versus annuity period p 7-1 Owner, annuitant and beneficiary

12. If an applicant for life insurance is considered a substandard risk by an insurer but still insurable, it will probably take

which of the following actions?

A. Decline the application C. Accept the risk and increase the number of exclusions

B. Accept the risk and rate up the premium D. Issue a standard policy

13. An application completed where the applicant and proposed insured are different persons would have to be signed by

which of the following?

A. The agent C. Proposed insured and applicant

B. The agent, applicant, and the insured D. The agent, applicant and proposed insured

14. If a premium is not paid at the time an application for life insurance is completed, when will coverage go into effect?

A. At the time of application

B. When the application is approved and the policy is issued

C. As of the date of the conditional premium receipt

D. When the producer delivers the policy and collects the premium

15. If no premium is paid with an application for life insurance, what responsibilities does an agent possess when the policy

is delivered?

A. Collect the premium and additional health history data

B. Collect the premium and issue a premium receipt

C. Collect the premium and a blood sample

D. Collect the premium and a statement of continued good health and issue a premium receipt

16. Tom Baney applies for life insurance but pays no initial premium. The application is submitted to underwriting, is

approved and a policy is issued. When the agent attempts to deliver the policy she learns that Tom is in the hospital.

What should she do?

A. Deliver the policy and collect the initial premium

B. Collect the premium and mail the policy to the owner later

C. Deliver the policy to the beneficiary

D. Do not deliver the policy nor collect the premium

NILA ©

L/H Licensing

Chapter 3—7

Page 43: FORWARD - WordPress.com · Return of premium p 6-8 6.0 Annuities p 7-1 Annuity principles and concepts p 7-1 Accumulation period versus annuity period p 7-1 Owner, annuitant and beneficiary

ANSWERS

1. B2. C3. C4. C

5. D6. A7. D8. A

9. A10. B11. D12. B

13. D14. D15. D16. D

CHAPTER 3 KEY CONCEPTS

ApplicationChanges to applicationField underwritingMIBPolicy delivery

Premium receiptsRepresentationsRequired signaturesRisk classificationsWarranty

NILA ©

L/H Licensing

Chapter 3—8

Page 44: FORWARD - WordPress.com · Return of premium p 6-8 6.0 Annuities p 7-1 Annuity principles and concepts p 7-1 Accumulation period versus annuity period p 7-1 Owner, annuitant and beneficiary

PREMIUM FACTORS— Some criteria which help to determine premiums:

1. Age of proposed insured.

2. Sex (females pay lower rates since they live longer).

3. Health history of proposed insured.

4. Family health history.

5. Personal activities or hobbies (i.e., avocations such as scuba diving, parachuting).

6. Personal habits (i.e., smoking or tobacco use of any kind, DWI or DUI).

7. Occupation.

8. Salary, earnings, wages.

9.* Travel outside the country (i.e., exposes proposed insured to additional risks, illnesses, etc.).

SELECTION CRITERIA

1. STANDARD RISKS — A proposed insured in good health who is assessed standard (i.e., normal) premium

rates.

2. SUBSTANDARD RISKS — A proposed insured who has health problems. Some may be declined. Others who

obtain coverage may be "rated up" and charged a higher premium.

3. PREFERRED RISKS — Proposed insured in excellent health receives a lower premium rate. These types of

risks die prematurely less often than standard or substandard risks.

INCREASED RATING METHODS

1. STEP-UP-IN-AGE OR RATE-UP-IN-AGE — The proposed insured is 45 years of age but because of

substandard classification is rated as a 50 year old.

2. FLAT EXTRA RATE (i.e., flat additional premium) — The proposed insured's rate per thousand is increased

from $12 per $1,000 of coverage to $15 per $1,000 of coverage.

3. PERCENTAGE OF THE NORMAL RATE — The proposed insured's rate is multiplied by 150% or 200%

(i.e., a stunt pilot) because of the substandard or higher risk classification.

FACTORS AFFECTING PREMIUMS,RATING OF LIFE INSURANCE

AND SELECTION CRITERIA

NILA ©

L/H Licensing

Chapter 3—9

Page 45: FORWARD - WordPress.com · Return of premium p 6-8 6.0 Annuities p 7-1 Annuity principles and concepts p 7-1 Accumulation period versus annuity period p 7-1 Owner, annuitant and beneficiary

SECTION II

Life InsurancePolicy

Provisions, Optionsand Riders

Page 46: FORWARD - WordPress.com · Return of premium p 6-8 6.0 Annuities p 7-1 Annuity principles and concepts p 7-1 Accumulation period versus annuity period p 7-1 Owner, annuitant and beneficiary

LIFEINSURANCE

POLICIES

Page 47: FORWARD - WordPress.com · Return of premium p 6-8 6.0 Annuities p 7-1 Annuity principles and concepts p 7-1 Accumulation period versus annuity period p 7-1 Owner, annuitant and beneficiary

INTRODUCTION

Life insurance policies are contractual arrangements between a policy owner and a life insurer. Thepolicy owner has transferred the risk of a possible financial loss (i.e., premature death) to theinsurance company. There are a myriad of life insurance policies in existence that provide anindividual with the mechanism to protect himself, herself and their family properly. This chapter willreview the various forms of life insurance including term life, whole life, and endowments.

LIFE INSURANCE POLICIES

TERM INSURANCE

TERM LIFE INSURANCE — Term life is a form of insurance which provides pure or temporary protection. The face amount is payable if the insured dies during a specified term, such as five years. Once the specified periodends, protection terminates unless coverage is renewable. Some policies are renewable while others are not. Terminsurance provides the maximum amount of life insurance at the lowest initial outlay of funds. This means thatinitially, term insurance is less costly than whole life. Premiums will increase as one gets older. Unlike permanentinsurance (i.e., whole life), there is no cash value or equity build-up in a term life policy. There are two types ofindividual term insurance policies:

1. LEVEL TERM INSURANCE — Level term is the most common form of term life insurance andprovides a constant or fixed amount of coverage for as long as the policy remains in force. It is characterizedby a level face amount (death benefit) and increasing premiums (at the end of the renewable period)according to one's age. Level term insurance is renewable up to a specified age (i.e., age 70). The fact thatlevel term life insurance is renewable means that at the end of the renewable period, the insured may extendcoverage for another renewable period without proving insurability. In other words, the insured is notrequired to pass a medical exam or demonstrate good health since that requirement was already satisfiedbefore the policy was originally issued.

For example, a ten-year renewable $200,000 level term policy may cost $500 in premium for a 40 year oldmale. Following the ten year period, if the policy owner wishes to continue coverage, the premium willincrease to $1,350 for the next ten years. Even though the premiums increase based on the attained age ofthe insured, the face amount remains level or constant.

2. DECREASING TERM INSURANCE — Also referred to as non-level term, the coverage amount of thispolicy is reduced each year. In other words, the death benefit or amount of protection decreases annually. The premium, however, remains level or fixed for as long as the policy remains in force. This type of policyis generally used when cash flow is the applicant's primary consideration or to provide funds at death so thatsurvivors are able to pay off a debt obligation. This type of policy enables an insured to leave the house, boator auto to his survivors free and clear of all debts in case of his premature death. The premium is lower thanlevel term due to the fact that the face amount decreases as the rate per $1,000 of insurance increases. A de-

NILA ©

L/H Licensing

Chapter 4—1

Page 48: FORWARD - WordPress.com · Return of premium p 6-8 6.0 Annuities p 7-1 Annuity principles and concepts p 7-1 Accumulation period versus annuity period p 7-1 Owner, annuitant and beneficiary

creasing term life insurance policy is generally used in connection with a mortgage or other indebtedness andmay also be described by insurers as a mortgage protection plan, mortgage redemption plan or non-levelterm. In addition, decreasing term insurance is the type used for credit life insurance situations that ispurchased to cover the amount of a loan or debt (i.e. car loan, boat loan, home equity loan, etc.).

RENEWABILITY — Level term policies may be renewed at a higher cost once the specified period ends. Renewalis effected without a medical examination. When the renewable period ends, the insured may continue the same levelamount of coverage for another renewable period without proving insurability. The renewal premium of the policyis based upon the insured's attained age at the time of renewal. The cost increases due to the increased risk of deathof the insured. The most common type of renewable term insurance is annual renewable or yearly renewable termlife. This means that the policy is renewable (at the policy owner's option) every year. Therefore, a new premiummust be derived each year as well. The new premium, again, will be based upon the insured's attained age. Theattained age premium will increase each year as the insured's age increases. The risk to the insurer is increased sinceevery year the insured lives, he or she is closer to death and the payment of the death benefit. Renewable periodssuch as 1, 5, 10, 15, 20 and 25 years are available today. Although it has been mentioned that level term life ischaracterized by "increasing" premiums at some point in the future, the cost is fixed during the renewable period.If the policy is renewed again for another renewable period, the cost will increase since the insured is older. Thepremium is then fixed for the next renewable period and so forth. Some renewable term policies include a re-entryfeature (i.e., re-entry term) which states that the premium can change at renewal. This means that to maintain thebest (i.e. lowest) premium rate, the insured may have to prove insurability again upon renewal. If there is aninsurability problem, coverage may be maintained but at a higher rate. Sometimes re-entry term is also referred toas revertible term.

CONVERTIBILITY — Term life policies include a provision which allows a policy owner to trade it for apermanent insurance plan. If the policy owner wishes to convert, it may be accomplished without the insured beingrequired to demonstrate good health. In other words, the conversion may be accomplished without a medical exam. The conversion may be accomplished in one of two ways: (1) The owner of the policy may convert at his presentor attained age premium rate. In this case, the owner is terminating the pure protection and simply purchasing anew whole life policy rated at his present age without providing any health history information. Most conversionsare accomplished in this manner; or (2) the owner may also convert at his or her original age premium rate. Thisis the age and premium rate at which the policy owner could have purchased whole life insurance but initiallypurchased the term insurance policy instead. The insurer will determine what the appropriate premium would havebeen had the owner purchased a whole life policy at the "original age" when life insurance was initially purchased. Therefore, according to the original age method of conversion, the policy owner may convert to a whole life policyand pay the original age whole life premium. However, if this method is selected, the owner must fund or depositan amount equal to what the cash value would have been had he or she purchased whole life at the original age. Asstated in Life Insurance, the premium and cash value deposit characterize the retroactive conversion that exists ifthis method is selected.

USES OF TERM INSURANCE — There are many uses, advantages, and disadvantages of term life insuranceincluding but not limited to: (1) advantages of term insurance include: it is less expensive, it protects the insured'sinsurability, it may be used in conjunction with debts or mortgages and it may be used as a supplement to whole lifeinsurance; (2) several disadvantages are present with regard to term life insurance including: that no protection isin effect once the term protection ends; premium rates rise as the insured ages; and it possesses no equity (i.e., cashvalue). Since it has no cash value, it does not mature as does a whole life policy.

NILA ©

L/H Licensing

Chapter 4—2

Page 49: FORWARD - WordPress.com · Return of premium p 6-8 6.0 Annuities p 7-1 Annuity principles and concepts p 7-1 Accumulation period versus annuity period p 7-1 Owner, annuitant and beneficiary

WHOLE LIFE INSURANCE

WHOLE LIFE INSURANCE — Whole life insurance provides for the payment of a death benefit or faceamount of coverage upon the death of the insured, whenever death occurs. This type of policy will cover an insuredfor his or her entire lifetime or "whole life." Whole life policies may also be referred to as straight life, continuouspremium life, permanent life insurance or ordinary life. A straight life or whole life insurance contract is one ofthe most common types of life insurance policies sold today. Whole life policies are based upon the assumption thatpremiums will be paid by the policy owner throughout the insured's lifetime or to age 100, whichever occurs first.This means that the whole life policy is designed by its manufacturer (i.e. the life insurance company) to "mature"or "endow" at age 100. “Mature” or “ endow” means that the cash value accumulations are equal to the face amount. Insurers use age 100 to calculate death benefits and premiums since they assume that most people will be deceasedby that age.

O STRAIGHT LIFE POLICY — A traditional whole life policy combines pure death protection with acash savings feature. Therefore, this policy is a combination of death protection plus a cash value. Whena premium for coverage is paid, a portion is used to pay for death protection or the “term” insurance cost. Following an initial period of time once the contract becomes effective, another portion of the premium isdeposited into the policy’s cash value. This cash value or “equity’ continues to build during the life of thecontract. In addition, a whole life policy protects an insured permanently for the remainder of his or her life. This form of life insurance policy never needs to be converted (as does a term life policy if one wishes tohave permanent protection) since it remains in force as long as all premiums are paid in a timely fashion. This continuous premium or whole life plan is characterized by level or fixed premiums as long as thecontract remains in force.

A cash savings feature is also provided as part of this contract. Once it begins to accumulate, this cashsavings value increases with each subsequent premium payment. The cash value is the equity portion of thepolicy. This policy possesses a savings aspect or savings element but is not “cash” owned by the policyowner. The owner certainly has access to the cash value but in order to receive any of this “equity,” thecontract owner must borrow it. The cash value is comprised of a portion of each premium plus a fixedinterest amount provided by the policy. A whole life policy maybe described as a fixed death benefit, fixedpremium life insurance contract. In other words, it is characterized by a level death benefit, a cash savingsvalue (i.e., equity build-up), permanent protection and a fixed, level or pre-determined premium. The deathbenefit, premium payment and interest rate paid on the cash value are all pre-determined for the insured's"whole life."

CASH VALUE — In most traditional whole life policies, a cash savings value begins to build after thefirst two to three years after premium payments begin. There are exceptions to this concept which will bereviewed later. After this initial period, the cash value accumulates with every subsequent premiumpayment. The cash value accumulates from the premiums paid plus a fixed interest rate which allows thesavings to increase further during the policy year. This cash value buildup will equal the policy's faceamount by age 100. Therefore, if a person purchases a whole life policy today and lives to age 100, he orshe will receive all their premiums back plus some interest. The premiums paid plus interest equal the totalcash value at age 100.

The policy's cash value is available to the policy owner at any time. The policy owner may surrender thepolicy thereby canceling coverage and receive the amount of the cash value. In addition, the policy ownermay borrow against the cash value and receive some of this equity while keeping the policy in force. Thisis considered a loan and interest is charged for it. The cash value buildup is not paid to a beneficiary in

NILA ©

L/H Licensing

Chapter 4—3

Page 50: FORWARD - WordPress.com · Return of premium p 6-8 6.0 Annuities p 7-1 Annuity principles and concepts p 7-1 Accumulation period versus annuity period p 7-1 Owner, annuitant and beneficiary

addition to the death benefit when the insured dies unless a specific benefit rider is added to the policyknown as a return of cash value rider. The cash value is equity that builds in the contract with everysuccessive premium payment. This can be compared to the equity a homeowner builds in a home with everysuccessive mortgage payment.

To review, whole life insurance provides a level death benefit or permanent protection with a cash savingsfeature. It is characterized by a fixed, level or predetermined premium for life (or up to age 100). Itspremiums are referred to as "bundled premiums." This means that the insurer is not required to explain tothe policy owner how the premium paid is distributed (i.e., for death protection, commissions and otherexpenses). The policy's cash value increases with every premium payment and a fixed interest rate is paidon it that is also fixed for the life of the policy. Therefore, the cash value buildup of a whole life policyequals the face amount at age 100. The contract also includes nonforfeiture options or values in the eventthe policy owner wishes to surrender the policy. Premium rates are based upon a dollar amount per $1,000of coverage. There are other forms of whole life insurance available including but not limited to:

O LIMITED-PAYMENT WHOLE LIFE POLICIES — The advantage of this type of traditional wholelife policy is to permit the policy owner to cease paying premiums at the conclusion of the "limited paymentperiod". At this point the policy is fully "paid-up" for life. No future premiums are required. However, likea straight life plan, the policy is designed to mature or "endow" at age 100. A limited payment whole lifepolicy is characterized by a pre-determined premium for a limited payment period. There are several typesof limited payment policies in existence such as ten-pay life, twenty-pay life, thirty-pay life and life paid-upat the insured's age 65. Premiums paid for these policies are higher initially than a straight life policy sincethey are only paid for a limited period (i.e., 10 years rather than to age 100). Once the payments are finished,the policy is "paid-up" for life. This means that no further premiums are due. Whenever the insured dies,the death benefit will be paid to the designated beneficiary. Even though the policy is paid-up earlier thana traditional whole life policy, it will still not "mature" until age 100 since the contract has been pre-determined. Since the initial premiums for these policies are higher than a traditional whole life policy, somesay that they possess a "heavier savings element" or a greater emphasis on savings than the traditional wholelife contracts.

SINGLE - PREMIUM WHOLE LIFE — This is the extreme form of limited payment policycharacterized by a lump sum or single premium payment. Therefore, the policy is fully paid-up upon thepayment of a lump sum premium. Some "single" premium plans are in existence which consist of twopremium payments such as a "dual premium" policy. Single premium plans may possess tax advantages andthe ability to borrow against the cash value at below market interest rates.

A single premium whole policy is the most expensive whole life policy initially. This is obvious due to itslump sum premium. But if compared to a straight life policy over the life of the contract, the single premiumlife is the least expensive. For example, assume that an insured is 30 years of age and purchases a $10,000traditional whole life policy. The annual premium is $120. For a single premium life policy in the same faceamount, the lump sum premium may be $3,800. If the insured lives for forty years he or she will have paid$4,800 in traditional whole life premiums (i.e., $120 X 40 years). Therefore, during the life of the policy,in most cases, traditional whole life policies are more expensive than the single premium life plan although,again, the single premium policy is more expensive initially.

O MODIFIED AND GRADED PREMIUM WHOLE LIFE — Additional types of whole life plansagain alter the method in which premiums are paid. The premium charged for these plans are less than thatfor a straight life insurance policy in the first few years of the policy. Cash values also accumulate with eachsuccessive premium payment. Insureds who need permanent protection but cannot afford the higher

NILA ©

L/H Licensing

Chapter 4—4

Page 51: FORWARD - WordPress.com · Return of premium p 6-8 6.0 Annuities p 7-1 Annuity principles and concepts p 7-1 Accumulation period versus annuity period p 7-1 Owner, annuitant and beneficiary

traditional whole life premiums required would buy this type of policy. Modified whole life is a type ofpolicy that is characterized by an initial premium that is lower than traditional whole life during anintroductory period of time. The policy owner will pay a lower initial premium, as compared to the wholelife premium, for the first few (i.e., five) years. After this time, the premium will increase to an amountgreater than what the initial whole life premium would have been. Once the premium "jumps" following theinitial period, it will remain level or constant for the life of the policy. Therefore, for example, modifiedwhole life is characterized by two fixed premiums — a lower initial premium for five (5) years that increasesin the sixth year to an amount higher than the traditional whole life premium would have been and remainslevel for life. A graded premium whole life plan is a contract characterized, like modified life, by a lowerpremium than whole life in the early years of the contract. However, premiums increase annually or everyyear for the initial period. It then jumps to an amount higher than the whole life premium and remains fixedfor life. The premiums for these policies are pre-determined but are not level, in the traditional sense, as theywould be in the traditional whole life or limited pay whole life plans.

O ENHANCED ORDINARY LIFE — Also known as economatic life or extraordinary life, it is a lowpremium based participating permanent insurance policy. The contract's face amount reduces each year. Any dividends paid are set aside and used to purchase either paid up additions or one year term insuranceequal to the reduction of death coverage. This policy provides a guaranteed death benefit in the early yearsof the policy even if dividends are insufficient to maintain level coverage.

O INDETERMINATE PREMIUM WHOLE LIFE — This is a type of whole life policy that provideslow current premium costs for a specified period. The insurer may then increase premiums. Thecharacteristics of and benefits provided by this policy are similar to other contracts. This type of plan allowsfor a change of premium due to a change in the investment income of the insurer. Therefore, future premiumadjustments are based upon the insurer's investment performance. These contracts are sometimes referredto as non-participating policies but with non-guaranteed premiums. Premiums may be raised or lowered bythe company but can never exceed the guaranteed maximum. This innovative type of policy was adoptedby insurers in order to offer lower cost life insurance. Term insurance, today, may also be written withindeterminate premiums.

O CURRENT ASSUMPTION WHOLE LIFE (CAWL) — This is a new nontraditional policy whichprovides the potential for low-cost whole life insurance protection. CAWL utilizes current interest rates incash value determination plus the use of an indeterminate premium structure. It is sometimes referred to asinterest sensitive whole life insurance:

According to Life Insurance by Black and Skipper, CAWL policies are either of the low premium or highpremium variety. Both possess several characteristics including but not limited to:

(1) The use of an accumulation account which is made up of the premium, less expense and mortalitycharges, and credited with interest based on current rates.

(2) Use of a surrender charge, fixed at issue, which is deducted from the accumulation account to derivethe policy's surrender value.

(3) Use of a fixed death benefit and maximum premium level at time of issue.

LOW PREMIUM TYPE — This type includes an indeterminate premium that is low initially. It alsocontains a redetermination provision which states that after a specified period, the insurance company canre-figure the premium.

NILA ©

L/H Licensing

Chapter 4—5

Page 52: FORWARD - WordPress.com · Return of premium p 6-8 6.0 Annuities p 7-1 Annuity principles and concepts p 7-1 Accumulation period versus annuity period p 7-1 Owner, annuitant and beneficiary

HIGH PREMIUM TYPE — The initial premium is relatively high. It possesses an optional pay-up orvanishing premium provision. This provision states that the policy owner may cease paying premiums oncethe policy's values are sufficient to pay-up the contract.

O INDEXED WHOLE LIFE — This policy possesses a face amount which increases along with rises inthe Consumer Price Index (CPI). These policies are classified according to whether the policy owner or theinsurer assumes the inflation risk. Under the type where the policy owner assumes the risk, the death benefitincreases each year in accordance with the CPI and the insurer bills the policy owner each year for the new,higher amount of insurance. The insurer agrees not to require the insured to demonstrate insurability for anyincrease in protection as long as each year's increase in coverage amount is accepted by the policy owner. The type where the insurer assumes the inflation risk is similar to the aforementioned approach, except thatthe premium charged initially by the insurer is loaded in anticipation of future face amount increases. Theseincreases in face amount do not alter the premium level paid. These policies possess a cap as to themaximum total permitted.

Another type of indexed life insurance product that is relatively new is equity indexed life insurance. Thistype of policy combines most of the features, benefits and security of traditional life insurance with thepotential of earned interest based on the upward movement of an equity index. Instead of the including aspecific interest rate as in a traditional whole life plan, interest earnings are credited based on increases inthe specific equity index (for example, the S&P index) to which the policy is linked. Therefore, creditedinterest is linked to an index without the downside risk connected with directly investing in the stock market. These policies are characterized by a guaranteed minimum interest rate, tax deferral of interest accumula-tions, and policy loan access. The equity index returns are designed to keep pace with or beat inflation whichprotects the policyholder against downside market risk. Equity indexed life insurance contracts combineterm life insurance with an investment feature, similar to a universal life plan. Death benefit amounts arebased upon the coverage amount selected by the contract owner plus the account value.

O MINIMUM DEPOSIT — This is a method of financing life insurance and not an actual type of policy. It is best suited for individuals in high marginal tax brackets. It allows the policy owner to use policy loansto pay premiums due each year. For example, the policy owner is allowed each year to borrow, subject tocertain tax restrictions, that year's cash value increase and use it to pay the premium. The policy owner onlypays the difference between the premium due and the amount borrowed (plus interest on the policy loan).

O ENDOWMENTS — Endowments emphasize savings more than a traditional whole life policy. Endowmentlife insurance contracts pay a death benefit to a named beneficiary upon the death of an insured during aspecified "endowment" period; or it pays a cash value equal to the face amount of the policy at the end ofthe endowment period if the insured is still living. Remember, an endowment pays a death benefit if theinsured dies during the endowment or specified period. It does not pay the cash value upon death. The cashvalue will be paid to the policy owner/insured when the contract matures or "endows." Therefore, anendowment pays at the earlier of death or at the end of a specified period (i.e., at the end of the endowmentperiod).

Endowments were originally designed to combine life insurance with a heavy savings element. Endowments may be issued for specified periods such as five, ten, twenty, twenty-five, or thirty years, or upto a specified age such as 65. An endowment provides an accelerated or quicker cash value build-up ofany whole life policy since it possesses higher premiums than other traditional whole life plans. Anendowment pays at the earlier of death or at the end of a specified period. An endowment provides the in-sured/policy owner with the specified amount needed at the end of a specified period.

NILA ©

L/H Licensing

Chapter 4—6

Page 53: FORWARD - WordPress.com · Return of premium p 6-8 6.0 Annuities p 7-1 Annuity principles and concepts p 7-1 Accumulation period versus annuity period p 7-1 Owner, annuitant and beneficiary

While a whole life policy "matures" or "endows" at age 100, an endowment matures or endows at the endof a specified period of time. For example, a $50,000 whole life policy endows at age 100. A $50,000, 20year endowment endows at the end of the 20 year period. If John wished to make sure that his life insurancepolicy paid him $50,000 at retirement (i.e., age 65), and that it would pay his beneficiary $50,000 if he diedprior to age 65, he would purchase a $50,000 endowment at age 65. The initial premium for an endowmentpolicy is greater than traditional whole life contracts. Since the policy "endows" or matures at the end of thespecified period, the initial premiums will be higher and the cash value increases quicker. Endowment hasthe highest premium cost per thousand dollars of coverage, all things being equal. If Tom buys a ten year$20,000 endowment and dies in the third year of its existence, his beneficiary will receive $20,000. If Tomlives the entire ten years, he will receive a check for $20,000. The cash value of an endowment equals thepolicy's face amount at the end of the endowment period.

Endowments, while not as popular today as they were thirty years ago, were primarily designed to providefunds for educational purposes or to supplement one's retirement. The policy would provide a specificamount of money at the end of a definitely known period of time.

! A variation of the endowment principle may be illustrated by a retirement income policy. In thisplan, the amount payable upon survival is greater than the face amount, and the amount payable atdeath is the face amount or cash value, whichever is greater. This type of policy is generally utilizedin pension plans.

! Semi-endowments are plans that pay upon survival only one-half the sum payable in the event ofdeath during the specified endowment period. Pure endowments pay nothing if the insured diesduring the endowment period. They only pay if the insured survives. Juvenile endowments arethose covering children which mature at a specified age. These plans help to meet the educationalgoals of the policy owner.

Recommendation – You are an agent or producer. Your potential client wishes to buy a life insurancepolicy that will provide her with $20,000 in twenty years. What do you recommend she purchase? The onlytype of life insurance policy that will provide the insured with $20,000 in 20 years is a $20,000 twenty yearendowment. The policyholder would not buy a twenty-pay life policy to satisfy her objective. Remember,this limited payment policy will not mature or "endow" until age 100. An endowment's cash value equals its face amount at the end of the endowment period.

FLEXIBLE PREMIUM POLICIES

Most whole life insurance policies are characterized by a fixed or level premium. This is why traditional whole lifepolicies are referred to as level death benefit, level premium life insurance. During the 1970's, insurers developeda new strain of policy which provided greater flexibility for the policy owner. These "non-traditional" policies arecharacterized by flexible coverage amounts and premiums.

ADJUSTABLE LIFE (AL) — This type of permanent insurance product combines elements of traditional fixedpremium whole life insurance with the potential to adjust the coverage or face amount based on the policy owner'schanging needs. The best description of adjustable life has been provided by the authors of "Life Insurance," Blackand Skipper, Prentice-Hall, 1985.

NILA ©

L/H Licensing

Chapter 4—7

Page 54: FORWARD - WordPress.com · Return of premium p 6-8 6.0 Annuities p 7-1 Annuity principles and concepts p 7-1 Accumulation period versus annuity period p 7-1 Owner, annuitant and beneficiary

"At any point in time, adjustable life is a level-premium, level-death benefit policythat may assume the form of any traditional type of life insurance."

Characteristics — AL provides an adjustable death benefit, cash value and possesses all the features of traditionalwhole life policies. Its distinguishing characteristic is a provision called the adjustment provision. The advantageof this policy is that it permits the policy owner to make prospective adjustments (i.e in the future) to the policy'scoverage amount. The policy premium is fixed for the policy year if an adjustment in coverage is made (obviouslyif more coverage is purchased the consumer pays more). An individual whose income has been fluctuating duringthe past several years or a couple who plans to have children over the next several years are illustrations ofprospective clients who may purchase an adjustable life policy so that they possess flexibility to satisfy their"changing needs." Therefore, this is the type of life insurance policy available to an individual who wishes to havethe opportunity to make changes or alterations to his or her contract each year.

There may be some confusion regarding premiums for adjustable life. While the policy owner may pay more or lessper year in the future than the original premium, the premium is adjustable. However, remember our originaldefinition: At any point in time Adjustable Life insurance is a level-premium level-death benefit policy. This meansthat whenever a coverage amount is increased, the premium then due is level for the coming policy year. If the policyowner decides to increase the coverage amount, the insured party must always prove insurability. To simplify, anadjustable life policy is a traditional whole life policy with an adjustable death benefit. This type of policy ischaracterized by prospective (i.e., future) adjustments only.

UNIVERSAL LIFE (UL) — Like Adjustable Life, Universal Life insurance provides its owner with moreflexibility than a traditional whole life plan. It may be referred to as an adjustable form or nontraditional type of lifeinsurance since it allows the policy owner to change the coverage amount at his or her discretion. This type of policyis sometimes characterized as an interest sensitive life product since it utilizes changing interest rates (or rate ofreturn) in order to determine cash values. These changing interest rates are not used to determine death benefits norfuture premiums. UL premiums pay for pure protection (i.e., term insurance) and a portion is deposited into its cashvalue. The cash value in this policy may be referred to as: (1) a cash value fund; (2) a cash savings plan; (3) policyequity; or (4) a savings aspect, element or feature. Just as in traditional whole life, a fixed interest rate is paid on thecash savings plans as it accumulates. The minimum fixed interest rate paid on the cash value of a universal life con-tract is equal to the maximum paid (4%) on a traditional whole life policy's cash value. This interest rate paid onthe cash savings plan will vary (i.e., interest sensitive) depending upon market conditions and is responsible for theyearly increase in the cash savings. Therefore, the interest rate is guaranteed for the policy year.

Most importantly, UL is also characterized by a flexible premium. This means that the policy owner may paywhatever amount of premium he or she wishes to each year. This flexibility could be a disadvantage for the ownerof the policy if it is not managed properly. For example, if premiums are not paid following the first year, in orderto keep coverage in force, the cost of death protection will be withdrawn from the cash savings plan. In this manner,the policy can pay for itself for as long as there is cash savings. If no additional premiums are paid, the policycontinues to use the cash value to keep coverage in force. However, if in the future there is not enough cash to payfor death protection, the policy could lapse. If only a small amount of cash value remains, the owner has the optionof using whatever cash is available to purchase as much protection as the cash will buy. Therefore, the death benefitin a universal life policy may not be guaranteed and in some cases may be dependent upon the amount of cash valueavailable (if the policy owner is not funding the policy by paying premiums). Again, remember that the death benefitmay not be guaranteed (i.e., if not managed properly), the interest rate paid on the cash value is guaranteed (i.e., forthe policy year) and sometimes the amount of coverage provided for the year will be dependent upon the amount ofcash value available. In a UL policy, the premiums, cash value and the face amount can be adjusted. However, itis not identical to adjustable life nor is it backed by equities as in variable life products.

NILA ©

L/H Licensing

Chapter 4—8

Page 55: FORWARD - WordPress.com · Return of premium p 6-8 6.0 Annuities p 7-1 Annuity principles and concepts p 7-1 Accumulation period versus annuity period p 7-1 Owner, annuitant and beneficiary

! UL provides flexible death benefit, flexible premium life insurance. UL policies are transparent since theyare characterized by unbundled or decoupled premiums. This means that the policy owner is provided withinformation describing where the policy costs are allocated. In other words, the policy owner receives abreak-down of premiums, death benefits, mortality charges, expenses and cash values. This shows the policyowner the disposition of the policy funds. Funds withdrawn from the cash savings plan may not be subjectto interest payments if they are used to pay premiums.

! UL offers two death benefit patterns including Option A and Option B. Under Option A, a level deathbenefit is provided. The net amount at risk (NAR) is adjusted after each month. This means that a mortalitycharge is deducted from the policy's cash savings plan on a monthly basis. Therefore, the cash value andNAR (benefit) together provide a fixed death benefit. Option B provides an increasing death benefit as thecash value increases. Therefore, at death the policy pays the face amount plus the cash value.

! Some insurers offer a target premium to allow some policy owners to plan premium payments on a regularbasis. Since the premium is flexible, many policy owners who do not manage the plan may see theircoverage lapse. To avoid possible tax problems, premium allocations to a universal life policy's cash valuemust comply with tax law (IRS) guidelines.

! Fixed interest rates paid on the cash value generally include a guaranteed minimum of 4%, although a fewplans in existence may possess a guaranteed minimum of up to 7% in some instances. Therefore, interestmay be paid at either the contract's guaranteed minimum rate or at the current interest rate as declared by theinsurer. The fixed interest rate may change each year based on money instruments (i.e., treasury bill, moneymarket rates, etc.). Since the interest rate paid on the cash value changes each year and the amount of cashvalue changes, an annual statement must be provided to the policy owner each year by the insurer. Thisannual statement identifies the upcoming interest rate for the coming year, the cost of coverage, the currentcash value amount, other expenses applicable and additional relevant information.

SEC REGULATED POLICIES

SEC REGULATED POLICIES — The Securities and Exchange Commission (SEC) regulates securitytransactions whether it involves the trading of stocks, bonds or variable insurance products. The Financial IndustryRegulatory Authority (FINRA) is the entity that oversees securities firms in the U.S. Formerly, the NationalAssociation of Securities Dealers (NASD), this regulatory body governs securities activities, administers securitiesexams and issues licenses. Therefore, producers who wish to solicit variable products must hold a life insurancelicense and a securities license.

VARIABLE LIFE — Variable life insurance (VL) also combines guaranteed death protection and a non-guaranteed savings feature. However, the cash value is invested in securities such as common and preferred stocks,bonds, mutual funds and other like forms. Therefore, due to the growth potential of the cash value, the policy mayout perform other plans (i.e., traditional whole life) characterized by a fixed rate of interest. In other words, the cashvalue of a VL policy is deposited into a separate account which, in turn, is invested in securities. Therefore, the cashvalue is not guaranteed. But this policy does provide the potential for a greater rate of return on its cash value basedon the performance of the securities in which the cash value is invested. Variable life insurance is a type of lifeinsurance contract where the death benefit or cash surrender value may vary depending upon the investmentperformance of the cash value (i.e., separate account) that is invested in securities. The annual premiums paid onVL are pre-determined or fixed. Other characteristics include:

NILA ©

L/H Licensing

Chapter 4—9

Page 56: FORWARD - WordPress.com · Return of premium p 6-8 6.0 Annuities p 7-1 Annuity principles and concepts p 7-1 Accumulation period versus annuity period p 7-1 Owner, annuitant and beneficiary

! A guaranteed minimum coverage amount (i.e., death benefit).! VL possesses cash values that fluctuate daily based on stock market performance and include the normal

policy provisions and rights found in traditional whole life policies.! VL is a product that its owner can use to keep pace with economic conditions or to use as a hedge against

inflation, although it does not provide a guaranteed hedge (since the cash value is invested in securities). ! This policy may be advantageous for a person who wishes to experience a potentially greater rate of return

or degree of growth of the cash savings value. It also provides a policy owner with flexibility and controlover the investment portion of the contract.

! A FINRA (formerly NASD) license is needed in order to sell this or any type of variable product. Thislicense is also known as a Series 6 license. A securities license is required because VL is considered asecurity as well as a life insurance policy. In addition, a life insurance license is also needed. Therefore,these products are regulated at the Federal and State level.

VARIABLE / UNIVERSAL LIFE — This plan combines some characteristics of variable life and universal lifeinsurance. VUL offers the policy owner a combination of investment options with a flexible premium payment /expense deduction method and a guaranteed minimum death benefit. It is the equivalent of a variable life policy witha flexible premium payment. It also possesses a guaranteed minimum coverage amount. It may also be referred toas Universal/Variable Life. It is an insurance and investment product with a flexible or variable premium.

SPECIALIZED FORMS OF LIFE INSURANCE

SPECIALIZED AND OTHER FORMS OF LIFE INSURANCE POLICIES — Additional life insurancepolicies that are in existence include but are not limited to:

FAMILY INCOME PLAN— This type of combination plan consists of a whole life policy with a decreasingterm insurance rider attached. Family income plans include a base plan of whole life insurance. In addition, adecreasing term insurance rider is added to it to provide additional income on a monthly basis if the insured diesduring the decreasing term period. If the insured dies during the specified term period (i.e., 10 years), the beneficiarywill receive a monthly income benefit for the remaining part of the period. Once this occurs, the lump sum wholelife death benefit will be paid to the beneficiary. In other words, income is paid to the beneficiary if the insured diesduring the term period and then the lump sum whole life face amount is paid out. Since decreasing term insuranceis involved, every month the insured lives, the FIP rider decreases in value. If the insured lives to the end of theselected period, the FIP rider ends. All that remains in this case is the whole life death coverage.

The coverage amount of the underlying whole life policy is generally paid after all income is exhausted from the terminsurance rider. For example, a family income rider is attached to a $50,000 whole life policy paying $1,000 permonth to a beneficiary for a potential 20-year period. This potential 20 year period is measured from the date ofpurchase of the contract. So if the insured dies after the policy was in effect for 10 years, his or her beneficiary wouldreceive $1,000 per month for 10 years and upon completion of the monthly income, the $50,000 whole life faceamount is paid. A few insurers pay the whole life face amount first and then the income until its exhausted.Therefore, if the insured dies during the twenty year period, the beneficiary receives monthly income for theremaining part of that twenty year period. Theoretically, a family income plan will pay a lump sum and monthlybenefits to a beneficiary during the period of time that the dependent children are at home and need care.

Another specialized type of policy available which is similar to the family income policy is the family maintenancepolicy. This is a whole life policy with a level term insurance rider attached. Using the same information as theprevious example, if the insured dies during the 20 year period, the beneficiary will receive monthly income from

NILA ©

L/H Licensing

Chapter 4—10

Page 57: FORWARD - WordPress.com · Return of premium p 6-8 6.0 Annuities p 7-1 Annuity principles and concepts p 7-1 Accumulation period versus annuity period p 7-1 Owner, annuitant and beneficiary

the date of death for 20 years. Again, the lump sum whole life death benefit may be paid at the time of death or atthe end of the income period (i.e., 20 years). This policy is more expensive than the family income policy since levelterm insurance will be more expensive than decreasing term coverage. Like the family income plan, if the insuredsurvives the initial 20 year period and dies thereafter, only the whole life death benefit is paid to the beneficiary.Again, in order for the beneficiary to receive any income payments from the rider, the insured must die during thetwenty year period. If this occurs, the beneficiary receives monthly income for twenty years (and then the lump sumwhole life death benefit is paid).

FAMILY POLICY — This contract covers all family members including husband, wife and all children whethernatural or adopted. Whole life insurance covers a primary insured while the rest of the family is covered by levelterm insurance (that is convertible in later years if desired). Whole life coverage will be purchased on the life of theprimary insured (i.e., breadwinner) with level term life insurance covering the spouse and all children ( as they areborn into the marriage, whether natural or adopted. Therefore, this policy is a combination of whole life and levelterm insurance. This type of contract is also known as a family protection policy or family plan.

! All children in the family are automatically covered even if born (or adopted) after the policy has beenissued. In other words, newborns are covered from the moment of birth. No rider has to be added to providethis coverage. Coverage is usually provided up to age 18 or 21, whatever the policy language states. Sinceterm life protection is provided, the spouse and children will possess a conversion right generally availableto those covered by term life coverage. The larger the family the more economical the family policy will besimply because more individuals will be covered by the plan. Momentarily you will find that the premiumis based solely on the life of the primary insured.

! Most family policies provide an amount of whole life coverage on the primary insured; 50% of that amountin term life coverage on the spouse; and 25% of the amount on the primary insured is provided to cover eachchild as he or she is born. Premiums charged are based on the life or age of the primary insured. The policydoes not terminate if the primary insured dies. Level term coverage continues for survivors as long aspremiums are paid. Some insurers provide paid-up protection for the surviving family when the primaryinsured dies. Therefore, this policy provides coverage for all family members although the benefit levels aredifferent.

! Individual Policy With a Family Rider — Rather than offering a family policy that covers the entirefamily under one contract, an insurer may allow an insured to purchase an individual whole life policy andlater add a family rider to it which will cover a spouse, all natural children and any adopted children. Thisaccomplishes the same objective as the family policy since everyone in the family will be covered. However,the scope of coverage is different when comparing both the family policy and the family rider. The familypolicy covers everyone in the family including the primary insured. The family rider only covers the spouse,natural children and adopted children. The family rider does not cover the primary insured since he or sheis covered by the individual policy.

JOINT WHOLE LIFE (FIRST-TO-DIE) POLICY — A joint life policy covers two or more individualsunder one contract. Generally it is purchased to cover a husband and wife. When the first spouse dies, the faceamount is paid to the named beneficiary. The contract also ends. In other words, protection ceases after the firstspouse dies. No further life insurance is provided under the policy for the survivor. This type of policy is generallyreferred to as "first to die" life insurance. Therefore, a joint life policy pays a death benefit and terminates when thefirst insured dies.

NILA ©

L/H Licensing

Chapter 4—11

Page 58: FORWARD - WordPress.com · Return of premium p 6-8 6.0 Annuities p 7-1 Annuity principles and concepts p 7-1 Accumulation period versus annuity period p 7-1 Owner, annuitant and beneficiary

SURVIVORSHIP WHOLE LIFE — This is a policy generally covering two insured lives, usually for a husbandand wife. No death benefit is paid when the first insured dies. When the second insured or "last survivor" dies, thedeath benefit is paid. This type of policy is also referred to as a "last to die" or "joint and last survivor" life policy.This type of policy would be purchased because it is less expensive than buying two separate whole life policies tocover the spouses and the death benefit is generally used to pay off any federal or state death taxes when the secondspouse dies. Therefore, this plan will cover two (or more) lives under one contract but only pays a death benefitupon the second (or last) insured's death.

JUVENILE LIFE INSURANCE — These types of policies are written on children. A parent will be the ownerof the policy (i.e., third party ownership) and the child is the insured. Insurance on a child may be purchased forburial expenses or to protect the child's insurability. Some parents purchase these plans to begin a savings plan fortheir child. A jumping juvenile policy is available which is characterized by an increasing death benefit when thechild reaches the age of majority or a specified age (i.e., age 18, 21 or 25). The coverage amount "jumps up" at thattime with no premium increase. Some insurers refer to such a plan as a junior estate builder.

CREDIT LIFE INSURANCE — This type of coverage protects a lender against the premature death of aborrower before the latter party has the opportunity to pay off a debt or loan. The amount of life insurance on thelife of the borrower cannot exceed the amount of the borrowed funds. Decreasing term life insurance is generallyused to provide such coverage. The lender or creditor (i.e., bank) is the policy owner, pays the premium, and is thebeneficiary. A lender or creditor cannot require a borrower or debtor to purchase credit life coverage as a conditionof securing a loan. The lender also has an insurable interest in the life of the borrower. By purchasing this type ofbusiness life insurance, the lender will not be left with an uncollectible debt should the borrower die before payingoff the loan. This is another illustration of third party ownership.

INDUSTRIAL LIFE INSURANCE — Industrial or home service life insurance was previously sold by debitagents who worked for an insurer and visited policy owners on a weekly or monthly basis to collect premiums.Policies were sold in face amounts of $1,000 or less. They were generally of a whole life type building some equityrather than term insurance.

QUIZ

1. A life insurance policy which matures at the termination of a specified period is known as:

A. Whole life policy C. Limited-pay policy

B. Variable life policy D. Endowment

2. Mrs. James wishes to purchase a life insurance policy that will help her survivors pay off her mortgage in case she dies.

What type of policy would you recommend she purchase?

A. Flexible premium annuity C. Level term

B. Modified life D. Decreasing term

3. A whole life policy owner's cash savings value will equal the policy's face amount at what age?

A. Age 55 C. Age 70

B. Age 65 D. Age 100

NILA ©

L/H Licensing

Chapter 4—12

Page 59: FORWARD - WordPress.com · Return of premium p 6-8 6.0 Annuities p 7-1 Annuity principles and concepts p 7-1 Accumulation period versus annuity period p 7-1 Owner, annuitant and beneficiary

4. Paul is insured by a family income plan. The policy, purchased in 1994, includes a whole life death benefit of $50,000

with an income rider of twenty years that will pay $1,000 per month. If Paul dies ten years after he buys this policy, what

will be paid to his beneficiary?

A. $50,000 at death and $1,000 per month for 20 years

B. $50,000 at death and $1,000 for month for 10 years

C. $1,000 per month until the year 2004 and then the $50,000 benefit

D. $1,000 per month until the year 2014 and then the $50,000 benefit

5. What portion of a mortgage redemption plan decreases annually?

A. The nonforfeiture values C. The annual premium

B. The cash values D. The death benefit

6. A family policy is a combination of:

A. Whole life and decreasing term C. Whole life and level term

B. Whole life and an annuity D. Whole life and increasing term

7. All of the following are characteristics of a universal life policy, EXCEPT:

A. Fixed interest paid on cash value C. Adjustable death benefit

B. Bundled premiums D. Transparency

8. What increases in a whole life policy with every successive premium payment?

A. Coverage amount C. Cash value

B. Nonforfeiture values D. Face value

9. A type of life insurance policy whose premium is initially lower than a traditional whole life plan and increases each year

for an introductory period best describes:

A. Universal life C. Variable life

B. Flexible premium life insurance D. Graded premium whole life

10. What type of life insurance policy allows insurers to adjust premiums to reflect any changes in investment income?

A. Limited pay whole life C. Indeterminate premium whole life

B. Permanent life insurance D. Current assumption whole life

11. Each of the following policies will lapse if an annual premium is not paid to the insurer, EXCEPT:

A. Ten pay life C. Current assumption life

B. Universal life D. Annual renewable term

12. A method of financing life insurance which is best suited for individuals who are in higher tax brackets is known as:

A. Split life C. Joint life

B. Minimum deposit insurance D. Deposit term insurance

13. John's client would like to purchase a whole life policy that will provide her with $30,000 in ten years. Which of the

following will help accomplish this objective?

A. Ten pay life policy C. Ten year endowment

B. Single premium life D. Ten year renewable term

NILA ©

L/H Licensing

Chapter 4—13

Page 60: FORWARD - WordPress.com · Return of premium p 6-8 6.0 Annuities p 7-1 Annuity principles and concepts p 7-1 Accumulation period versus annuity period p 7-1 Owner, annuitant and beneficiary

14. A variation of the endowment principle may be illustrated by a retirement income policy. In this plan, the amount

payable upon survival is greater than the face amount. The death benefit payable is the greater of the face amount or the:

A. Mortality factor C. Performance of the securities

B. Bundled premium D. Cash value

15. Which of the following would not be covered by a family rider?

A. A spouse C. A primary insured

B. A natural child D. An adopted child

16. The primary portion of an adjustable life policy that is adjustable is the:

A. Beneficiary designation C. The surrender values

B. Policy expenses D. The coverage amount

17. What type of license must a producer hold in order to engage in the sale of variable life insurance?

A. A securities license issued by FINRA C. A non-resident agent license

B. A broker's license D. A life insurance license and a securities license.

18. A policy covering a child characterized by an increasing death benefit at a future age with no premium increase best

describes:

A. Split life policy C. Payor benefit life

B. Jumping juvenile policy D. Junior endowment

19. Which of the following types of life insurance is characterized by guaranteed coverage but provides no guarantees with

regard to cash value build-up?

A. Variable life policy C. Joint life policy

B. Universal life policy D. Modified life policy

20. Bob purchased a policy that will pay him $50,000 at retirement. If he dies prior to this time, the policy will pay his

family $50,000. What type of policy did Bob purchase?

A. Retirement income policy C. Endowment

B. Retirement annuity D. Variable life

21. Insurance companies use age 100 to calculate life insurance death benefits because:

A. It allows them to charge higher premiums

B. It makes sure they never have to pay a death benefit

C. They assume everyone will have died by that age

D. They assume that mortality rates will decrease by then

22. A 40 year old desires a life insurance policy that will allow him to increase or decrease the death benefit in the future

would purchase a:

A. Whole life policy C. Adjustable life policy

B. Modified life policy D. Renewable term life policy

NILA ©

L/H Licensing

Chapter 4—14

Page 61: FORWARD - WordPress.com · Return of premium p 6-8 6.0 Annuities p 7-1 Annuity principles and concepts p 7-1 Accumulation period versus annuity period p 7-1 Owner, annuitant and beneficiary

23. Jack buys a $100,000 20-year Family Income Policy in 1990. The policy includes an income rider of $1,000 per month.

He dies in 1995. What will Jack's beneficiary receive?

A. $1,000 per month for life

B. $100,000

C. $1,000 per month until the year 2010 and then a $100,000 lump sum

D. $1,000 per month for twenty years and then a $100,000 lump sum

24. Family policies usually provide death benefit coverage:

A. For the primary insured only

B. For the working spouse only

C. For all family members although the benefits differ

D. That may be assigned to the tertiary beneficiary

25. Interest sensitive whole life policies, such as universal life, use varying interest rates:

A. In order to determine future premiums

B. In order to determine death benefits

C. In order to determine cash values

D. In order to determine non-forfeiture values

26. What type of policy did Bill buy if the contract states that the cash value will be $326 per $1,000 of coverage in year

eighteen?

A. Variable life C. Level term

B. Convertible life D. Whole life

27. Which of the following life insurance policies provides a policy owner with flexibility and control over the investment

portion of the contract?

A. Whole life policy C. Adjustable life policy

B. Variable life policy D. Continuous premium life policy

28. Ralph wants a life insurance policy that will pay him $10,000 in ten years. Which of the following would you

recommend to Ralph to help him meet his goals?

A. Ten pay life policy C. Ten year endowment

B. Twenty pay life policy D. An endowment at age 65

29. An insurance policy that is characterized by a level death benefit, a level premium and a cash savings value is known

as a:

A. Universal life policy C. Adjustable life policy

B. Straight life policy D. Convertible term life policy

30. What type of policy would you suggest your client purchase if she desires death protection and a cash value build-up but

is presently of limited means?

A. Whole life C. Universal life

B. Modified life D. Straight life

NILA ©

L/H Licensing

Chapter 4—15

Page 62: FORWARD - WordPress.com · Return of premium p 6-8 6.0 Annuities p 7-1 Annuity principles and concepts p 7-1 Accumulation period versus annuity period p 7-1 Owner, annuitant and beneficiary

UNIVERSAL LIFE

ANSWERS

1. D2. D3. D4. D5. D

6. C7. B8. C9. D10. C

11. B12. B13. C14. D15. C

16. D17. D18. B19. A20. C

21. C22. C23. C24. C25. C

26. D27. B28. C29. B30. B

CHAPTER 4 KEY CONCEPTS

Adjustable lifeConvertibilityCurrent assumption whole lifeDecreasing termEndowmentsEnhanced lifeEquity indexed lifeIndeterminate premiumIndexed whole life

Level termRenewabilitySingle premium lifeSpecialized life insuranceTerm insuranceUniversal lifeVariable lifeVariable universal lifeWhole life

NILA ©

L/H Licensing

Chapter 4—16

Page 63: FORWARD - WordPress.com · Return of premium p 6-8 6.0 Annuities p 7-1 Annuity principles and concepts p 7-1 Accumulation period versus annuity period p 7-1 Owner, annuitant and beneficiary

TYPES OF WHOLE LIFE INSURANCE

EXHIBIT 1

Ë Traditional Whole Life (WL)/Straight Life/Continuous Premium Life

! Death benefit ! Cash value (i.e., equity) is a savings feature or element ! Permanent insurance ! Level/Fixed premium ! All premiums returned if you live to age 100

Ë Limited Payment Whole Life

! Death benefit ! Cash value ! Permanent insurance ! Pre-determined premium for limited payment period

Ë Modified/Graded Premium Whole Life

! Death benefit ! Cash value ! Permanent insurance ! Lower premium than whole life in the early years ! Higher premium than whole life in year six and fixed thereafter

Ë Adjustable Whole Life

! Death benefit! Cash value ! Permanent insurance

! Adjustable premium (although fixed for the year after adjustment made) ! Flexible/Adjustable death benefit based on prospective changing needs

NILA ©

L/H Licensing

Chapter 4—17

Page 64: FORWARD - WordPress.com · Return of premium p 6-8 6.0 Annuities p 7-1 Annuity principles and concepts p 7-1 Accumulation period versus annuity period p 7-1 Owner, annuitant and beneficiary

Ë Universal Life (UL)

! Flexible death benefit ! Cash value (money market rates) ! Guaranteed interest rate each year ! Combines term insurance and cash savings plan ! Permanent insurance product but death benefit is not guaranteed ! Flexible premium ! Death protection may depend upon the amount of available cash value each year

Ë Variable Life (VL)

! Cash value deposited in separate account and then invested in securities ! Permanent insurance ! Fixed premium ! Death benefit and cash value will vary based on investment performance ! Guaranteed minimum death benefit ! No guarantees on the cash value ! Potential for greater rate of return on cash value based on performance

Ë Universal Variable Life (or Variable Universal Life)

! Hybrid of UL and VL ! Flexible premiums

Ë Endowments

! Death benefit ! Cash value ! Higher premiums (than WL) ! Accelerated or quicker cash value build-up ! Pays if insured dies or if he/she survives endowment (i.e., specified) period ! 10-year; 20-year; 30-year, endowment at age 65

Ë Term Insurance

! Temporary protection! Lowest initial premium! Level or decreasing death benefit! Level term is renewable

! No cash value

NILA ©

L/H Licensing

Chapter 4—18

Page 65: FORWARD - WordPress.com · Return of premium p 6-8 6.0 Annuities p 7-1 Annuity principles and concepts p 7-1 Accumulation period versus annuity period p 7-1 Owner, annuitant and beneficiary

EXHIBIT 2

COMBINATION PLANSAND

VARIATIONS

FAMILY INCOME FAMILY POLICYJOINT WHOLE LIFE

LAST-SURVIVORWHOLE LIFE

! WL and Decreasing

Term

! WL and Level Term

! Does not terminate

when primary insured

dies

! Pays when first insured

dies and then policy ter-

minates

! WL policy is used

! Covers two lives and

pays only when second

insured dies and then

policy ends. WL used

C Family Rider does not cover

primary insured. It only

covers spouse, natural children

and adopted children.

NILA ©

L/H Licensing

Chapter 4—19

Page 66: FORWARD - WordPress.com · Return of premium p 6-8 6.0 Annuities p 7-1 Annuity principles and concepts p 7-1 Accumulation period versus annuity period p 7-1 Owner, annuitant and beneficiary

LIFE INSURANCE POLICIES

DEATH

BENEFITS

CASH VALUE PREMIUM POLICY

LOANS

PARTIALWITHDRAWALSOF CASH VALUE

SURRENDERCHARGES

TERM Level or

Decreasing

None L increases at each

renewal. D is fixed.

NO NO NO

WHOLE LIFE Fixed, Level or

Predetermined

Predetermined

and guaranteed

Level for period se-

lected

YES NO - to get

cash you

must borrow

NO

ADJUSTABLE

LIFE

Level but

changeable by

request

Predetermined

but new schedule

needed after each

negotiated policy

change

Level - Level may

change when policy

change is requested

YES, if

there is a

cash value

NO - to get

cash you

must borrow

NO

VARIABLE

LIFE

Guaranteed min-

imum DB but

may increase

from

investments

May increase or

decrease based

on investment

performance

Fixed, Level or Prede-

termined

YES NO - to get

cash you

must borrow

YES

UNIVERSAL

LIFE

Flexible. May not

be guaranteed if

owner is not

funding the pol-

icy with premi-

ums

Guaranteed mini-

mum interest rate

(4%). Interest rate

will vary each

year based on

money market

index

Flexible premium. Re-

quired 1st year target

(i.e., suggested level

premium) and then the

owner may pay flexi-

ble amount each year

or nothing at all.

YES.

Loans af-

fect inter-

est rate

credited to

cash value

YES YES

VARIABLE

UNIVERSAL

LIFE

Guaranteed pre-

mium level or

minimum + cash

value

Depends on in-

vestment perfor-

mance

Flexible premium. Re-

quired 1st year target

(suggested level pre-

mium)

YES YES YES

INTEREST

SENSITIVE

CURRENT

ASSUMPTION

Fixed level Scheduled and

guaranteed + ac-

cumulation fund

from excess inter-

est

May vary based on

experience. Guaran-

teed maximum level

YES YES YES

SINGLE

PREMIUM

WHOLE LIFE

Fixed, Level and

Predetermined

Predetermined

and guaranteed

Lump sum premium

paid at issue

Yes. Inter-

est rate on

loan tied to

interest

rate on

cash value

Generally not

available

YES

NILA ©

L/H Licensing

Chapter 4—20

Page 67: FORWARD - WordPress.com · Return of premium p 6-8 6.0 Annuities p 7-1 Annuity principles and concepts p 7-1 Accumulation period versus annuity period p 7-1 Owner, annuitant and beneficiary

LIFEINSURANCE

POLICYPROVISIONS

Page 68: FORWARD - WordPress.com · Return of premium p 6-8 6.0 Annuities p 7-1 Annuity principles and concepts p 7-1 Accumulation period versus annuity period p 7-1 Owner, annuitant and beneficiary

INTRODUCTION

All life insurance policies include several standard provisions which identify the duties, obligationsand rights of the parties to the contract. A provision may also be referred to as a policy clause. Theseprovisions are also intended to protect the owner of the policy as well.

LIFE INSURANCE POLICY PROVISIONS

POLICY PROVISIONS

The standard or usual provisions found in a life insurance contract are almost identical in language no matter whatlocale in which the policy is issued. The standard provisions found in a life insurance contract include but are notlimited to:

INCONTESTABLE CLAUSE — This provision states that, except in instances of fraud, the insurer may challengeinformation supplied by the applicant on the application only during the first two years that the policy is in force. Ifthe insurer does so, it can cancel the policy and return any premiums paid. This provision protects the insured andthe beneficiary after the first two years in case of any misstatements on the application. Therefore, this provisionallows an insurer to deny a claim or void a policy if the insured conceals material information or lies about healthhistory on an application, as long as the insurer contests it within the initial two years that the policy is in force.

PAYMENT OF PREMIUM — Also known as the premium paying provision, this provision states thatpremiums must be paid to an insurer or its representative in order for coverage to be provided. Insurers vary withregard to the payment modes made available. Some policyowners wish to pay on an annual basis which is the leastexpensive method. There are less insurer administrative or maintenance charges associated with an annual premium.The more costly methods of paying premiums include quarterly, semi-annual or monthly, the latter being the mostexpensive mode. Therefore, this provision permits the insurer to charge a small extra amount if the policy premiumis paid quarterly, semi-annually or monthly (i.e., any mode other than annual).

GRACE PERIOD — A grace period is an integral part of a life insurance policy. It is the period of time followingthe premium due date. Even though the premium has not been paid by the due date, coverage does not lapse. If thepayment is not received by the 31st day after it's due, coverage will lapse. The grace period is one provision thatprotects the policyowner against the unintentional lapse of the policy. This period is generally 31 days (in a fewStates it is 30 days) and coverage remains in effect during the days following the due date. The insurer offers thisgrace period because it wishes to keep business "on the books". If the insured dies during the grace period and beforea premium has been paid, the death benefit will be paid less a pro-rata share of the owed premium. A primarypurpose of the grace period, as well as the reinstatement and automatic premium loan provision, is to keep a lifeinsurance policy in force even when a premium payment is late.

REINSTATEMENT PROVISION — If a policy lapses because premiums are not paid, many life contracts allowreinstatement as long as it is requested within three (in some States five years) after the policy lapse. The request,however, is not the predominant factor. The insurer will require proof of insurability or good health and all back

NILA ©

L/H Licensing

Chapter 5—1

Page 69: FORWARD - WordPress.com · Return of premium p 6-8 6.0 Annuities p 7-1 Annuity principles and concepts p 7-1 Accumulation period versus annuity period p 7-1 Owner, annuitant and beneficiary

premiums (plus interest) owed must be paid to the insurer before reinstatement is granted. Once a reinstatementapplication is sent to the insurer, coverage will automatically be reinstated within 45 days unless the insurer statesotherwise. Whenever a policy is surrendered (i.e., given up by the policyowner), it cannot be reinstated. Anystatements appearing on a reinstatement application will be "contestable" for another two year period. Once the owedpremiums are paid and proof of insurability is provided, coverage will be reinstated as if it had never lapsed.

OWNER'S RIGHTS PROVISION — The life insurance contract includes an ownership provision. Thisprovision states that the policyowner possesses all rights contained in the policy. In any insurance policy or contract,the owner may name or change the beneficiary. In addition, if the policy is a cash value plan, the owner has theright to borrow against the savings feature. Further, if the contract is of a participating nature, the policyownerpossesses the right to receive any dividends payable. The policyowner may also select the frequency with whichpremiums may be paid (i.e., the premium mode such as annual, monthly, etc.). Finally, the owner may transfer one,some or all of these owner's rights to another party. This is known as assignment. The types of assignment availableinclude "absolute" and "collateral" or conditional assignment.

ASSIGNMENT — The assignment provision basically reiterates one of the policyowner's rights as stated in thecontract. It enables the policyowner to convey any or all of his or her policy rights to someone else. In other words,the owner possesses all the rights listed in the policy. If he or she transfers all policy rights, an absolute assignmenthas been effected. In this case, the entire contract has been transferred to another party. Absolute assignmentinvolves a complete transfer of the policy to another.

If the owner assigns one or some of the rights to another, but not all, he or she has engaged in a collateral orconditional assignment. This is a partial and temporary transfer of policy rights to another. In a conditionalassignment, the policyowner or assignor transfers a policy right to an assignee. Most conditional assignmentsinvolve the use by the assignor of a whole life policy's cash value as collateral in order to secure a loan from afinancial institution. If the cash value or a portion of it is "assigned" to the bank or "assignee", this party actuallybecomes the primary beneficiary with regard to it's interest. The assignee is only entitled to be reimbursed out ofthe policy proceeds for the amount of the outstanding loan (i.e., credit) balance. If the insured dies before theassignee is repaid, the death benefit is divided between the assignee and the stated primary beneficiary according totheir interest in the policy proceeds. In other words, under a conditional assignment, policy proceeds in excess of the"collateral" amount (i.e., the amount owed to the lender), pass to the insured's beneficiary. Therefore, pledging thecash value of a whole life policy as security for a loan involves a "collateral" assignment The following is adescription from the assignment provision of an actual policy:

"The policyowner may assign this policy. The insurer will not be responsible for the validity ofan assignment. The insurer will not be liable for any payments it makes or any actions it takesbefore notice of assignment is provided to it from the policyowner."

ENTIRE CONTRACT — The entire life insurance contract consists of the actual policy forms and a photocopyof the original life insurance application. According to Law and the Life Insurance Contract published by Irwinand Company, this provision "assures the policyowner that the policy includes every document affecting theprivileges and benefits of the contract. It also prevents the policyowner from contending that he or she was not awareof the statements made on the application." Therefore, the insurer cannot deny a claim in the future by stating thatit did not provide the policyowner with the entire contract. The application attached to the policy consists ofstatements concerning the proposed insured's health that are considered to be representations. Some insurancecontracts include a separate provision describing the "representations in the application." The "entire contract"actually includes the policy and all amendments, riders, attachments or endorsements attached to it. The followingis an example of an actual "entire contract" provision appearing in a life insurance policy:

NILA ©

L/H Licensing

Chapter 5—2

Page 70: FORWARD - WordPress.com · Return of premium p 6-8 6.0 Annuities p 7-1 Annuity principles and concepts p 7-1 Accumulation period versus annuity period p 7-1 Owner, annuitant and beneficiary

"The insurer has issued the policy in consideration of the application and payment of thepremium. A copy of the application is attached and is part of the policy. The policy with theapplication makes up the entire contract. All statements made by or for the insured will beconsidered representations and not warranties. This insurer will not use any statements in defenseof a claim unless it is made in the application and a copy of the application is attached to thepolicy when issued."

CONVERSION OPTION — This provision allows the policyowner the right to make changes in the type of policyowned. It allows the policyowner to convert: a term insurance policy to another term policy or a whole life contract;or a whole life policy to a term insurance policy. When converting from term to whole life, no insurability isrequired. If a conversion from whole life to term is requested, proof of insurability is needed in order to avoidadverse selection. This is due to the fact that the same face amount (i.e., whole life) is provided for a smaller (i.e.,term) premium.

MISSTATEMENT OF AGE OR SEX — This provision states that if the age of the insured is "misstated" onthe application, the policy will not be void or canceled. As mentioned in Life Insurance by Black and Skipper, "theamount of insurance shall be adjusted to be that which would have been purchased by the premium had thecorrect age been known". In other words, the amount of death proceeds will be adjusted (up or down) to reflectwhatever benefit the premium paid would have purchased at the correct age. A death benefit adjustment is involvedwhether the age was misstated higher or lower. This means that even if the applicant lied intentionally about his ageto save premium dollars, the insurer will not cancel or void the policy nor deny a death claim when it discovers theincorrect age. It will simply adjust the death benefit or the death claim amount. Therefore, if the age of the insuredis understated on the application, the insurer will pay a lower death benefit amount at death. If the age of the insuredis overstated, the insurer will pay a higher or additional face amount at death. If the sex of the insured is misstated,the insurer will take the same action of adjusting the face amount. Again, the insurer will not cancel the policy dueto these inaccuracies but will adjust the policy benefit.

For instance, assume that a 35-year old insured purchased a $10,000 policy for $100 and at his death it is determinedthat the actual age of the insured at the time of application was 40. The premium should have been $125. Therefore,since his age was misstated, the insurer will adjust the death benefit or pay 100/125 or 4/5ths of the original deathbenefit (i.e., $8,000). Whether the applicant understates or overstates his or her age on an application for lifeinsurance, the insurer will not deny a claim nor cancel the policy. It will simply adjust the death benefit up or downdepending whether or not the age of the insured was under or overstated.

MODIFICATION PROVISION — This provision states that any changes made to the contract must be endorsedand attached to the policy. It also states that only an officer of the insurer or authorized home office personnelpossess the authority to make any changes or modifications, or waive a policy provision.

FREE LOOK PROVISION — When a policy is delivered, the new owner possesses the ability to review thecontract for a specified period of days. If she changes her mind about keeping the policy, she may return it to theinsurer as long as this is accomplished within ten days (10) of the delivery date. If the new policyowner takes thiscourse of action and decides to return the policy to the insurer, he or she may do so and receive a full return ofpremium. The free look provision may also be referred to as the "right to examine" provision. This provision allowsthe policyowner to return the policy without giving a reason to the insurer. A free-look provision must be includedin all forms of life insurance except for flight or aviation insurance.

! Free look periods available vary in each state but generally are a minimum of 10 days. For the applicant toreceive a premium refund, the policy must be returned within 10 days from the date he or she receives it. The free look period begins when the policyowner receives the policy. It ends 10 days later.

NILA ©

L/H Licensing

Chapter 5—3

Page 71: FORWARD - WordPress.com · Return of premium p 6-8 6.0 Annuities p 7-1 Annuity principles and concepts p 7-1 Accumulation period versus annuity period p 7-1 Owner, annuitant and beneficiary

! For example, if a policy is delivered to the new owner on January 25th, the ten-day free look period beginson the 25th. It will end ten days later. This means that it would end on February 4th. To arrive at the correctanswer you should count day 1 as the 26th and so on.

EXCLUSION PROVISION — Life insurance policies contain an exclusion provision which allows the insurerthe right to deny a death claim if death is caused by any of the listed exclusions:

1. WAR — This specific clause or exclusion is contained in the exclusion provision. It allows the insurer todeny a death claim if the insured is killed in a war whether declared or undeclared. Obviously, the chanceof death increases in a war zone for military participants and civilians. Therefore, the possibility for adverseselection is increased dramatically. That is why this cause of death is excluded. Insurers may "retire" theirwar clause at their discretion and later reinstate it. There are two common war clauses in existence, one ofwhich will appear in most policy. The status war clause is a restrictive type clause which states that theinsured will not possess coverage under an individual life insurance policy while he or she is in the militaryeven if killed while away on furlough. The results clause states that coverage is not provided by anindividual policy if the insured dies while participating in military activities or during military maneuversof some sort. If the insured was on a furlough and killed he or she would be covered under an individualpolicy.

2. AVIATION — Most life insurance policies also deny death claims if an insured is killed during certain typesof high risk aviation activities including but not limited to the activities of:

(1) A stunt pilot.(2) A test pilot (i.e., experimental or armed forces pilots).(3) Student pilot.(4) Flight instructor.(5) Aircraft used for agricultural purposes (i.e., crop dusting).

These activities are not covered since the risk is greater to the insurer. However, passengers, crew membersor pilots of commercial aircraft will generally be covered. Other exclusions sometimes appearing in a lifeinsurance policy include hazardous occupations (i.e., auto racing, Hollywood stuntmen, etc. ) or hobbiesand avocations (i.e., sky diving, hang-gliding, parachuting or bungy jumping) although many policies coverthis latter group if an extra premium is collected.

BENEFICIARY PROVISION — This provision allows the policyowner to designate the name of a beneficiaryor beneficiaries who will receive the death benefit when the insured party dies. Individual, estate, minor, trust andclass designations may be utilized as a beneficiary. There are several options and classifications:

! PRIMARY BENEFICIARY — The primary beneficiary is the individual who receives the death benefitwhen the insured dies. This is the first or principal person in line to receive income tax free policy proceeds. In most cases, spouses designate one another as their primary beneficiary. The death benefit is paid to theprimary (or other) beneficiary income tax free. A beneficiary need not possess an insurable interest in thelife of the policyowner. The policyowner must possess an insurable interest in the life of the proposedinsured.

If a policyowner designates two (or more) primary beneficiaries who are to receive equal shares of the deathbenefit but one of them dies prior to the insured's death, the face amount is paid to the surviving primarybeneficiary. The estate of the deceased beneficiary does not receive any payment in this case (unless thiswas specifically requested as part of the contract).

NILA ©

L/H Licensing

Chapter 5—4

Page 72: FORWARD - WordPress.com · Return of premium p 6-8 6.0 Annuities p 7-1 Annuity principles and concepts p 7-1 Accumulation period versus annuity period p 7-1 Owner, annuitant and beneficiary

! CONTINGENT BENEFICIARY — This is the individual(s) who receives the death benefit only if theprimary beneficiary has died prior to the insured. The primary beneficiary must predecease the insured inorder for this secondary beneficiary to receive any proceeds. If the contingent beneficiary survives theprimary beneficiary, he or she will receive the income tax free death benefit when the insured dies. Atertiary beneficiary is third in line to receive policy proceeds when the insured dies if he or she survived theprimary and contingent beneficiaries.

The policyowner must designate all parties that he or she wishes to receive possible death benefits. If theowner fails to designate as beneficiary certain children in the family, no benefits will be paid to them evenif they are all legitimate heirs to the insured's estate. For example, assume that Mr. Jones purchases a lifeinsurance policy covering his life in the amount of $100,000. He designates his spouse as the primarybeneficiary. His only daughter is designated as the contingent beneficiary. However, Mr. Jones fails todesignate any of his four sons as beneficiaries. If his spouse and daughter are killed in an auto accident andfive years later Mr. Jones dies himself and has not made any alterations to the life insurance contract, thedeath benefit or policy proceeds will be paid to his estate. The proceeds are then subject to possible federaland state death taxes and probate fees before any of it is passed along to the sons according to Mr. Jones' will. This potential problem for the survivors could have been avoided if Mr. Jones had changed the beneficiarydesignation following the deaths of the primary and contingent beneficiaries and listed his four sons asprimary beneficiaries to receive an equal share of the proceeds in the event of his death. Even if Mr. Joneshad listed his sons as contingent beneficiaries along with his only daughter when he initially purchased thepolicy, the proceeds would have been payable to them immediately when he died (and would not have beentied up in probate).

Minors are generally named as contingent beneficiaries. However, proceeds are not paid to a minor upon thedeath of the insured. It is generally necessary for a guardian to be appointed on behalf of the minorbeneficiaries.

! SUCCESSION — The policyowner may designate any succession of beneficiaries he or she desiresincluding primary, contingent, tertiary, fourth in line, and so forth. Other persons, minors, trusts, or estatesmay also be named as beneficiaries. The policyowner may name anyone he or she desires as the beneficiary. For example, the owner may name his or her alma mater as beneficiary and no one will be able tosuccessfully contest the designation since the owner may name anyone or anything he or she desires. A trustcan also be named as a beneficiary and can be used to provide for the care of a minor child.

! REVOCABLE BENEFICIARY — Most designations are revocable. This means that the policyownermay modify, alter or change them at any time at his or her discretion. This is one of the owner rightsprovided by the policy. If the owner wishes to change the beneficiary he or she may simply request a"change of beneficiary form", complete it and return it to the insurer. The policyowner is not legallyrequired to notify anyone other than the insurer when changing a beneficiary designation.

! IRREVOCABLE BENEFICIARY — The policyowner or a court of law may designate an irrevocablebeneficiary. In this case, the beneficiary designation cannot be changed or modified without the permissionor consent of the named beneficiary. In other words, the irrevocable beneficiary must approve any revisionof the beneficiary designation. In addition, even though the policyowner retains all other ownership rightsin the policy, he or she must secure permission from the irrevocable beneficiary in order to exercise anypolicy rights such as borrowing against the cash value in a whole life policy. In addition, an irrevocablebeneficiary has the right to receive a copy of the policy if he or she desires.

NILA ©

L/H Licensing

Chapter 5—5

Page 73: FORWARD - WordPress.com · Return of premium p 6-8 6.0 Annuities p 7-1 Annuity principles and concepts p 7-1 Accumulation period versus annuity period p 7-1 Owner, annuitant and beneficiary

! COMMON DISASTER CLAUSE — How a policy responds to common disaster deaths is governed bythe Uniform Simultaneous Death Act. The common disaster clause or provision states that if the insured andprimary beneficiary die in a common disaster (i.e., plane crash) and it cannot be determined who died first,the insured will be considered to have survived the primary beneficiary (or died last). In other words, theprimary beneficiary will be considered to have pre-deceased the insured. Therefore, when the insured dies,the face amount is paid to the contingent beneficiary. Otherwise, if the insured died first, the death benefitis payable (i.e., due) to the primary beneficiary. If the primary beneficiary then died, the face amount willbe paid to the estate of the primary beneficiary and not directly to a contingent beneficiary. Again, possibledeath taxes and probate charges may be assessed before the heirs receive what remains. Therefore, thisprovision protects a contingent beneficiary. Some life insurance policies include a survivorship clausewhich states that if the beneficiary dies simultaneously with the insured or during the fourteen days (i.e., 14days) immediately following the date of death of the insured, the policy will pay the death benefit as if thebeneficiary had died before the insured.

! SPENDTHRIFT CLAUSE — This provision protects a beneficiary from creditors with regard to lifeinsurance proceeds. When the death benefit is left with the insurer, no creditors can attach a lien of any kindto the proceeds. The spendthrift clause also protects a beneficiary by minimizing or restricting the use ofproceeds as long as the insurer holds them. Only after proceeds have been distributed can the beneficiaryuse or assign benefits. Therefore, this provision states that proceeds payable to a beneficiary may not betransferred, attached, encumbered or commuted in any way if they are left with the insurer once an insureddies!

! TRUSTS AND CLASSES — Trusts may be named as beneficiary of a life insurance policy and managethe proceeds upon the insured's death. This is the most advantageous designation to use when a policyownerwishes to leave policy proceeds to a "minor" child. While any entity can be named as a beneficiary, manyStates do not permit proceeds to be paid to a minor since he or she lacks “legal capacity”. In this case, atrustee will manage the trust for the benefit of the child or children. Testamentary trusts are created at theinsured's death according to a will. Inter vivos trusts or living trusts are created during the life of the insured. Class designations may also be included in a life insurance contract. For example, "all children born intothe marriage of the insured to receive an equal share" is a common class designation to include all childrenin the policy. Therefore, class designations allow the policyholder to leave proceeds to children withoutidentifying them individually. In our previous example concerning Mr. Jones, his daughter and four sons,this designation description would have included all of his children and thus the proceeds would have beenpaid to the surviving sons rather than to his estate, thereby avoiding possible death taxes and probate feeson life insurance proceeds.

! A per capita beneficiary means that distributions of equal amounts are paid to the surviving children of theinsured “per head” or “per person”. A per stirpes beneficiary designation means that distributions are madeaccording to family line, branch or root (i.e., children and grandchildren).

! NO BENEFICIARY / ESTATE — If no beneficiary is listed or alive when the insured dies, the policyproceeds will be paid to the insured's estate. The death benefit, in this case, is added together with all otherassets of the insured including but not limited to real property (i.e., a home), liquid assets such as a savingsaccount, mutual funds or other like investments, rental property, antiques and collectibles. If the total valueof the estate exceeds $1,000,000, the excess is federally estate taxable. Most States also assess a death taxon the estate's value. Additional probate fees are also going to be charged by attorneys as well.

NILA ©

L/H Licensing

Chapter 5—6

Page 74: FORWARD - WordPress.com · Return of premium p 6-8 6.0 Annuities p 7-1 Annuity principles and concepts p 7-1 Accumulation period versus annuity period p 7-1 Owner, annuitant and beneficiary

! FACILITY OF PAYMENT CLAUSE — This provision stipulates that an insurer may select asbeneficiary an individual who appears to be equitably entitled to a portion or all of the life insuranceproceeds since he or she has incurred final medical or funeral expenses of the insured. This may occur whena death claim is not filed within one to two months following the death of the insured. The insurer then willgenerally provide such a payment to a relative who may have incurred any of the final expenses. It alsopermits the insurer to pay another party (i.e. non-relative) who paid for the funeral expenses of thedeceased insured.

INSURING CLAUSE — This provision appears on the first page of the policy. The initial page of the policy isalso referred to as the face page of the contract. This provision provides a summary of the entire contract betweenthe parties involved. The insuring provision is also referred to as the insuring clause or insuring agreement. It willidentify several important pieces of information in addition to the parties involved in the contract. This provisionalso identifies the scope and limits of coverage provided by the policy. In other words, it specifies the death benefitor face amount. The amount of the annual premium is also identified as is the frequency with which the premiumis paid. The "frequency" of premium payment means the "mode" or method by which the policyowner decides tobudget or pay the annual premiums. The insuring provision also lists the name of the beneficiary as well. The factthat the scope and limits of coverage appear in the insuring clause indicates that this is the area of the policy thatidentifies the promise provided by the insurer. For example, the following is an illustration of an actual insuringprovision which appears in a life insurance policy:

"This agreement has been made between the policyowner and insurer. It provides a coverage limitof $100,000 payable to the primary or other beneficiary in the event of the insured's death. Theannual premium is $400 to be paid in the method or mode selected by the policyowner."

CONSIDERATION CLAUSE — In order for the contract to be enforceable, there must be an exchange of valuebetween the parties to the contract. The policyowner provides truthful statements and a premium "in consideration"of the insurer's promise to pay the death benefit if the insured dies while the policy is in force. The materialstatements of the applicant must be true. Therefore, the policyowner's consideration in a life insurance contract isthe premium paid and his or her representations regarding health history which appear in the application. Again, theinsurer's consideration in this life insurance agreement is its promise to pay a legitimate death claim once it receivesa completed proof of loss (i.e., claim form) accompanied by a notarized death certificate.

If for some reason the consideration is not complete on the part of the policyowner, the contract will be void. Thismeans that there was never a valid contract nor coverage in effect. For example, if the policyowner's check bouncesor is returned for insufficient funds, there is no coverage because there is no consideration. If the check clears thebank but the insurer later finds that the applicant engaged in material misrepresentations concerning his or her health,again, there will be no coverage since there is no valid consideration. In this latter instance, the insurer will void orcancel the policy and return the premiums to the policyowner. A "representation" should not be confused with a"warranty." As mentioned in an earlier chapter, a warranty is a statement made by the policyowner that is guaranteedto be absolutely true and coverage is provided based upon the guaranteed information.

CHANGE OF BENEFICIARY PROVISION — If a policyowner wishes to change or modify the beneficiary,he must simply complete an appropriate form and submit it to the home office. As long as the beneficiary designationis revocable, it may be changed at the owner's discretion.

SUICIDE CLAUSE — For many years insurers did not cover death as a result of suicide. Today, an insurercontinues to protect itself against this contingency of a person taking his or her own life. Therefore, a suicideprovision is inserted in most life insurance contracts. The provision or clause stipulates that death caused by suicideis excluded during the initial two years after the policy becomes effective. Therefore, if the insured dies as a result

NILA ©

L/H Licensing

Chapter 5—7

Page 75: FORWARD - WordPress.com · Return of premium p 6-8 6.0 Annuities p 7-1 Annuity principles and concepts p 7-1 Accumulation period versus annuity period p 7-1 Owner, annuitant and beneficiary

of suicide during the first two years of the policy's existence, the insurer will deny the claim and refund allpremiums paid up to that point to the beneficiary. Furthermore, coverage for death as a result of suicide will becovered after the initial two years and the coverage limit will be paid to the beneficiary.

For instance, John is covered by a life insurance policy. Eighteen months after he buys it he commits suicide. Willthe insurer pay the death benefit amount to the beneficiary? No it will not because suicide is not a covered cause ofdeath in the initial two years. However, if John commits suicide in the first two years will anything be paid to thebeneficiary? Yes, something will be paid and that is a return of the premium.

INTEREST ON INSURANCE PROCEEDS — If a beneficiary decides to leave life insurance proceeds withan insurer following the death of the insured, the insurance company must pay interest on the proceeds. The interestcredited or paid to the beneficiary is taxable as ordinary income.

CASH VALUE RELATED PROVISIONS

PROVISIONS ASSOCIATED WITH THE CASH VALUE OF STRAIGHT LIFE INSURANCE —The following provisions are associated with the cash value of a whole life insurance policy. Other policy optionsto be reviewed later in this text known as nonforfeiture options are also associated and based upon a whole lifepolicy's cash value.

1. POLICY LOAN PROVISION — Since a whole life policy builds "equity," the owner has the right toaccess that equity at his or her discretion. This provision, which is supported by the previously reviewedowner's rights provision, permits the owner to receive an advance against the cash value build-up of thewhole life policy. When this advance or cash loan is borrowed against the savings value, the insurer willassess a fixed interest rate of no more than 8%. The loan need not be repaid but if the insured dies, anyunpaid loan plus interest will be deducted from the death benefit. If the owner wishes to pay off the interesteach year rather than add it to the loan amount, he or she may do so as long as it is paid to the insurer on theanniversary date of the policy (not the anniversary date of the loan). The maximum loan value of a wholelife policy is generally its cash value less any projected interest. Policyowners may make withdrawals orpartial surrenders in amounts that do not exceed the cash value less the interest. Any outstanding policyloans or outstanding interest at the time of the insured's death will reduce the policy's face amount.

According to Life Insurance by Black and Skipper, "interest which is not paid by the owner when due shallbe added to the amount of the indebtedness. If this indebtedness ever equals or exceeds the current cash valuein the whole life policy, the contract will terminate, subject to proper notice provided by the insurer." Thecash value of the whole life policy is not synonymous with money deposited in a savings account at FirstNational Bank, for instance. As premiums are paid, allocations to the cash value build equity in much thesame manner as we build equity in our home by making mortgage payments. If we wish to keep our homebut withdraw some of its equity, we secure a second mortgage against the property. In other words, weborrow against its equity. The same principle applies with regard to a whole life policy. If we wish to keepthe policy in force but need cash, we are able to "borrow against" the policy's equity. In addition, interestpaid to the insurer on a policy loan is not tax deductible. All permanent insurance policies include a policyloan provision.

2. AUTOMATIC PREMIUM LOAN (APL) — This provision is not automatically included in a policyand must be elected at the time of application by the applicant. According to Life Insurance by Black andSkipper, "it offers another way for the policyowner to avoid or guard against the unintentional lapse of the

NILA ©

L/H Licensing

Chapter 5—8

Page 76: FORWARD - WordPress.com · Return of premium p 6-8 6.0 Annuities p 7-1 Annuity principles and concepts p 7-1 Accumulation period versus annuity period p 7-1 Owner, annuitant and beneficiary

policy as long as there is sufficient cash value present in the policy." For example, if the policyowner'spremium due date is the first of the month and he or she forgets to pay the premium, coverage could lapsefollowing the 31day grace period. However, if the provision has been inserted in the policy and if there isenough cash value present, once the grace period elapses, the insurer "automatically" withdraws anamount from the cash value in order to pay the premium. The funds taken from the cash value constitutea loan and are used to pay the premium. This provision, if elected, provides a means by which the policy willpay for itself if the owner inadvertently forgets to pay the premium within the grace period. However, ifthere is not sufficient cash value present, this provision will not work when needed. Again, this provisionprevents a policy from lapsing due to non-payment of premium as long as there is enough equity in the policyto pay the overdue premium. In other words, this provision, if included in a whole life policy, will make surethat coverage continues beyond the grace period in the event of a policy lapse. It prevents a premium defaultin a cash value policy

QUIZ

1. An insurer will take what type of action if the applicant misstates his age on a term life insurance application?

A. Cancel the policy C. Adjust the death benefit

B. Cancel the policy and return the premium D. Adjust the cash value

2. Willie purchases a policy on June 11, 2010 and it is delivered 18 days later. The ten day free-look would commence

on:

A. June 28, 2010 C. June 30, 2010

B. June 29, 2010 D. June 31, 2010

3. A type of assignment where some but not all rights are transferred to another party best describes:

A. Debt consolidation C. Collateral assignment

B. Lender assignment D. Absolute assignment

4. The insured's consideration in a life insurance contract is the:

A. Premiums paid

B. Promise to pay

C. Warranties attached to the policy

D. Premiums paid and statements made on the application

5. What provision allows the insurer to void a contract and return premiums if it is determined during the initial two years

that the insured lied about his or her health?

A. Reinstatement provision C. Grace period provision

B. Incontestable provision D. Suicide clause provision

6. The policy plus a copy of the application makes up the:

A. Insuring clause C. Policy loan provision

B. Free look provision D. Entire contract

NILA ©

L/H Licensing

Chapter 5—9

Page 77: FORWARD - WordPress.com · Return of premium p 6-8 6.0 Annuities p 7-1 Annuity principles and concepts p 7-1 Accumulation period versus annuity period p 7-1 Owner, annuitant and beneficiary

7. A policyowner conditionally assigns the cash value of a whole life policy to a lender in order to secure a loan. If the

insured dies before the loan is paid off, how will the death benefit be divided?

A. The death benefit listed in the policy will be paid to the beneficiary.

B. The proceeds are paid to the beneficiary and the loan amount is forgiven.

C. The assignee receives its interest with the remainder paid to the primary beneficiary.

D. The lender receives the proceeds of the policy.

8. What life insurance policy provision enables the policyowner to name or change the beneficiary?

A. Beneficiary provision C. Assignment provision

B. Irrevocable provision D. Ownership provision

9. If a policyowner fails to pay the required premium by the end of the grace period, the policy will lapse. The policyowner

may request reinstatement of the lapsed policy as long as such action is requested within what period of time following

the lapse?

A. One year C. Three years

B. Two years D. Five years

10. Most grace periods which appear in life insurance policies provide for a period of:

A. 30 days C. 35 days

B. 31 days D. 60 days

11. Joe Gerber owns a life insurance policy with an annual premium due date of February 14th. Joe does not pay the

premium by the due date and dies on March 16th. What action will the insurer take when the claim is filed?

A. Deny the claim C. Pay the claim

B. Deny the claim and return the premium D. Pay the claim less the owed premium

12. The most expensive mode of premium payment is the:

A. Annual mode C. Semi-annual mode

B. Quarterly mode D. Monthly mode

13. Bob applies for life insurance stating his age as 28. His annual premium is $150. Bob dies twenty years later and the

insurer discovers that he was really 35 years of age at the time of application and should have been paying $200 annually.

What will the insurer do in this instance?

A. Deny the claim due to misrepresentation C. Pay the face amount

B. Reduce the claim by 28 / 35 ths D. Reduce the claim by 150/200

14. If a policyowner wishes to convert a policy to another type, she is allowed to do so because of the conversion option.

Which of the following will result if a policy type is changed from whole life to term insurance?

A. The death benefit will be adjusted C. The face amount will be increased

B. Proof of insurability will be required D. The premium will be increased

15. Which of the following provisions allow the insurer to deny a claim if the insured is killed while performing stunts in

an air show?

A. Aviation exclusion C. Exclusion provision

B. War exclusion D. Modification provision

NILA ©

L/H Licensing

Chapter 5—10

Page 78: FORWARD - WordPress.com · Return of premium p 6-8 6.0 Annuities p 7-1 Annuity principles and concepts p 7-1 Accumulation period versus annuity period p 7-1 Owner, annuitant and beneficiary

16. Jack delivers the policy to Jill on February 24th. When does the ten-day free-look period end?

A. February 24 C. March 5

B. March 4 D. March 6

17. Each of the following types of life insurance must include a right to examine provision, EXCEPT:

A. Annual renewable term insurance C. Whole life insurance

B. An endowment D. Flight insurance

18. Tim Devine purchases a $100,000 life insurance policy and names his daughters Maureen and Kathleen as primary

beneficiaries to receive an equal share of the death benefit. Tim's wife Ellen is the contingent beneficiary. Kathleen dies

in 2007 and Tim dies in 2008. How are the proceeds distributed?

A. Maureen receives $50,000 and Kathleen's estate receives $50,000

B. Maureen receives $100,000

C. Maureen receives $50,000 and Ellen receives $50,000

D. Ellen receives $100,000

19. Which of the following is protected by the common disaster clause?

A. The primary beneficiary C. An irrevocable beneficiary

B. A revocable beneficiary D. The contingent beneficiary

20. A policy loan made against the cash value does not have to be repaid. However, when the insured dies and there is an

outstanding unpaid loan, how is the death benefit treated?

A. The full face amount is paid

B. The proceeds are reduced by the outstanding loan

C. The death benefit is reduced by the interest owed on the loan

D. The death benefit is paid less any outstanding premium owed.

21. Which of the following would make sure that coverage continues in the event of a premium lapse for more than the grace

period?

A. Insuring clause C. Automatic premium loan provision

B. Entire contract clause D. Consideration clause

22. An insured dies and the death benefit is paid to the named beneficiary. The beneficiary dies three months later. There

are three surviving children. Any remaining proceeds from the policy covering the insured would be paid to which of

the following?

A. To the estate of the deceased insured C. To the oldest surviving beneficiary

B. To the surviving children D. To the estate of the deceased beneficiary

23. Which of the following is true regarding an irrevocable beneficiary?

A. This designation may be changed at any time by the policyowner

B. This beneficiary has the right to receive a copy of the policy

C. This beneficiary has no rights in the policy

D. This beneficiary is also known as a tertiary beneficiary

NILA ©

L/H Licensing

Chapter 5—11

Page 79: FORWARD - WordPress.com · Return of premium p 6-8 6.0 Annuities p 7-1 Annuity principles and concepts p 7-1 Accumulation period versus annuity period p 7-1 Owner, annuitant and beneficiary

24. Theresa is covered by a $100,000 whole life policy. She borrows $4,000 against the cash value without repayment. Her

$1,000 premium is due on January 2 but is not paid. If Theresa dies on January 15 , what will be paid to her primarynd th

beneficiary?

A. Nothing C. $96,000

B. $95,000 D. $99,000

25. Which of the following policy provisions prevents an insurer from denying a death claim in the future after the purchase

of a life insurance policy?

A. Entire contract clause C. Consideration clause

B. Insuring clause D. Owner's rights clause

26. Which of the following policy provisions prevents a premium default in a cash value policy ?

A. Paid up additions C. Default provision

B. Automatic premium loan D. Premium payment provision

ANSWERS

1. C

2. B

3. C

4. D

5. B

6. D

7. C

8. D

9. C

10. B

11. D

12. D

13. D

14. B

15. C

16. D

17. D

18. B

19. D

20. B

21. C

22. D

23. B

24. B

25. A

26. B

CHAPTER 5 KEY CONCEPTS

AssignmentAutomatic premium loanBeneficiary provisionConversion contractEntire contractExclusion provisionFree-lookGrace period

Incontestable clauseMisstatement of age or sexModification provisionOwnershipPayment of premiumPolicy loan provisionReinstatementStandard provisions

NILA ©

L/H Licensing

Chapter 5—12

Page 80: FORWARD - WordPress.com · Return of premium p 6-8 6.0 Annuities p 7-1 Annuity principles and concepts p 7-1 Accumulation period versus annuity period p 7-1 Owner, annuitant and beneficiary

LIFE INSURANCEPOLICY OPTIONS

ANDBENEFIT RIDERS

Page 81: FORWARD - WordPress.com · Return of premium p 6-8 6.0 Annuities p 7-1 Annuity principles and concepts p 7-1 Accumulation period versus annuity period p 7-1 Owner, annuitant and beneficiary

INTRODUCTION

Whole life insurance policies contain non-forfeiture options or guaranteed values which are availableto a policyowner who wishes to surrender a whole life policy. Other options contained in a whole lifepolicy include dividend options and settlement options. In addition, benefit riders, which provide extracoverage, are also available to a policyowner for an additional premium. All of these relevant topics willbe reviewed in this chapter.

LIFE INSURANCE POLICY OPTIONSAND BENEFIT RIDERS

CONTRACT OPTIONS

LIFE INSURANCE POLICY OPTIONS — The following section identifies and describes the optionsavailable to policyowners who possess either whole life or term life insurance. The initial two options that will bereviewed are available in whole life policies. These are the nonforfeiture options and dividend options. The finaloption section of a life insurance policy is the settlement options which are provided for a beneficiary when aninsured dies.

NON-FORFEITURE OPTIONS — Three options are available to a policyowner who decides to surrender a wholelife policy. If this scenario arises, the policyowner can stop paying premiums and he will not "forfeit" the cash valueor equity that has built up in the policy. In other words, when a policyowner decides to terminate the policy, he orshe has the "option" of utilizing the cash surrender value in any of three ways. None of these options can be effectedunless a cash value is present. These nonforfeiture options available prevent the loss of cash value or equity if thepolicy is surrendered by the owner. Generally, an option will be selected by the owner. However, in one instancean option will go into effect automatically. In addition, once a policy is surrendered, it cannot be reinstated. Nonforfeiture options may also be referred to as nonforfeiture benefits or nonforfeiture values since they provide forguaranteed values. In addition, a policy cannot lapse as long as there is cash value present. Nonforfeiture optionsrecognize the equity build up in a whole life policy.

It is the grace period, in general, that protects against the lapse of a policy if premiums are not paid by the due date.The automatic premium loan provision protects against the lapse of a policy if premiums have not been paid by thedue date and sufficient cash value is present. However, the policy OPTIONS available in a whole life policy thatprevent the policy from lapsing if enough cash value is present are the nonforfeiture options. The three optionsavailable include surrender for cash, extended level term insurance and reduced paid-up permanent insurance (i.e.,remember the acronym "SER").

1. SURRENDER FOR CASH — Also called "cash surrender," this option allows a policyowner to returnor send the policy back to the insurer in return for its cash savings value. The cash value, like the deathbenefit and the annual premium, is pre-determined. Whenever the policyowner selects this option, he or sheknows the exact amount that will be received upon surrender. If the amount received in a surrender is greaterthan the amount of premiums paid less dividends paid, the excess is taxable as ordinary income. Surrendercharges are generally assessed when a policy is terminated by the owner and they are normally incorporated

NILA ©L/H LicensingChapter 6—1

Page 82: FORWARD - WordPress.com · Return of premium p 6-8 6.0 Annuities p 7-1 Annuity principles and concepts p 7-1 Accumulation period versus annuity period p 7-1 Owner, annuitant and beneficiary

in the contract. This is the option available when the policyowner no longer needs to maintain the wholelife policy and exchanges it for the cash value or equity. The surrender value returned to the owner underthis option includes interest paid on the cash value. The interest is not taxable as long as the amount receivedat surrender is not greater than the amount paid in premiums.

2. EXTENDED LEVEL TERM INSURANCE — This option allows the policyowner to surrender thewhole life in exchange for a level term plan with the same face amount. This option, as stated in LifeInsurance by Black and Skipper, "permits the policyowner to surrender the policy and exchange the cashvalue for a paid-up level term policy." The policyowner may send the policy back to the insurer and informit to purchase level term insurance with the cash value. The coverage amount of the new paid-up term lifeinsurance policy that is purchased with the cash value will be identical to the face amount of thesurrendered whole life policy. The whole life policy provides information which will indicate for how longthe level term insurance will last (e.g., 11 years and 140 days). At the end of this time frame coverage ends. Since the coverage provided is level term insurance, it can be renewed for another specified period.

Again, from the time the whole life policy was originally purchased, the extended term insurance periods arepre-determined. However, since level term insurance is provided, once the extended period ends, coveragemay be renewed based on the attained age of the insured at that time. If a whole life policy lapses becausethe policyowner fails to pay the premium and there is still an amount of cash value present, the extended termoption will routinely or automatically be activated if the insurer is unable to make contact with thepolicyowner. The extended term insurance option provides the insured with the most life insuranceprotection in the event of a voluntary policy surrender or non-payment of premium (i.e., greatest faceamount). Again, it utilizes the contract's cash value to purchase a new policy. This is the option availablethat allows the owner to purchase term insurance when he or she decides to surrender a whole life policy thathas cash value.

3. REDUCED PAID-UP PERMANENT INSURANCE — This option allows the policyowner tosurrender the policy and use the cash value to buy a smaller or reduced amount of permanent paid-up lifeinsurance. Therefore, this nonforfeiture option provides for the longest period of protection since it involveswhole life insurance. The cash value is being used to make a single premium purchase. No further premiumsare due. This smaller policy is now in force for life and, like most whole life policies, will mature at age 100.The reduced face amount that is purchased by the owner upon surrender is based upon (1) the insured'scurrent or attained age and (2) the cash value available since this is the single premium that is paid. Whenan insured selects this option, he or she has recognized the need for some type of permanent life insurancebut no longer wishes to continue premium payments. Therefore, this is the option that provides thepolicyholder / insured with life insurance coverage for the greatest length of time (i.e. permanent whole lifeprotection).

DIVIDEND OPTIONS

Life insurers that pay dividends to policyowners are known as mutual insurers. They issue “participating” lifeinsurance policies. Therefore, insurers that are incorporated as mutual companies, such as ABC Life, may paydividends to policyowners in a particular year. Insurers that do not pay dividends are known as stock companies. These insurers issue “non-participating” insurance policies and do not pay dividends to policyowners. A lifeinsurance dividend is not considered to be taxable income since it is actually a return of an overpayment ofpremium. Therefore, life insurance dividends are tax exempt. Producers may not, during the sale of life insurance,inform an applicant that dividends are guaranteed. They are not. This would be an example of misrepresentationby a producer. However, a producer is allowed to provide illustrations or documentation to an applicant which

NILA ©L/H LicensingChapter 6—2

Page 83: FORWARD - WordPress.com · Return of premium p 6-8 6.0 Annuities p 7-1 Annuity principles and concepts p 7-1 Accumulation period versus annuity period p 7-1 Owner, annuitant and beneficiary

verifies dividends paid in previous years by the insurer. If a policyowner is entitled to a dividend, he or she canchoose how to receive it in any of the six ways identified below. An easy way to remember these options is by usingthe acronym or mnemonic CRAPO.

CASH — If the policyowner is entitled to a $50 dividend, he or she may request that the insurer send paymentdirectly to them. Again, life insurance dividends are tax exempt.

REDUCE PREMIUMS — If the policyowner's annual premium is $250 and she learns that she is entitled to a $50dividend, she simply informs the insurer to retain the dividend and subtract that amount from the premium she owes. The policyowner then pays $200 for the year's premium. This dividend option assists the policyowner whose primaryobjective is to conserve cash since he or she does not have to remit the entire annual premium. This is the dividendoption utilized by the policyowner when he or she is concerned about or wishes to minimize their current outlayof funds.

ACCUMULATE AT INTEREST — The policyowner, under this option, informs the insurer to retain the $50dividend in a designated account. When this occurs, the insurer must pay interest on the dividend(s) it holds. Whilethe dividend is tax exempt or not taxable, any interest earned on it when left with the insurer is taxable as ordinaryincome. As related in Life Insurance, if death occurs, the face amount is paid plus any dividend accumulations. Inthe event of a policy surrender, the cash value plus the dividend accumulations will be paid to the policyowner.

PAID-UP PERMANENT ADDITIONS — The policyowner may elect to use the $50 dividend to purchaseadditional permanent whole life insurance. The amount that can be purchased will be based upon two criteria: (1)the current age of the insured; and (2) the amount of the dividend. The dividend amount is the premium that will beused to purchase a small amount of permanent paid-up life insurance. In other words, the dividend is being spent topurchase a small face amount of single premium life insurance. This paid-up addition has its own cash value. Therefore, this option provides for an increase in cash value for the policyowner. If this option is used, it increasesthe total death coverage.

PAID-UP OPTION — A policy dividend may be utilized to pay up a policy earlier than normal by suspending orterminating premium payments. If the situation arises where the policy's cash surrender value plus dividends payable,equals or exceeds what the single premium at the attained age of the insured would be for a coverage amount equalto the current face amount of the policy, the policy will be "paid-up" for life.

ONE-YEAR TERM — The $50 dividend can be used to simply purchase any type of term insurance that the insureroffers. This is the dividend option that provides the policyowner with "a different type of life insurance (i.e., termlife insurance) than that provided by the primary policy (i.e., whole life) paying the dividend. This option may beused to provide a face amount of life insurance equal to the amount of a policy loan taken against the cash value ofthe whole life policy. For example, the policyowner who has an outstanding loan can use this option to buy morelife insurance just in case the insured dies before the loan is repaid.

NOTE ABOUT DIVIDENDS AND DIVIDEND OPTIONS

! Dividends may be paid on participating policies. ! Mutual companies (owned by policyowners) issue participating policies. ! Dividends from life insurance policies are not taxable. ! Interest paid on dividends is taxable. ! To remember these 5 options remember the acronym — CRAPO.

NILA ©L/H LicensingChapter 6—3

Page 84: FORWARD - WordPress.com · Return of premium p 6-8 6.0 Annuities p 7-1 Annuity principles and concepts p 7-1 Accumulation period versus annuity period p 7-1 Owner, annuitant and beneficiary

SETTLEMENT OPTIONS

All life insurance policies include a variety of settlement options that are available to a beneficiary when an insureddies. As mentioned in Life Insurance by Black and Skipper, "one of the most important things a policyowner shouldconsider at the time they purchase life insurance is the manner in which death proceeds will be paid. Failure toarrange for the proper payment of proceeds may defeat the very purpose for which the insurance was intended." Asettlement option may be selected by the policyowner at the time of application. The spendthrift clause, mentionedearlier, also stipulates that the owner may designate how the proceeds will be distributed to a beneficiary upon thedeath of the insured. In most cases, however, the settlement selection is made by the beneficiary at the time of theinsured's death. Therefore, settlement options provide for various methods of paying policy proceeds. The settlementoptions available provide the beneficiary with more flexibility with which to receive proceeds. The principal methodused to pay death proceeds is lump sum. In this manner, the beneficiary receives policy proceeds income tax freein one payment (remember the acronym "FFILL"). If another option is selected and proceeds remain with the insurer,any interest paid on the proceeds will be taxable.

CASH PAYMENT / LUMP-SUM — In most cases, the proceeds of a policy are paid to a beneficiary in thismanner. This is the most commonly used form of death proceeds distribution to a beneficiary.

FIXED - PERIOD — As described in Life Insurance by Black and Skipper, "this is one of the two options basedupon the concept of systematically liquidating principal and interest over a period of years, without references to lifecontingencies." Under this option, the beneficiary leaves the death proceeds with the insurer. Interest is paid on theproceeds (i.e., principal) by the insurer and monthly income is then paid to the beneficiary for a specified period oftime as selected by the beneficiary (i.e., ten years). Life Insurance further relates, "this option provides for thepayment of proceeds in installments over a definite number of years not to exceed 25 or 30." Further, "the amountof each installment is determined by: (1) the amount of proceeds: (2) the period of time selected: (3) the guaranteedrate of interest; and (4) the frequency of payments."

The fixed period option "is valuable when the most important consideration is to provide income for a definite periodof time. Many insurers allow the beneficiary, at any time, the right to discontinue all remaining installments andreceive a lump-sum of the remaining balance." * Therefore, this settlement option allows the beneficiary to receivethe policy proceeds in equal installments over a specified period of time. This settlement option is determined bythe length of time that proceeds are paid to the beneficiary. In other words, the face amount and the length of timefor which payments will be made will determine the monthly income amount paid to the beneficiary.

FIXED - AMOUNT — This option permits the death proceeds to be left “at interest” with the insurer and to be paidout in installments of specified amounts for an indefinite amount of time until all funds are exhausted. Under thisoption, Life Insurance again states that "the amount of income is the primary consideration rather than the periodof time over which the proceeds and interest are to be liquidated. Here the beneficiary designates an amount ofincome to be paid, such as $1,200 per month, and these payments continue until the principal and the interest areexhausted." Further, "this settlement option is more advantageous than the fixed period option, since it is much moreflexible. Insurers may permit the beneficiary to receive varying amounts of income at various times. The beneficiarypossesses the right to withdraw proceeds at any time." The fixed amount and fixed period options both pay theprincipal and interest until the proceeds are exhausted. The amount of monthly income selected by the beneficiary,the amount of proceeds and the interest rate paid by the insurer will all determine the length of time in which thebeneficiary receives the monthly income.

INTEREST ONLY — Under this option, the proceeds of the policy remain with the insurer and only the interestis paid to the beneficiary. This option also provides the beneficiary with flexibility since the proceeds may be leftwith the insurer which frees him or her of any investment worry while guaranteeing both principal and a minimum

NILA ©L/H LicensingChapter 6—4

Page 85: FORWARD - WordPress.com · Return of premium p 6-8 6.0 Annuities p 7-1 Annuity principles and concepts p 7-1 Accumulation period versus annuity period p 7-1 Owner, annuitant and beneficiary

rate of return (i.e., interest). The beneficiary always possesses the right to withdraw proceeds at any time in thefuture. Under any option selected where the policy proceeds are left "at interest" with the insurer, the beneficiaryis protected against the claims of creditors. Protection of the beneficiary in this manner is also provided by thespendthrift clause as well. As always, interest paid by an insurer on policy proceeds is taxable. Again, even whenthe policy proceeds are left with the insurer and this option is selected by the beneficiary, he or she continues topossess the right to withdraw the proceeds in the future at his or her discretion.

LIFE INCOME — This is another settlement option which liquidates policy proceeds (i.e., principal) and interestwith regard to life contingencies. This option is actually a form of single premium immediate straight life annuitysince the death benefit amount is being utilized to purchase immediate monthly income that will be paid to thebeneficiary for life. When the monthly installment commences, the beneficiary becomes the annuitant. LifeInsurance again states that "the amount of each installment paid depends upon: (1) the type of life income selected;(2) the amount of proceeds; (3) the rate of interest assumed; and (4) the age and sex of the beneficiary when incomebegins."

According to Life Insurance, "there are several life income options available from which the beneficiary may selectincluding: (1) the single, pure or straight life income option; (2) the refund life income option; and (3) the lifeincome option with a period certain (i.e., for a specified period such as ten years). Under the single, pure or straightlife income option, like a straight life annuity to be reviewed in chapter 7, monthly installments are paid to thebeneficiary for as long as he or she lives. In other words, income payments end upon the death of the recipient (i.e.,the beneficiary). No refund nor any other payments are made once the beneficiary dies. This option potentiallyprovides the greatest amount of income per $1,000 of proceeds." However, as mentioned, it also possesses thegreatest amount of risk since there is no survivorship. Further "the refund option, such as a joint and survivor option, guarantees the return of an amount equal to theprincipal less any payments already made. In other words, it provides a minimum guaranteed return. Once theprimary beneficiary dies, his or her survivors may receive the refund on an installment basis or in a lump sum whichis referred to as a cash refund." The former is also known as the installment refund. The life income option with aperiod certain pays a monthly income for as long as the beneficiary lives, but should he or she die before apredetermined number of years have elapsed, the insurer will continue monthly payments to a second beneficiary forthe remainder of the designated period certain."

LIFE INSURANCE BENEFIT RIDERS

Benefit riders may be added to a life insurance policy for an additional premium. The cost of the policy increasessince an extra benefit is being provided. Available benefit riders include but are not limited to:

WAIVER OF PREMIUM — Sometimes referred to as "waiver of stipulated premium or waiver of cost," thisbenefit rider may be added to a whole life or term insurance policy. It is not automatically included in a lifeinsurance policy. It requires an extra premium but is actually quite inexpensive. Its cost is generally cents per $1,000of coverage. This rider, when part of the policy, prevents the policy from lapsing when the policyowner becomestotally disabled. If an insured becomes totally disabled for six consecutive months, the insurer promises to waiveany future premium payments until he or she returns to work. In a whole life policy, the cash value accumulates likeit normally would because the insurer is actually paying the premium. In addition, any premiums paid within theinitial six months of disability will be refunded to the policyowner from the first day of disability. This rider maybe utilized until the policyowner reaches the traditional age of retirement (i.e., age 65). It does not provide anyincome nor does it modify in any way the coverage amount.

NILA ©L/H LicensingChapter 6—5

Page 86: FORWARD - WordPress.com · Return of premium p 6-8 6.0 Annuities p 7-1 Annuity principles and concepts p 7-1 Accumulation period versus annuity period p 7-1 Owner, annuitant and beneficiary

Disability Income Benefit Rider — Many insurers offer a waiver of premium rider that also includes a disabilityincome benefit. Most riders provide a benefit of one-percent of the face amount of the policy which is payable if theinsured is totally disabled. For example, if Jim owns a $20,000 life insurance policy with this rider included and hebecomes totally disabled, he would be paid $200 per month (1% of $20,000). The monthly income paid is generallylimited to no more than $1,000 per month. Income benefits begin after the six month waiting period as previouslydescribed.

ACCELERATED BENEFITS RIDER — This benefit rider may be added to a life insurance policy for anadditional premium. It is also referred to as a living benefits or terminal illness rider. Certain conditions forpayment must be satisfied in order for a benefit to be paid. This rider allows an insured to "accelerate" the deathbenefit of a life insurance policy while still living if a physician diagnoses and verifies that the insured is sufferingfrom a terminal illness and is likely to die within twelve, or in some cases, twenty-four months or less. The insuredmay receive up to a specific percentage (i.e., 25%) of the death benefit. For example, if the insured is covered for$40,000 and has added an accelerated benefits rider with a 25% living benefits limit and he is diagnosed with aterminal illness, up to $10,000 may be paid to the insured prior to death as long as medical corroboration is provided. Therefore, when the insured dies, $30,000 is paid to the beneficiary. This demonstrates the effect on the deathbenefit. Another type of accelerated benefit is provided by catastrophic illness coverage (i.e., dread diseasecoverage). The terms of this coverage are similar to the terminal illness rider except that the covered disease mustbe identified or listed in the policy (i.e., cancer, heart disease, renal failure, stroke, etc.).

Long-Term Care Rider — Some accelerated benefits are available by adding a long-term care rider to a lifeinsurance policy. For example, if an insured is permanently confined to a nursing home and requires long term care,the policy rider will pay a benefit. These riders may be added to individual or group policies. To qualify for anaccelerated benefit under a long-term care rider, the (long term) confinement must be covered by the rider oradditional requirements must be satisfied. For example, the long-term care rider, like an individual LTC policy, willgenerally pay benefits when the insured is unable to perform at least two activities of daily living (ADLs) such aseating, dressing, bathing, toileting / continence, walking / ambulation, transferring or taking medication. Like theaccelerated benefits rider, the long-term care benefits received will reduce the total death benefit payable upon thedeath of the insured. For instance, if the life insurance policy has a death benefit of $100,000 and the long-term carerider pays out $20,000 prior to the death of the insured, the final death benefit paid will be $80,000. Therefore,accelerated benefits may be paid in appropriate situations by a terminal illness rider, catastrophic illness coverageor a long-term care rider.

RIDERS COVERING ADDITIONAL INSUREDS — Additional riders may be added to a life insurance policywhich provide term insurance coverage for the spouse, children or entire family. The family policy, previouslyreviewed in Chapter 4, essentially provides the same protection as the family (term) rider. It provides whole lifeon the primary insured (i.e., breadwinner) and level term insurance on the spouse and children (i.e., the rest of thefamily). The family rider when added to an insured's individual policy covers the rest of the family but not theprimary insured who is protected by the individual policy. Dependent riders may be added to a primary policy tocover a spouse or “another insured”, children or adopted children. Therefore, several types of dependent ridersavailable include but are not limited to a spousal rider, a child rider or a family rider. For example, assume you arean agent or producer. Your potential client wishes to purchase a life insurance policy covering her life. She alsowishes to cover her husband by adding a rider to her policy in case he dies in an accident. What type of rider wouldyou suggest she add to the policy to meet her needs? The reference to "in case he dies in an accident" gives one theimpression that the accidental death rider would be added to the policy to satisfy this need. But it is a dependent riderthat would be added to cover the spouse. The accidental death rider is added to the primary insured's policy to coverhim or her. It does not cover his or her spouse.

NILA ©L/H LicensingChapter 6—6

Page 87: FORWARD - WordPress.com · Return of premium p 6-8 6.0 Annuities p 7-1 Annuity principles and concepts p 7-1 Accumulation period versus annuity period p 7-1 Owner, annuitant and beneficiary

RIDERS AFFECTING THE DEATH BENEFIT AMOUNT — Additional benefit riders are available whichprovide increased or additional amounts of coverage when added to an existing or underlying policy. These ridersgenerally require an additional premium. These types of riders include but are not limited to:

1. ACCIDENTAL DEATH RIDER — This type of rider may also be attached to a life insurance policyfor an additional premium. This benefit provides multiple indemnity and provides an additional deathbenefit if the cause of death listed on a death certificate is "accidental." This rider includes a restriction thatthe insured must die within 90 days of the accident in order for the additional death benefit to be paid. Whenan accident occurs and the insured dies more than 90 days later, no coverage is provided by the rider. Statistically, the cause of death in such cases is not accidental but probably due to heart failure, kidney orliver failure, pneumonia or some other type of "non-accidental" cause. Remember the cause of death whichappears on the death certificate will indicate whether or not the insurer pays the claim. So if the insuredsuffers a fatal heart attack or stroke while driving a car and the auto crashes into a tree, the policy will notpay the rider benefit.

Policies that pay a multiple of two times the policy face amount are called double indemnity and those thatpay three times the death benefit for death due to accidents are called triple indemnity and so forth. Thisrider provides the least expensive form of life insurance available today. The primary reason that it ischeaper is due to the fact that it is limited in scope since it only covers death as a result of an accident anddeath must occur within 90 days of the accident. Therefore, coverage under this rider is less expensive thanwhole life insurance or any type of individual term life insurance whether characterized by a level ordecreasing face amount. At the very least, this rider provides an extra death benefit amount at the lowest costand will be payable if the insured dies in an accident. The addition of an accidental death rider to a wholelife policy does not impact the policy's cash value.

2. GUARANTEED INSURABILITY RIDER — This rider allows its purchaser to acquire more lifeinsurance at some point in the future without requiring a medical exam. When the rider is added to thepolicy, it will include specific (future) option dates based on the insured's future birthdays. As the optiondates arrive, the insured may elect or skip them. If elected, no physical exam will be required since coverageis "guaranteed issue." Some insurers offer this option each year after the policy is purchased up to age 50. Whenever an option is elected, the premium rates are based upon the insured's current, attained or presentage. Options, with a few exceptions, may only be elected on the option dates listed in the rider. In otherwords, if an option is available and is not used, it is lost and the insured must wait until the next option (i.e.,birthday) arrives.

3. COST OF LIVING RIDER — This rider automatically increases the face amount of the policy atspecified intervals based upon increases in the Consumer Price Index (CPI). The CPI measures the inflationrate each year. If there were a 2% rise in this index, the policyowner's face amount would increase by 2%for the next year.

4. RETURN OF PREMIUM — This is a type of increasing term insurance rider that is added to a wholelife policy. When the insured dies, the beneficiary receives the face amount plus an additional (terminsurance) death benefit equal to the cumulative total of all premiums paid during the life of the policy. Therefore, the amount of coverage under this rider increases each year based on the cumulative total of allpremiums paid.

NILA ©L/H LicensingChapter 6—7

Page 88: FORWARD - WordPress.com · Return of premium p 6-8 6.0 Annuities p 7-1 Annuity principles and concepts p 7-1 Accumulation period versus annuity period p 7-1 Owner, annuitant and beneficiary

NOTE

These latter two riders are referred to as increasing term riders that provide an additional term insurance faceamount at death which is equal to either all premiums paid or the amount of cash value. Therefore, the primarycharacteristic of an increasing term rider is that it provides a greater amount of coverage each year.

5. RETURN OF CASH VALUE — This is another type of increasing term rider that provides anincreasing amount of term insurance that equals the cash value as it accumulates in a whole life policy. Thisrider allows the cash value to be paid in addition to the face amount. Again, the rider provides an additionalterm insurance benefit equal to the cash value amount at the time of death.

PAYOR RIDER — This rider is only added to a policy an adult purchases to cover a child. It provides thatpremiums will be waived until the child reaches age 21 (some policies use age 18 or 25) if the premium payor (i.e.,parent or guardian) dies or becomes totally disabled. In addition, if the parent wanted to make sure that his or herchild could buy more life insurance in the future, a guaranteed insurability rider could be added to the child's policyas well. Do not confuse the payor rider with the guaranteed insurability rider. The guaranteed insurability rider maybe added to a policy covering an adult or child and allows the insured to buy more insurance without a medical exam. The payor rider waives premiums or the cost on a policy covering a child. This rider may also be referred to as thepayor benefit or payor clause.

VIATICAL SETTLEMENTS

A viatical settlement provider is a business firm that purchases life insurance contracts from their owners, thusbecoming the new policyowner. Since the new owner now possesses all the rights of ownership, it changes thebeneficiary to itself so that when the insured dies, the policy proceeds are paid to the firm. Upon the transfer ofownership, the business entity is now responsible for paying the premium and (hopefully) experiences a profit whenthe insured party dies. The former policyowner/insured received a lump sum amount in return for "selling" thecontract to the viatical firm. This amount will vary depending upon the terminal condition of the insured.

! ADVANTAGES — While upon an initial inspection of these arrangements it may appear that those in needof cash when terminally ill are being taken advantage of, viatical settlement arrangements involve the saleof property just as if one was selling his or her home. When a person is suffering from a terminal illness suchas AIDS, cancer or heart disease and cash is needed to pay for continuing medical or extended care, a viaticalsettlement provides a way for the afflicted individual to raise needed funds prior to death. During the pastdecade, insurers began to offer a rider that could be added to a life insurance policy to provide for theaforementioned contingency. This rider, as reviewed earlier in this chapter, is called an accelerated benefitsrider. Generally, this rider limits withdrawals, which are fully taxable, to a maximum of 50% of the faceamount while numerous insurers only permit anywhere from 25-40% of the face amount to be withdrawn.Therefore the viatical settlement arrangement allows terminally ill insureds to receive a larger "livingbenefit." In addition, the payment received by the policyowner / viator is tax free as long as the insuredmeets the definition of terminally ill which is that death will probably result within twenty-four months.

! PROVIDER — Any individual, corporation, partnership or firm who pays a sum to the owner of a lifeinsurance contract in return for the ownership of the contract is a viatical settlement provider (i.e., viatee).The owner of the policy may be referred to as the viator. A viatical settlement broker is an individual who

NILA ©L/H LicensingChapter 6—8

Page 89: FORWARD - WordPress.com · Return of premium p 6-8 6.0 Annuities p 7-1 Annuity principles and concepts p 7-1 Accumulation period versus annuity period p 7-1 Owner, annuitant and beneficiary

attempts to negotiate such a settlement between the firm (i.e., provider) and the policyowner (i.e., viator).A viatical settlement provider is not an insurer nor any financial institution. Therefore, a life insuranceprovider does not engage in viatical settlements. Brokers may not receive greater than 2% of the amountpaid to the viator as compensation. A life insurance company shall respond to a request for verification ofcoverage from a viatical settlement provider or viatical settlement broker within 30 calendar days of the datethe request is received.

! CONTRACT PROVISIONS — These agreements must be in writing and must conform with State lawwhile being approved by the Commissioner or Superintendent of Insurance. The contract must include thesum to be received by the policyowner and additional terms of the transaction. The contract must alsoinclude additional disclosure provisions such as: (1) information regarding the ill policyowner's right oravailability of benefits in the existing policy; (2) the fact that the sum received may be taxable or subject tocreditor's claims; (3) the equivalent of a 7 day free-look period to change their mind and receive the policyback (this 7 day period will be extended to the next business day if it falls on a weekend). In addition, theill person must also be informed that receipt of a sum of money may affect benefits for which he or she areentitled under government sponsored programs such as Medicaid.

! RULES OF CONDUCT — Two witnesses must be present when the agreement is signed by the providerand the viator. Once the 7 day period expires, the provider must remit the agreed upon sum to the viator incare of an escrow or other type of account until the policy is assigned absolutely. Once this is effected, thefunds in escrow will be remitted to the viator.

QUIZ

1. All of the following are types of dividend options, EXCEPT:

A. Paid-up additions C. One year term insurance

B. Reduced paid-up insurance D. Accumulate at interest

2. Which of the following policy options allows a policyowner to effectuate a single premium purchase?

A. Cash refund insurance C. One year term

B. Reduced paid-up insurance D. Reduced premium payments

3. Which of the following non-forfeiture options provides the greatest amount of life insurance protection?

A. Paid-up additions C. Extended term insurance

B. Surrender for cash D. Reduced paid-up insurance

4. Each of the following riders affect the face amount of a life insurance policy, EXCEPT:

A. Cost of living rider C. Waiver of premium

B. Return of premium D. Accidental death

5. Whole life policies include non-forfeiture options that are available in the event the policyowner wishes to surrender the

policy. Once a policy is surrendered it cannot be:

A. Renewed C. Reissued

B. Voided D. Reinstated

NILA ©L/H LicensingChapter 6—9

Page 90: FORWARD - WordPress.com · Return of premium p 6-8 6.0 Annuities p 7-1 Annuity principles and concepts p 7-1 Accumulation period versus annuity period p 7-1 Owner, annuitant and beneficiary

6. Which non-forfeiture option indicates that insurance coverage no longer exists?

A. Cash surrender C. Reduced paid-up insurance

B. Extended term insurance D. Paid-up additions

7. If a policyowner exercises a paid-up additions dividend option, the amount of coverage purchased will be based upon

the insured's attained age and:

A. The amount of cash value C. The face amount of the policy

B. The amount of the dividend D. The reduced paid-up amount

8. Each of the following is a dividend option available in a participating policy, EXCEPT:

A. Reduce the premium C. Accumulate at interest

B. Reduced paid-up insurance D. Paid-up additions

9. All of the following are settlement options available in a life insurance contract, EXCEPT:

A. Fixed period C. Interest only

B. Paid-up additions D. Life income

10. Which non-forfeiture option is automatically exercised if a policy lapses with current cash value and the insurer is unable

to contact the policyowner?

A. Reduced paid-up insurance C. Extended term insurance

B. Surrender for cash D. Return of premium

11. Which of the following settlement options pays a specified monthly income for as long as the policy proceeds plus

interest last?

A. Lump sum C. Life income

B. Fixed period D. Fixed amount

12. John selects a settlement option paying a life income of $2,000 per month with ten years certain. If he dies five yearslater, his beneficiary would receive a cash refund of:

A. $24,000 C. $120,000B. $96,000 D. $240,000

13. Dividends paid on a life insurance policy may be referred to as a return of an overpayment of premium. Which of thefollowing best describes the tax treatment of life insurance dividends?

A. Not taxable as ordinary income C. Tax deductibleB. Taxable as ordinary income D. Not tax deductible

14. Which of the following riders prevents a policy from lapsing if the insured becomes disabled?

A. Accelerated benefits rider C. Accidental death riderB. Waiver of premium rider D. Cost of living rider

15. A rider that permits the policyowner to purchase additional amounts of insurance without taking a physical examinationbest describes:

A. Waiver of premium rider C. Cost of living riderB. Payor benefit rider D. Guaranteed insurability rider

NILA ©L/H LicensingChapter 6—10

Page 91: FORWARD - WordPress.com · Return of premium p 6-8 6.0 Annuities p 7-1 Annuity principles and concepts p 7-1 Accumulation period versus annuity period p 7-1 Owner, annuitant and beneficiary

16. A rider that pays an increasing amount of term insurance equal to the amount of total payments made for coverage, inaddition to the death benefit at the time of death, is known as a:

A. Return of cash value C. Increasing benefits riderB. Return of premium D. Accelerated benefits rider

17. Alex owns a $50,000 life insurance policy. It also includes a waiver of premium and disability income rider. If Alexis disabled for six straight months he will begin to receive what amount of income per month while he is disabled?

A. $250 C. $1,000B. $500 D. $5,000

18. Alice owns a $100,000 life insurance policy with an accelerated benefits rider which includes a 40% clause. Five yearsafter purchasing the policy, Alice is diagnosed with a serious illness. She notifies the insurer and fourteen months laterdies. How much will the policy pay her beneficiary at the time of death?

A. $0 C. $60,000B. $40,000 D. $100,000

19. An accelerated benefits rider may be added for an additional premium to a life insurance policy. Living benefits maybe paid for each of the following, EXCEPT:

A. Long-term care C. Terminal illnessB. Disability income D. Dread disease

20. Which of the following riders provides an increasing amount of coverage each year based upon changes to the consumerprice index?

A. Living benefits rider C. Return of cash value rider B. Return of premium rider D. Cost of living rider

21. Jack is covered by a $100,000 whole life policy. The policy includes a term life insurance rider providing an extra$50,000 of coverage until Jack retires at age 65. If he dies sixteen years following his retirement, what will the insurerpay the primary beneficiary?

A. $0 C. $100,000B. $50,000 D. $150,000

22. If a whole life policy premium is not paid by the end of the contract's grace period and the policy contains severalthousand dollars of equity, what policy option will protect the insured against a coverage lapse?

A. The grace period C. The nonforfeiture benefitB. The automatic premium loan provision D. The settlement options

23. Beverly Rogers owns a permanent life insurance policy with a death benefit of $200,000. She has also added to it anaccelerated benefits rider with a 25% clause. The policy also includes an accidental death rider for $500,000. If Beverlyis diagnosed with terminal cancer and receives the maximum amount from the appropriate rider, what will be paid to herbeneficiary following her death?

A. $50,000 C. $175,000B. $150,000 D. $200,000

24. Which of the following provides the least expensive form of life insurance protection?

A. Level term insurance C. Modified life insuranceB. Decreasing term life insurance D. Accidental death protection

NILA ©L/H LicensingChapter 6—11

Page 92: FORWARD - WordPress.com · Return of premium p 6-8 6.0 Annuities p 7-1 Annuity principles and concepts p 7-1 Accumulation period versus annuity period p 7-1 Owner, annuitant and beneficiary

25. Bill Miles buys an individual whole life policy with a face amount of $160,000. He adds to the policy a family rider tocover his 43 year old wife, his daughter and an adopted son. All of the following will be covered by the rider, EXCEPT?

A. The insured C. The daughterB. The spouse D. The adopted son

26. A life insurance producer sells a straight life policy to a 55 year old male in excellent health. The producer also convincesthe applicant to add an accelerated benefits rider and a waiver of premium rider to the policy as well. What additionaltype of rider can be added to this policy to cover the applicant's spouse if she dies in an accident?

A. Accidental death rider C. Guaranteed insurability riderB. Payor rider D. Dependent rider

27. An individual purchases a whole life policy with an accelerated benefits rider of 40%. The contract, which provides a

death benefit of $250,000, also includes a $100,000 accidental death rider and a waiver of premium rider. A dependent

rider is added to cover the remaining family members. Twenty seven years after the policy is purchased, a loan of

$13,000 against the policy's cash value is effected. If the insured dies from a terminal illness within a month after

receiving the equity, what will be paid to the beneficiary?

A. $137,000 C. $237,000

B. $150,000 D. $250,000

28. What type of rider can be added to a policy covering a child which will allow him or her to purchase additional amounts

of life insurance in the future without requiring a medical examination?

A. A payor rider C. A dependent rider

B. A guaranteed insurability rider D. A family rider

29. Which of the following settlement options potentially provides a beneficiary with the greatest benefit per $1,000 of

proceeds?

A. Fixed period C. Life income

B. Interest only D. Fixed amount

30. Alex owns a graded premium life policy with a face amount of $500,000. A family term rider is added to the contract

in addition to an accelerated benefits rider of 30%. Alex is suffering from a terminal disease and notifies the insurer. The

insurer activates the latter rider and sends the maximum amount allowable to Alex. If he dies three months later, what

will be paid to his beneficiary?

A. $500,000

B. $470,000

C. The death benefit plus the accelerated amount of 30%

D. The remainder of the death benefit

31. Ben has a $200,000 whole life policy with a $50,000 AD&D rider. Also attached is an accelerated benefits rider. A

couple of years later Ben becomes terminally ill and receives $25,000 from the policy. Six months later he dies. What

will the beneficiary receive?

A. The principal sum C. $175.000

B. $150,000 D. $225,000

NILA ©L/H LicensingChapter 6—12

Page 93: FORWARD - WordPress.com · Return of premium p 6-8 6.0 Annuities p 7-1 Annuity principles and concepts p 7-1 Accumulation period versus annuity period p 7-1 Owner, annuitant and beneficiary

32. The waiver of premium rider, when added to a life insurance policy, would provide coverage in which of the following

circumstances?

A. The insured breaks his leg which permits him to work part time

B. The insured is unemployed for six consecutive months

C. The insured suffers a total disability lasting eight months

D. The insured suffers a partial disability lasting twelve months

33. A rider attached to a life insurance policy is part of the:

A. Face page of the policy C. Owner's rights

B. Incontestable provision D. Entire contract

34. In order to be eligible for long-term care benefits under a life insurance policy, the insured must be unable to perform

how many activities of daily living?

A. 2 C. 4

B. 3 D. 5

35. All of the following are considered to be an activity of daily living, EXCEPT?

A. Dressing C. Working

B. Bathing D. Feeding

36. Jack owns a life insurance policy with an accelerated benefits rider. If a benefit from this rider is paid, how does it affect

the policy’s death benefit ?

A. The benefit will be increased C. The benefit will be eliminated

B. The benefit will be decreased D. The benefit will be surrendered

ANSWERS

1. B2. B3. C4. C5. D6. A7. B8. B9. B10. C11. D12. C

13. A14. B15. D16. B17. B18. D19. B20. D21. C22. C23. B24. D

25. A26. D27. C28. B29. C30. D31. C32. C33. D34. A35. C36. B

NILA ©L/H LicensingChapter 6—13

Page 94: FORWARD - WordPress.com · Return of premium p 6-8 6.0 Annuities p 7-1 Annuity principles and concepts p 7-1 Accumulation period versus annuity period p 7-1 Owner, annuitant and beneficiary

BENEFIT RIDER

CHAPTER 6 KEY CONCEPTS

Accelerated benefitsAccumulate at interestBenefit ridersCashCash surrenderCatastrophic illness coverageDividend optionsDread disease coverageExtended term insuranceFixed amountFixed period

Interest only Life incomeLong-term care riderNon-forfeiture optionsPaid-up additionsReduced paid-up insuranceReduce the premiumSettlement optionsTerminal illness riderViatical settlementsViatee and Viator

NILA ©L/H LicensingChapter 6—14

Page 95: FORWARD - WordPress.com · Return of premium p 6-8 6.0 Annuities p 7-1 Annuity principles and concepts p 7-1 Accumulation period versus annuity period p 7-1 Owner, annuitant and beneficiary

SECTION III

AdditionalLife

InsuranceProducts and

Concepts

Page 96: FORWARD - WordPress.com · Return of premium p 6-8 6.0 Annuities p 7-1 Annuity principles and concepts p 7-1 Accumulation period versus annuity period p 7-1 Owner, annuitant and beneficiary

ANNUITIES

Page 97: FORWARD - WordPress.com · Return of premium p 6-8 6.0 Annuities p 7-1 Annuity principles and concepts p 7-1 Accumulation period versus annuity period p 7-1 Owner, annuitant and beneficiary

INTRODUCTION

An annuity is a series of periodic benefits or payments made to an annuitant. It is an insurancecontract between a contract owner and insurer. The contract owner funds the contracts and the insurerpromises to pay, at some point in the future, a series of periodic income payments for either a fixedperiod or for the rest of the annuitant's life. This chapter will focus on a description of the variousclassifications of the annuity product.

ANNUITIES

ANNUITY CONTRACTS

DEFINITION OF AN ANNUITY — An annuity contract and a life insurance policy possess some commoncharacteristics in that they both include some of the same provisions such as the ten day free-look, entire contractprovision and consideration clause. In addition, both are characterized by pre-paid financing (i.e., payment ofpremiums). An annuity is a contract only sold by a life insurance company that provides a mechanism foraccumulating cash. It is designed to help protect an individual from outliving his or her income. It is a savings typevehicle that is primarily used to set aside funds for the future. An annuity may be defined as the liquidation of anestate. This is the opposite of the definition of life insurance which involves the immediate creation of an estate.An annuity has as its primary function the systematic reimbursement or liquidation of funds (i.e., savings) for aspecified period or for life. Therefore, an annuity is a systematic approach to liquidating funds. An individualdeposits in or makes contributions to an annuity during the pay-in or accumulation phase. During this phase, theindividual making the interest amassing payments is known as the contract owner. The policy holder possessescontractual rights in the annuity contract when the contract is purchased. As soon as the contract owner receives theinitial periodic installment, he or she is now known as the annuitant and the annuity phase begins. The owner mustalso designate a beneficiary who will have access to the accumulated funds if the contract owner dies. This amountis really not a death benefit in the whole life or term insurance sense since it only includes the amount contributedand accumulated due to interest credited. This indicates that an annuity possesses an insurance aspect. During theaccumulation phase, the principal grows at interest. The interest earned as the principal grows is tax deferred. Whena periodic payment is received at some point in the future, it is considered to represent a combination of principalplus interest.

With regard to tax purposes, annuities are classified as qualified or non-qualified. A non-qualified annuity is onethat is not tax qualified. The contract owner receives tax deferral of interest earned but there is no tax deductionof premiums (or yearly tax savings through a salary reduction). Annuities purchased outside qualified pension plansdo not receive tax-favored treatment of premium payments. In other words, premiums are not tax deductible. Non-qualified annuities may be purchased by any individual or entity, but again, the premium payments are not taxdeductible. A qualified annuity is one that is purchased as part of a tax qualified retirement plan. If the premiumpaid for a qualified annuity is in the form of a contribution by an employer to a qualified retirement plan, thepremium is tax deductible. Some qualified annuities permit employees to fund the plan through a salary reduction

NILA ©

L/H Licensing

Chapter 7—1

Page 98: FORWARD - WordPress.com · Return of premium p 6-8 6.0 Annuities p 7-1 Annuity principles and concepts p 7-1 Accumulation period versus annuity period p 7-1 Owner, annuitant and beneficiary

(i.e., TSA). In this case, the plan is funded with pretax dollars which lowers the employee’s yearly taxable income.The interest earned is tax deferred. Whether the annuity is qualified or non-qualified, accumulations (i.e., interest earned) are tax deferred. Mostindividuals purchase an annuity contract in order to receive income in the future (i.e., retirement). It is attractive toinvestors since insurers generally pay higher interest rates than other traditional savings vehicles (i.e., certificates ofdeposit or money market funds). However, if the contract owner withdraws funds prior to a stated period, withdrawalpenalties could be assessed. If the contract owner dies or becomes disabled, funds can be withdrawn withoutpenalty. The parties involved in an annuity contract include the insurer, the contract owner, the annuitant and thebeneficiary. The contract owner has the right to name a beneficiary who will have access to the funds in the eventof the owner’s death prior to annuitization (i.e., the annuity or pay out phase). An annuity possesses some insuranceaspects in that a mortality factor is used to determine periodic payments, although it is not the same mortality factoras used in whole life or term life insurance. In addition, as mentioned previously, a beneficiary must be named inthe event that the contract owner dies prior to the annuity phase. If no beneficiary is listed on an annuity contractand the owner dies prior to the “annuitization, the proceeds are paid to the owner’s estate. Annuities may beclassified in several categories including: (1) according to the type of premium paid; (2) according to whenbenefits commence; (3) according to the units expressed (i.e., type of income); (4) according to the disposition ofproceeds; and (5) according to the number of lives covered.

ACCORDING TO HOW THE PREMIUM IS PAID — Annuities may be funded with a single or periodicpremium. There are two classifications of periodic premium plans.

! SINGLE PREMIUM ANNUITIES — This class of annuity is characterized by a lump sum or singlepayment. In other words, the annuity is entirely funded by a single premium. Monthly income paymentsmade to the annuitant may begin immediately (i.e., 30 days following the single premium) or beginning atsome time in the future (i.e., deferred). When an annuity is funded with a single, lump sum payment, theprincipal is created immediately. Generally, this type of annuity does not permit the contract holder to makeany additional deposits into the contract. This means that the contract is fully funded with one lump sumpayment.

! PERIODIC PREMIUM ANNUITIES — This class is made up of two types. The level premium annuityis characterized by level or constant payments each year which funds the annuity. For example, John, age35, purchases a level premium annuity with an annual premium of $1,200. John will pay that level sum eachyear until retirement (i.e., age 65). At that time, he will begin to receive monthly income payments. Thistype is also known as an annual premium annuity. A flexible premium annuity is characterized by periodicpremiums that may be variable amounts each year. In this case, the contract owner pays-in whatever he orshe desires each year. These premiums are paid until the contract owner wishes to begin receiving incomeat retirement time. As long as a minimum payment is made, the contract owner may contribute whatever heor she can afford each year. The future income benefit will be based upon the total amount of funds savedwhen the plan annuitizes (i.e., when income begins).

! ANNUITY PREMIUMS — Annuities possess their own mortality tables which are different from thoseused for life insurance. Items taken into consideration include the interest rate paid, the amount of totalcontributions or accumulations and the settlement options selected. Premiums paid are deposited into theinsurer’s general account and not the individual policyholder’s contract. An annuitant's occupation orhobbies hold no influence since these items will not affect the liquidation of funds.

ACCORDING TO WHEN BENEFITS BEGIN — Annuities may be described according to when the pay-outphase commences including:

NILA ©

L/H Licensing

Chapter 7—2

Page 99: FORWARD - WordPress.com · Return of premium p 6-8 6.0 Annuities p 7-1 Annuity principles and concepts p 7-1 Accumulation period versus annuity period p 7-1 Owner, annuitant and beneficiary

! IMMEDIATE ANNUITY — This class of annuity is designed to generate an income stream to theannuitant soon after it is purchased. It provides for the first installment payment to an annuitant thirty (30)days after the annuity is funded or purchased. Only lump sum premiums will be accepted by an insurer forthis type of plan. This means that there is no accumulation period since only a single payment is made. No income is paid to the annuitant until the lump sum has been provided to the insurer. The incomepayments made to the annuitant consist of principal and interest. In addition, as in any annuity, how longthe annuity generates income to the annuitant will depend upon a couple of factors including: (1) the totalamount of the account; and (2) the settlement or distribution option that the owner selects. The longer theperiod of income payments to the annuitant and the more guarantees provided (i.e., period certain, etc.), thelower the amount of each installment. The income from the annuity may be either a fixed dollar amount eachmonth or a variable sum. An immediate annuity is best suited for a person who needs "immediate" income(i.e., someone who is totally disabled or someone who is ready to retire).

! DEFERRED ANNUITY — This class of annuity may be funded with any type of premium plan. Thisclassification of an annuity is different from the immediate annuity since it does include an accumulationperiod. This means that there is a lengthy period between the time the contract is purchased and when theincome or annuity phase commences. A deferred annuity is useful for those who wish to defer income untilthe future (i.e., retirement). Contributions may accumulate over time and every year the insurer credits thefunds with a certain rate of interest which is tax deferred. When the owner wishes to receive cash from thefund in the future (i.e., retirement), he or she has three options including: (1) a lump sum distribution, ofwhich the interest credited is taxable; (2) systematic or periodic withdrawals; or (3) convert the fund to theannuity phase and begin to receive a monthly income stream. For example, if a new physician is justbeginning his or her practice and wants to set aside funds for the future but his or her current expenses arehigh, one method available in order to achieve this objective is to purchase a flexible premium deferredannuity. A deferred annuity emphasizes safety of principal, asset accumulation and tax deferral of interest.

ACCORDING TO THE SOURCE OF INCOME — Some annuities may be classified by the source ofincome payments provided.

! A FIXED ANNUITY guarantees a predetermined income or level benefit payment amount which is paideach month for the life of the annuitant. The recipient (i.e., the annuitant) will receive the same monthlyincome or fixed dollar amount each month for the rest of his or her life. Fixed annuities are derived fromthe insurer's general account assets since it is this account that provides an interest rate guarantee as wellas the fixed dollar or income guarantee. A fixed annuity also guarantees safety of principal (as long asthe insurer remains solvent). This type of annuity is a conservative product in comparison to a variableannuity. Since it is characterized by predetermined amount of income, its purchasing power would be mostaffected by inflation. Once the predetermined income begins in a fixed annuity, the beneficiary receives aguaranteed refund when the annuitant dies (if a period certain has been selected). This type of annuity isdesigned to limit the contract owner’s investment risk since the contract provides a guaranteed return. Thismeans that the insurer invests the funds in safe and conservative investments which allow it to guarantee theannuity benefit. Since the values are guaranteed to the owner of the contract, it is the insurer that assumesthe investment risk. The insurer is required by the contract to provide the promised benefit whether or notit earns its assumed interest rate. The investment account used is the general account which guarantees areturn. The general account of an insurer holds all of its assets.

Again, a fixed annuity guarantees a minimum amount of interest to be credited to the purchase payment. Income payments do not vary from one payment to the next. The insurer can afford to make guaranteesbecause the money in a fixed annuity is placed in the general account of the insurer, which is part of theinvestment portfolio. The insurer makes investments that are conservative enough to ensure a guaranteedrate to the annuity owners.

NILA ©

L/H Licensing

Chapter 7—3

Page 100: FORWARD - WordPress.com · Return of premium p 6-8 6.0 Annuities p 7-1 Annuity principles and concepts p 7-1 Accumulation period versus annuity period p 7-1 Owner, annuitant and beneficiary

! A VARIABLE ANNUITY is a contract issued by an insurer that provides the contract owner with theoption of having premiums invested and managed differently. This type of annuity generally consists of twoinvestment accounts including a general account and a separate account. A guaranteed return is providedwhen funds are invested in the general account. When funds are invested in a "separate account,” they arebeing invested in equity products such as common and preferred stocks, bonds, and other such vehicles.There is a greater potential for higher returns in a variable annuity but there is no guarantee present as thereis in the general account. The separate account holds all of the variable account options of the insurer. Thevariable annuity allows the contract holder to control the investment of his or her premiums. This means thatthe contract holder assumes the investment risk when funds are directed to the separate account. However,more flexibility is provided since the contract owner can determine how much risk he or she is willing toassume. The benefits ultimately paid by the contract will be determined by the performance of the separateaccount (i.e., stock market performance). If an equity fund (i.e., a mutual fund) performs well, the monthlyincome amount paid to the annuitant will increase. If the fund does poorly, the monthly installment paymentwill decrease. In other words, just like the cash value in a variable whole life insurance policy, the cash ina separate account of a variable annuity contract is not guaranteed.

A FINRA (formerly NASD) license is needed in order to sell this or any type of variable product. Thislicense is also known as a Series 6 license. A securities license is required because variable products areconsidered a security. In addition, a life insurance license is also needed. Therefore, these products areregulated at the Federal and State level. This product allows the owner to keep pace with economicconditions and create a bit of a hedge against inflation. A variable annuity is characterized by variable orflexible premiums, variable rates of return. Its performance advances or declines with economic and marketconditions. This product also provides the contract owner with flexibility and control over the investmentportion of the contract.

ACCORDING TO DISPOSITION OF PROCEEDS (SETTLEMENT OPTIONS) — An annuity may alsobe described according to life contingency settlement options available including:

! STRAIGHT LIFE ANNUITY — This contingency option, also referred to as a pure life annuity or "life"annuity, is classified according to the length of time for which the annuitant will receive income. It providesincome to the recipient, once it commences, for life with no refund paid to the annuitant's family upon hisor her death. This settlement option possesses the greatest amount of risk to the annuitant as well since thereis no survivorship (i.e., no refund). The purpose of a straight life annuity is to protect against outliving onesincome. This means that a straight life annuity protects against superannuation. In other words, a straightlife annuity protects the annuitant against using up income due to longevity. Insurers paying out underlife annuities may suffer adversely if there is a sudden decrease in the mortality rate. This means that morepeople are living longer and therefore, insurers are paying life incomes longer as well. In addition, sincewomen have a longer life expectancy than men, monthly payments would be smaller to a female, all thingsbeing equal. For example, Joe and Joan are twins and inherit an equal amount of money from their favoriteaunt. If they both purchase an annuity with the funds and each contract includes the same life income option,Joe’s monthly income payments from the annuity contract will be higher since his life expectancy is shorterthan that of Joan. Straight life annuities may be funded with any available type of premium plan (e.g., lumpsum, periodic or flexible).

! STRAIGHT LIFE ANNUITY WITH PERIOD CERTAIN — A life annuity with a period certain(i.e., a guaranteed minimum) pays a benefit (i.e., income) for life but pays a survivor benefit if the annuitantdies before the end of the period certain (i.e., ten years). In other words, the annuitant or survivors areentitled to a guaranteed income of at least a specified number of years (i.e., period certain), or a refund ifthe annuitant dies before the end of the guaranteed refund period (i.e., refund life annuity). This refundavailability indicates that there will be a guaranteed minimum returned to a beneficiary if the annuitant dies. For example, if an annuitant is to receive $2,000 per month for five years certain, and she dies after the

NILA ©

L/H Licensing

Chapter 7—4

Page 101: FORWARD - WordPress.com · Return of premium p 6-8 6.0 Annuities p 7-1 Annuity principles and concepts p 7-1 Accumulation period versus annuity period p 7-1 Owner, annuitant and beneficiary

second year, her beneficiary possesses two options to choose from that will provide a refund or return ofannuity (income) proceeds. Remember that a refund annuity or annuity certain classification guarantees adefinite number of income payments.

1. INSTALLMENT REFUND OPTION — This refund will pay the beneficiary the samemonthly income benefit that the annuitant was receiving until the end of the period certain. In ourprevious example, this would be $2,000 per month for three (3) years.

2. CASH REFUND OPTION — This option is available where the beneficiary of the annuitantchooses to receive the refund in a lump sum. In our previous example, the cash refund amountwould be $72,000 ($2,000 per month for three years). The beneficiary does not receive a refundequal to the beginning annuity funds plus interest. He or she receives a "refund" of the remainingguaranteed amount.

An annuity certain is a description of income or installments for a fixed period of time if that is what theowner decides. This means that the monthly income will be paid for a specified period only (i.e., not for life). If the annuitant dies prior to the end of that period, payments continue to the designated beneficiary for theremainder of the specified period. An annuity certain may also be paid for life but with a guaranteed (i.e.,certain) income period. This means that if the annuitant lives for thirty years, he or she will receive themonthly payments during that time. However, if the annuitant dies within the guaranteed period, aninstallment or lump sum refund will be paid to the beneficiary. Again, the purpose of any "life annuity" isto make sure that the annuitant will not outlive his or her income.

NUMBER OF LIVES — Annuities may be classified according to the number of lives covered, whether singlelife or multiple life types. There are three basic types including:

! INDIVIDUAL OR SINGLE LIFE ANNUITY — The most common form, this type covers one lifeonly. Generally, there is no survivorship with this type of annuity.

! JOINT LIFE ANNUITY — A type of multiple life contract, this type of annuity is designed to paybenefits to two or more annuitants at the same time. All benefits, however, will end once the first annuitantdies. In this manner, it is similar to a joint life insurance policy.

! JOINT AND SURVIVOR ANNUITY — Another form of multiple life contract, benefits under this typeof annuity are paid throughout the lifetime of one or more annuitants. Therefore, payments continue untilthe last annuitant dies. In other words, joint and survivor annuities guarantee income payments for theduration of two lives.

ADDITIONAL ANNUITYCHARACTERISTICS AND ASPECTS

ACCUMULATION PERIOD — During the pay-in phase, the insurer is obligated to return all (or a portion) ofthe annuity's value if the contract owner dies. This value will be equal to the amount of any contributions (lesswithdrawals or other expenses), plus interest. Although the contract does not identify the proceeds available at deathas a death benefit per se, the owner must name a beneficiary who is entitled to proceeds if the owner dies during theaccumulation period. Again, the interest earned during the accumulation phase is tax deferred. Additional surrendercharges may also be assessed at withdrawal. The accumulation period will cease when any of the following occur:

NILA ©

L/H Licensing

Chapter 7—5

Page 102: FORWARD - WordPress.com · Return of premium p 6-8 6.0 Annuities p 7-1 Annuity principles and concepts p 7-1 Accumulation period versus annuity period p 7-1 Owner, annuitant and beneficiary

(1) the contract owner dies; (2) the annuity or "pay out" phase begins; or (3) the policy is surrendered. This periodwill not cease if a premium payment has not been made.

ANNUITY PERIOD — This is the income phase. Sometimes referred to as “annuitization,” this is the period oftime beginning when the contract owner gives up the right to the funds in the contract, in return for a promise ofmonthly income.

SURRENDER CHARGES — The charges assessed when an annuity is cancelled by the contract owner are calledsurrender charges. These are sometimes referred to as back-end loads. A surrender charge (i.e., penalty) is assessedwhenever a cash withdrawal is made in excess of a specified percentage (i.e., 10%), in any policy year. If the totalannuity is surrendered, the surrender charges are subtracted from the annuity value. For any withdrawals of lessthan the specified percentage, no surrender charge is assessed. This surrender charge generally decreases eachyear. For instance, an insurer may assess a surrender charge of 8% if any withdrawals in excess of 10% of theaccount balance are effected in the first year. This penalty will decrease 1% per year for the next eight years. In yearnine, there will be no surrender charge for excess withdrawals. In other words, after this time period expires, theinsurer effects a waiver of surrender charges. Annuity contracts also identify the non-forfeiture values which willbe the value of the fund less any surrender charges if funds are being withdrawn. As in certain types of qualifiedretirement plans available today, funds may be withdrawn with no surrender charges if the owner dies or becomesdisabled or requires specific types of extended medical care in a skilled nursing or other type of extended carefacility. Surrender charges are designed to make moving money out of an annuity less attractive to the contractowner. Such charges are generally waived in case of death or disability. Surrender charges assessed by an insurerare different than the 10 % federal tax penalty for a premature withdrawal. Therefore, a withdrawal from an annuitycan be subject to both a surrender charge and a tax penalty.

FLEXIBLE PREMIUM DEFERRED ANNUITY (FPDA) — This is the most popular annuity product soldtoday. It provides flexibility regarding payments and allows for the supply of income to an annuitant in the future. Interest earned is tax deferred. This type of annuity has effectively replaced the annual premium retirement annuitycontract. This older contract provided for a fixed schedule of annual premiums and possessed a high load (i.e.,expenses). It also included bundled premiums. Today's FPDAs have no (or little) front-end loads due to thetremendous competition between insurers. Many FPDAs do have back-end or surrender charges.

SINGLE PREMIUM ANNUITY — These types may provide immediate income (SPIA) or deferred income(SPDA). SPIA require a lump sum payment and income begins thirty (30) days later. The SPDA is paid for witha lump sum but income will be paid in the future. SPDAs sometimes include a bailout provision that allows theowner to withdraw funds without a penalty if the interest rate falls below a specified rate.

FIXED AND VARIABLE ANNUITIES — A fixed annuity pays a guaranteed, predetermined or level benefitpayment amount during the annuity phase. Premiums are placed in the insurer's general premium asset account withother non-variable product premiums. These premiums are invested in fixed rate vehicles (i.e., CDs, money market,etc.) to provide a "fixed" return based upon interest rate guarantees. These contracts include minimum interest rateguarantees and may pay higher rates based on current economic market conditions. Premiums paid on variableannuities are placed in a separate account. These funds are invested in securities or equities such as common stock,preferred stock or bonds. This type of annuity provides the potential for an increasing income if the securitiesperform well. A life insurance and FINRA securities license is needed in order to solicit this product.

ANNUAL PREMIUM RETIREMENT ANNUITIES — This is a vehicle that provides tax deferred incometo the owner. The income earned on the annual premium paid into the contract will not be taxed until it is removedfrom the account. Amounts deposited into the account are not tax deductible. However, again, the interest or

NILA ©

L/H Licensing

Chapter 7—6

Page 103: FORWARD - WordPress.com · Return of premium p 6-8 6.0 Annuities p 7-1 Annuity principles and concepts p 7-1 Accumulation period versus annuity period p 7-1 Owner, annuitant and beneficiary

earnings paid on the principal are tax deferred. As mentioned earlier, these older types of contract werecharacterized by high loads.

USES OF ANNUITIES — An annuity may be used for any reason in which to accumulate cash. This type ofcontract is primarily used to provide income at retirement. Again, it is a savings vehicle with tax deferral or taxdeferred growth. Contract owners may elect to receive a lump sum payout (i.e., settlement) when the annuity phasebegins. Receiving a lump sum settlement possibly represents a large tax liability on the part of the recipient. Annuities may also be used to fund qualified retirement plans, on an individual or group basis, which receive taxdeferral. Contributions may be tax deductible if the contract is qualified. Annuity contracts may also be used to fundvehicles such as Keogh Plans, Simplified Employee Pensions (SEPs), 401K Plans, pension plans and profit sharingplans. These latter two plans may be of the defined contribution type which specifies the amount that each employeewill contribute to the plan; or of the defined benefit type which specifies the benefit amount the (retired) employeewill receive in the future. However used, annuities may be used through an employer for employees in a group or,as previously reviewed, on an individual basis.

Annuities may also be utilized for personal use. They may be used to fund Individual Retirement Accounts (IRAs)and thus are known as Individual Retirement Annuities. Annuities may also be used to fund non-qualified retirementplans but such plans do not receive the same tax advantaged treatment as a qualified plan. Some annuities are usedto provide funds for a child's education or to pay out lottery winnings. The primary use of an annuity, for mostindividuals, is to set aside funds for retirement while receiving tax deferred growth.

NEWER TYPES OF ANNUITIES — Insurers also offer equity indexed annuities as well as market valueadjusted annuities. A non-variable annuity product whose renewal interest rate is linked to (but the funds are notdirectly invested in) a stock market-related index (i.e., Standard & Poor's 500 Index) is referred to as an equity-indexed annuity. Also referred to as an indexed annuity, this is a form of fixed annuity and provides the contractowner with safety of principal (since the principal is guaranteed) and a guaranteed minimum return (i.e., 3%) sincea high percentage of the contract owner’s premium is invested in high grade government bonds. This provides adownside guarantee if the market performs poorly. In other words. This type of contract allows the owner toparticipate in market gains without assuming the risk of a market decline. It also provides the opportunity forappreciation (i.e., upside potential) in the stock market. Generally, the contract owner is obligated to remain in thecontract for some minimum period of time, such as three years, and then return a percentage of the appreciation (i.e.,10%) in the selected equity index over that time. This "percentage of the appreciation" may also be referred to asthe participation rate.

A market value-adjusted annuity (MVA), also called a modified guaranteed annuity, shifts some but not all of theinvestment risk from the insurer to the contract owner since the annuity account value will fluctuate as market interestrates fluctuate. In other words, it is a type of single premium-deferred annuity that allows contract owners to lockin a guaranteed interest rate over a specified maturity period ( i.e., usually two to ten years). They function in asimilar fashion to that of bonds with regard to bond value fluctuations (i.e., when interest rates fall, bond prices rise,etc.). MVAs generally provide higher interest rates than traditional annuities. They also possess lower reservingrequirements and pass on more risk to the contract owner since, when surrendered, there will generally be both amarket value adjustment and a surrender penalty assessed to the owner. In other words, when the contract issurrendered early the owner will receive the value of the contract minus a surrender charge.

NILA ©

L/H Licensing

Chapter 7—7

Page 104: FORWARD - WordPress.com · Return of premium p 6-8 6.0 Annuities p 7-1 Annuity principles and concepts p 7-1 Accumulation period versus annuity period p 7-1 Owner, annuitant and beneficiary

QUIZ

1. Which of the following types of annuities provides a pre-determined income amount for life?

A. Deferred and fixed annuity C. Joint life annuity

B. Immediate annuity D. Fixed life annuity

2. Which of the following settlement options would provide an annuitant with the greatest amount of income if he or she

were to live until age 90?

A. Installment refund annuity C. Deferred annuity

B. Life annuity with period certain D. Straight life annuity

3. If the beneficiary of a deceased annuitant selects a cash refund, the funds are paid in:

A. A lump sum C. Policy proceeds

B. Loan values D. Monthly installments

4. Bill inherits $1 million from his great aunt Betty. Three years prior to receiving this sum, Bill was involved in an

accident and was totally disabled for life. If he wishes to use all or a portion of his inheritance to purchase an annuity,

what would you, his agent, recommend?

A. A level premium annuity C. An immediate annuity

B. A deferred annuity D. A cash refund annuity

5. In order to solicit a variable annuity, a sales representative must hold which of the following licenses:

A. A securities license issued by the National Association of Securities Dealers

B. A broker/dealer license

C. A life insurance license

D. A securities license and a life insurance license

6. An annuity is a savings vehicle which protects against the possibility of outliving one's income. Which of the following

best describes an annuity?

A. The immediate creation of an estate C. The systematic creation of an estate

B. The liquidation of funds D. The transfer of risk through the accumulation of funds

7. An annuity provides its owner tax deferral during which of the following phases?

A. The pay-out phase C. The accumulation phase

B. The liquidation phase D. The annuity phase

8. If an individual wishes to purchase an immediate annuity, what type of premium payment plan must he or she utilize?

A. A level premium plan C. A flexible premium plan

B. A single premium plan D. An immediate premium plan

9. When the series of periodic benefits are paid to the recipient, the installments are comprised of:

A. A return of principal C. Principal and interest

B. Interest only D. Tax deferred interest

NILA ©

L/H Licensing

Chapter 7—8

Page 105: FORWARD - WordPress.com · Return of premium p 6-8 6.0 Annuities p 7-1 Annuity principles and concepts p 7-1 Accumulation period versus annuity period p 7-1 Owner, annuitant and beneficiary

10. Which of the following types of annuities involves the use of a separate account?

A. Variable annuity C. Life annuity

B. Fixed annuity D. Deferred annuity

11. Which of the following best describes a fixed straight life annuity?

A. Fixed premium paying income for life C. Predetermined income with ten years guaranteed

B. Predetermined income for life D. Predetermined income for life with no survivorship

12. An individual who purchases an annuity is referred to as the contract owner during the accumulation phase. Once income

is paid from this contract, the contract owner is known as the:

A. Beneficiary C. Payor

B. Principal D. Annuitant

13. Many annuities include surrender charges. These are charges assessed if the owner withdraws more than specified

percentage of the account value. Surrender charges are also referred to as:

A. Withdrawal expenses C. Back-end loads

B. Assessments D. Loading factors

14. Premiums paid to fund fixed annuities are deposited in which of the following:

A. Separate account C. Deferred account

B. General account D. Contingent account

15. An annual premium retirement annuity contract is characterized by which of the following?

A. Fixed death benefit C. Defined benefits

B. Flexible assessments D. High loads

16. All of the following statements regarding annuities are true, EXCEPT:

A. An annuity is a systematic approach to liquidating funds

B. Annuities may only be funded with flexible premium payments

C. When an annuity is funded with a single premium the estate is created immediately.

D. Life insurance companies may offer annuities

17. Which of the following is true regarding a cash refund classification of an annuity?

A. The refund amount equals the original amount of the annuity plus interest

B. It guarantees a certain number of payments

C. It guarantees income for the duration of two lives

D. The interest received as part of the refund is taxable in the year received

18. Harry owns the Pumpkin Pub. He purchased an annuity years ago in order to provide him with income when he retires.

If Harry has a Pure Life annuity that begins to provide life income at retirement, what will be paid to his beneficiary if

he dies during the annuity phase?

A. A guaranteed income for a period certain C. Monthly income for the life of the beneficiary

B. The amount of the principal plus interest D. Nothing, since there is no survivorship

NILA ©

L/H Licensing

Chapter 7—9

Page 106: FORWARD - WordPress.com · Return of premium p 6-8 6.0 Annuities p 7-1 Annuity principles and concepts p 7-1 Accumulation period versus annuity period p 7-1 Owner, annuitant and beneficiary

19. When does the policyholder of an annuity possess contractual rights in the contract?

A. At annuitization C. When insurable interest is present

B. At amortization D. When the contract is purchased

20. The primary purpose of a straight life annuity is to protect against:

A. Premature death C. Superannuation

B. The difficultly of saving D. The reduction of purchasing power

21. An annuity is different from a life insurance contract in each of the following ways, EXCEPT:

A. An annuitant receiving benefits is living whereas death has to occur for a life insurance benefit to be paid

B. An annuity has no stipulated benefit when the contract is purchased but a life insurance contract specifies a

death benefit

C. An annuity is characterized by pre-paid financing whereas a life insurance contract is characterized by a pre-

paid premium

D. An annuity can be used to fund a qualified plan but a life insurance policy cannot be used in such a manner

22. If a beneficiary is not identified in an annuity contract, to whom are proceeds paid if the owner or annuitant die?

A. The heirs of the owner C. The estate of the annuitant

B. The State general revenue fund D. The insurer who issued the contract

23. How are the accumulations or appreciation credited to an annuity during the life of the contract taxed?

A. They are not taxed C. They are tax free

B. They are taxable as income D. They are tax deferred

24. Bob desires an annuity that will provide him with periodic income for a short period of time without a lifetime guarantee

of income. Which of the following would help him achieve this objective?

A. A refund annuity C. A variable annuity

B. An annuity certain D. A flexible premium deferred annuity

25. Alice leaves $50,000 each to her twin grandchildren, George and Gracie. Each spends the money wisely on the purchase

of an annuity with a life income option. Which of the following statements is true concerning this scenario?

A. Gracie’s monthly benefit will be larger than that of George

B. George’s monthly benefit will be greater than that of Gracie

C. Monthly income paid to either will be influenced by their health history

D. The monthly payments to both George and Gracie will be the same

26. The monthly income paid by an annuity once the contract annuitizes is based upon which of the following?

A. The rate of inflation C. The life expectancy of the annuitant

B. The life expectancy of the contract owner D. The life expectancy of the beneficiary

27. Which of the following is generally used to payout lottery winnings ?

A. A universal life policy C. A qualified plan

B. A 401- K plan D. An annuity

NILA ©

L/H Licensing

Chapter 7—10

Page 107: FORWARD - WordPress.com · Return of premium p 6-8 6.0 Annuities p 7-1 Annuity principles and concepts p 7-1 Accumulation period versus annuity period p 7-1 Owner, annuitant and beneficiary

ANNUITIES = INCOME

28. Which of the following is permitted to issue annuity contracts ?

A. A life insurance company C. Financial institutions

B. An investment firm D. A trust

29. Which of the following best describes a straight life annuity with twenty years certain ?

A. It pays income to an annuitant for twenty years and then pays a beneficiary for life

B. It pays income for life with a minimum guarantee of twenty years of income to an annuitant or beneficiary

C. It pays income for life with a maximum guarantee of twenty years of income to an annuitant or beneficiary

D. It pays income for life if the annuitant lives twenty year past life expectancy

30. Which of the following may take a loan or distribution from a fixed annuity?

A. Contract owner C. Beneficiary

B. Annuitant D. A non-living entity

ANSWERS

1. D2. D3. A4. C5. D6. B7. C8. B

9. C10. A11. D12. D13. C14. B15. D16. B

17. D18. D19. D20. C21. C22. C23. D24. B

25. B26. C27. D28. A29. B30. A

CHAPTER 7 KEY CONCEPTS

Accumulation phaseAnnuityAnnuity phaseDeferred annuityEquity indexed annuityFixed annuityFlexible premiumImmediate annuityLife annuityMarket value adjusted annuityNumber of livesPeriod certainQualified vs non-qualifiedRefund annuitySuperannuationSurrender chargesTax considerationsVariable annuity

NILA ©

L/H Licensing

Chapter 7—11

Page 108: FORWARD - WordPress.com · Return of premium p 6-8 6.0 Annuities p 7-1 Annuity principles and concepts p 7-1 Accumulation period versus annuity period p 7-1 Owner, annuitant and beneficiary

EXHIBIT 1

CLASSES OF ANNUITIES

HOW PREMIUMSARE PAID

WHEN BENEFITS(INCOME) BEGIN

SOURCE OR TYPEOF INCOME

DISPOSITION OFPROCEEDS

! Single (lump sum)! Level! Flexible

! Immediate! Deferred

! Fixed! Variable

! Straight Life! Straight Life w /

period certain* Refunds in lump sumor installments

!An indexed annuity isa form of fixed annuity.Also referred to as non-variable

Protects against outliv-ing one's income. Inother words, a "life an-nuity" protects againstsuperannuation.

ANNUITIES MAY BE

! Individual! Joint Life! Joint and Survivor

NILA ©

L/H Licensing

Chapter 7—12

Page 109: FORWARD - WordPress.com · Return of premium p 6-8 6.0 Annuities p 7-1 Annuity principles and concepts p 7-1 Accumulation period versus annuity period p 7-1 Owner, annuitant and beneficiary

TAX TREATMENTOF LIFE

INSURANCE andANNUITIES

Page 110: FORWARD - WordPress.com · Return of premium p 6-8 6.0 Annuities p 7-1 Annuity principles and concepts p 7-1 Accumulation period versus annuity period p 7-1 Owner, annuitant and beneficiary

INTRODUCTION

Life insurance and annuity products receive several types of tax benefits. This chapter will identifythe tax benefits, liabilities and other ramifications of these products.

TAX TREATMENT OF LIFEINSURANCE AND ANNUITIES

TAX CONSIDERATIONS

TAX TREATMENT OF LIFE INSURANCE — According to the Internal Revenue Code (IRC 101-a-c),premiums paid for individual life insurance policies are not tax deductible since, like auto insurance premiums, theyare considered to be a personal expense. This same tax treatment applies to any type of insurance premiums whetherthey are paid for individual disability insurance, auto insurance or homeowners insurance. Just because a premiumis used to buy life insurance on a spouse or in a third-party ownership situation does not mean that the premiums willbe tax deductible. Since the premiums paid are not tax deductible, the proceeds from the life insurance policy (i.e.,the death benefit) are paid income tax free to the named beneficiary. In some instances, premiums paid on lifeinsurance may be tax deductible if they are used to pay for business insurance purposes or to provide for charitablecontributions. The Tax Equity and Fiscal Responsibility Act (TEFRA) of 1982 and subsequent tax reform acts (i.e.,TRA of 1984, etc.) continually modify rules with regard to taxation issues in connection with life insurance, annuitiesand qualified plans.

O CASH VALUES, DIVIDENDS AND POLICY LOANS — The equity that builds within a wholelife policy is referred to as the cash value. This is the value that, once it begins to accumulate, grows witheach subsequent premium payment. The interest paid on the cash value as it increases or accumulates is taxdeferred unless the cash value exceeds the net single premium required to fund future contract benefits. Inother words, according to the cost recovery rule, if the cash surrender value exceeds the cost basis (i.e., totalpremiums paid less dividends received), the excess (i.e., the gain) is considered ordinary income and taxable. Therefore, a tax consequence may be created with regard to the cash value when a contract is surrendered.

Dividends paid on a whole life policy are tax exempt. Dividends are considered to be a return of overpaidor excess premiums. However, if dividends are left with the insurer to accumulate at interest, the interestearned will be taxable as ordinary income in the year received. If the life insurance policy is determinedby the IRS to be a Modified Endowment Contract (MEC), dividends will be taxable unless they are used topurchase paid-up additional insurance. In addition, dividends payable under a MEC may be subject to a 10%penalty tax (applying to premature distributions prior to age 59 ½). If the total of dividends received undera life insurance policy (i.e., non-MEC policy), exceed the total premiums paid, all dividends received inexcess of the premiums paid is taxable as ordinary income.

With regard to policy loans, if a contract owner borrows against the cash value in the contract, there are notax consequences in most situations. However, if a policy is a MEC, distributions are subject to the interestfirst rule which states that they are taxable as income if the cash value of the contract immediately prior tothe payment exceeds the cost basis in the contract. Borrowing against the cash value is sometimes referred

NILA ©

L/H Licensing

Chapter 8—1

Page 111: FORWARD - WordPress.com · Return of premium p 6-8 6.0 Annuities p 7-1 Annuity principles and concepts p 7-1 Accumulation period versus annuity period p 7-1 Owner, annuitant and beneficiary

to as a partial surrender and such action on the part of the owner, while not resulting in a taxable event,does lower the owner's equity in the policy. If a total surrender occurs, again, the cash value received is nottaxable as long as it does not exceed what the owner paid in premium. What the owner paid into the contractis sometimes referred to as the "cost basis." For instance, assume Bob is the owner of a $100,000 lifeinsurance policy on his own life. The cash surrender value is $45,000. He has paid $35,000 in premiumsover the years. If Bob surrenders the policy, he will have a taxable gain of $10,000 (that he must report asordinary income). In addition, when a contract owner borrows against the cash value of a whole life policy,the interest paid to the insurer is not tax deductible.

O DEATH BENEFITS — The death proceeds from a life insurance contract include the policy’s faceamount of coverage plus any additional insurance amounts paid as a result of the insured’s death such aspaid-up additions, term riders or accidental death benefits. The beneficiary receives a lump sumsettlement of policy proceeds income tax free. Whenever another settlement option is selected by thebeneficiary where the face amount is retained by the insurer, interest must be paid to the beneficiary. Thismeans that under any settlement option selected by the beneficiary, when the proceeds or principal is leftwith the insurer, interest will be paid on the principal by the insurer.

Again, the proceeds are tax free but the interest paid is taxable as ordinary income. However, accordingto the transfer for value rule, if any policy is transferred to another party for a valuable consideration, it willlose its tax-free status. In other words, if a policyowner transfers a life insurance policy to another partywithin three years of his or her death, the entire face amount is included in the former owner’s estate. If nobeneficiary is alive when the insured dies or if one has not been designated, the death proceeds will bepayable to the insured's estate and subject to possible Federal estate and / or State death taxes.

O VALUES INCLUDED IN INSURED'S ESTATE — When an individual dies, all his or her assets arecompiled and a monetary value is placed upon them. This is known as one's estate. Federal and state deathtaxes may be levied on the estate value. These death taxes are also known as estate taxes. These taxes arebased upon the value of the estate at the time of the property owner's death. The following items areexamples of property that is generally included in the estate:

! Real property.! Personal property.! Life insurance death benefits.! Annuities.! Gifts to others.! Any interest or rights in other property.

Real property includes a home or rental property. Personal property could include collectibles, antiques,cash, mutual funds or other like items. The (death benefit) value of a life insurance policy is included in thepolicy owner's estate, even though the face amount is paid to the beneficiary income tax free. The value ofan annuity is also an asset and is also included in one's estate. Gifts or property transfers, in most cases,made to others are included in the estate if the decedent dies within sixty (60) months of making the gift. Gifts to others that are made at death or revocable gifts are also included. Any other interests or rights inproperty (i.e., community, joint or marital property) may be included as well. The total value of all theseassets is referred to as the gross estate. In 2011, if the estate value exceeds $1,000,000, ($2 million forcouples) the excess amounts are taxable and the applicable federal estate tax must be paid within nine (9)months of the decedent's death (maximum rate 55%). State inheritance taxes are applicable in many Statesin addition to federal estate taxes. Further costs associated with death, as mentioned in an earlier chapter,include probate, administrative and other fees (i.e., attorney fees).

NILA ©

L/H Licensing

Chapter 8—2

Page 112: FORWARD - WordPress.com · Return of premium p 6-8 6.0 Annuities p 7-1 Annuity principles and concepts p 7-1 Accumulation period versus annuity period p 7-1 Owner, annuitant and beneficiary

MODIFIED ENDOWMENT CONTRACTS

A life insurance policy is considered to be a Modified Endowment Contract if the premiums paid for it are not inproportion to the death benefit provided. Any life insurance policy purchased after 6/20/88 is considered by the IRSto be a MEC if it does not satisfy the seven-pay test reviewed below. The reason that the IRS modified the rulesconcerning life insurance policies is that many insurers began marketing them as investment vehicles rather than lifeinsurance plans providing protection against premature death. A MEC is not a life insurance policy. It is an IRSclassification of an insurance contract if certain conditions are not satisfied with regard to the funding of the contract.

SEVEN PAY TEST — To avoid being classified as a modified endowment contract, a life insurance policy mustsatisfy a seven-pay test. If a whole life insurance policy does not satisfy the seven pay test, it will be considered tobe a modified endowment contract. For example, according to The American College Study Guide for LifeInsurance, "if the accumulated premiums paid at any time during the initial seven years of the contract exceed thesum of net level premiums that should have been paid during this time frame (i.e., if it was a seven pay life plan), thepolicy is considered to be a MEC." In other words, a whole life policy will be considered to be a MEC if thecumulative amount paid (i.e., premiums) under the contract at any time during the first seven years of the contract’sexistence exceeds the cumulative amount that would have been paid had the policy’s annual premium equaled thenet level premium for a seven-pay life insurance policy. The MEC rules were initiated under TAMRA of 1988 (i.e.,the Technical and Miscellaneous Revenue Act). This rule or seven pay test was designed to reduce or prevent thepurchase of whole life insurance (i.e., universal life) for the purpose of short-term investment gains. In other words,the primary purpose of federal regulations governing MECs was to reduce incentives for the use of life insuranceas a short term investment vehicle.

DISTRIBUTIONS — Like non-MEC life insurance policies, a Modified Endowment Contract receives favorabletax treatment on interest credited to the policy’s cash value (i.e., tax deferral) as long as no distributions are made. However, cash distributions from a whole life policy classified as a MEC are subject to taxation as income in theyear received if the cash value of the contract prior to pay out exceeds the investment (i.e., premiums) in the policy. Distributions include surrendering the contract for the cash value, receiving dividends or a policy loan. In addition,the contract owner will be penalized in that a ten percent (10%) premature distribution and penalty tax is assessedon any distribution from a MEC that is included in the recipient's taxable income when withdrawn prior to age 59½. However, there is no penalty if the distribution is: (1) received because of a disability; (2) received at or afterage 59 ½; or (3) paid-out like a life annuity or a series of periodic payments. According to tax law, once a policy isclassified as a Modified Endowment Contract (MEC), it can never revert back to an ordinary (i.e., whole) lifeinsurance contract with regard to tax treatment. Once again, if a policy is declared to be a Modified EndowmentContract and it is surrendered by the policyholder, any gains received will be taxable. For example, John has aninsurance policy with $50,000 of cash value when it is designated as a modified endowment contract. The contract’scash value later grows to $100,000. The policy also includes a $500,000 death benefit. If John withdraws or receivesa distribution of $25,000, the entire amount of the distribution (i.e., 100 %) will be subject to taxation. The followingare illustrations of tax treatment with regard to distributions from a MEC:

1. Taxation only occurs when any cash is distributed to the contract owner or money is withdrawn whetherby surrender, loan or dividends.

2. The gain (i.e., interest or appreciation) is taxable first when a distribution is made (i.e., LIFO).3. The first dollars received by the contract owner are considered excess amounts of cash value beyond

premiums and are taxable.4. If a policy is a MEC and cash is withdrawn, even if it will be used for a legitimate reason such as financial

hardship or to pay medical expenses, applicable amounts are still taxable and if effected prior to age 59 ½,a 10% penalty tax will be assessed.

NILA ©

L/H Licensing

Chapter 8—3

Page 113: FORWARD - WordPress.com · Return of premium p 6-8 6.0 Annuities p 7-1 Annuity principles and concepts p 7-1 Accumulation period versus annuity period p 7-1 Owner, annuitant and beneficiary

TAXATION OF ANNUITIES

As mentioned in Chapter 7, a qualified or non-qualified annuity receives tax deferral during the accumulation phaseor period. The accumulation phase may terminate if the contract owner is disabled or dies and then the annuity phasecommences (i.e., the contract enters the annuity (i.e., payout) phase or the annuity contract is surrendered). Asmentioned earlier, contributions to a qualified individual (or employer-sponsored) annuity are generally taxdeductible. However, contributions to or premiums paid for a non-qualified individual annuity are not taxdeductible. The cost basis of an annuity is generally the amounts contributed (i.e., premiums paid) by the contractowner. Penalties are assessed on premature distributions except if: (1) the owner becomes disabled; (2) the ownerhas reached age 59½ (3) the owner has died; (4) an immediate annuity was purchased; or (5) funds are received undera qualified pension plan. If a lump sum cash surrender is effected when the contract owner dies during theaccumulation phase, any amount received in excess of the payments made to the contract is taxable. Also, assumethe owner invests a specific amount into the contract (i.e., $15,000) and several years later the contract value is higherdue to interest credited (i.e., $22,000). If he or she withdraws $10,000 from the account, the interest or $7,000 willbe taxable. Prematurely withdrawn amounts are generally taxed on a Last In First Out (LIFO) basis which meansthat accumulations or interest earned is considered withdrawn first when distributions are made. Again, they are alsosubject to the 10 % federal age-based tax penalty.

The beneficiary is also able to minimize tax liability, for instance, if he or she elects a life income option or aninstallment option based upon the exclusion ratio. However, the choice must be made within 60 days of theannuitant / contract owner's death. When the annuity phase commences, each payment (i.e., benefit) is comprisedof a non-taxable return of principal and a taxable return of interest. To determine what is taxable, an exclusionratio is utilized. This is the ratio of the total investment in the contract to the expected return and again it determinesthe portion of the payment that will be non-taxable and the portion that will be taxable.

For instance, Ed buys a $100,000 immediate annuity which is designed to pay him $1,000 per month for life. He is57 years of age. If he lives at least to life expectancy (i.e., 74 years of age), Ed will receive $204,000. According tothe exclusion ratio, Ed will be able to determine what portion of his monthly annuity payment will be taxable andobviously what portion will be a nontaxable return of proceeds.

Yearly annuity income X Contributions = Nontaxable income per yearExpected income

$12,000 X $100,000 = $5,882$204,000

This means that $6,118 per year will be taxable income.

Therefore, annuity income payments are taxable to the extent that they represent interest earned rather than capitalreturned. The exclusion ratio is the mechanism used to determine the taxable portion of each income payment.

DISTRIBUTIONS AT DEATH — If the owner dies during the accumulation phase and prior to the annuity phase,the beneficiary will receive the greater of the accumulated value of the annuity or the amount of contribution (i.e.,premium payments). Any amounts received in excess of premiums paid is taxable as ordinary income to the recipient(i.e., beneficiary). Therefore, whatever gain is realized will be taxable (whether resulting from the contract owner’sdeath or contract surrender). The annuity must also provide that if the owner dies prior to the annuity phase, theentire interest in the contract must be distributed within five years following the owner’s death. If the beneficiarychooses to annuitizes the contract proceeds within one year of the owner’s death, the IRS considers the five-year

NILA ©

L/H Licensing

Chapter 8—4

Page 114: FORWARD - WordPress.com · Return of premium p 6-8 6.0 Annuities p 7-1 Annuity principles and concepts p 7-1 Accumulation period versus annuity period p 7-1 Owner, annuitant and beneficiary

distribution requirement satisfied. The amount paid to the beneficiary when the contract owner dies is the amountaccumulated (i.e., contributions plus interest). The death benefit paid by life insurance is the face amount no matterhow few premiums have been paid. Since an annuity is a piece of property, if the contract owner / annuitant diesduring the accumulation period, its proceeds are generally includible in the deceased's estate.

CORPORATE OWNED ANNUITIES AND LIFE INSURANCE — Contributions to a corporate annuityare taxed differently than individually owned annuities. If the corporate owned annuity qualifies as a tax retirementplan, it will be taxed differently. Prior to 1986, interest earned in an annuity was tax deferred regardless of whetheran individual or corporation owned the contract. The federal government then enacted legislation designed to preventcorporations and other entities from taking advantage of the tax deferral provisions of the tax code. Current tax lawstates that if a corporation owns an annuity it must name a “natural person” as an annuitant. If a nonnatural entityis named an annuitant (i.e., the corporation), interest earned is taxable as ordinary income in the year credited. If a natural person is named as annuitant, the interest credited to the annuity each year is may be tax deferred in somecases. Sometimes this natural person is referred to as a “measurable life.” There is an exception to this nonnaturalperson rule. If the annuity is held by a trust, corporation or other “nonnatural person,” as an agent for a naturalperson (i.e., a human being), interest earned continues to be tax deferred. Other exceptions to this rule include butare not limited to:

1. An annuity contract that is acquired by a person’s estate following the death of that person;2. An annuity contract that is held under a qualified retirement plan, a TSA or an IRA; or3. A contract which is an immediate annuity purchased with a single premium, with periodic payments

to commence within a year.

Corporate owned life insurance is generally treated as a deductible business expense. The proceeds are paid tax freeup to a certain level (i.e., $50,000). If more than this amount is provided to an employee, the excess premium usedto purchase it must be reported by the employee as taxable income. An annuity could also be owned by a nonlivingentity such as a trust and the tax considerations will be based upon whether it is qualified or non-qualified.

INDIVIDUAL RETIREMENT ANNUITIES — These plans are also referred to as Individual RetirementAccounts (i.e., IRAs). Like other types of property, the value of IRAs are generally included in one’s estate.Information regarding contributions, deductibility, premature distributions, penalty taxes and rollovers will bereviewed in chapter 9.

SECTION 1035 EXCHANGE — The Internal Revenue Code (IRC) permits an individual to trade or exchangea life insurance policy, endowment or annuity contract for another of like kind. Since a life insurance policy is aform of property, exchanges of policies are allowed. When a less competitive policy is being replaced or exchangedwith a more competitive policy, a Section 1035 exchange may be utilized which allows for the postponement of taxconsequences. In other words, the IRC allows a tax-free exchange if it is done from insurer to insurer where thepolicy owner never receives any cash. This regulation allows the policy owner to move cash value from one contractto another without current tax considerations. As long as the transfer is transacted within (i.e., intra-company) orbetween insurance companies and the policy owner receives no money, the exchange is permitted (without taxramifications). This means that, theoretically, the cost basis remains the same. Therefore, the following types ofexchanges are allowed under this section of the IRC; (1) a life insurance policy for another life insurance policy,endowment or annuity; (2) an endowment policy for another endowment or an annuity; or (3) an annuity contractfor another annuity contract. It is not permissible to exchange an annuity for a life insurance policy. The reasonfor this is that the IRS is not willing to allow a policy owner to exchange policies which will improve the income taxtreatment of the benefits. If this were permitted, the policy owner could transfer funds from the annuity, which incursincome taxation on at least a portion of the benefits (i.e., tax deferral means that the annuitant will be taxed oninterest sometime in the future), to a life insurance policy which pays a death benefit free of income taxation. If this

NILA ©

L/H Licensing

Chapter 8—5

Page 115: FORWARD - WordPress.com · Return of premium p 6-8 6.0 Annuities p 7-1 Annuity principles and concepts p 7-1 Accumulation period versus annuity period p 7-1 Owner, annuitant and beneficiary

were allowed, an annuity to life insurance exchange would move a tax-deferred characteristic to a tax-freecharacteristic. With regard to a whole life insurance policy, a 1035 exchange involves the vesting of cash value fromone permanent insurance plan to another. Some of the reasons why an individual considers such a transfer include:(1) the financial condition or rating of the current insurer; (2) the enhancement of benefits in newer policies; (3)higher interest rates being offered in the newer policies; or (4) the desire for additional investment alternatives notavailable under the older contract. When a policy owner decides to make a permitted exchange, the transfer,assignment or surrender of the policy to the insurer and the subsequent replacement with the new contract must becompleted within sixty days. To ensure that there are no tax considerations because of the exchange, the policyowner should receive no cash proceeds.

QUIZ

1. According to the Internal Revenue Code, premiums paid on life insurance policies are:

A. Tax deductible C. Income taxable

B. Not tax deductible D. Not income taxable

2. Which of the following is income taxable as ordinary income?

A. Death benefit C. Interest paid on life insurance proceeds

B. Return of principal D. Estate taxes

3. Death taxes assessed by the government upon the assets of a decedent are known as:

A. Income taxes C. Probate fees

B. Federal estate taxes D. Estate allocation taxes

4. Federal and state death taxes must be paid within what period of time following the decedent's death?

A. Six months C. Nine months

B. Eight months D. Twelve months

5. If a life insurance policy becomes a modified endowment contact, it may revert back to an ordinary life policy within:

A. One year C. Seven years

B. Three years D. It is not allowed to revert back to an ordinary life

contract

6. Which of the following was designed to reduce or prevent the purchase of whole life insurance for the purpose of short-

term investment gain?

A. The Tax Reform Act of 1987 C. The one-sixth rule

B. Exclusion ratio D. Seven pay test

7. An annuity grows tax deferred during:

A. The annuity phase C. The distribution period

B. The payout period D. The accumulation period

NILA ©

L/H Licensing

Chapter 8—6

Page 116: FORWARD - WordPress.com · Return of premium p 6-8 6.0 Annuities p 7-1 Annuity principles and concepts p 7-1 Accumulation period versus annuity period p 7-1 Owner, annuitant and beneficiary

8. When the annuity phase commences, each payment made is comprised of a non-taxable return of principal and a taxable

return of:

A. Periodic benefits C. Interest

B. Premiums D. Expenses

9. If an annuity contract owner dies during the pay-in phase and prior to the pay out phase, the beneficiary will receive the

greater of the accumulated value of the annuity or the amount of contributions. Any amount received in excess of

premiums paid is:

A. Non-taxable C. Taxable in the year following receipt

B. Taxable D. Taxable as ordinary income

10. Tad surrenders his whole life policy and receives $10,000. His total premium paid during the past fifteen years was

$8,000. How much of the surrender value will be taxable?

A. None C. $8,000

B. $2,000 D. $10,000

11. Section 1035 of the Internal Revenue Code allows for the exchange of certain life insurance or annuity products. All

of the following would be permitted exchanges under this IRS regulation, EXCEPT?

A. Exchanging a variable life policy for another variable life policy

B. Exchanging a life insurance policy for an annuity

C. Exchanging an annuity for a life insurance policy

D. Exchanging an endowment for an annuity

12. Which of the following statements is not true with regard to a whole life insurance policy loan?

A. A policy loan that is not repaid will be deducted from the face amount upon the death of the insured

B. Funds borrowed against the cash savings value are taxable income

C. An unpaid loan may be subtracted from the cash surrender value

D. An insurer may assess an interest charge on a policy loan

13. A Modified Endowment Contract is not a type of life insurance policy. It is an IRS classification of a life insurance

policy meaning that the contract has not satisfied the seven-pay test. All of the following statements are true regarding

a MEC, EXCEPT?

A. Any gain received is taxed first with the investment received after the gain

B. The policy owner may receive distributions at any time without being penalized

C. Funds received by the policy owner are considered to be excess amounts of cash value over premiums paid and

are taxable

D. Funds received by the policy owner are taxable in the year received

14. When the beneficiary of an annuity receives monthly payments consisting of both principal and interest, which of the

following will be taxable?

A. Principal only C. Principal and interest

B. Interest only D. Neither principal nor interest

NILA ©

L/H Licensing

Chapter 8—7

Page 117: FORWARD - WordPress.com · Return of premium p 6-8 6.0 Annuities p 7-1 Annuity principles and concepts p 7-1 Accumulation period versus annuity period p 7-1 Owner, annuitant and beneficiary

15. The continual increase of cash value in a whole life policy:

A. May be subject to taxation when withdrawn if the policy fails the seven-pay test

B. Will never be subject to taxation

C. Will always be subject to taxation

D. Will be subject to taxation as soon as the policy is classified as a Modified Endowment Contract

16. Albert transfers his life insurance policy to Walter two years before Albert’s death. Which of the following statements

is true with regard to this scenario?

A. The interest paid on the cash value is taxable to Walter upon transfer

B. The entire face amount will be included in Albert’s estate

C. The transfer of the policy is considered a 1035-A exchange

D. Since the policy has been transferred, the value will not be included in the estate if either

17. Mr. Gleason is covered by a $150,000 life insurance policy. Mrs. Gleason is the primary beneficiary. Following the

death of Mr. Gleason, Mrs. Gleason leaves the death proceeds with the insurer but wishes to have receive interest

payments. During the current year she receives $500 per month in interest. Which of the following is going to be

taxable?

A. $500 C. $150,000

B. $6,000 D. $156,000

18. Which of the following is not true with regard to a 1035 Exchange?

A. It allows for the transfer or assignment of an existing policy to the insurer who replaces it with another of like

kind

B. Once begun, the exchange must be completed within thirty days

C. This involves the exchanging of a less competitive policy with a more competitive one

D. It is an IRS rule permitting the exchange of similar policies with no immediate tax consequences

19. A corporation may purchase an annuity. However, it must identify a natural person as the annuitant. If the firm fails to

do this, which of the following will occur?

A. Interest earned during the year will be taxable to the firm

B. Distributions may not be made until an annuitant is named

C. Funds in the account will be forfeited to the insurance company

D. An annuitant need not be named if the contract is owned by a corporation

20. Under what condition is the cash value in a Modified Endowment Contract not taxable?

A. When funds are distributed to the policy owner

B. When the policy is surrendered for its cash value

C. While it accumulates

D. While proceeds are capitalized

NILA ©

L/H Licensing

Chapter 8—8

Page 118: FORWARD - WordPress.com · Return of premium p 6-8 6.0 Annuities p 7-1 Annuity principles and concepts p 7-1 Accumulation period versus annuity period p 7-1 Owner, annuitant and beneficiary

ESTATE TAXES ON

ALL PROPERTY

ANSWERS

1. B2. C3. B4. C5. D

6. D7. D8. C9. D10. B

11. C12. B13. B14. B15. A

16. B17. B18. B19. A20. C

CHAPTER 8 KEY CONCEPTS

Annuities

Annuity distributions

Cash values

Corporate owned annuities

Death benefits

Estate taxes

Exclusion ratio

Life insurance

Modified Endowment Contracts

Policy loans

Seven pay test

Tax Equity and Fiscal Responsibility Act (TEFRA)

Tax treatment

Tax Reform Acts

1035 exchange

NILA ©

L/H Licensing

Chapter 8—9

Page 119: FORWARD - WordPress.com · Return of premium p 6-8 6.0 Annuities p 7-1 Annuity principles and concepts p 7-1 Accumulation period versus annuity period p 7-1 Owner, annuitant and beneficiary

QUALIFIEDRETIREMENT

PLANS and GROUPLIFE INSURANCE

Page 120: FORWARD - WordPress.com · Return of premium p 6-8 6.0 Annuities p 7-1 Annuity principles and concepts p 7-1 Accumulation period versus annuity period p 7-1 Owner, annuitant and beneficiary

INTRODUCTION

This chapter will focus on most of the qualified retirement plans available today. These plans are "qualified"in the sense that they are approved by the IRC to receive advantageous tax treatment. Such tax advantagesinduce individuals to set aside funds for retirement years. This chapter will also review basic group insurancecharacteristics and principles as well.

QUALIFIED AND OTHER RETIREMENT PLANS, GROUP LIFE INSURANCEAND SOCIAL SECURITY

QUALIFIED PLAN REQUIREMENTS

GENERAL REQUIREMENTS OF A QUALIFIED PLAN — As previously mentioned, qualified plansare those approved by the Internal Revenue Service to receive favorable tax treatment. This means that contributionsto such plans are generally tax deductible. On the other hand, plans that are classified as non-qualified are thosewhose contributions are not tax deductible. In order to receive "qualified" treatment, the pension or profit plan mustmeet several criteria including but not limited to: (1) be permanent and in writing and be communicated toemployees; (2) be provided for the exclusive benefit of employees and their beneficiaries; (3) satisfy minimum ageand service standards; (4) can be discriminatory if fair; (5) ensure that contributions do not exceed calendar yearlimitations; (6) must provide survivor benefits; (7) must meet minimum vesting standards; and (8) must satisfy top-heavy plan rules (i.e., 60% rule). Non-qualified plans, while legal, do not provide for tax deductions of contributionssince they are not approved under the Internal Revenue Code (IRC) for such benefits. An employer may discriminatewith regard to a non-qualified deferred compensation plan.

The primary purpose of the plan must be to provide retirement funds for the employee and beneficiaries. Manyqualified plans include death, disability or other supplemental benefits. Life insurance and disability income benefits covering an employee that are part of a qualified plan are subject to the incidental benefits rule which states that suchsupplemental benefits must be incidental to the plan.

FEDERAL TAX CONSIDERATIONS — Tax incentives are provided under the Internal Revenue Code (IRC)to persuade or promote savings for retirement years. Tax advantages are provided to employers and employees andfor distributions, income or rollovers.

1. EMPLOYER AND EMPLOYEE ADVANTAGES — Contributions made by employers to qualifiedplans are tax deductible since they are considered to be a legitimate business expense. Wheneverdistributed, the amounts received are taxable as ordinary income. Contributions made to a qualified planare not counted as income to an employee and funds accumulate in the plan on a tax deferred basis. Inother words, contributions and earnings of the plan are exempt from current income taxation.

2. TAXATION OF DISTRIBUTIONS — Any distributions from qualified plans prior to age 59½ are taxedon a pro-rata recovery of cost basis. A 10% penalty is also imposed on the premature distribution exceptin the cases of death, disability, divorce, financial hardship, loans from the plan or if the distribution ispart of a qualified rollover. These are sometimes referred to as pre-retirement distributions. Age related

NILA ©

L/H Licensing

Chapter 9—1

Page 121: FORWARD - WordPress.com · Return of premium p 6-8 6.0 Annuities p 7-1 Annuity principles and concepts p 7-1 Accumulation period versus annuity period p 7-1 Owner, annuitant and beneficiary

distributions made after age 59½ are referred to as qualified retirement distributions. Any distribution thatrepresents an employee’s cost basis is received tax free. Any amount received that is appreciation (or anemployer contribution) is taxed as ordinary income. Therefore, distributions from qualified plans areconsidered qualified if they occur at age 59 ½ or beyond (or if due to a qualifying event such as death,disability, first time home purchase, etc.).

3. ROLLOVER — This is the transfer of funds from one qualified plan to another. When the rollover isaccomplished from plan to plan (i.e., trustee to trustee), the owner does not receive any funds and funds arenot subject to the 20% withholding rule. If the employee/individual withdraws such funds in order to depositthem in another plan, a 20% withholding tax may apply. This tax is not applicable if one premium rolls overfunds into a second pension. In addition, to avoid any other penalties or adverse tax considerations, arollover must be effected within 60 days of withdrawal.

4. VESTING — All qualified plans must satisfy specific standards that set forth a vesting schedule. Vestingmeans the right each employee has to employer contributions in his or her fund. This means that benefits thatare vested belong to an employee even if he or she terminates employment prior to retirement. An employeealways, at all times, has a full or 100 % vested interest in his or her own contributions.

INDIVIDUAL RETIREMENT ACCOUNTS

TRADITIONAL INDIVIDUAL RETIREMENT ACCOUNTS — An individual retirement account or IRAis an investment vehicle available to all individuals who possess earned income. Earned income, as defined by theIRS, does not include income from rents or royalties which is unearned income. Also referred to as an IndividualRetirement Arrangement or “deemed IRA,” this plan is a tax qualified and IRS approved plan which provides theadvantages of tax deferral and a possible tax deduction for those who qualify. IRA's are offered by banks, insurancecompanies, and investment firms. They can be funded by cash or cash type vehicles (i.e., an annuity) and accumulatebased upon a fixed interest rate. When an IRA is funded, proceeds can be used to purchase any of severalinvestments from certificates of deposit to stocks and mutual funds. Individual Retirement Annuities are also offeredby insurers and usually pay a higher fixed interest rate than an Individual Retirement Account. However, they arealso characterized by heavy withdrawal penalties in the early years after they are purchased, even if funds arewithdrawn in order to be rolled over. "Tax Facts," provides the following excerpts with regard to IRAs. A deemedIRA is subject to IRA rules and not qualified plan rules. Although it has some common characteristics of a qualifiedplan, it is not classified as such. Some of the more important characteristics of IRAs include:

! Annual contributions to an IRA may be equal to 100% of earned income not to exceed $5,000 per year foryear 2011 ($6,000 for age 50 and above). A spousal IRA contribution may not exceed $5,000 per year. Thisis an IRA created for an eligible individual and a non-wage earning spouse. Even if a person earns part-timeincome he or she may contribute up to the maximum amount to an IRA.

! IRAs may be rolled over into another higher yielding (i.e., higher interest rate) account if desired withoutpenalty. In other words, a rollover is a tax-free event. However, the roll over must be accomplished within60 days after withdrawal. Again, an Individual Retirement Annuity generally may not be rolled over duringthe first several years of existence without penalty according to annuity rules. If a beneficiary is entitled toproceeds from an IRA because the owner dies, the beneficiary is permitted to rollover this amount into hisor her own IRA (or maintain the deceased spouse’s account as his or her own).

NILA ©

L/H Licensing

Chapter 9—2

Page 122: FORWARD - WordPress.com · Return of premium p 6-8 6.0 Annuities p 7-1 Annuity principles and concepts p 7-1 Accumulation period versus annuity period p 7-1 Owner, annuitant and beneficiary

! Withdrawals (with no intent to roll over) may not be made without penalty prior to age 59 ½. If a withdrawalis made prior to this age, a 10% penalty will be charged based on the amount of the withdrawal. Forexample, if the contract owner withdraws $2,500 prior to age 59 ½, 10% or $250 will be deducted from thewithdrawal request. Therefore, the contract owner receives $2,250. In addition, the $2,500 withdrawal mustbe included as taxable income on the recipient's tax return in the year received. Withdrawals from theaccount must also begin prior to age 70 ½ as well. When the owner begins to receive benefit payments ormonthly retirement income, an annuity phase commences. If the annuitant dies, the value may be includedin his or her estate. However, if there is a beneficiary to whom proceeds are paid, there will be a taxableevent.

! Excess contributions to an IRA will be subject to an additional 6% penalty.

! A tax deduction is available to individuals who contribute to an IRA and are not covered by an IRS qualifiedpension plan or an employer sponsored retirement program, no matter what their annual taxable income.In year 2011, if the wage earner is single and earns less than $56,000 per year, a full deduction of thecontribution amount is allowed even if the individual participates in an employer sponsored pension plan. If this same wage earner has an adjusted gross income (AGI) of between $56,000 and $66,000, he willreceive a partial deduction based on a formula. If this single wage earner has an AGI of $66,000 or more,no deduction is allowed.

In the year 2011, joint filers whose AGI is less than $90,000 may deduct any IRA contribution in a particulartax year. If their AGI is between $90,000 and $110,000 and either participates in a company sponsoredpension plan, a partial deduction will be allowed. If joint income is $110,000 or more, no deduction isallowed. If the couple is not actively participating in an employer sponsored retirement plan, anycontribution will be deductible no matter what the AGI.

! A new “catch-up” provision allows investors age 50 and over to contribute an additional $1,000 per yearbeginning in year 2006 and thereafter. IRAs may be funded by cash payments or any type of annuity. Theymay not be funded by a life insurance policy itself, although a cash portion of a death benefit or cashsurrender value may be utilized to fund an IRA.

Education IRAs (also referred to as a Coverdell Education Savings Account) are also available where an investorcan make non-deductible contributions of up to $2,000 per child under age eighteen. The funds saved can be usedfor primary and secondary school expenses (i.e., tuition, books) in addition to higher education fees (i.e., college). Any funds left over (i.e., if a child does not attend college), may be rolled over into another Education IRA prior toage 30.

ROTH IRA — This newer IRA allows one to make non-deductible contributions which are tax free whenwithdrawn. The owner forfeits deductibility when he or she contributes, but does not have to pay any taxes on theearnings. A Roth IRA is beneficial for individuals with large savings that may leave them in the same or even ahigher tax bracket after retirement. It is also advantageous for those who will continue to work after age 70 ½ andwould prefer to delay IRA distribution. Additionally, it is also good for those who are looking for estate planningbenefits for their beneficiaries since distributions to beneficiaries will pass income tax free.

Eligibility — A person must have an adjusted gross income of less than $107,000 if single or $169,000 if filingjointly to contribute up to the full allowable limit.

Age Requirements — An individual (with earned income) may continue to contribute to the Roth IRA after reachingage 70 ½. There is no mandatory distribution age.

NILA ©

L/H Licensing

Chapter 9—3

Page 123: FORWARD - WordPress.com · Return of premium p 6-8 6.0 Annuities p 7-1 Annuity principles and concepts p 7-1 Accumulation period versus annuity period p 7-1 Owner, annuitant and beneficiary

Restrictions — In year 2011, a person may contribute the lesser of $5,000 or 100% of earned income per year (age50 and above is $6,000). This is a combined limitation of all IRAs owned. Catch-up allowances are also availablein the same amounts as a traditional IRA.

Distributions — Penalty-free withdrawals may be made from the Roth IRA prior to age 59 ½ for qualifiededucation expenses for the taxpayer, spouse, or any child or grandchild of either. Qualified higher educationexpenses include tuition, fees, supplies, books, room and board, and graduate courses. However, even though thesedistributions are penalty-free, they may still be subject to income taxation. Federal income tax AND penalty-freedistributions may be made from the Roth IRA in the following situations: The Roth IRA must have existed for fiveyears after the first taxable year for which a contribution was made, and must either be made: (1) after the attainmentof age 59 ½; or (2) due to the owner’s death or disability; or (3) for qualifying first-time home buyer expenses. Withdrawals that are made from a Roth IRA that do not meet the IRS requirements for qualified distributions areincluded in income. Nondeductible contributions are always distributed first and are always tax and penalty-free.

QUALIFIED PLANS

TYPES OF PLANS AND THEIR CHARACTERISTICS — There are numerous plans available whichassist an individual in saving funds for retirement and provide tax advantages including but not limited to:

SIMPLIFIED EMPLOYEE PENSION (SEP) — According to "Tax Facts," these are defined contributionretirement vehicles for small businesses, self-employed individuals and their eligible employees. They offeremployers a method in which to provide funds for their own and an employee's retirement years, as long as certaineligibility requirements have been satisfied. SEPs can also be described as employer-sponsored IRAs.

! The plan requires an employer to make contributions, based on a formula contained in the plan, for allemployees who have reached the age of 21. In addition, the employee must have been employed for at leastthree out of the past five years as well.

! Contributions to the SEP by the employer are deductible and are deposited directly into the plan for eachemployee. The amount of employer contributions permitted each year is the lesser of 25% of earned incomenot to exceed $49,000. Contributions for employees are allowed up to $16,500 in year 2010 and 2011. Thisfigure is now indexed to reflect cost of living increases. Earnings accumulate on a tax deferred basis.

! SEPs were primarily created as an alternative to Keogh Plans which have strict and complicated reportingrequirements. SEPs have simplified the administration and reporting requirements. Therefore, SEPs offerthe ease of administration with minimal reporting requirements. An individual simply records the amountof the SEP contribution on the appropriate area of his or her tax form just as is done with an IRA.

KEOGH PLANS — This is a non-corporate retirement savings vehicle also referred to as an HR-10 plan, namedfor the House of Representatives bill passed by Congress. A Keogh Plan is designed for the self-employed personor the sole proprietor and employees. Someone who owns his or her own business and is not incorporated is eligiblefor a Keogh Plan. Like an IRA or SEP, a Keogh Plan provides a combination of tax deferral and a tax deductionwhich reduces the participant's annual tax liability.

! Contribution limits are 25% of earned income not to exceed $49,000 per year (in the year 2011).

NILA ©

L/H Licensing

Chapter 9—4

Page 124: FORWARD - WordPress.com · Return of premium p 6-8 6.0 Annuities p 7-1 Annuity principles and concepts p 7-1 Accumulation period versus annuity period p 7-1 Owner, annuitant and beneficiary

! Withdrawals are not permitted without penalty prior to age 59 ½. They must begin prior to age 70 ½. Penalties will be assessed for a violation of these requirements (i.e., 10% penalty and inclusion in taxableincome).

! Keogh Plans are characterized by strict reporting requirements. A great deal of documentation must beprovided to the IRS to verify contribution and deductible amounts.

401k PLANS — Section 401 of the IRC provides favorable tax treatment to those who participate in a qualifiedplan for retirement purposes. The 401k allows contributions by employees to be invested in various vehicles. Contributions made to this qualified plan are excluded from the individual's gross income up to a maximum limit. This limit is adjusted annually for inflation. Some 401k plans provide for a matching of the employee contributionby an employer. This plan, therefore, utilizes a salary reduction which reduces the employee's tax liability as well. Interest earned on contributions are tax deferred. Like most qualified plans, withdrawals are not permitted prior toage 59½ unless the employee becomes disabled, dies, retires, changes jobs or demonstrates financial hardship. A401k plan provides current as well as future tax savings for employees. A 401k plan may be arranged by using a thriftplan, a salary reduction plan or a bonus plan. Employers match a percentage of the dollar amount that an employeecontributes in the aforementioned plans. Like all qualified plans, an employee may borrow from such plans withoutreceiving a distribution penalty. A 401k plan provides current as well as future tax savings for employees. Themaximum annual pretax contribution limit has increased to $16,500 in year 2010 and 2011.

PROFIT SHARING PLAN — These qualified plans are generally of a defined contribution type in which theemployer can share profits with employees. These plans provide for no definite benefits to participants. If no profitsare realized, no contributions will be made.

PENSION PLANS — These plans are employer sponsored and maintained. Generally utilized by firms with alarger number of employees, they provide retirement income benefits to participants. Employees must satisfy certainminimum service requirements in order to be eligible for benefits. A retirement age will be identified in the pensionplan which designates when benefits will begin. There will also be a provision for early retirement if desired whichwould result in a benefit decrease. However, if the employee desires to put off or defer retirement to a later age, heor she would receive a benefit increase.

Benefit Formula — Defined benefit and contribution terminology is used to describe the benefits or contributionlimits that will be provided in the future by qualified (IRS approved) retirement or pension plans. A defined benefitplan identifies the amount of the installment benefit to be paid to the retired employee upon retirement. A definedcontribution plan is characterized by an individual account for each employee. The plan specifies the contributionamount the employer must make each year but does not promise any specific future installment (i.e., monthly) benefit.

TAX-EXEMPT EMPLOYER PLANS — These are retirement plans receiving special tax treatment since theyare available to certain tax exempt organizations.

O SECTION 457 DEFERRED COMPENSATION — Section 457 of the IRC allows any State or localgovernment entity to provide a deferred compensation program for its employees. According to LifeInsurance by Black and Skipper, in this arrangement, the employer agrees with each employee to reduce hisor her pay by a specified amount and to invest the deferrals in one or more investment outlets that mayinclude insurance products. Deferred amounts cannot exceed certain limitations.

The maximum amount in year 2011 is 25% of one’s post-deferral taxable compensation up to $16,500 (upto $22,000 for age 50 and over). As stipulated in "Life Insurance," by Black and Skipper, the primary

NILA ©

L/H Licensing

Chapter 9—5

Page 125: FORWARD - WordPress.com · Return of premium p 6-8 6.0 Annuities p 7-1 Annuity principles and concepts p 7-1 Accumulation period versus annuity period p 7-1 Owner, annuitant and beneficiary

difference between Section 457 plans and 403(b) plans is that investments in a Section 457 plan are ownedby the employer. In many cases, the vehicles used in these deferred compensation plans are fixed or variabledeferred annuities.

O 403 (b) TAX-SHELTERED ANNUITIES (TSA) — This type of retirement plan is available toemployees of qualified or tax exempt organizations. The Internal Revenue Code, 403(b), stipulates thatfunds may be contributed to a qualified annuity with pre-tax contributions which are also referred to asbefore tax dollars. According to IRC guidelines, ("Tax Facts, “) this type of plan is available to "public,private, and parochial school teachers; social workers; college professors; school superintendents; clergymenand clergy women; and other employees of qualified organizations (i.e., school systems, nonprofitorganizations, welfare agencies, research foundations or church or religious organizations, etc.). Employeesof the government are not eligible." A TSA is characterized by a salary reduction rather than a taxdeduction. Therefore, the participant receives two benefits: (1) a reduction of tax liability; and (2)contributions to the TSA accumulate on a tax deferred basis. The maximum allowable annual contributionis $16,500 in year 2011 (for those age 50 and above, $22,000). TSAs are characterized by a salary reductionwhich provides tax deferral and a reduction of annual tax liability. This means that the participant does notpay income tax on interest earned by contributions while funds are accumulating. Later, when interestincome is received it is taxable as income. In addition, because the salary has been reduced, the participant'sannual income tax payable will also decrease. The same rules apply to a TSA with regard to a prematuredistribution (i.e., 10 percent penalty if the withdrawal is made prior to age 59 ½).

SIMPLE PLANS— This type of plan is a savings incentive match plan for employees who work for smallemployers. It provides for a retirement option that is not required by the IRC to satisfy many of the qualified planrules. Assets of the plan are not taxed until they are distributed. Contributions to the plan are tax deductible to theemployer making them. An employer will be eligible to contribute if he or she has 100 or fewer employees whoreceived at least $5,000 each in compensation for the preceding year. In addition, the employer will not be eligiblefor participation in a Simple Plan if he or she already maintains another employer-sponsored plan to whichcontributions were made (or where benefits have already accrued). This type of plan may be structured as a 401kplan or as an IRA. The latter would be known as a Simple IRA. In 2010 and 2011, the limit is $11,500 ($14,000 forage 50 and over).

The employer is required to: (1) match any elective contributions by an employee on a dollar for dollar basis up toa limit of 3% of the employee’s compensation; or (2) make a non-elective contribution of 2% of each employee’scompensation. Non-elective means that the employee has no choice in the matter. An advantage of this type of planis that all employees are vested as soon as contributions are made. Employees are taxed on distributions from SimplePlan accounts. Early distribution penalty taxes apply for withdrawals prior to age 59 ½. If the distributions are madewithin the first two years of participation in the Plan, the employee is subject to a 25% penalty tax. If thedistributions are made after the first two years of participation in the Plan, the employee is subject to a 10% penaltytax. There are exceptions to the early distribution penalties including but not limited to the permanent and totaldisability of a participant or his or her death. An employer is also provided with a two-year grace period to maintainthe Plan once he or she becomes ineligible to participate because the number of employees exceeds 100.

SECTION 529 SAVINGS PLANS — The Economic Growth and Tax Relief Act of 2001 created the qualified529 savings plan. This is a State operated investment plan that gives families a federal tax-free method to save moneyfor college and other qualified, post-secondary higher education expenses (.i.e, vocational school, graduate schoolor trade school). There are two types of 529 plans. A college savings plan allows parents to use their plan fundsfor college expenses at any college. A prepaid tuition plan allows parents to “lock-in” future tuition at instate publiccolleges at current prices. The following are some 529 plan facts:

NILA ©

L/H Licensing

Chapter 9—6

Page 126: FORWARD - WordPress.com · Return of premium p 6-8 6.0 Annuities p 7-1 Annuity principles and concepts p 7-1 Accumulation period versus annuity period p 7-1 Owner, annuitant and beneficiary

Just like a Roth IRA, earnings from a 529 plan are exempt from federal taxes as are any withdrawals as long as theygo toward paying college costs. Some States waive State taxes for residents while others allow deductions oncontributions. Qualified expenses include but are not limited to tuition, room and board, books and supplies.

529 plans have generous maximum contribution limits (some up to $250,000 per beneficiary). If funds are withdrawnfor purposes other than education (i.e., unqualified distributions), the earnings are subject to a 10% penalty as wellas federal income tax. States may assess their own penalties. Grandparents can use this plan to make gifts tograndchildren. The amounts contributed by grandparents are excluded from their estate.

The owner of a plan may switch investment options within the same plan once per year. A rollover is permitted fromone 529 plan into another once per year as well. Savings are treated as a parental asset when aid is determined butonly 5.6 % or less of the account’s value is factored into calculating the expected family contribution (EFC) for eachacademic year. The account holder controls the money for the life of the account, even after the beneficiary reachesage 18. Beneficiaries may also be changed at any time as well.

NON - QUALIFIED PLANS

These types of plans are generally provided by employers for highly paid or key employees, or directors and officersof a firm. The contributions to such plans are not tax deductible since the employer is legally discriminating infavor of higher paid employees. In other words, the employer makes no effort to satisfy the qualificationrequirements under the Internal Revenue Code (i.e., IRC) or under ERISA for tax favored treatment of qualified plancosts or benefits. Providing this type of additional compensation to an employee allows the firm to attract and retainthe services of key employees.

Common types of nonqualified plans in existence include nonqualified deferred compensation plans, supplementalexecutive retirement plans and incentive compensation plans. For instance, in a nonqualified deferred compensationplan, compensation for services provided by an employee is postponed until retirement. No tax will be paid by theemployee on the deferred amounts, generally, until received. The employer is not able to deduct the deferredpayments until they are actually received by the employee, usually at retirement. Nonqualified plans may be fundedor unfunded. A funded plan is one where assets are maintained by the employer in some sort of trust or escrowaccount as security for the promise of future benefit payments. An unfunded plan exists when no actual funds orassets have been designated to fund the plan. Therefore, the employee is relying on the unsecured promise of theemployer.

GROUP LIFE INSURANCE

CHARACTERISTICS GROUP TERM LIFE INSURANCE — Group life insurance differs fromindividual life insurance contracts in several ways. One of the differences between the two is that group insuranceis most often comprised of annual renewable term life insurance whereas individual insurance contracts may beterm life or whole life insurance. Underwriting is handled differently and varying types of policy provisions appearin a group life policy. Group life insurance coverage is characterized by the underwriting of numerous individualsrather than one. Group term life insurance, like all insurance contracts previously mentioned, is a two party contractbetween the policyholder and the insurer (just like an individual contract). The employer providing the group lifecoverage pays all or a portion of the premium and is the policyholder. The employer or plan sponsor receives themaster policy while the covered employee receives a booklet or certificate of coverage which describes the benefits,the coverage provided and how long coverage will last. The covered employee is also known as the certificate

NILA ©

L/H Licensing

Chapter 9—7

Page 127: FORWARD - WordPress.com · Return of premium p 6-8 6.0 Annuities p 7-1 Annuity principles and concepts p 7-1 Accumulation period versus annuity period p 7-1 Owner, annuitant and beneficiary

holder. Types of groups that eligible include employees of a single employer, credit groups, labor unions and multipleemployer groups.

GROUP LIFE UNDERWRITING REQUIREMENTS — Sound group underwriting can prove profitable toan insurer especially since it reduces adverse selection. Adverse selection or anti-selection is the tendency or dangerof an insurer to write (i.e., approve) more bad risks than good risks. It is the tendency of poorer risks to seekinsurance coverage. Since more individuals are covered under group policies there is a greater probability that a"bad" risk will be included. The insurer may continue to earn a profit, however, if the good risks far outweigh thebad. This is what an insurer is depending on by writing group life coverage. Writing large groups of individualsalso helps to reduce adverse selection based on the law of large numbers. Other group term life characteristicsinclude:

! Proof of insurability may not be required of larger groups. Insurers may require some type of insurabilityfor smaller groups since the risk is greater. An employee with a physical impairment (i.e., paralysis) willnot be excluded from coverage under a group life plan.

! A number of people may not form a group just to secure group insurance coverage. The securing of suchcoverage must be incidental to the group's formation. In other words, a group of people cannot form anorganization whose main purpose is to secure life insurance coverage. Businesses are operated in order toproduce a product or provide a service and earn a profit, and therefore, is eligible to purchase groupinsurance. A group of persons who are engaged in occupations of a common industry may form anassociation (i.e., all hat manufacturers) and later purchase group coverage. There generally must be anemployment relationship present in order for group coverage to be secured.

! Underwriters take persistency into account as well. The insurer may shy away from groups that changeinsurers regularly. Therefore, the insurer feels that writing such groups do not represent a good risk.

! All group policies contain a conversion privilege. A covered employee has the option of converting his or

her group term life coverage to his or her own individual plan when employment is terminated. Terminationof employment includes an employee who is laid-off or who leaves a job voluntarily. In most cases, whenan employee is leaving an employer, he or she may take advantage of the conversion privilege. However,most insurers only allow the terminated employee to convert the group coverage to an individual whole lifepolicy.

The period of time during which the terminated employee may convert to an individual plan of insurancewithout proof of insurability is within 31 days after termination. If death occurs during the conversion period(within 31 days after termination), even if the employee does not plan to convert to an individual policy, thedeath claim will be paid by the group policy. No medical exam or other proof of insurability is requiredto convert coverage to an individual policy. In other words, an insured employee may exercise theconversion privilege regardless of his or her insurability. If a conversion occurs, the premium is based onthe employee's current or attained age.

! While the employer is the policyholder in a group life policy, he or she retains all rights of ownership exceptthe right to name or change the beneficiary. This means that the covered employee or "certificate holder"possesses an "incident of ownership" in the group plan. In addition, in recent years, many insurers have beenpermitted by modifications of State laws to include an assignment provision in group life policies. Of course,any assignment must be in writing and be filed with the insurer.

NILA ©

L/H Licensing

Chapter 9—8

Page 128: FORWARD - WordPress.com · Return of premium p 6-8 6.0 Annuities p 7-1 Annuity principles and concepts p 7-1 Accumulation period versus annuity period p 7-1 Owner, annuitant and beneficiary

PREMIUM PAYMENT OF GROUP PLANS — Group life insurance may be sponsored by or premiumsmay be paid by an employer or may be shared by the employer and employees. Group plans whose premiums arepaid by the employer are called non contributory group plans. They are also known as employer pay-all plans. Ifthe premium is shared by the employer and employees, the plan is referred to as a contributory group plan.

If a group plan is contributory, the insurer will not write coverage unless at least 75% of the eligible employeeschoose to be covered. For example, if there are 100 eligible employees, 75 of them must choose to be covered or theinsurer will not write the policy. Under some contributory plans there exists specific time frames when newemployees may secure coverage without taking a medical exam or answering any health questions. This type ofperiod is known as an enrollment period. The reason for a contribution requirement is to protect against or reduceadverse selection. If a group plan is non contributory, the insurer will not write the coverage unless 100% of alleligible employees participate.

ADVERSE SELECTION — This underwriting concept involves the tendency of poorer risks to seek insurancecoverage or the chance that an insurer will accept applicants who are bad risks (i.e., those in poor health, moralhazards, etc.) . Sound and competent individual or group underwriting will reduce the probability or chance ofadverse selection.

TAX TREATMENT OF GROUP LIFE — Premiums paid by employers for group life insurance are taxdeductible as a legitimate business expense as long as certain requirements are satisfied. However, a sole proprietoror partner may not deduct premiums for group life covering his own life since he is not considered to be an employee. The cost of the first $50,000 of group term life is tax exempt to an employee. The cost of coverage amounts in excessof $50,000 may be taxable (as ordinary income) to the employee. Death proceeds under group life are receivedincome tax free (whether term or whole life insurance is used).

GROUP VS INDIVIDUAL INSURANCE

GROUP INDIVIDUAL

One master policy is issued to the group Each person covered possesses his or her own policy

Covered members have the same benefits andcoverages

Each individual selects his or her benefits andcoverages

Only eligible group members can apply Any individual can apply

Group underwriting Individual underwriting

Coverage terminates when the member leaves thegroup

Coverage continues as long as premium is paid

Less expensive with few restrictions More expensive with more restrictions

The essentials of group versus individual forms of insurance apply to life and health insurance

NILA ©

L/H Licensing

Chapter 9—9

Page 129: FORWARD - WordPress.com · Return of premium p 6-8 6.0 Annuities p 7-1 Annuity principles and concepts p 7-1 Accumulation period versus annuity period p 7-1 Owner, annuitant and beneficiary

SOCIAL SECURITY

Social insurance is provided by Social Security, also referred to as OASDHI. This is the Old Age SurvivorsDisability Health Insurance program. Social Security is "funded" by payroll taxes collected from employees,employers and those who are self employed. Social Security provides several benefits to those who are eligibleincluding but not limited to retirement income, disability income, a lump sum death benefit and survivor benefits.To receive maximum benefits the eligible individual must be "fully insured.” This means that the eligible personmust have forty quarters of coverage. In other words, a person must have worked for ten years to be fully insured.

! Primary Insurance Amount (PIA) — Benefits payable by Social Security are based upon what anindividual worker has contributed to the program. In other words, this is the average indexed monthlyearnings (i.e., average income over one's working lifetime). This is known as the worker's primary insuranceamount. Therefore, retirement income payable by Social Security is based upon a worker's primaryinsurance amount or PIA. Retirement income begins on the first day of the month in which the individualbecomes age 65 (and the individual must apply for benefits). An individual who begins to collect retirementincome benefits at age 65 is still allowed to work and there is no longer any reduction or offset from SocialSecurity income. The program also provides for a lump sum death benefit to be paid to the eligible spouseof a deceased worker. The lump sum death benefit is $255.

IRA AND SELECTED QUALIFIED RETIREMENT PLAN VEHICLES

PLAN CONTRIBUTION LIMITS ELIGIBILITY TAX TREATMENT

Traditional

Individual Retirement

Account (IRA)

100% of earned income not to ex-ceed $5,000 per year. $5,000 limitfor non-wage earning spouse.

Anyone with earned income.Even those working part-time.

Tax deferred. May be taxdeductible.

Simplified Employee

Pension (SEP)

25% of self-employment income peryear not to exceed $49,000(employee up to $16,500 in 2011).

Self-employed. Employermay provide for employees.

Tax deferral. Tax deductiblecontributions.

Keogh Plan (HR-10) 25% of self-employment income peryear not to exceed $49,000 in 2011.

Non-corporate self-em-ployed (sole proprietor).

Tax deferred. Tax deductiblecontributions.

403(b) Tax Sheltered

Annuity (TSA)

$16,500 in year 2011. Employees of charitable,non-profit, religious oreducational organizations.

Tax deferral. Reduced taxliability through salary reduc-tion.

Section 457 Deferred

Compensation

In 2011, the lesser of 25% of taxablecompensation or $16,500.

Employees of governmententities (i.e., state, city,county, etc.).

Tax deferral. Reduced tax lia-bility through salary reduc-tion.

401(k) Plan Adjusted annually for inflation (2011limit is $16,500). Employer match-ing generally.

Employees of businesseswhich desire to provide thistype of plan.

Tax deferral. Contributionsexcluded from income.

NILA ©

L/H Licensing

Chapter 9—10

Page 130: FORWARD - WordPress.com · Return of premium p 6-8 6.0 Annuities p 7-1 Annuity principles and concepts p 7-1 Accumulation period versus annuity period p 7-1 Owner, annuitant and beneficiary

QUIZ

1. In a contributory group life insurance plan, what portion of eligible employees must participate before a policy is issued?

A. 25% C. 75%

B. 50% D. 90%

2. Each of the following statements is true concerning group term life insurance, except:

A. Group life coverage must be incidental of the formation of the group.

B. Group life underwriting encourages adverse selection.

C. A certificate holder possesses the right to name a beneficiary.

D. A conversion privilege is provided in most group life plans.

3. Which of the following may be used to fund an individual retirement account?

A. Whole life policy C. Universal life policy

B. Life paid-up at 65 policy D. Flexible premium annuity

4. A tax-sheltered annuity is characterized by a salary reduction and tax deferral. Which of the following would be eligible

to participate in this type of retirement vehicle?

A. A fifty-eight year old male who is totally disabled

B. An administrative assistant to a company officer

C. A minister

D. An employee of the federal government

5. A Keogh Plan is designed for which of the following parties?

A. High salaried directors of a corporation

B. A sole proprietor and employees

C. Partners with a business interest in excess of $500,000

D. An employee of a nonprofit organization

6. Pre-retirement pension plan distributions may occur prior to age 59½. What penalty amount may be assessed for an early

distribution?

A. 5% C. 15%

B. 10% D. 25%

7. To avoid any penalties or adverse tax consequences, rollovers of qualified plans must be effected within how many

months of withdrawal?

A. One month C. Three months

B. Two months D. Sixty months

8. A tax-sheltered annuity provides reduced tax liability by means of a salary reduction. All of the following are eligible

to participate in a 403(b) tax-sheltered annuity, EXCEPT?

A. Private school teacher C. Employee of a town

B. Minister D. Employee of a non-profit agency

NILA ©

L/H Licensing

Chapter 9—11

Page 131: FORWARD - WordPress.com · Return of premium p 6-8 6.0 Annuities p 7-1 Annuity principles and concepts p 7-1 Accumulation period versus annuity period p 7-1 Owner, annuitant and beneficiary

9. Which of the following qualified plans is more inclined to utilize a fixed or variable deferred annuity as its primary

investment vehicle?

A. A defined contribution plan C. Section 457 plan

B. Group term life insurance plan D. Buy-sell agreement

10. Under a group life insurance policy, an insured is provided as a tax-free benefit up to what coverage amount?

A. $20,000 C. $50,000

B. $25,000 D. $100,000

11. Which of the following types of pension plans do not permit a tax deduction of contributions to the plan?

A. A defined benefit plan C. A defined contribution plan

B. A qualified plan D. Nonqualified plan

12. When an employee terminates employment she may choose to take her group life insurance coverage with her according

to the conversion privilege. Generally, the insurer allowing the conversion will require the employee to convert to which

of the following types of life insurance?

A. Individual term life insurance C. Group permanent life insurance

B. Individual whole life insurance D. Group term life insurance

13. Social Security provides several types of benefits to those who are eligible. One of these benefits is retirement income.

If an eligible recipient is fully insured, which of the following statements is true?

A. A lump sum benefit of $255 will be paid to an eligible dependent child when the eligible worker dies

B. The primary insurance amount will be paid to an eligible recipient at retirement

C. A recipient may receive monthly income benefits at age 65 and continue to work

D. Income benefits may begin during the blackout period if the eligible spouse is over age 60

14. Which of the following is true regarding group life insurance ?

A. Benefits paid to a beneficiary are taxable

B. An employee with a previous physical impairment is eligible for group life coverage

C. Group life insurance is not portable

D. Group life policies include a savings feature which permits policy loans

15. Each of the following are characteristics if a Roth IRA, EXCEPT?

A. Distributions may be deferred beyond age 70 ½

B. Contributions to this account are tax deductible

C. Contributions may continue after age 70 ½

D. Qualified distributions are not income taxable

16. A direct rollover made from one qualified plan to another without the owner receiving any cash is:

A. Subject to the 20% withholding tax

B. Not subject to the 20% withholding tax

C. Subject to a withholding tax if not effected within thirty days of the rollover

D. Subject to a penalty unless completed within one year

NILA ©

L/H Licensing

Chapter 9—12

Page 132: FORWARD - WordPress.com · Return of premium p 6-8 6.0 Annuities p 7-1 Annuity principles and concepts p 7-1 Accumulation period versus annuity period p 7-1 Owner, annuitant and beneficiary

17. Which of the following is best suited for providing retirement benefits for a firm with 800 employees ?

A. A 403 (b) plan C. A 457 deferred compensation plan

B. A traditional IRA D. A pension plan

18. No cash is assumed to be distributed when money is withdrawn from a traditional IRA if the funds are reinvested within:

A. Thirty days C. Ninety days

B. Sixty days D. One hundred and eighty days

19. Permitted withdrawals from a Roth IRA are:

A. Tax free C. Taxable as ordinary income

B. Tax deductible D. Partially taxable depending upon income

20. The conversion period in a group life insurance policy is:

A. Twenty days C. Thirty days

B. Twenty-one days D. Thirty-one days

21. Which of the following investment vehicles does not take into consideration one’s income?

A. Individual retirement account C. Variable annuity

B. 403(b) plan D. Roth IRA

22. How do the earnings of a 529 savings plan grow?

A. Tax deferred C. Tax deductible

B. Tax exempt D. Taxable

23. All of the following are qualified plans, EXCEPT:

A. 403(b) plan C. 401k plan

B. 501c Trust D. Section 457 deferred compensation

ANSWERS

1. C

2. B

3. D

4. C

5. B

6. B

7. B

8. C

9. C

10. C

11. D

12. B

13. C

14. B

15. B

16. B

17. D

18. B

19. A

20. D

21. C

22. B

23. B

NILA ©

L/H Licensing

Chapter 9—13

Page 133: FORWARD - WordPress.com · Return of premium p 6-8 6.0 Annuities p 7-1 Annuity principles and concepts p 7-1 Accumulation period versus annuity period p 7-1 Owner, annuitant and beneficiary

CHAPTER 9 KEY CONCEPTS

401(K) plans

457 deferred compensation

College savings plan / 529 plans

Conversion period

Group life

Incidental benefits rule

IRA

Keogh plans

Pension plans

Profit sharing plans

Retirement plans

Rollovers

Roth IRAs

Simple plans

SEPs

Tax considerations

TSAs

NILA ©

L/H Licensing

Chapter 9—14

Page 134: FORWARD - WordPress.com · Return of premium p 6-8 6.0 Annuities p 7-1 Annuity principles and concepts p 7-1 Accumulation period versus annuity period p 7-1 Owner, annuitant and beneficiary

BUSINESS USESOF

LIFE INSURANCE

Page 135: FORWARD - WordPress.com · Return of premium p 6-8 6.0 Annuities p 7-1 Annuity principles and concepts p 7-1 Accumulation period versus annuity period p 7-1 Owner, annuitant and beneficiary

INTRODUCTION

Life insurance can be used to provide funds in a business situation. It can be utilized to protect orindemnify a business owner and provide benefits to employers, employees, and dependents. Corporations or partnerships regularly purchase life insurance for business uses including keyemployee insurance, credit insurance, business continuation, and employee compensation plans.

BUSINESS USES OF LIFE INSURANCE

KEY PERSON LIFE

KEY EMPLOYEE LIFE INSURANCE — The principal reason that key employee insurance was developedis to compensate a business for the loss of earnings (or increase in expenses) due to the death (or disability) of a keyemployee. This type of plan is also referred to as key person insurance.

Purpose — A firm is sometimes dependent upon a key person whose management skill, technical knowledge, andexperience makes him or her an invaluable asset of the business. In a sense, the company is dependent on this keyperson for its success. The proceeds of a life insurance policy covering a key employee will provide the business withthe necessary funds to find and train a new employee and continue the business without further interruption. Keyemployee insurance covers an employee and not the business owner.

! Third Party Ownership — Key employee insurance is a common illustration of third party ownership. The business possesses an economic and financial interest in its key employee. Therefore, insurable interestis present in such a relationship. The potential economic loss of the business can be protected against thedeath of a key employee if the business is made the beneficiary of the life insurance policy. Therefore, thepolicy will indemnify the business for the financial loss it suffers due to the death of the covered keyemployee. The business will be indemnified for its loss of the key manager, director or officer and policyproceeds will help it to continue while a replacement is sought. The employer or business is the policy ownerand the employee is the insured. This situation involves third party ownership. Since the employer is thepolicyholder, it possess all ownership rights including the right to name the beneficiary.

! Ownership — The corporation, firm, partnership, or sole proprietorship will be the applicant, policy owner,premium payor, and beneficiary (i.e., third party ownership). Therefore, the business possesses the ownerrights under the policy such as naming or changing the beneficiary, borrowing from the cash value, receivingdividends, or assigning benefits. Whole life or universal life contracts are commonly used to fund a keyemployee life insurance plan. Term life insurance may be used for short term needs. As "Tax Facts" states,premiums paid by a business for key employee life insurance are generally not tax deductible. In addition,none of the death benefit paid is taxable when a key employee dies. The death proceeds will not be includedin the deceased employee's estate as long as he or she has no incidents of ownership in the contract. Remember that a Key Employee or Key Person life insurance plan does not provide life insurance coverageon the employer's life. It covers the life of the key person and indemnifies the employer (i.e., the business)if the key employee dies.

NILA ©

L/H Licensing

Chapter 10—1

Page 136: FORWARD - WordPress.com · Return of premium p 6-8 6.0 Annuities p 7-1 Annuity principles and concepts p 7-1 Accumulation period versus annuity period p 7-1 Owner, annuitant and beneficiary

BUSINESS CONTINUATION

Sole proprietorships, corporations, or partnerships are faced with the challenge of business stability and continuationfollowing the death of one or more of its owners or partners. The surviving family of the deceased individual alsoholds a personal and economic interest in the business as well, or more specifically, the deceased's share of the firm. The partners of a business wish to make sure that if one of them dies, the survivors of the deceased partner willreceive funds equal to his or her financial interest in the firm. Therefore, partners or members of a corporation, ora sole proprietor and a key employee will enter into a formal business continuation agreement known as a buy-sellagreement.

BUY-SELL AGREEMENT — A buy-sell agreement is a legal agreement which provides for: (1) an orderlycontinuation of the business; (2) an amount of money to be paid to the deceased's survivors. Funds to be paid to thesurviving family may come from life insurance. Life insurance may be purchased to fund a buy-sell agreement.

In a partnership, law states that any change in its membership will cause its dissolution. Therefore, if a partner dies,the partnership ends. The remaining partners must now wind up the business and pay to the deceased partner's estate an amount equal to the deceased's fair share of the liquidated value of the business. If a forced sale results whereassets are sold for less than they are worth, this fair share of the business may be less than anticipated. Therefore,life insurance which funds the buy-sell agreement will help to maintain the value of the business. A buy-sell agree-ment used in a partnership binds the surviving partners to purchase the partnership interest of the first partner to die,at a prearranged price identified in the agreement. The agreement obligates the estate of the deceased partner to sellits interest to the surviving partner(s). This agreement permits the surviving partners, officers or stockholders tomaintain control of the business. This agreement, supported by life insurance, is designed to protect the businessor firm.

There are two types of partnership buy-sell agreements. An entity plan specifies that the partnership is obligated tobuy out the ownership interest of the deceased partner. In other words, the agreement made is between thepartnership and each of the partners. Therefore, if the partnership consists of four partners, it will purchase, own,and pay for a life insurance policy covering each of the four partners. In other words, four policies will be purchasedto fund the agreement. Policy proceeds will be paid to the partnership. A cross-purchase plan specifies that theagreement will exist between the partners themselves and not between the partnership and the partners as in the entityplan. For example, if the partnership consists of four partners, each of the partners will purchase, own, and pay fora policy covering each of the other partners. In this case, there would be a total of twelve policies.

Buy-sell agreements used for corporations may also be funded by life insurance. An entity plan used for acorporation is known as a stock redemption plan. The corporation is bound to purchase the stock of the deceasedstockholder at a prearranged price. If funded by life insurance, the corporation buys a policy on each of thestockholder's lives. A cross-purchase plan financed by life policies involves each stockholder buying policies on eachof the other stockholders. Both corporation plans function in a similar manner to agreements available to partnersand partnerships. An estate comprised mostly of stock which possesses potential estate tax problems (i.e., forcedsale) may utilize a 303 redemption funded by life insurance. To qualify for this type of plan the value of the stockmust represent at least 35% of the deceased's adjusted gross estate. Buy-sell agreements can be funded for use in asole proprietorship, partnership or in a closely-held corporation.

NILA ©

L/H Licensing

Chapter 10—2

Page 137: FORWARD - WordPress.com · Return of premium p 6-8 6.0 Annuities p 7-1 Annuity principles and concepts p 7-1 Accumulation period versus annuity period p 7-1 Owner, annuitant and beneficiary

SPLIT DOLLAR

This type of plan is a funding method and not a specific type of life insurance policy. It is characterized by an ar-rangement between an employer and employee. The plan can only be funded with whole life, cash value, orcontinuous-premium life insurance. The death benefit is split as is the cash value (i.e., living benefit). In some casesthe premium may be split as well.

PURPOSE AND USE — The purpose of a SDP is to join together the needs of one person (i.e., the employee)with the premium paying ability of another party (i.e., employer). A SDP can provide an employee with life in-surance protection that he cannot afford on his own. The employer may discriminate when providing such plans. In other words, the employer can provide a SDP for any employee he or she chooses (does not have to provide forall).

The most common type of SDP is characterized by an employer providing funds to pay that part of each annualpremium which is equal to the annual increase in cash value. The employee pays the balance. For example, if theannual premium was $500 and the increase in cash value was $420 after the premium was paid, the employer pays$420 and the employee $80. The employer is entitled to receive death proceeds in an amount equal to the policy'scash value with the balance paid to the employee's beneficiary. SDPs can also be used among family members (i.e.,parent/child) or stockholders in a corporation. Other split dollar plan variations include single bonus plans, reversesplit dollar plans and employer (non-contributory) pay-all plans.

DEFERRED COMPENSATION

DEFERRED COMPENSATION FUNDING — This generally refers to non-qualified retirement plans. Plansthat do not receive tax advantages according to the IRC as do qualified plans. These types of arrangements aregenerally between an employer and employee where compensation is paid to the employee in later years. Someemployers use cash value life insurance or annuity products to provide promised funds.

CORPORATE OWNED LIFE INSURANCE — According to Life Insurance by Black and Skipper, "someinsurers now include a "change of insured provision" in life insurance policies utilized for business purposes whichallow for a change of insureds. This is useful primarily in corporate-owned life insurance policies". When anemployee who is covered by the policy either retires or his employment is terminated, the employer may change thename of the insured with that of a new or replacement employee, subject to insurability requirements. Theavailability of this provision eliminates the need to write a completely new policy which would result in additionalpolicy fees, commissions or other expenses that are incurred when purchasing new life insurance.

EXECUTIVE BONUSES — Also referred to as a Section 162 bonus plan, an executive bonus plan is a nonqualified employee benefit arrangement in which an employer pays a compensation bonus to a selected employee

who uses the bonus payment to pay the premiums on a life insurance policy covering his or her life. The employeeowns the policy personally. The employer may use the amount of the bonus as a tax deduction and the employee,whether he or she uses it to buy insurance or not, must include the amount of the bonus in his or her gross income. The proceeds of the policy are paid, in the event of the employee / insured's death, to the designated beneficiaryincome tax free. Any policy withdrawals, surrenders or loans made by the employee are taxed as they would be ifthe employee had purchased the policy without the benefit of the bonus arrangement.

NILA ©

L/H Licensing

Chapter 10—3

Page 138: FORWARD - WordPress.com · Return of premium p 6-8 6.0 Annuities p 7-1 Annuity principles and concepts p 7-1 Accumulation period versus annuity period p 7-1 Owner, annuitant and beneficiary

QUIZ

1. Tristar Computers is a partnership comprised of John, Bob, and Tim. They hold an equal financial interest in the

partnership. After deciding to enter into a buy-sell agreement, their agent suggests they utilize a cross-purchase plan and

fund it with life insurance. If this plan is utilized, how many life insurance policies will be purchased?

A. Three C. Six

B. Four D. Eight

2. Which of the following parties has the right to change the name of the primary beneficiary under a key employee life

insurance plan?

A. The covered key employee C. The irrevocable beneficiary

B. The revocable beneficiary D. The employer

3. All of the following statements are correct with regard to a split dollar plan, except:

A. A split dollar plan must utilize a cash value policy

B. The split dollar plan is a specific type of life insurance policy

C. An employer may discriminate when providing a split dollar plan to employees

D. Split dollar plans join together the needs of a key employee and the employer

4. Which of the following is an indication of third party ownership?

A. Deferred compensation funding C. Key employee life insurance

B. Industrial life insurance D. Split dollar plans

5. The ABC Organization is comprised of five partners. If it wishes to enter into an entity type of buy-sell agreement

funded potentially by life insurance, how many policies need to be produced?

A. 3 C. 5

B. 4 D. 6

6. An applicant for life insurance, who is not the proposed insured, possesses an insurable interest in the life of:

A. Her legal representative C. Her next of kin

B. Her attorney D. Her business partner

7. Which of the following policies will assist a business in paying expenses and other necessary bills should a manager or

director of a firm die?

A. A buy-sell agreement C. Split dollar plan

B. Key employee insurance D. A buy-out policy

8. The purpose of a buy-sell agreement funded by a life insurance policy is to:

A. Make sure that the insured does not outlive his income

B. Protect directors and officers of a corporation should they be liable for the mismanagement of funds

C. Permit the surviving officers of a firm to maintain control of the business

D. Provide an employee with life insurance protection not otherwise affordable

NILA ©

L/H Licensing

Chapter 10—4

Page 139: FORWARD - WordPress.com · Return of premium p 6-8 6.0 Annuities p 7-1 Annuity principles and concepts p 7-1 Accumulation period versus annuity period p 7-1 Owner, annuitant and beneficiary

9. A buy sell agreement is designed to protect which of the following ?

A. The survivors of a deceased partner

B. A deceased partner

C. The firm

D. Dependents of the deceased business partner

ANSWERS

1. C 6. D 2. D 7. B3. B 8. C4. C 9. C5. C

NILA ©

L/H Licensing

Chapter 10—5

Page 140: FORWARD - WordPress.com · Return of premium p 6-8 6.0 Annuities p 7-1 Annuity principles and concepts p 7-1 Accumulation period versus annuity period p 7-1 Owner, annuitant and beneficiary

LIFEINSURANCE

SAMPLE EXAM

Page 141: FORWARD - WordPress.com · Return of premium p 6-8 6.0 Annuities p 7-1 Annuity principles and concepts p 7-1 Accumulation period versus annuity period p 7-1 Owner, annuitant and beneficiary

LIFE INSURANCESAMPLE EXAMINATION

1. An annuity which provides lifetime benefits for the annuitant and a lump sum settlement of any undistributed cost of the

annuity upon the annuitant's death is a:

A. Life annuity with 20 years certain and continuous

B. Life annuity with installment refund

C. Joint and survivor annuity

D. Cash refund annuity

2. A contract which provides protection against the risk of living too long is a(n):

A. Endowment C. Annuity

B. Renewable term policy D. Whole life contract

3. All of the following are true regarding "insurable interest", EXCEPT:

A. The beneficiary does not need to have an insurable interest in the insured

B. Usually, an individual may not insure the life of another without the consent of the other party

C. Generally, it is assumed that an individual does not have an insurable interest in his/her own life

D. The policy owner must have an insurable interest in the insured

4. Life insurance might be used in all of the following situations, EXCEPT:

A. To fund a buy-sell agreement C. As a charitable contribution

B. To help fund a child's college education D. To cover the risk of living too long

5. The elements of a contract include all of the following, EXCEPT:

A. Offer and acceptance C. Consideration

B. Conditional receipts D. Competent parties to the contract

6. An agent's responsibility for notification of an adverse underwriting decision includes all of the following, EXCEPT:

A. Providing the applicant with a summary of rights regarding investigative consumer reports

B. Providing some reasons for the adverse decision and informing the individual that specific reasons will be given

upon written request

C. Providing the applicant with a new policy from a different insurer

D. Providing names of institutional sources which provided information regarding the applicant

7. An individual has applied for life insurance and given the agent the initial premium. The agent provides the applicant

with a receipt which specifies that the individual is covered immediately for 60 days from the date of the application.

Which type of receipt was issued?

A. Approval C. Tentative

B. Unconditional D. Insurability receipt

NILA ©L/H Licensing

Life Insurance Sample Exam 11-1

Page 142: FORWARD - WordPress.com · Return of premium p 6-8 6.0 Annuities p 7-1 Annuity principles and concepts p 7-1 Accumulation period versus annuity period p 7-1 Owner, annuitant and beneficiary

8. Which element of an insurance contract requires that the proposed insured be competent and have the capacity to enter

into a contract?

A. Legal capacity C. Legal intent

B. Competent consideration D. Legal relevance

9. Which of the following best defines a warranty?

A. A failure to disclose a material fact

B. Any statement made by the applicant as part of the insurance contract

C. A statement or condition that the applicant represents to be absolutely true

D. A statement that the contract is in accordance with state law

10. An applicant who sends the application to the company without any premium is:

A. Making an offer C. Accepting an offer

B. Offering an acceptance D. Making an invitation

11. Under the Uniform Simultaneous Death Law, when the insured and the primary beneficiary are killed in a common

disaster, who is considered to have died last?

A. The beneficiary

B. The insured

C. The beneficiary and the insured are presumed to have died simultaneously

D. Neither. Policy proceeds are paid to the policyholder

12. A life insurance policy which insures two or more individuals with the face amount paid upon the death of the first to

die is a:

A. Modified whole life policy C. Joint life policy

B. Joint survivor policy D. Family maintenance plan

13. Settlement options in a life insurance policy:

A. Provide for various methods of paying policy proceeds

B. Are the same as annuities

C. Have various modes for premium payments

D. Are similar to non-forfeiture provisions

14. The settlement option which pays principal and interest until exhaustion is:

A. Interest only C. Fixed income option

B. Fixed amount option D. Life with period certain option

15. A contingent beneficiary is one who:

A. Shares equally with the deceased's estate

B. Receives benefits contingent on surviving the insured

C. Is the same as a primary beneficiary

D. Receives the proceeds if the primary beneficiary has predeceased the insured

NILA ©L/H Licensing

Life Insurance Sample Exam 11-2

Page 143: FORWARD - WordPress.com · Return of premium p 6-8 6.0 Annuities p 7-1 Annuity principles and concepts p 7-1 Accumulation period versus annuity period p 7-1 Owner, annuitant and beneficiary

16. The insured wishes to provide for the care of his minor child in the event of his/her premature death. To accomplish this,

the insured may designate as beneficiary:

A. The estate C. A trust

B. A favorite charity D. A spouse as an irrevocable beneficiary

17. Which of the following would affect an insurer the most if there were a sudden decrease in mortality?

A. Life annuities C. Modified life insurance

B. Whole life insurance D. Group life

18. Pledging a life insurance policy as security for a loan involves a(n):

A. Collateral assignment C. Absolute assignment

B. Contingent assignment D. Credit assignment

19. Who may change a beneficiary?

A. The contingent beneficiary C. The insured's estate representative

B. The insured D. The policyholder

20. All of the following will reduce the face amount of the policy, EXCEPT:

A. An unpaid loan at the time of death

B. Unpaid loan interest at the time the policy matures

C. Waiver of premium

D. Automatic premium loan outstanding at the policy maturity

21. Which of the following is true regarding the one-year term dividend option?

A. It provides additional life insurance protection equal to the annual premium

B. It provides additional life insurance protection equal to a policy loan

C. It provides additional life insurance protection equal to the double indemnity benefit

D. It is an optional settlement option

22. Which of the following is true regarding Universal Life?

A. Premiums, cash values and the face amount can be adjusted

B. There are no interest rate guarantees

C. It is identical with Adjustable Life

D. It is backed by equities such as a common stock portfolio

23. Which of the following is true regarding life insurance policy riders?

A. Accidental death benefits are twice the face amount of the policy

B. Guaranteed insurability provides the insured with the option to purchase additional amounts of insurance

without proof of insurability

C. The cost of the automatic premium loan provision is related to the age of the insured and the face amount of

the policy

D. Waiver of premium benefits protect the insured by providing disability income benefits

NILA ©L/H Licensing

Life Insurance Sample Exam 11-3

Page 144: FORWARD - WordPress.com · Return of premium p 6-8 6.0 Annuities p 7-1 Annuity principles and concepts p 7-1 Accumulation period versus annuity period p 7-1 Owner, annuitant and beneficiary

24. All of the following are true regarding term insurance, EXCEPT:

A. It provides temporary insurance for a certain period of time

B. The policy matures at the end of the term period

C. It may be converted to whole life

D. It does not build cash value

25. What is the principal difference between the life income and life income with period certain settlement options?

A. Life income with period certain provides a higher monthly income

B. Life income payments end with the death of the recipient whereas life with period certain provides income to

someone for the period certain

C. Life income with period certain provides for a greater number of payments than life income only

D. Life income is guaranteed whereas life with period certain is not fully guaranteed

26. All of the following are important underwriting factors, EXCEPT:

A. Medical information C. The amount of money deposited with the application

B. The agent's statement D. The application

27. Which of the following is true regarding the designation of minors as beneficiaries?

A. Usually minors cannot be named as beneficiaries

B. If a minor is named as beneficiary, policy proceeds must be paid to the minor

C. Most often, minors are designated as contingent beneficiaries

D. It is not necessary for a guardian to be appointed on behalf of a minor beneficiary

28. An insured recognizes the need for "some permanent life insurance" but no longer wishes to continue premium payments.

Which of the following might an agent recommend?

A. The extended term option C. The annuity option

B. Cash surrender option D. Reduced paid up option

\

29. An insured purchased a 20-year family maintenance policy and dies in the 19th year of the plan. How long will monthly

income benefits be provided for the family?

A. One year C. Twenty years

B. Five years D. Cannot be determined

30. The insured purchased a 20-year family income plan at age 30. She dies as age 52. How long will monthly income

benefits be provided?

A. Five years C. Twenty years

B. Ten years D. None of the above

31. All of the following are true regarding endowments, EXCEPT:

A. Endowments mature after a specified period of time

B. If the insured dies during the endowment period, the beneficiary receives the cash value as of the date of death

C. Endowments offer death protection for a specified period of time

D. Endowments can be used for retirement purpose

NILA ©L/H Licensing

Life Insurance Sample Exam 11-4

Page 145: FORWARD - WordPress.com · Return of premium p 6-8 6.0 Annuities p 7-1 Annuity principles and concepts p 7-1 Accumulation period versus annuity period p 7-1 Owner, annuitant and beneficiary

32. All of the following are true regarding variable life, EXCEPT:

A. Variable life policies are both state and federally regulated

B. Variable life policies are products of life insurance companies

C. Variable life policies are a guaranteed hedge against inflation

D. Variable life policies are considered a security

33. Which of the following best describes a graded premium life plan?

A. The premiums and face amount are adjustable

B. Lower premiums are paid for the first few years followed by a higher, level premium thereafter

C. The face amount of the policy changes annually

D. Premiums are relatively low but increase annually during the first 5 or 10 years followed by a level premium

thereafter

34. If the insured no longer wishes to pay premiums for his/her 10 year term policy but wishes to have the full face amount

of the policy remain in force for a certain period of time, he/she should:

A. Place the policy on extended term

B. Use the automatic premium loan provision

C. Place the policy on reduced paid up

D. None of the above

35. The Fair Credit Reporting Act:

A. Provides guidelines on premium financing

B. Is not applicable to the heath insurance industry

C. Establishes procedures for consumer reporting agencies

D. Prohibits the use of information on an individual's character and general reputation

36. A family policy or family plan would cover all of the following, EXCEPT:

A. Permanent insurance on the father

B. A single policy covering all family members

C. All children are covered by term insurance

D. Children's coverage terminates without a conversion privilege

37. All of the following are true regarding industrial life, EXCEPT:

A. It is usually sold in amounts of $1,000 or less

B. Premiums are usually collected on a weekly basis

C. It may be issued as an endowment

D. It may be issued as term to age 70

38. All of the following are methods of determining rates for substandard risks, EXCEPT:

A. Rated-up age C. Tabular Rating

B. Graded premium D. Flat Additional Premium

39. Which life insurance provision recognizes the equity build up in the early years of a whole life policy?

A. Nonforfeiture provisions C. Payment of premiums provision

B. Grace period D. Incontestable provision

NILA ©L/H Licensing

Life Insurance Sample Exam 11-5

Page 146: FORWARD - WordPress.com · Return of premium p 6-8 6.0 Annuities p 7-1 Annuity principles and concepts p 7-1 Accumulation period versus annuity period p 7-1 Owner, annuitant and beneficiary

40. Decreasing term insurance might best be used for covering:

A. A child's education C. The financial obligations which arise at death

B. A home mortgage D. Retirement years

41. The law of large numbers provides that:

A. Predictions become more accurate as the number of exposure units increases

B. Large losses are easier to predict than small losses

C. Most losses are not predictable

D. All losses are predictable

42. All of the following are powers or the authority of an agent, EXCEPT:

A. The express authority contained in the agency contract

B. The apparent authority acquired through usage of the industry

C. The implied authority exercised by an agent

D. The authority to change policy provisions when the policy is delivered to the insured

43. A salaried employee of a profit making corporation may be covered by which of the following?

A. A Keogh plan C. A tax sheltered annuity

B. A pension plan D. An HR-10 plan

44. Which of the following is not one of the dividend options available in a whole life policy?

A. Fixed period installments C. Accumulate at interest

B. Paid up additions D. Reduce the premium

45. Which dividend option will provide for increased cash value in the policy?

A. Paid up additions C. Reduce the premium

B. Accumulate at interest D. Paid up policy

46. All of the following are true regarding dividend options, EXCEPT:

A. The dividend may be taken in cash subject to taxation

B. The dividend may accumulate at interest which is taxable

C. The dividend may be used to pay part of the premium

D. The dividend may be used to purchase one year term insurance

47. The provision which permits the policyholder to cancel the policy within a certain period of time and receive a full refund

of premium is the:

A. Ten day free-look C. Cash surrender option

B. Return of premium rider D. Probationary provision

48. A life annuity with ten years certain means:

A. The annuitant must make payments for ten years

B. The annuitant will be paid for ten years

C. The annuitant or his survivor is guaranteed ten years of benefits

D. The annuitant or his survivor is guaranteed ten years of payments and if the annuitant survives ten years,

payments continue for the lifetime of the annuitant

NILA ©L/H Licensing

Life Insurance Sample Exam 11-6

Page 147: FORWARD - WordPress.com · Return of premium p 6-8 6.0 Annuities p 7-1 Annuity principles and concepts p 7-1 Accumulation period versus annuity period p 7-1 Owner, annuitant and beneficiary

49. The difference between a 20 pay life and a 20 year endowment is:

A. Endowments do not require evidence of insurability since they are considered savings plans

B. A 20 pay life policy matures after 20 years of premium payments

C. Cash values and the premium are higher for an endowment

D. The 20 pay life policy offers lifetime death protection whereas the 20 year endowment has no death benefit

50. In the course of a sales interview, an agent informs the prospect that policy dividends are guaranteed. This is an example

of:

A. Defamation C. Unfair disclosure

B. Misrepresentation D. Rebating

51. An insured who no longer feels the need to maintain a whole life policy and no longer wishes to pay premiums due to

the need to invest more heavily in a stock portfolio, might exercise which of the following options?

A. Cash surrender C. Reduced paid up

B. Automatic premium loan D. Extended term

52. An insured has a $50,000 participating whole life policy with an outstanding loan of $3,000. What would be the best

solution if the insured wishes to maintain the full face amount of the policy?

A. Nothing. At death, the policy matures for $47,000 because of the outstanding loan

B. Use dividends to reduce the premium payments and place this savings in the bank until it accumulates to $3,000

C. Use the dividends to accumulate at interest until they total $3,000

D. Use the dividends to buy one-year term insurance under the one-year term option

53. Which settlement option has been elected if a beneficiary is receiving the insurance proceeds in equal installments over

a specified period of time?

A. Interest only C. Life income

B. Fixed period D. Joint-survivor

54. Which nonforfeiture option provides for the longest period of protection?

A. Reduced paid up insurance C. Life with period certain

B. Extended term insurance D. Cash surrender

55. The insured dies during the grace period of a life policy. The beneficiary will receive:

A. Nothing since the premium was not paid

B. The full face amount of the policy

C. The face amount of the policy less a pro-rata share of the unpaid premium

D. The face amount of the policy less one month's premium

56. The nonforfeiture option which provides for the full face amount of the policy to remain in force for a specified period

of time is the:

A. Reduce paid up option C. Automatic premium loan

B. Extended term option D. Cash surrender option

57. Which dividend option would increase the total death benefit to the greatest amount possible?

A. Accumulation at interest C. Paid-up additions

B. Accelerated endowment D. Application to reduce the premium

NILA ©L/H Licensing

Life Insurance Sample Exam 11-7

Page 148: FORWARD - WordPress.com · Return of premium p 6-8 6.0 Annuities p 7-1 Annuity principles and concepts p 7-1 Accumulation period versus annuity period p 7-1 Owner, annuitant and beneficiary

58. Level premiums, guaranteed cash value and a constant face amount best characterizes:

A. Modified life C. Term life to age 65

B. Life paid-up at 65 D. Graded premium whole life

59. A minor son is to receive the policy proceeds at the death of the insured. How could this best be achieved?

A. Name the minor as the primary beneficiary

B. Name the insured's estate as the beneficiary because the son is a minor

C. Designate a trust as the beneficiary for the benefit of the minor son

D. Do not name a beneficiary

60. A 25 year old doctor wishes to invest in an annuity with the ultimate goal of providing for an early retirement at age 50.

Currently, the doctor is just beginning her medical practice. If the doctor's expenses are high and income is modest but

the future is very bright financially, which type of annuity do you think she should purchase?

A. Single premium immediate annuity C. Single premium deferred annuity

B. Flexible premium deferred annuity D. Flexible premium immediate annuity

61. The value of an individual's earning potential over a period of time is known as:

A. The capitalization of earnings approach C. Deferred compensation approach

B. Single needs approach D. The human life value approach

62. Sources of underwriting life insurance include all of the following, EXCEPT:

A. The agent's report C. The application

B. Medical Information Bureau D. Income tax statements

63. A beneficiary is receiving $300 per month for an indefinite period of time. This is an example of which of the following?

A. Life income settlement option C. Fixed amount installment option

B. Fixed period installment option D. Interest only settlement option

64. Which of the following situations would not result in a 10% penalty tax for withdrawal of annuity proceeds?

A. An annuitant, age 40, withdraws part of the annuity proceeds for vacation expenses

B. An annuitant becomes totally disabled at age 48

C. An annuitant who owns a tax sheltered annuity makes a partial withdrawal due to the death of a spouse

D. An annuitant, age 33, surrenders the annuity for cash

65. When an insured withdraws money from a Modified Endowment Contract before age 59 ½ , what amount of the

distribution will be subject to taxation?

A. 25% B. 50% C. 75% D. 100%

66. The Internal Revenue code that allows a tax sheltered annuity for certain types of employees is known as:

A. 401K B. HE-10 C. 1035A D. 403-B

67. Which of the following would not qualify for a tax free exchange under 1035-A, Internal Revenue code?

A. Variable annuity for a variable annuity

B. A variable life insurance policy for a universal life covering two lives

C. Limited pay whole life for a whole life

D. Fixed annuity for a fixed annuity

NILA ©L/H Licensing

Life Insurance Sample Exam 11-8

Page 149: FORWARD - WordPress.com · Return of premium p 6-8 6.0 Annuities p 7-1 Annuity principles and concepts p 7-1 Accumulation period versus annuity period p 7-1 Owner, annuitant and beneficiary

68. The accumulation phase of a fixed annuity may terminate for each of the following reasons, EXCEPT:

A. Death of annuitant C. Premium payments cease

B. The annuity phase begins D. Surrender of the annuity

69. All of the following should be eligible to establish a Keogh retirement plan, EXCEPT:

A. A dentist in private practice C. A sole proprietor of a jewelry store

B. Partners in a furniture store D. A major stockholder-employee in a family corporation

70. All of the following statements regarding survivor financial needs are correct, EXCEPT:

A. The term "dependency period" refers to the 20-year period immediately following the insured's death during

which the widowed spouse must depend on Social Security

B. The period for which there are no Social Security benefits for the surviving spouse is known as the blackout

period

C. A final expense fund addresses a breadwinner's last illness and funeral costs, death taxes and outstanding debts

D. A housing fund addresses a family's rental or mortgage needs

71. Three business partners individually agree to acquire the interest of a deceased partner and own life insurance on each

of the other partners in the amount of his or her share of the business's buy-out value. What is described here is:

A. An entity buy-sell plan C. A cross-purchase buy-sell plan

B. A stock redemption buy-sell plan D. A 401(k) plan

72. Which of the following statements regarding key-person insurance is NOT true?

A. Key-person life insurance indemnifies a business for financial loss caused by the death of a key-employee or

key executive

B. The business may borrow from the cash value of a permanent key-person life insurance policy

C. The policy's death proceeds received by the business are not taxable

D. Premiums for a key-person life insurance policy are a tax deductible expense to the business

73. Jack Quinn and his employer agree on the purchase of a split-dollar life insurance plan and the usual split-dollar approach

to premium payments. Each year, the employer will contribute to the premium an amount equal to:

A. One-half the premium C. The increase in the policy's cash value

B. The annual dividend D. Two-thirds of the premium

74. Which of the following examples pertaining to fixed annuities is correct?

A. Brian would like to protect his wife and son against the contingency of his premature death and also supplement

future retirement income. Therefore, he should purchase an annuity contract rather than a life insurance policy

B. A $40,000 single payment immediate annuity would have a lower total payment cost than a $40,000 single

payment deferred annuity

C. Hazel, who plans to retire in 12 years, inherited $10,000 from an aunt and wants to put the funds into an

annuity. A single-payment deferred annuity should be recommended

D. Randolph and Gertrude, both age 60, will receive the first monthly payment on their annuities next month.

Gertrude will receive the larger monthly check per $1,000 of annuity funds

75. A survivorship life insurance plan is a specialty type of contract. This type of policy is generally purchased in order to

cover how many individuals?

A. 2 B. 3 C. 5 D. 7

NILA ©L/H Licensing

Life Insurance Sample Exam 11-9

Page 150: FORWARD - WordPress.com · Return of premium p 6-8 6.0 Annuities p 7-1 Annuity principles and concepts p 7-1 Accumulation period versus annuity period p 7-1 Owner, annuitant and beneficiary

76. A credit life insurance plan is available to a borrower to cover the amount of an indebtedness. The type of life insurance

used in a credit insurance situation is:

A. Modified life C. Level term

B. Yearly renewable term D. Decreasing term

77. Permanent insurance policies include non-forfeiture options. Which of the following permits the policy owner to use

the cash value to purchase more insurance?

A. Extended term insurance C. One year term

B. Reduce the premium D. Life income option

78. All of the following statements regarding annuities are correct, EXCEPT:

A. An annuity is a systematic approach to liquidating an estate

B. When an annuity is funded with a single, lump-sum payment, the principal is created immediately

C. Generally, annuity contracts issued today require fixed, level funding payments

D. Only insurance companies are able to offer annuities that provide guaranteed lifetime income for the annuitant

79. Which of the following statements regarding annuity income options is NOT correct?

A. Under a straight life annuity option, all annuity payments stop when the annuitant dies

B. In a cash refund annuity, the annuitant's beneficiary always receives an amount equal to the beginning annuity

fund plus all interest

C. A period certain annuity guarantees a definite number of payments

D. Joint and survivor annuities guarantee payments for the duration of two lives

80. "Annuity payments are taxable to the extent that they represent interest earned rather than capital returned." What

method is used to determine the taxable portion of each payment?

A. The exclusion ratio C. The surtax ratio

B. The marginal tax formula D. The annuity ratio

81. Before he died, Gary received a total of $9,200 in monthly income payments from his $15,000 straight life annuity. He

also was the insured under a $25,000 life insurance policy that named his wife, Darlene, as primary beneficiary.

Considering the two contracts, Darlene would receive death benefits totaling:

A. $15,000 C. $30,800

B. $25,000 D. $40,000

82. Which of the following statements pertaining to the Medical Information Bureau (MIB) is correct?

A. The MIB is operated by a national network of hospitals

B. Information obtained by the MIB is available to all physicians

C. The MIB provides assistance in the underwriting of life insurance

D. Applicants may request that MIB reports be attached to their policies

83. Which of the following statements regarding the Fair Credit Reporting Act (FCRA) is correct?

A. Applicants must be notified within a short period of time that their credit report has been requested

B. If an applicant for insurance is rejected based on a consumer report, the name of the reporting agency must be

kept confidential

C. If requested to do so, the insurance company must provide the actual consumer report to the applicant

D. Consumer reports are final in nature and cannot be disputed by an applicant

NILA ©L/H Licensing

Life Insurance Sample Exam 11-10

Page 151: FORWARD - WordPress.com · Return of premium p 6-8 6.0 Annuities p 7-1 Annuity principles and concepts p 7-1 Accumulation period versus annuity period p 7-1 Owner, annuitant and beneficiary

84. Earl signed an application for a $100,000 life insurance policy on September 2nd and took a required medical exam on

September 4th. He gave the agent a check for the initial premium and received a conditional insurability receipt at the

time of application. The home office approved the policy and sent it to the agent on September 30th and the agent

delivered the policy to Earl on October 5th. Earl's insurance protection actually began on:

A. September 2 C. September 30

B. September 4 D. October 5

85. All of the following statements pertaining to the conversion privilege of group term life insurance are correct, EXCEPT:

A. An insured employee has 31 days following termination of employment in which to convert the group insurance

B. An insured employee must convert to the same type of term life coverage as was provided under the group plan

C. Insureds who convert their coverage to individual plans pay a premium rate according to their attained age

D. An insured employee may exercise the conversion privilege regardless of his or her insurability

86. Nancy has a life insurance policy stating that her husband, Joe, is to receive the full death benefit. If he predeceases her,

their three children are to share the benefit equally. If her husband and all three children predecease her, the benefit is

payable to the First Community Church. All of the following statements are correct, EXCEPT:

A. Joe is the primary beneficiary

B. The three children are all secondary beneficiaries

C. The First Community Church is the tertiary beneficiary

D. The designation of the Church can be contested by any of Sam's relatives who survive the children

87. Christine's policy has a clause that reads as follows: "Should the primary beneficiary and the insured die in the same

accident and the primary beneficiary fails to survive the insured by 14 days, it will be assumed that the beneficiary

predeceased the insured." Which of the following is responsible for this clause?

A. Secondary beneficiary provision C. Uniform Simultaneous Death Act

B. Facility-of-payment provision D. Common disaster provision

88. Which of the following statements pertaining to life insurance premiums is correct?

A. The premiums for a policy that insures a spouse are tax deductible

B. A company may purchase key-person life insurance and deduct the premiums as a business expense

C. Premiums for group term insurance covering employees are tax deductible, assuming certain rules are met

D. Premiums for policies in which the insured is someone other than the policyholder are tax deductible

89. Art, the owner and insured under a $75,000 life policy, is killed in an accident. He has paid total premiums of $26,000.

How much of the death benefit will be included in his gross estate for estate tax purposes?

A. $26,000 C. $50,000

B. $49,000 D. $75,000

90. Sarah, age 65, the owner of a $150,000 whole life policy, decides to surrender the policy and take the $90,000 cash value

in a lump sum. Over the years, she has paid a total of $54,000 in premiums. How much of the payment will be taxed?

A. $0 B. $36,000 C. $54,000 D. $90,000

91. Which of the following statements regarding the assignment of a life insurance policy is NOT correct?

A. Absolute assignment involves a complete transfer, giving the assignee full control over the policy

B. Under a collateral assignment, a creditor is entitled to be reimbursed out of the policy's proceeds only for the

amount of the outstanding credit balance

C. Under a collateral assignment, policy proceeds in excess of the collateral amount pass to the insured's

beneficiary

D. All beneficiaries must expressly approve any assignments of life insurance policies

NILA ©L/H Licensing

Life Insurance Sample Exam 11-11

Page 152: FORWARD - WordPress.com · Return of premium p 6-8 6.0 Annuities p 7-1 Annuity principles and concepts p 7-1 Accumulation period versus annuity period p 7-1 Owner, annuitant and beneficiary

92. All of the following phrases describe life insurance policy nonforfeiture options, EXCEPT:

A. Cash surrender option C. Extended term insurance option

B. One-year term insurance option D. Reduced paid-up (permanent) insurance option

93. Which of the following statements best describes the role of policy dividends?

A. Policy dividends represent earnings to shareholders who hold stock in insurance companies

B. Policy dividends impact the costs of virtually all insurance policies issued today

C. Policy dividends are an intentional return of a portion of the premiums paid

D. Policy dividends provide policy owners with a level, known annual cash flow

94. If an insurance company determines that the insured is totally disabled, the policyholder is relieved of paying the policy

premiums as long as the disability continues. This statement describes:

A. The premium suspension clause C. The disability income rider

B. The waiting period exemption D. The waiver of premium rider

95. All of the following statements regarding basic forms of whole life insurance are correct, EXCEPT:

A. Generally, straight life premiums are payable, at least annually, for the duration of the insured's life

B. The owner of a 30-pay life policy will owe no more premiums after the thirtieth year the policy is in force

C. Limited payment life provides protection only for the years during which premiums are paid

D. A single-premium life policy is purchased with a large one-time only premium

96. Which of the following statements regarding modified endowment contracts (MECs) is correct?

A. A 1988 revenue act, commonly known as TAMRA, greatly increased the popularity of MECs

B. Congress has granted the modified endowment contract the most favorable tax status among all life insurance

policies

C. To avoid being classified as a modified endowment contract, a life insurance policy must satisfy the "7-pay test"

D. According to the "7-pay test," if the total amount a policyholder pays into a life contract during its five years

exceeds the sum of the net level premiums that would have been payable to provide paid-up future benefits in

seven years, the policy is a modified endowment

97. Which of the following statements regarding universal life insurance is NOT correct?

A. Although premiums and cash value allocations are flexible in a universal life policy, the amount of death

protection always remains the same

B. A mortality charge is deducted from the policy's cash value monthly

C. Interest may be available at either the contract's guaranteed minimum rate or at the current interest rate as

declared by the company

D. To avoid tax problems, premium allocations to a universal life insurance policy's cash account must fall within

guidelines

98. An error in age is discovered after the death of an insured, but before any policy death proceeds are distributed. The

insured was older than previously assumed. How would an insurance company handle such a situation?

A. No adjustment would be made because the contestable period had passed

B. The amount of death proceeds would be reduced to reflect the statistically diminished mortality risk

C. The amount of death proceeds would be reduced to reflect whatever benefit the premium paid would have

purchased at the correct age

D. The survivors would receive any overpaid back premiums

NILA ©L/H Licensing

Life Insurance Sample Exam 11-12

Page 153: FORWARD - WordPress.com · Return of premium p 6-8 6.0 Annuities p 7-1 Annuity principles and concepts p 7-1 Accumulation period versus annuity period p 7-1 Owner, annuitant and beneficiary

99. The grace period, the reinstatement clause and the automatic premium loan provision exist in a policy for the same

essential purpose. What is that purpose?

A. To keep a life insurance policy in force even when the premium is late

B. To allow for adjustments when distributions of participating policy dividends are late

C. To provide temporary coverage for situations in which an insured's health status is questionable

D. To permit changes in the ownership of a life insurance policy

100. All of the following are situations in which an incontestable clause does NOT apply, EXCEPT:

A. Impersonation of the applicant by another C. Intent to murder

B. No insurable interest D. Concealment of smoking

101. Which of the following provides malpractice like insurance for producers and independent agents?

A. Workers’ Compensation C. Professional liability

B. Errors and omissions D. Group term life

102. Which of the following types of insurance is provided for covered employees in a group life insurance plan?

A. Annual renewable term C. Increasing term

B. Decreasing term D. Limited payment life

103. Which of the following occurs if the cash value of an ordinary life insurance policy equals or exceeds the face amount?

A. The insurer voids the contract

B. The insurer waives its claim of the cash value

C. The insurer pays the amount to the policy owner

D. The insurer pays the death benefit

104. Which of the following helps to avoid a moral hazard?

A. A policy summary C. Outline of coverage

B. Buyer’s guide D. Field underwriting

105. The underwriting department of a life insurer engages in each of the following activities, EXCEPT:

A. Protects the insurer’s against adverse selection

B. Develops the premium rate for the insured

C. Selects risks randomly

D. Reviews applications for coverage

106. When the concept of agency is applied to the insurance business, which of the following is the principal?

A. Insurer C. Producer

B. Agent D. Captive

107. When a contract owner receives an annuity but returns it to the insurer within ten days, which of the following occurs?

A. A pro-rated return of premium is provided

B. A full premium refund is provided

C. A partial return of premium is provided

D. The free-look period ends

NILA ©L/H Licensing

Life Insurance Sample Exam 11-13

Page 154: FORWARD - WordPress.com · Return of premium p 6-8 6.0 Annuities p 7-1 Annuity principles and concepts p 7-1 Accumulation period versus annuity period p 7-1 Owner, annuitant and beneficiary

108. Which of the following best describes a source of information available to an underwriter known as the Attending

Physician Statement (APS) ?

A. A survey promoted by a personal physician C. A medical information bureau report

B. An annual premium statement D. A medical report from an applicant’s personal physician

109. Which of the following life insurance provisions protects the interest of a contingent beneficiary?

A. Beneficiary provision C. Incontestable clause

B. Interest only provision D. Common disaster clause

110. Which of the following indicates that death proceeds from a life insurance policy will be paid “through the root” ?

A. Per occurrence C. Per stirpes

B. Per capita D. Per status

111. Which of the following would be best served by purchasing an immediate annuity?

A. An individual who is terminally ill C. The beneficiary of a life insurance policy

B. A new physician just out of medical school D. A person who is permanently disabled

112. Which of the following would not be covered by a family term rider?

A. Spouse of primary insured C. The primary insured

B. Adopted child of primary insured D. A natural child of the primary insured

113. A qualified annuity differs from a non-qualified annuity in that the premiums paid to the insurer are:

A. Before tax dollars C. Tax deductible

B. Taxable income D. Tax exempt

114. Once an individual is paid funds from an IRA in order to roll it over into another like account or qualified plan, the

rollover must be accomplished within:

A. Thirty days C. Six months

B. Sixty days D. One year

115. Which of the following involves a return of premium?

A. Paid-up additions C. Reduce the premium

B. Fixed amount D. Dividend

116. Which of the following is a not method available to accelerate benefits in an insurance policy prior to an insured’s death?

A. Accelerated benefits rider C. Accidental death rider

B. Long-term care rider D. Viatical settlement

117. In a viatical settlement, which of the following is the party that transfers ownership of a life insurance policy?

A. Viatical settlement broker C. Viator

B. Viatee D. Conveyor

NILA ©L/H Licensing

Life Insurance Sample Exam 11-14

Page 155: FORWARD - WordPress.com · Return of premium p 6-8 6.0 Annuities p 7-1 Annuity principles and concepts p 7-1 Accumulation period versus annuity period p 7-1 Owner, annuitant and beneficiary

18. Which of the following types of annuity has no accumulation period?

A. Flexible premium annuity C. Deferred annuity

B. Level premium annuity D. Immediate annuity

119. Once a whole life policy is designated as a Modified Endowment Contract and the owner withdraws funds prior to age

59 ½ , a 10% penalty will be assessed. In addition, the amount withdrawn is:

A. Subject to taxation C. Tax exempt

B. Taxable D. A short-term gain

120. Which of the following beneficiary designation means that equal amounts are paid to the children of an insured?

A. Tertiary beneficiary C. Per capita beneficiary

B. Testamentary beneficiary D. Per stirpes beneficiary

121. A penalty-free withdrawal may be made from a Roth IRA prior to age 59 ½ for which of the following?

A. To contribute to a qualified plan C. Estate creation

B. Economic necessity D. Education expenses

122. Interest paid by a policy owner of a whole life insurance contract on a policy loan is:

A. Tax deductible C. Not tax deductible

B. Taxable D. Not taxable

123. When the owner of an annuity contract makes premium payments, the funds are deposited in which of the following?

A. The owner’s account C. The general account

B. The separate account D. The investment account

124. In which of the following ways is an annuity similar to a life insurance policy?

A. Both pay a specified benefit to a beneficiary

B. Both are characterized by prepaid financing

C. Both earn interest that is tax exempt

D. Both may be used to fund a qualified plan

125. Which of the following annuity contracts promises a guaranteed rate for a specified period of time?

A. Variable annuity C. Level premium annuity

B. Market value adjusted annuity D. Whole life annuity

126. An age based penalty will be applied to premature distributions from a Modified Endowment Contract or qualified plan

if the withdrawal is made prior to age:

A. 55 C. 62 ½

B. 59 ½ D. 65

127. Jack and Joe are twins. They inherit a large amount of money from a distant relative. With their share they each buy

an annuity with a life income option. Which of the following is true with regard to the income payments that Jack and

Joe will receive in the future?

A. The income paid to both is tax exempt C. Jack will receive a greater benefit than Joe

B. Joe will receive a greater benefit than Jack D. They will receive the same periodic benefit

NILA ©L/H Licensing

Life Insurance Sample Exam 11-15

Page 156: FORWARD - WordPress.com · Return of premium p 6-8 6.0 Annuities p 7-1 Annuity principles and concepts p 7-1 Accumulation period versus annuity period p 7-1 Owner, annuitant and beneficiary

128. A tax deferred annuity is available to accumulate cash for any reason. If this contract is non-qualified, which of the

following is true?

A. Tax deferral of interest only

B. Tax deferral of interest and deductibility of premiums

C. Tax deductibility of premiums only

D. Tax deductibility interest and premiums

LIFE INSURANCESAMPLE EXAM ANSWERS

1. D 2. C 3. C 4. D 5. B 6. C 7. B 8. A 9. C10. D

11. B12. C13. A14. B15. D16. C17. A18. A19. D20. C21. B

22. A23. B24. B25. B26. C27. C28. D29. C30. D31. B32. C

33. D34. D35. C36. D37. D38. B39. A40. B41. A42. D

43. B44. A45. A46. A47. A48. D49. C50. B51. A52. D53. B

54. A55. C56. B57. C58. B59. C60. B61. D62. D63. C64. B

65. D66. D67. B68. C69. D70. A71. C72. D73. C74. C

75. A76. D77. A78. C79. B80. A81. B82. C83. A84. B85. B

86. D87. D88. C89. D90. B91. D92. B93. C94. D95. C96. C

97. A 98. C 99. A100. D101. B102. A103. C104. D105. C106. A

107. B108. D109. D110. C111. D112. C113. C114. B115. D116. C117. C

118. D119. A120. C121. D122. C123. C124. B125. B126. B127. D128. A

NILA ©L/H Licensing

Life Insurance Sample Exam 11-16

Page 157: FORWARD - WordPress.com · Return of premium p 6-8 6.0 Annuities p 7-1 Annuity principles and concepts p 7-1 Accumulation period versus annuity period p 7-1 Owner, annuitant and beneficiary

SECTION IV

Accident & HealthInsurance

Basics

Page 158: FORWARD - WordPress.com · Return of premium p 6-8 6.0 Annuities p 7-1 Annuity principles and concepts p 7-1 Accumulation period versus annuity period p 7-1 Owner, annuitant and beneficiary

ACCIDENT & HEALTHINSURANCE BASIC

CONCEPTS andPRINCIPLES

Page 159: FORWARD - WordPress.com · Return of premium p 6-8 6.0 Annuities p 7-1 Annuity principles and concepts p 7-1 Accumulation period versus annuity period p 7-1 Owner, annuitant and beneficiary

INTRODUCTION

The field of health insurance is confusing and complex to many. The term "health" insuranceencompasses numerous types of insurance contracts which are each designed to protect an insuredand/or family against various types of risks. This chapter will begin with a review of basic insuranceterms and principles with regard to health insurance.

The term "health" insurance is one that includes accident and health insurance, accident and sicknessinsurance, sickness insurance, disability income insurance, accident only insurance, travel accidentinsurance and many other forms.

The primary forms of health insurance with which the license candidate must be familiar include (1)disability income insurance and (2) medical expense insurance. Disability income insurance involvesthe payment of periodic (i.e., monthly) income benefits when an insured is unable to work and earnan income due to an accident or illness. Medical expense insurance covers the cost of medical careresulting from an accident or illness. In other words, disability income replaces lost income as aresult of illness or injury. Medical expense insurance provides for the payment or reimbursement ofmedical expenses incurred. An injury is covered by a health insurance policy since it is caused by acovered peril known as an accident. Accident and health policies also cover expenses incurred as aresult of a sickness or illness.

In preparation for the state examination, the license candidate must also become familiar with aworking knowledge of health benefits provided by Social Security and Medicare. In addition, thefuture health insurance professional must also recognize the various mandatory and optional policyprovisions which appear in a health insurance contract. Finally, all candidates should possess a soundunderstanding of underwriting principles and responsibilities.

ACCIDENT AND HEALTH INSURANCEBASIC CONCEPTS AND PRINCIPLES

DEFINITIONS

DEFINITION OF INSURANCE — The concept of insurance, in generic terms, may be defined as the transferof risk from one party to another through the pooling or accumulation of funds. This means that one person istransferring the chance of a possible loss to another party. The transfer is accomplished through an insurance contractbetween two parties. The two parties, an applicant and insurer, ultimately arrive at an agreement for insuranceprotection. Insurance helps to reduce uncertainty with regard to possible financial losses. Insurance spreads the riskof loss from one person to a large number of persons through the pooling of premiums. In other words, insurancereduces financial risk. The concept of risk may be identified as the uncertainty or chance of financial loss. A purerisk is the only type that may be insured since it involves a chance of loss only; whereas a speculative risk (i.e.,

NILA ©

L/H Licensing

Chapter 12—1

Page 160: FORWARD - WordPress.com · Return of premium p 6-8 6.0 Annuities p 7-1 Annuity principles and concepts p 7-1 Accumulation period versus annuity period p 7-1 Owner, annuitant and beneficiary

placing a wager on the Super Bowl) provides the opportunity for loss or gain. A speculative risk may not be insuredsince it possesses the chance for loss or financial gain. Again, by purchasing insurance an individual’s financialuncertainty with regard to a possible loss is reduced.

O RISK MANAGEMENT — This is a process used by an insurer of treating and managing loss exposures. An exposure is defined as a hazardous condition brought about by the nature of the activities of an insured.This may be accomplished by: (1) detecting the potential loss exposure; (2) selecting a method or tool toreduce risk; (3) executing a course of action, and (4) periodically reviewing the measures taken. Risk maybe reduced or managed by purchasing an insurance contract. There are many methods of risk managementavailable to handle risk such as avoidance, retention, sharing, reduction and transfer (please refer to page1-14). The primary peril or hazard covered by a life insurance policy is premature death. A peril is thecause of a loss while a hazard is a condition present that increases the chance of a loss occurring (i.e., aslippery floor, icy steps, reckless driver, etc.)

O ELEMENTS OF INSURABLE RISK — It is not possible to insure every type of risk. In order for apure risk to be insurable it must involve a chance of loss that is accidental, measurable and definable. Inaddition, the law of large numbers must also apply. This is a mathematical law of probability that states thatthe larger the number of occurrences (i.e., the number of lives covered), the more predictable losses will be. As the number of exposures increase, the more the actual results will approach the results expected for aspecific event. In other words, the larger the number of homogeneous units (i.e., similar risks), losspredictability will increase. Other elements of insurable risk include the fact that: the loss must be largeenough to cause a financial hardship (i.e., death); the loss must not be catastrophic (i.e., loss due to war);the cost for coverage (i.e., the premium) must not be unreasonable; and the loss must be accidental. Again,methods available to manage risk include but are not limited to: (1) avoidance; (2) retention (i.e., adeductible); (3) reduction; (4) sharing; or (5) transfer. The most common way to manage risk is when anindividual transfers risk to an insurer by purchasing an insurance policy. The concept of insurance mayalso be referred to as an economic or social device used for the transfer of risk from one party to anotherthrough the accumulation or pooling of funds (i.e., premiums).

HEALTH INSURANCE BASICS

HEALTH INSURANCE — Health insurance involves the transfer of the risk from one party (i.e., policyowner/ insured) to another party (i.e., insurer). When a health insurance contract performs (upon the sickness, illness orinjury of the insured), it provides funds to reimburse or indemnify the medical provider or pay a loss of income (i.e.,disability income).

O PARTIES INVOLVED IN THE INSURANCE CONTRACT — There are two parties involved inany insurance contract. The policyowner is responsible for the payment of premiums and possesses allownership rights of the contract. These rights will be reviewed later. The insurer issues the policy once theapplication is approved. When a policyowner is different from the insured party, third party ownership isin existence. For example, a business owning disability insurance on a key employee are illustrations of thirdparty ownership.

O INSURABLE INTEREST — An individual may not purchase insurance covering another person unlesshe or she possesses an insurable interest in that person. This involves a financial or economic interest in theperson to be covered. This doctrine or rule was designed to prevent one person from insuring another andthen profiting from another's illness. Insurable interest must exist at the time of application whereas in

NILA ©

L/H Licensing

Chapter 12—2

Page 161: FORWARD - WordPress.com · Return of premium p 6-8 6.0 Annuities p 7-1 Annuity principles and concepts p 7-1 Accumulation period versus annuity period p 7-1 Owner, annuitant and beneficiary

property insurance it is required to exist at the time of loss. Insurable interest also exists in maritalrelationships, between parents and children, in a business situation between a business and a key employee,or in a debtor-creditor relationship. An individual would possess an insurable interest in a nephew or nieceif the latter lived in the individual's household and he or she was their guardian. Again, the basic requirementwhich must exist when insurance is purchased on another person is insurable interest. Insurable interest isclosely associated to the concept of indemnity. This concept states that the purpose of insurance is to restorean insured to his or her original financial position. This principle makes the insured "whole" again followinga loss.

O ADVERSE SELECTION — The concept of adverse selection is paramount to a health insurer. This isa process that involves the danger to which an insurer is exposed when approving more bad risks than goodones or the tendency of poorer risks seeking insurance. When an applicant who is in poor health applies forinsurance and the insurer's underwriters allow the application to somehow be approved, the insurer will beadversely affected. In other words, if the underwriting department of an insurer does not select risksaccording to insurer guidelines, this action (or lack thereof) may adversely affect the financial condition ofthe insurer. Therefore, sound and competent underwriting may reduce the chance for adverse selection. All insurers attempt to reduce or avoid adverse selection by thoroughly underwriting all individual riskssubmitted. Even some policy provisions attempt to reduce adverse selection after underwriting such as thesuicide clause, the incontestable clause or the pre-existing condition limitation / provision. War, aviationand intentionally self-inflicted injury exclusions also assist in avoiding adverse selection as well.

TYPES OF INSURERS

Insurers may be classified according to domicile, ownership or authorization.

ACCORDING TO DOMICILE — A domestic insurer is one which has its principal or home office in a statewhere it is authorized. In other words, a domestic insurer is chartered in or formed under the laws of the State whereit is authorized. For instance, Nationwide Insurance Company of Columbus, Ohio operates as a domestic insurer inthe State of Ohio. A foreign insurer is one that is authorized in one State but its charter or principal office is inanother State. Nationwide Insurance Company is also licensed or authorized in the State of Illinois. Therefore, itoperates throughout the State of Illinois as a foreign insurer. An alien insurer is one which is authorized in any Statewithin the U.S. but its principal office is located outside this country. Nippon Life of Tokyo, Japan or Sun FinancialServices of Toronto, Canada are examples of alien insurers.

ACCORDING TO OWNERSHIP — A mutual company is one which is owned by its policyholders. A mutualcompany is an organization formed without capital stock that issues participating life insurance policies and mayallow all policyholders to share (i.e., participate) in its surplus in any particular year by receiving a dividend. A stockcompany is one owned by its stockholders. Any profits realized by the stock company are distributed to thestockholders. Stock companies generally do not pay dividends. Therefore, a mutual company and a stock companydiffer according to ownership.

ACCORDING TO AUTHORIZATION — An insurer which is allowed or authorized to conduct insurancebusiness in a particular State is referred to as an authorized insurer. Also known as admitted insurers, an authorizedcompany is issued a certificate of authority. An unauthorized insurer, also referred to as a nonadmitted company,is not permitted nor allowed to conduct insurance operations in a particular State.

NILA ©

L/H Licensing

Chapter 12—3

Page 162: FORWARD - WordPress.com · Return of premium p 6-8 6.0 Annuities p 7-1 Annuity principles and concepts p 7-1 Accumulation period versus annuity period p 7-1 Owner, annuitant and beneficiary

OTHER INSURER CLASSIFICATIONS — There are several additional classifications of insurers includingbut not limited to: (1) Fraternal associations are benevolent societies or social organizations which provideinsurance only for their members. These organizations operate on a lodge system and due to their charitable naturethey are exempt from federal income and state taxes. Organizations such as the Independent Order of Foresters orthe Knights of Columbus are examples of a fraternal; (2) A reciprocal is an unincorporated organization in whichall members insure one another. Reciprocals are managed by an attorney-in-fact (i.e., manager) who attempts tosecure additional subscribers for the organization. Other duties of the attorney-in-fact include collecting premiumsfrom members, administration and adjusting losses; (3) Self-insuring is a method used where an individual orbusiness assumes the total risk of possible losses. An individual may choose to self-insure, or in the case of WorkersCompensation, permission must be granted by a State authority and proof of the ability to self-insure must also beprovided (i.e., financial statements). Self insurance groups are not for profit unincorporated associations generallyconsisting of five or more private or public employers who are engaged in the same trade or professional associationwhich has not been in existence for less than five years, and who enter into agreements to pool their liabilities forWorkers Compensation and other specified forms of liability insurance; (4) risk retention groups provide productliability coverage to businesses that make up the group; and (5) surplus lines insurance is available to those who needprotection but it is not available through private or commercial carriers. "Surplus lines" refers to the nontraditionalinsurance market. A person will seek coverage through a surplus lines broker in order to secure coverage forsubstandard or unusual risks (i.e., hole in one insurance or nonappearance coverage). In order to qualify for surpluslines coverage, an effort has to be made to secure coverage in the authorized market. An individual may not attemptto secure coverage just because it may be less expensive. Surplus lines brokers must also maintain adequatedocumentation of all transactions and a list of the standard insurers who have rejected coverage. A surplus linesbroker is also responsible for collecting a premium tax from the insured in return for placing this business and mustpass this fee along to the appropriate State regulatory authorities.

PRIVATE VERSUS GOVERNMENT INSURERS — Private or commercial insurance companies are ownedby private citizens or groups. They may be proprietary or cooperative. An example of a proprietary insurer wouldbe a profit motivated stock company. Private insurers offer individual, group, industrial or blanket insurance policies. Government insurers are owned and operated by a Federal or State entity. Government insurers may either: (1) writeinsurance to cover perils that are not insurable by commercial insurers (i.e., war, flood, nuclear reaction) or (2) writeinsurance on risks that are insurable and thus compete with the commercial marketplace (i.e., Social Insurance suchas Workers' Compensation). Government insurers generally write insurance to cover catastrophic perils (i.e., flood).

INSURER FINANCIAL STATUS AND RATING SERVICES — All States throughout the country haveas their main objective the regulation of insurers in order to protect the insurance buying public. State insurancedepartments regularly examine (i.e., audit) insurers to ascertain their financial stability. In addition to insuranceregulatory authorities, there are numerous independent rating services which evaluate the financial stability ofinsurers including but not limited to:

! A.M. Best Review (A++ to F)! Moody's Investment (Exceptional to Lowest)! Standard and Poor's (AAA to D)! Weiss Research (Excellent to Very Weak)! Fitch Review

These rating services, in the past, may not have included as much data on an insurer's "risky investments" as it shouldhave. Today, the organization that appears to be the most discerning insurer rating service is Weiss Research.

NILA ©

L/H Licensing

Chapter 12—4

Page 163: FORWARD - WordPress.com · Return of premium p 6-8 6.0 Annuities p 7-1 Annuity principles and concepts p 7-1 Accumulation period versus annuity period p 7-1 Owner, annuitant and beneficiary

INSURER MARKETING AND DISTRIBUTION SYSTEMS — There are four principal marketing systemsused to distribute insurance products to the public. They include (1) Independent agency system which placesbusiness with an assortment of insurers. These independent agents own and control this business. In other words,independent agents own the renewals or expirations of the business; (2) Direct writers are insurers who employcaptive agents that sell the products of the one individual direct writing insurer they represent. The insurer owns therenewals or expirations; (3) Exclusive agency system is a distribution system made up of independent contractorswho agree to market products of one exclusive insurer; (4) Direct mail systems which market their product throughthe mail or media (i.e., newspaper, T.V., radio). The initial three systems are the principal ones involved in themarketing of insurance products.

Insurers manage their distribution systems through the use of the branch office system or the general agency system. These management systems generally provide technical support and evaluate the production of its sales force. Thebranch office system is made up of insurer employees including a branch manager, sales staff and clerical employees. The general agency (GA) system may be of the managing general agent (MGA) type or the personal producing type(PPGA). The MGA generally appoints (i.e., hires) and supervises producers while also performing some servicework such as underwriting and claims. A PPGA does not appoint producers and is generally a personal producer.

PRODUCERS

This moniker refers to the wide range of individuals who solicit insurance products to the public. The various typesof producers include: (1) Agents who represent the insurer that sponsors them; (2) Brokers who represent themselvesand the insured (i.e., the client or customer); (3) Solicitors who represent and solicit insurance on behalf of an agent;and (4) Service Representatives who are employees of an insurer. Service representatives who do not engage in salesactivities which pay commissions need not be licensed. If commissions are paid, the recipient must possess aninsurance license. In some locales any person who solicits or countersigns policies or collect premiums frompolicyowners must be licensed.

CONCEPT OF AGENCY — This is the legal relationship between two parties when one of the parties acts onbehalf of another. This concept does not just apply to the insurance field. However, it may be applied to theinsurance business to illustrate the relationship between an insurer and a producer. For example, this legalrelationship exists when a producer (i.e., an agent) represents an insurer. The insurer is referred to as the principal. The producer may be referred to as the agent or authorized representative. When the producer / agent acts withinthe scope of his or her authority, the insurer or principal is bound or responsible for such action. There are severaltypes of agent authority in existence:

1. ACTUAL OR EXPRESS AUTHORITY — This is the type of authority an agent possesses which isdefined or contained in the agent contract or agreement. When this contract is in force, the insurer isresponsible for the acts of its agents / producers. This agreement specifies the actual powers and functionsof the producer. This type of authority is generally in written form. Producers or agents are allowed to solicitappointments, select or reject risks in the field, complete applications, collect premiums, submitapplications and premiums to the insurer and other various duties. Express authority is extended to aproducer in an agency contract or agreement.

2. IMPLIED AUTHORITY — This type of authority is not specifically identified in the agency agreement. However, it is an extension of a producer's regular duties (i.e., authority exercised by the producer). Anybusiness activities that are usual and customary are implied. For instance, it is implied that a producer who

NILA ©

L/H Licensing

Chapter 12—5

Page 164: FORWARD - WordPress.com · Return of premium p 6-8 6.0 Annuities p 7-1 Annuity principles and concepts p 7-1 Accumulation period versus annuity period p 7-1 Owner, annuitant and beneficiary

solicits insurance is permitted to sign the application, or it is implied that the producer may ask an applicantquestions concerning the health history of the proposed insured contained in the application.

3. APPARENT AUTHORITY — This is the type of authority the insurance buying public perceives theproducer possesses. In other words, the public believes that the agent or producer possesses this form ofauthority.

PRODUCER RESPONSIBILITIES — Any producer is required to act in a fiduciary capacity when collectingpremiums or dealing with the public. This means that all producers possess a fiduciary responsibility when engagingin insurance transactions. The producer possesses the duty to act honestly with applicants for insurance coverage andto represent his or her insurer faithfully. For instance, a producer must account for all funds collected from aconsumer. Therefore, the producer possesses financial and professional responsibilities. Producers must also striveto increase their knowledge with regard to financial services topics. This is why most States now have in placecontinuing education laws to ensure that the producer continually increases his or her knowledge for the benefit ofthe consumer. A producer who has made an unintentional error or honest mistake has committed a tort known asan error and omission. This is why an insurer purchases Errors and Omissions (E+O) insurance to cover themalpractice or negligence of producers. Independent producers or agents purchase their own E+O coverage.

Producers must represent the insurer and their clients in a fiduciary capacity. For example, any premiums collectedby an agent must be submitted to the insurer. They cannot be used for personal use or commingled with a personalaccount. Producers are also required to follow and adhere to the insurer's underwriting rules and guidelines. Unlesspremium notices are billed directly to an insured by an insurer or agency, the agent generally collects insurancepremiums.

PREMIUM COLLECTION AND RESERVES — Producers generally collect premiums from the insured atthe time of application. All future premiums are billed to the insured by the insurer and are remitted by the insuredto the company. Insureds who cannot afford premiums may sometimes use a premium financing organization whichwill function similar to an installment loan. Earned premium is the amount an insurer is entitled to since it providedcoverage for a specific period of time. Unearned premium is an amount that is returned to an insured by the insurersince it was paid and no coverage was provided. The earned and unearned premium make up the total premium. Thereserves of an insurer are funds set aside to pay future claims.

FEDERAL REGULATIONS PERTINENT TO INSURANCE — Federal Regulation 18 USC 1033 and 1034stipulates that anyone who is engaged in the business of insurance whose activities affect interstate commerce andknowingly, with the intent to deceive, makes any false material statement or report, may be subject to a monetarypenalty and imprisonment. Interstate commerce means any business conducted within the District of Columbia, anyState within the U.S. or any of its territories or possessions. The term “State” includes any U. S. State, the Districtof Columbia, the Commonwealth of Puerto Rico, the Northern Mariana Islands, the Virgin Islands, American Samoa,and the Trust Territory of the Pacific Islands.

Any violation of this federal law will subject an individual to a monetary fine of up to $50,000, or imprisonment forup to ten (10) years, or both. In addition, if the material misrepresentation jeopardized the safety and soundnessof an insurer and was a significant cause of the insurer being placed in conservation, rehabilitation, insolvency orliquidation, the agent making the false statements may be subject to imprisonment of not more than fifteen (15)years. In other words, if the insurer’s solvency is threatened due to the material misrepresentations of a licensee, aprison sentence of up to 15 years may be assessed the guilty individual.

NILA ©

L/H Licensing

Chapter 12—6

Page 165: FORWARD - WordPress.com · Return of premium p 6-8 6.0 Annuities p 7-1 Annuity principles and concepts p 7-1 Accumulation period versus annuity period p 7-1 Owner, annuitant and beneficiary

CONTRACT LAW

According to the law of contracts, elements that must be present in order for any contract to be legal and enforceablein a court of law include: (1) offer and acceptance, (2) consideration, (3) competent parties, and (4) legal purpose. If one of these elements is missing, the contract is void.

AGREEMENT — A valid offer and unconditional acceptance must be present in order for the contract to beenforceable. The offer and acceptance is sometimes referred to as the agreement. An offer is a proposal by oneparty that, if accepted by another, will create an agreement. In an insurance contract the "offer" is generally madeby the applicant for insurance. The insurer either accepts or declines (rejects) the offer based on its underwritingcriteria. If the insurer accepts the offer, both parties have arrived at a meeting of the minds which, in effect, is theagreement. Therefore, the acceptance can be demonstrated by the insurer issuing or the producer delivering thepolicy. There must be a genuine assent between the parties which means that neither party must be under duress orany undue influence. If an applicant submits an application without an initial premium, he or she is making aninvitation. The offer is not complete unless the premium is included. If the insurer makes a counter-offer, theoriginal offer made by the applicant has been rejected by the insurer. The original offer is off the table so to speak. No contract will exist unless the applicant accepts the insurer’s counter-offer.

CONSIDERATION — In order for the agreement to be binding, both parties must provide each other withsomething of value. In other words, there must be an exchange of values between the parties to make the agreementbinding. The applicant's / insured's consideration is the premium paid and the representations (i.e., statements) heor she makes on the application. The insurer's consideration is the promise to pay legitimate claims for coverageprovided during the policy period. Sometimes a consideration is referred to as a bargained exchange. Therefore, theconsideration is the binding force in the insurance policy.

COMPETENT PARTIES — The parties (i.e., applicant and insurer) arriving at an agreement must possess the"legal capacity" to enter into that contract. If a person is competent, he or she has such capacity. Most people (andinsurers) are considered competent to enter into a contract with some exceptions including but not limited to:

! Minors, except those entering into agreements for necessities such as food, clothing and shelter. Laws will vary by state as far as determination of "age of majority."

! Insane or mentally incompetent individuals.! Individuals under the influence of alcohol or drugs at the time of application.! Persons forced or coerced to enter into a contract.! Enemy aliens (i.e., a foreign national of a country with which the U.S. is at war).! Convicts (based on state law).

LEGAL PURPOSE — A contract must be in existence with the public interest in mind. Contractual arrangementsmay not be contrary to public policy and must be created in the best interest of the public. Therefore, an organizedcrime "hit" contract is not valid nor is it in the best interest of the public.

DISTINCT CHARACTERISTICS OF AN INSURANCE CONTRACT — Insurance contracts also possessadditional features which distinguish them in the "world of contract law" including but not limited to:

O UNILATERAL —An insurance policy may be described as a unilateral contract since its language isdeveloped by one party – the insurer. Therefore, it is sometimes considered to be a one-sided contract. Anoffer for a unilateral contract generally requires the performance of an act in order for the contract to bebinding. For example, John completes a life insurance application and submits it to ABC Mutual. The

NILA ©

L/H Licensing

Chapter 12—7

Page 166: FORWARD - WordPress.com · Return of premium p 6-8 6.0 Annuities p 7-1 Annuity principles and concepts p 7-1 Accumulation period versus annuity period p 7-1 Owner, annuitant and beneficiary

insurer informs John that it has accepted the offer by the issuance of the policy. The act performed by theinsurer is the "promise" to provide coverage and pay valid claims. A unilateral contract may also be describedas a promise in exchange for an act already performed or an act in exchange for a promise. This also meansthat it is a contract where only one party must perform (i.e. the insurer makes the promise).

O ADHESION —An insurance policy is a contract of adhesion. This means that it is drafted by the insurerand accepted by an applicant. In other words, the insurer "draws up" all the contract language and theapplicant (if he or she desires coverage) must act according to the language of the agreement. Since theinsurer drafts the language of the contract, any language that is unclear or ambiguous that later becomesapparent may be decided by a court in favor of the person who did not draft the language (i.e., the insured). A contract of adhesion is drawn up by the insurer and the applicant has the option of purchasing it or notpurchasing it. The fact that an insurance contract is a contract of adhesion and unilateral makes it "one-sided". The possibility of ambiguities arising in a contract of adhesion involve legal interpretations affectingcontracts. For example, a common legal interpretation of this situation entails the doctrine of reasonableexpectations. According to this doctrine, a court of law will generally state that an insurance contract maybe interpreted by a "reasonable" consumer to mean what the producer or insurer has indicated it means orwhat he or she has interpreted or expected it to mean.

O ALEATORY —An insurance contract is an aleatory contract since one party may recover more in value thanhe or she has parted with based upon a possible future event (i.e., death). The potential value (i.e., claimpayment) received by the insured is generally greater than the value (i.e., premium) received by the insurer. An insurance policy is considered to be aleatory in nature since the insurer promises to pay funds if a coveredloss results. Performance is based upon an uncertain future event involving unequal bargaining value. Inother words, an aleatory contract is one characterized by an unequal payment or consideration. This aleatoryfeature of an insurance contract involves a policyowner obtaining a lot of coverage in return for a small fee(i.e. premium).

O CONDITIONAL —An insurance policy may also be described as a conditional contract since its functionis conditional upon the performance of the parties arriving at an agreement. The insurer promise orperformance (i.e., paying claims) is conditional upon the payment of premium to keep coverage in force. In addition, the policyowner and insurer both must satisfy certain aspects before the contract can beexecuted.

O PERSONAL — A property and casualty contract is between an insurer and a named insured (i.e., a person). Even though it provides protection for a life, a home or a car, the agreement is between the company and aperson.

O UTMOST GOOD FAITH — This means that both parties rely on the statements of one another and mustact in accordance with standards of honesty and sincerity. Insurance policies are contracts of utmost goodfaith.

OTHER CONTRACTUAL DOCTRINES AND PRINCIPLES — An insurance contract may also bedescribed as a contract of utmost good faith. Trust by both parties is important when finalizing the agreement. Theapplicant for insurance must provide to the insurer, full and fair disclosure of the exposure being insured. In otherwords, the risk or exposure that the insurance company is assuming must be the exact risk or exposure that theinsured is transferring. Most insurance contracts are policies of indemnity which means that the policyowner / insuredis made financially whole after a loss is paid. Therefore, the insurer “indemnifies” the policyowner / insured. Therefore, the insured should make truthful statements on the application and refrain from concealing any important

NILA ©

L/H Licensing

Chapter 12—8

Page 167: FORWARD - WordPress.com · Return of premium p 6-8 6.0 Annuities p 7-1 Annuity principles and concepts p 7-1 Accumulation period versus annuity period p 7-1 Owner, annuitant and beneficiary

information from the insurer. Some additional legal principles regarding the utmost good faith of an applicant /insured for insurance coverage include but are not limited to:

O REPRESENTATIONS —These are statements made by an applicant for insurance that are recorded on theapplication. They are generally answers to specific questions on the application regarding the insured andthe exposure (i.e., health history etc.) to be insured. A representation is a statement made by an applicantthat is true to the best of his or her knowledge. All statements recorded on an insurance application are(hopefully) true representations and not warranties.

O MISREPRESENTATIONS — This is a false statement made by an applicant. If an applicant incorrectlystates his or her residence address on an application, this would be considered a misrepresentation — simply, a false fact. Misrepresentations generally do not affect coverage unless the applicant has made falsestatements regarding his or her health status. This would be an indication of a material misrepresentation.

A material misrepresentation involves a material fact that, had it been known by the insurer at the time ofapplication, would have caused the insurer to reject the application. For instance, an applicant for insuranceis asked on the application if he has suffered any illness during the past five years. He answers no when, infact, he has been seriously ill on three occasions in the past two years. This involves a materialmisrepresentation. If the policy is issued and the insurer later learns of the material misrepresentation (i.e.,the lie), it will void the policy. Insurers use this principle in varying degrees and depending on theirunderwriting guidelines.

O WARRANTY — According to Life Insurance, by Black and Skipper, a warranty is "a representation madeby the applicant/insured that is guaranteed to be absolutely true." Such a statement is made part of thecontract and then becomes a warranty — a promise made by the owner / insured upon which coverage isbased.

For example, due to several robberies at a small branch office of a bank, an insurer informs the bank that itwill not provide crime coverage unless the bank installs a new alarm system and hires a guard who willalways be on duty during banking hours. In order to receive coverage, the bank warranties (i.e., guaranteesin writing) that it will comply. This warranty is physically made part of the policy. If the bank is robbedwhen the guard left the premises to buy lunch, the insurer could deny coverage because the warranty wasbreached.

O CONCEALMENT — An applicant for insurance is also obligated to voluntarily disclose any other materialfact concerning the exposure to be insured, even if the insurer and its producer do not ask questions regardingit. The failure to disclose material facts is considered to be concealment. In other words, an insured maynot withhold any material facts that he should realize the insurer would want to know. Concealment isfraudulent in nature and is grounds for voiding the policy. For example, if an applicant withholdsinformation about his health history or status that he should have provided, he has probably engaged inconcealment.

O FRAUD — This involves deliberate or intentional deceit with the objective of making false statements inorder to be compensated by an insurance contract (i.e., filing a false claim).

O WAIVER AND ESTOPPEL — These are two legal principles that may prevent an insurer from denyinga claim. A waiver is the voluntary surrendering of a known right. It may also be defined as "the deliberate,voluntary or intentional abandonment of a known right by the insurer". It usually involves conduct of aninsurer or its sales representative which intentionally relinquishes a defense against a claim. For instance,

NILA ©

L/H Licensing

Chapter 12—9

Page 168: FORWARD - WordPress.com · Return of premium p 6-8 6.0 Annuities p 7-1 Annuity principles and concepts p 7-1 Accumulation period versus annuity period p 7-1 Owner, annuitant and beneficiary

assume that a life insurer issues a policy which states that if the insured enters the military the policy is void. The insured joins the army and is killed during a battle. A company officer informs the insured's beneficiarythat since the insured died in defense of his country the company will waive its defense of military servicedeath. Later the insurer denies the claim. However, they will have to pay the claim since the companyofficer's communication (written or verbal) constitutes a waiver and prevents the insurer from denying theclaim. Another example of an insurer engaging in a waiver is when it mistakenly accepts an incompleteapplication and issues a policy. If the insurer later attempts to rescind the policy or deny a claim becausethe application was incomplete, they will be prevented from doing so since they have engaged in a waiver.Their mistake prevents them from denying the claim or attempting to take back or rescind the policy. Awaiver can also occur if an insurer fails to enforce a provision in the policy. If an insurer finds after it issuesa policy that an individual lied about his health and the insurer does not inform him within a reasonable timethat the contract is going to be void or rescinded, it has engaged in a “waiver by silence.”

Estoppel is a legal principle involves a broken promise. It prohibits an insurer from denying a claim due tospecific actions by the insurer or its representatives or due to inaction by the insurer. One party makes a falserepresentation (i.e., an agent who states that the insured is fully covered). Another party relies on thatstatement (i.e., the insured). Harm then results to the insured (i.e., the insurer attempts to deny the claim). If these three elements are present the insurer will be "estopped" or prevented from denying the claim.

REINSURANCE

This is a device which helps an insurer avoid catastrophic losses. Reinsurance is actually insurance from insurers. In a reinsurance transaction, the insurer seeking reinsurance is known as the ceding insurer. The insurer assumingthe risk is known as the reinsurer. The portion of the risk that the ceding insurer retains is called the net retention(or net line). Transferring the risk to the reinsurer is called "ceding" while the risk itself is referred to as cession. For example, assume that Maine Mutual writes property insurance. Maine Mutual is a small insurer and cannotafford to cover any particular risk for more than $1 million. If an applicant seeks coverage of $5 million, MaineMutual will retain coverage up to its line limit (i.e., $1 million), and will "cede" the excess $4 million of protectionto Richmond Reinsurance Company. Therefore, Maine Mutual can continue to provide protection for its client butit does not assume the entire risk. Reinsurance can be defined as the transfer of risk from one insurance companyto another. There are two basic types of reinsurance including:

FACULTATIVE — Under a facultative agreement, the risks are considered individually by both parties (i.e., cedingcompany and reinsurer). In other words, the ceding insurer may submit the risk to the reinsurer and the latter mayeither accept or reject it on a case by case basis.

AUTOMATIC — Under an automatic or treaty agreement, the reinsurer agrees, in advance, to automaticallyaccept or assume a portion of a risk written by the ceding insurer. Conversely, the ceding insurer has agreed to"cede" a portion of the risk automatically to the reinsurer.

OTHER FORMS — A quota share agreement involves a ceding insurer agreeing to pay a reinsurer a specificpercentage (i.e., 40%) of the premium collected if the reinsurer agrees to pay the same percentage (i.e., 40%) of anyloss that occurs. An excess loss agreement involves the reinsurer agreeing to pay only when a loss exceeds a specificamount.

NILA ©

L/H Licensing

Chapter 12—10

Page 169: FORWARD - WordPress.com · Return of premium p 6-8 6.0 Annuities p 7-1 Annuity principles and concepts p 7-1 Accumulation period versus annuity period p 7-1 Owner, annuitant and beneficiary

TYPES OF LOSSES AND BENEFITS

In addition to potential income losses from disability and hospital and medical expenses losses referred to in theintroduction, other forms of losses involve accidental death and dismemberment (AD&D), dental expenses, long-term care, home health care, accelerated death benefits, vision care and limited or special health exposures (i.e.,cancer only policy). All of these forms of health losses and benefits available to offset such losses will be reviewed.In addition, health insurance policies may be classified in several ways as well including individual plans, franchiseand groups plans, coverage offered by the government or private (i.e., commercial) carriers, limited plans (i.e., thosethat cover one or a few types of loss) and comprehensive plans (i.e., those that cover a broad range of medicalservices such as Blue Cross / Blue Shield).

CONSIDERATIONS IN REPLACING HEALTH INSURANCE — Like life insurance, whenever a producerrecommends to a client that he or she should replace a health insurance policy with a new one, several considerationsmust be weighed and provisions reviewed in order to make a sound decision whether or not to replace an existingpolicy. The policyowner / insured must review each of the following in order to make a more informed decision.

! PRE-EXISTING CONDITIONS — A pre-existing condition is a health condition due to an accident orillness that existed prior to the effective date of a health insurance contract. Most policies include a pre-existing condition waiting period of anywhere from six to twelve months. Insurers are generally notpermitted to include this provision with a waiting period of more than one year. If the pre-existing conditionis severe enough, the insurer may issue an impairment rider which will restrict coverage for the illness orinjury in question for a period of years or for the life of the policy. This provision allows an insurer to denya claim for a health condition existing prior to coverage being provided under a new policy. Many Statesstipulate that insurers must reduce this period if the insured has prior creditable coverage (i.e., HIPAA) orprovide credit to an applicant for a previously satisfied pre-existing condition. However, pre-existingcondition exclusions do not apply in the case of newborns, adopted children (if under age 18) orpregnancies existing on the effective date of coverage. In addition, the use of genetic information as a pre-existing condition is prohibited unless there is a diagnosis of a pre-existing medical condition related to theinformation.

! WAITING PERIOD — A waiting period, also known as the elimination period, is common to most typesof disability income insurance. It functions like a deductible. On each occasion that an insured becomestotally disabled, a waiting period must be satisfied during which no monthly benefit is payable. The longerthe waiting period selected, the lower the policy premium.

! PROBATIONARY PERIOD — A one time event which must be satisfied before a health insurancepolicy's coverage becomes effective. Accidents would be covered immediately but sickness or illnesseswould not be covered until the expiration of a one-time 30 day period. The probationary period helps theinsurer avoid paying claims for illnesses that an insured may have contracted prior to the effective date ofthe policy. In other words, the probationary period allows an insurer to deny small pre-existing claims.

! NO LOSS / NO GAIN PROVISIONS — These are state laws that supposedly prevent a policyownerfrom profiting from the purchase of insurance. They state that the purpose of insurance policies are primarilyto "indemnify" a person for a loss. The principle of indemnity involves making the policyowner "whole"again; or restoring her to the same financial position which was present before an illness.

! BENEFITS, EXCLUSIONS AND LIMITATIONS — Health insurance policies include sectionsdescribing benefits covered and types of losses that are not covered or limited in scope. They also possesslimitations or restrictions with regard to covered losses. For example, a pre-existing condition provision is

NILA ©

L/H Licensing

Chapter 12—11

Page 170: FORWARD - WordPress.com · Return of premium p 6-8 6.0 Annuities p 7-1 Annuity principles and concepts p 7-1 Accumulation period versus annuity period p 7-1 Owner, annuitant and beneficiary

a limitation placed upon the insured for a specified period once he or she is covered by a new policy. Theseareas must be included in an outline of coverage provided to a policyowner or prospective purchaser. Theseareas will be reviewed in a later chapter.

! UNDERWRITING REQUIREMENTS — The function of an underwriting department is to determine if a health insurance risk is eligible for coverage. Sound underwriting will reduce the danger of adverseselection. Following insurer guidelines according to its experience will assist in reducing the chance foradverse selection.

! COMPENSATION LIMITATIONS FOR INTRACOMPANY REPLACEMENT — Regulatorsand insurers place compensation or commissions limits on replacements of existing policies within the sameinsurer. Normally, compensation paid to the producer for the new policy may not exceed the amount ofrenewal commission that he or she is entitled to with regard to the contract being replaced. This regulationis intended to discourage or dissuade a producer from replacing policies simply to receive a new first yearcommission.

! PRODUCER LIABILITY FOR ERRORS AND OMISSIONS — Producers are exposed to liability for negligence in their duties as producer. Insurers purchase errors and omissions coverage in casethey are sued by the insurance buying public due to the negligence or incompetence of their producers. Intentional acts by a producer would not be covered by this coverage.

MANAGED CARE

During the past two decades, the cost of health care has skyrocketed. More and more insurers and other health careproviders have pursued cost containment efforts to control this escalation. Policy benefits have been redesigned,more associations and employers have decided to self-insure to varying degrees and alternative delivery systems haveemerged as a way to control costs. Managed care involves medical expense plans that attempt to control costs by controlling the behavior of participants (i.e., those insured). These plans involve the (prepaid) financing, managingand delivery of health care services by a group of medical providers who share in the financial risk of the plan (orwho have an incentive to deliver cost effective medical services). Managed care plans generally involve several basiccharacteristics including: (1) Controlled access to providers — This encourages or forces subscribers to usepredetermined providers. This utilizes primary care physicians (PCP) who act as gatekeepers to determine thenecessity of care; (2) Case management — This involves reviewing individual cases at all levels to determine thetype of treatment needed, monitoring ongoing care and determining the success of the treatment provided; (3)Preventive care — This type of care attempts to prevent or detect medical conditions before symptoms appear; (4)Risk sharing — This involves the plan and the providers sharing in the financial consequences of medical decisions(i.e., a doctor gets a bonus for not ordering unnecessary tests); and (5) Quality care — The plan will carefully selectmedical providers who are competent and experienced. These features allow an insurer or provider organization toexert influence over the delivery, use and costs of services in order to achieve the purpose of managed care whichis to control health insurance costs and claim expenses.

A type of managed care plan in existence is an HMO. Many of these organizations do not provide coverage fortreatment outside of the managed care network since they required subscribers to utilize the services of pre-approvedproviders. Many consumers have shied away from these organizations because of a lack of choice of providers. However, the popularity of managed care plans increased since insurers and employers recognized that the escalatingcost of health care must be controlled. Fee for service plans and other types of indemnity plans may not includeprovisions or mechanisms geared to containing costs. Therefore, many insurers are currently committed todeveloping and maintaining managed care programs.

NILA ©

L/H Licensing

Chapter 12—12

Page 171: FORWARD - WordPress.com · Return of premium p 6-8 6.0 Annuities p 7-1 Annuity principles and concepts p 7-1 Accumulation period versus annuity period p 7-1 Owner, annuitant and beneficiary

TYPES OF PROVIDERS

Health insurance providers include but are not limited to commercial insurance companies, health maintenanceorganizations, preferred provider organizations, Blue Cross and Blue Shield plans, employer-sponsored plans andgovernment insurers. Commercial life, health and casualty insurers provide health insurance coverage on a traditionalfee-for-service basis. This is a billing method for health services where a provider charges separately for each servicerendered. Most physicians, hospitals and clinics bill on this basis. Commercial insurance company’s fee for serviceplans generally allow an insured to choose health care from any qualified provider. This is not the case with regardto HMOs, Blue Cross / Blue Shield or managed care where there may be a limited choice of providers.

Some policies issued by commercial insurers function on a reimbursement basis and require a prepaid premium (i.e.,prepaid financing). In other words, an insured is paid directly or reimbursed by the insurer after the submission ofa claim form. The insured may also assign benefits payable to a hospital or physician who provided the medical care. This may also be referred to as an indemnity basis as well. A prepaid plan may also be described as one where healthcare providers are paid for medical services in advance, whether or not any are provided. The amount paid to theproviders is based upon the projected annual cost determined by the provider. A traditional service type of provider(i.e., Blue Cross) pays covered amounts directly to the hospital or physician on a usual, customary and reasonable(UCR) charge basis. Any remaining balance is billed to and paid by the insured. Usual, customary and reasonablecharges refer to the standard range of fees that a physician charges. This may be contrasted with policies thatprovide benefits on a scheduled basis, such as surgical expenses. Traditional forms of medical expense insuranceprovided by commercial carriers will be reviewed in Chapter 15. Additional providers of health insurance will bereviewed here which include but are not limited to:

BLUE CROSS / BLUE SHIELD (BC/BS) plans are community or service organizations that provide health careto subscribers (i.e., members). Blue Cross covers hospital charges on a UCR basis including but not limited to: room and board, lab fees, diagnostic testing, anesthesia charges, physical therapy, chemotherapy, outpatient testing,and skilled nursing care. Blue Shield covers physicians’ fees. BC/BS plans may be written on an individual or groupbasis.

Blue Cross and Blue Shield plans function on either a for-profit or nonprofit basis depending upon State legislationin a particular locale. They receive favorable tax treatment as well since many States exempt them from premiumand income taxation. BC/BS organizations enter into written agreements with physicians and hospitals to providemedical care and services to subscribers (i.e., members). Through these contracts they agree to reimburse providersfor medical services dispensed although they do set limits on the benefits provided. BC/BS offers coverage on abasic and major medical expense basis. In addition, coverage may be offered through HMO's as well. Individual andgroup plans are offered to the insurance buying public. Generally, an insured or subscriber is allowed to choose hisor her own health care providers. The BC/BS organization is also required to disseminate to the subscriber allpertinent information with regard to the operation and procedures involved in securing care. While the corporatestructure of BC/BS plans differs from commercial health insurance companies with regard to delivery and favoredtax status, the nature of benefits and types of coverage offered are similar. Premium rates for these plans, likeother health care insurers, will vary based upon the benefit selected by the insured. The primary types of benefitsoffered by BC/BS plans are similar to other health care insurers such as basic and major medical and dental expenses. Limitations and exclusions among all health insurance companies or organizations are generally very similar. Again,those insured by a BC/BS plan are referred to as a subscriber, participant or member.

Sometimes referred to as a nonprofit hospital service corporation, these plans generally include an open enrollmentperiod of at least two months. The method by which the “Blues” reimburse the health care provider generallyresults in an advantage compared to insurance companies. With regard to hospitalization, most plans pay a

NILA ©

L/H Licensing

Chapter 12—13

Page 172: FORWARD - WordPress.com · Return of premium p 6-8 6.0 Annuities p 7-1 Annuity principles and concepts p 7-1 Accumulation period versus annuity period p 7-1 Owner, annuitant and beneficiary

participating hospital an amount per day (i.e., per diem) for every day a member is confined to a hospital no matterthe actual charge. Therefore, the BC/BS plan will make money on some confinements but lose money on others.

HEALTH MAINTENANCE ORGANIZATIONS (HMO) are regarded to be organized systems of health carethat provide comprehensive health care services to enrollees (i.e., members) living within a specified region. Formerly referred to as prepaid group practice plans, a prepaid premium must still be paid and members are thenallowed to seek treatment when needed at an HMO facility. HMOs do not function on a reimbursement basis. Amember provides proof of enrollment to the facility, pays a small co-payment (i.e., $10, $20, etc.), and then receivesneeded treatment. The medical services are provided at the facility. No claim forms need be processed. Therefore,HMO'S combine health care delivery with prepaid financing. There are several characteristics that distinguishHMOs from traditional forms of medical expense insurance. As mentioned previously, an HMO offers members avariety of comprehensive health care services including outpatient services and hospitalization.

In addition, HMOs emphasize preventive care which was not a covered benefit by traditional insurers nor the Blues. Preventive care attempts to detect medical problems before symptoms appear. This includes such services as routinephysical examinations, immunizations, mammograms, pap smears, well baby care and family planning services. Other basic benefits that must be provided by an HMO include:

1. Physician’s services2. Inpatient and outpatient hospital services (usually requires subscribers to use the HMO facility)3. Emergency services4. Limited outpatient mental health services5. Treatment for drug and alcohol abuse or addiction6. Diagnostic lab and radiological services7. Home health care services (some plans offer this service as an option)8. Medical social services

Emergency care is covered by health maintenance organizations. This type of medical care is also covered if evenif provided by a non-plan physician or hospital as long as it is necessary (i.e., urgency). Hospital services are alsocovered in full for the length of a stay. Optional coverages may also be added and require additional premiumincluding intermediate and long-term care (nursing home) services, vision care, dental care, prescription drugs andpharmacy services or rehabilitative services. Some HMOs offer home health care as an option as well.

For the most part, HMOs provide for the delivery of medical services by salaried physicians and other personnel (i.e.,nurse practitioner). This may be contrasted with a commercial insurance plan that traditionally functions on a feefor service basis. Subscribers are required to seek treatment from providers affiliated with the plan. HMOs offermedical services to those living in specific geographic boundaries or territories (i.e., county). This organization alsoemphasizes treatment by a primary care physician (PCP) as well. A PCP acts as a gatekeeper to control access tospecialists. Generally, the HMO only covers the cost of specialist services if referred by the PCP. HMOs alsopractice utilization review and other cost savings methods, such as second surgical opinions, to attempt to keep costsdown. If the HMO is provided for a group, the employer must allow any participating HMO to have access to eligibleemployees during an open enrollment period. A disadvantage of an HMO is that it limits the choice of physiciansand the areas where care can be received.

HMOs have been formed by employers, labor unions, private investors, insurers, physicians, hospitals and medicalschools. There are several types of HMOs. They may be structured or formed on a group practice plan basis whichis also known as a closed-panel plan. This type of plan is characterized by doctors who are directly contracted withor employed by the HMO or they are employed by a provider who is contracted with the HMO. The subscriber isrequired to use the physicians who are employed by the HMO. HMO's also function on an individual practice

NILA ©

L/H Licensing

Chapter 12—14

Page 173: FORWARD - WordPress.com · Return of premium p 6-8 6.0 Annuities p 7-1 Annuity principles and concepts p 7-1 Accumulation period versus annuity period p 7-1 Owner, annuitant and beneficiary

association basis. These plans offer more flexibility with regard to subscribers selecting physicians and are alsocalled open-panel plans. Many physicians participating in this type of plan provides services at their own office. They are able to treat any HMO subscribers and any other patients who are not HMO subscribers. The latter, ofcourse, are treated on a fee-for-service basis. Other HMO's function on an open-ended basis. Do not confuse thiswith open-panel. An HMO that is open-ended means that up to 10% of the medical services it provides may bedelivered by physicians who are not affiliated nor formally contracted with the HMO. HMO's may also operate ona for-profit or nonprofit basis. To summarize:

! An HMO is an alternative to traditional commercial health care insurance. Both HMOs and commercialinsurers finance health care (i.e., collect premiums). However, an HMO also delivers the health care aswell. An HMO's objective is to reduce the cost of health care by providing services which are aimed atpreventing serious illness. By providing early treatment through various testing modes and regularexaminations, the cost of health care may be reduced. One of the primary objectives of an HMO is to providepreventive care services. Like most private carriers, an HMO also provides hospital services, emergencycare and other basic health services.

! An HMO, like commercial insurers, provides comprehensive health services in return for a stipulated prepaidpremium. In many cases, all services are provided at the HMO facility. Many HMOs own their ownhospitals. This type of operation is different from the traditional "fee-for-service" approach used by doctorsand hospitals (i.e., reimbursement basis). Therefore, HMOs do not provide coverage on a reimbursement(i.e., indemnity) basis. HMO premiums are generally higher than other providers because of the compre-hensive nature of the services and benefits provided. Members or subscribers make a small co-payment whenseeking treatment. Normally, members will be provided with a list of physicians to choose from and onlythose on the list who participate in the HMO may be used by subscribers.

! Most HMOs are group practice plans. Sometimes referred to as closed-panel types, they employ memberphysicians who are paid a salary for their services. Individual practice plans or open panel types allowmember physicians, to provide in their own office, medical services to subscribers. They then bill the HMOon a fee basis for the subscribers treated. These physicians are not employees of the HMO. Therefore,subscribers may choose from a list of participating physicians. Many HMOs require that members select aprimary care physician (PCP) as a “gatekeeper.” The physician acts as a gatekeeper to control utilizationand refer members to other physicians who are specialists within the HMO. Keeping members away fromhigher cost specialists, unless its necessary, helps control costs.

! Types of sponsors for HMOs include consumer groups, health care providers, insurance companies, BlueCross/ Blue Shield, and labor unions. Plan designs include comprehensive care, including basic andsupplemental benefits. Basic benefits include doctor services, emergency services, inpatient and outpatientcare, lab and other testing fees and additional preventive services all designed to reduce the cost of medicalcare. Supplemental or optional benefits include dental and vision care, home health and long term care,prescription medicines, physical and speech therapy, mental health and alcohol related treatment and footcare. Usual exclusions appearing in medical expense policies are also present in HMO coverage. A primarycare physician is generally selected by the covered subscriber / member (i.e., insured). Employer groupplans may function with a dual choice provision and an annual open enrollment period for new prospectivemembers. The dual choice provision is placed upon employers who employ 25 or more workers. If an HMOis functioning in the area and the employer is providing health coverage to employees, it must offer at leastone HMO option to its employees. An open enrollment period must be held by HMOs on an annual basisfor individuals who desire coverage. The open enrollment is available for anyone regardless of healthhistory.

NILA ©

L/H Licensing

Chapter 12—15

Page 174: FORWARD - WordPress.com · Return of premium p 6-8 6.0 Annuities p 7-1 Annuity principles and concepts p 7-1 Accumulation period versus annuity period p 7-1 Owner, annuitant and beneficiary

PREFERRED PROVIDER ORGANIZATIONS — A PPO functions very similarly to an HMO in some waysbut vastly different in others. A PPO is an organization, generally administered by a third party administrator (i.e.,TPA), which enters into a contractual arrangement with doctors or hospitals (i.e., providers) in order to securemedical coverage for its insureds (i.e., members). A PPO is not a traditional comprehensive health care provider. The provider is the physician or hospital "providing" medical services. Physicians are paid a fee for the services theyprovide. Covered insureds may choose any primary care physician (i.e., gatekeeper) they desire for treatment. Theprovider (i.e., doctors) offers to discount medical service fees while organizations promise to increase patientvolume. One of the objectives of a PPO is to channel patients to providers that discount medical services. Therefore,to entice more members to participate, the organization may offer concessions with (i.e., lower) deductibles or coin-surance provisions. PPOs provide health care through its network but also allow members to seek treatment out ofthe network if desired. This aspect of a PPO functions on an indemnity basis (i.e., reimbursement). However,coverage and benefits may be reduced when treatment is sought outside the network. Therefore, a range of serviceis provided rather than comprehensive type care as in an HMO.

Some PPOs may include the same gatekeeper concept as an HMO. These gatekeeper PPOs require that a subscriberselect a primary care physician in the same manner as an HMO. The doctor then manages care through utilizationreview and specialist referrals. However, subscribers may seek treatment outside the network and still receivecoverage, although the benefits are reduced.

PPOs may function on an open panel or closed panel basis. Multiple option plans may also be available. These plansare generally sponsored by a single provider who offers the employer a variety of plans characterized by loweradministrative costs. In other words, they provide a choice of physicians and hospitals to the member or subscriberas well as medical services at a reduced cost. PPOs may be run by Blue Cross/Blue Shield, by an insurance companyor as a result of an employer-employee relationship. New providers may be added to the plan if they agree to followPPO standards and charge the applicable fees. The arrangement or contract between the individual physician andthe preferred provider organization is called a PPA — Preferred Provider Arrangement. All PPAs must conform withstate law and generally must be approved by the Division or Department of Insurance in any state where it operates. Again, a member may seek treatment outside a PPO network. However, the coverage and benefits may be reduced.

A new type of managed care arrangement in existence today is a point-of-service plan (POS). A POS is a hybridarrangement that combines aspects of a traditional HMO and a PPO. These plans are characterized by a higherdegree of managed care than that provided by a PPO. In this plan, members elect whether to receive treatment withinthe network or outside the network. Subscribers are allowed to use the services of physicians within an HMO oroutside the plan. If a member seeks treatment outside the network from a nonmember physician, the doctor will bepaid on a fee for service basis. In addition, the benefits may be reduced as in the previously described PPO. Inaddition, some POS plans require that a member inform the POS that they are going to seek treatment outside thenetwork. Then the PCP can attempt to persuade the member to seek treatment inside the network. However, if themember decides to look for outside treatment it is permitted. There are two types of POS plans including an open-ended HMO type and a gatekeeper PPO type.

EMPLOYER ADMINISTERED PLANS — These plans provide coverage for employees and their dependents. Employers administer and manage their health related exposures by using commercial insurers or other entities whichprovide health care services such as HMOs or PPOs. Employer plans may be self-funded or self-insured. Underthis approach, the employer retains a portion of the exposure by self- insuring up to a specified limit (i.e., dollaramount). For instance, the employer may self-insure up to a stop-loss limit of $2,000 per employee. This meansthat the employer pays the first $2,000 of medical expenses. Any additional (i.e., excess) medical expenses arecovered by the insurer. Self-insuring allows the employer to reduce the cost of providing health care to employees. In order for self-insuring to work adequately: (1) the group or employees must be large enough; (2) an agreementmust be in place for the insurer to cover the excess charges over and above the stop-loss limit; and (3) there must be

NILA ©

L/H Licensing

Chapter 12—16

Page 175: FORWARD - WordPress.com · Return of premium p 6-8 6.0 Annuities p 7-1 Annuity principles and concepts p 7-1 Accumulation period versus annuity period p 7-1 Owner, annuitant and beneficiary

a third party administrator (TPA) to manage claims. Employers who wish to self-insure must also provide financialproof and other statistical evidence that they are economically sound.

MULTIPLE EMPLOYER TRUSTS / MULTIPLE EMPLOYER WELFARE ARRANGEMENTS(METS/MEWAS) — The multiple employer trust (MET), also known as a multiple employer welfare arrangement(MEWA), has recently provided a vehicle in which to secure group health insurance coverage for small businesses. METs actually provide a mechanism for the employees of small businesses with common employments to bandtogether to purchase more affordable group health insurance. Due to the increased cost of medical care andinsurance premiums, it has been extremely difficult for small companies to afford needed coverage.

! METs have provided small companies with a means with which to secure health coverage. For example,several shoemakers may be unable to afford the cost of an individual group health plan offered by an insurer. If these shoemakers band together to become a larger group, they may secure coverage through a MET at amore advantageous premium rate.

! METs may be sponsored by an insurance company and therefore are considered to be (fully) "insured"METs. Some are administered by a third party (i.e., TPA) and others may be uninsured or self-funded. Thislast type of MET or MEWA is not yet subject to state regulatory authority in many locales.

! MEWAs or METs may involve groups of employers who pool risks in order to self-insure. Some may becomprised of individuals or firms from similar industries. An employer wishing to secure coverage for itsemployees from a MET or MEWA must subscribe and become a member. The employer is issued a joinderagreement which spells out the relationship between the MET / MEWA and the employer and specifies thecoverages to which the employer has subscribed.

GOVERNMENT INSURERS — The government provides health insurance as well. Social Security helps tofund programs providing health care such as Medicare. Federal and State governments provide funds for Medicaid. State governments generally administer the Medicaid programs and provide disability benefits through Workers'Compensation programs. These areas will be reviewed in greater detail in a later chapter.

! CHAMPUS — This acronym stands for the Civilian Health and Medical Program of the UniformedServices. According to the CHAMPUS booklet published by the U.S. Government, this is "a governmentorganization that provides health care benefits for the families of members of the armed services." Covereddependents include those of active service members, retired service members, deceased service members orretired service members themselves. Active duty service members who are eligible for Part A of Medicareare not eligible.

! TRICARE — Tricare is a health benefit program for all seven uniformed services and their dependentsincluding the Army, Navy, Air Force, Marines, Coast Guard, Public Health Service and the National Oceanicand Atmospheric Administration. This is a regionally managed program for active duty and retired membersof the uniformed services and their families and survivors. Tricare offers eligible persons three choices fortheir health care including: (1) Tricare Prime where military treatment facilities are the principal source ofhealth care; (2) Tricare Extra which is a preferred provider option that saves money; and (3) Tricare Standardthat operates on a fee for service option (this is the Champus coverage).

NILA ©

L/H Licensing

Chapter 12—17

Page 176: FORWARD - WordPress.com · Return of premium p 6-8 6.0 Annuities p 7-1 Annuity principles and concepts p 7-1 Accumulation period versus annuity period p 7-1 Owner, annuitant and beneficiary

QUIZ

1. Which of the following provides for the availability of health insurance benefits for small companies with common

employments which band together?

A. Multiple funded plans C. Multiple employment associations

B. Multiple employer trusts D. Long term care associations

2. Which of the following supports the theory of indemnity?

A. Exclusions C. The law of large numbers

B. Reductions in coverage D. No loss, no gain statutes

3. A waiting period in a disability income policy is also referred to as:

A. Probationary period C. Recurrent period

B. Elimination period D. Waiver period

4. All of the following are true regarding an HMO, EXCEPT:

A. Requires a prepaid premium

B. Focuses on preventive care

C. Functions on a reimbursement basis

D. Requires co-payment before service is provided

5. What is the function of a third party administrator?

A. To market insurance products C. To issue insurance polices

B. To process commissions D. To manage claims

6. An insurer that is chartered in the State of New Jersey and is authorized in Pennsylvania is referred to in the latter State

as which of the following?

A. Domestic insurer C. Alien insurer

B. Foreign insurer D. Nonadmitted insurer

7. An authorized company whose principal office is located in this State and operates in this State is known as:

A. A domestic company C. An alien company

B. A foreign company D. A nonadmitted company

8. An insurer owned by its policyholders without capital stock best describes?

A. A stock company C. An alien company

B. A mutual company D. A reciprocal company

9. A reciprocal is managed by which of the following?

A. A branch manager C. A fraternal

B. A company officer D. An attorney-in-fact

NILA ©

L/H Licensing

Chapter 12—18

Page 177: FORWARD - WordPress.com · Return of premium p 6-8 6.0 Annuities p 7-1 Annuity principles and concepts p 7-1 Accumulation period versus annuity period p 7-1 Owner, annuitant and beneficiary

10. A fraternal offers various types of insurance programs for which of the following?

A. For those who cannot obtain standard coverage

B. For those over age 65

C. For their members only

D. For labor unions and their members

11. Direct writers employ:

A. Captive agents C. Professional insurance agents

B. Independent agents D. Independent contractors

12. Each of the following are supplements or optional coverages that may be added to HMO coverage, EXCEPT?

A. Home health services C. Long-term care

B. Emergency treatment D. Pharmacy services

13. A legal relationship between two parties when one of the parties acts on behalf of another is known as:

A. Authority C. Fiduciary capacity

B. Agency D. Declaratory judgement

14. Which of the following types of authority is an extension of a producer’s regular duties?

A. Express C. Apparent

B. Implied D. Warranted

15. All of the following are legally competent to enter into an insurance contract, EXCEPT?

A. An individual who is past retirement age

B. An emigre who has lived in this country for five years

C. An individual recently laid off from his job

D. A twelve year old girl

16. Which of the following best describes insurance?

A. The transfer of risk

B. The increase in the chance of loss

C. The uncertainty of loss

D. The pooling of funds in order to receive appreciation

17. Which of the following best identifies the parties involved in an insurance contract?

A. Beneficiary and policyowner C. Producer and policyowner

B. Insurer and producer D.. Policyowner and insurer

18. The goal of a Preferred Provider Organization is:

A. Make health care available to all

B. Channel patients to providers that discount medical services

C. Reduce the preexisting condition period of a new health plan

D. Limit the cost of health care paid by employers

NILA ©

L/H Licensing

Chapter 12—19

Page 178: FORWARD - WordPress.com · Return of premium p 6-8 6.0 Annuities p 7-1 Annuity principles and concepts p 7-1 Accumulation period versus annuity period p 7-1 Owner, annuitant and beneficiary

19. If a POS member seeks medical care from a nonmember physician, payment is made to the provider on a:

A. Reimbursement basis C. Fee for service basis

B. 80 / 20 basis D. Coinsurance basis

20. Which of the following is not a type of preventive care?

A. Family planning C. Well baby care

B. Chemotherapy D. Prescription drugs

21. Each of the following must be present in order for an insurance contract to be legal and enforceable, EXCEPT:

A. Offer C. Competent underwriting

B. Acceptance D. Legal purpose

22. If one party makes an offer and another party makes a counter offer, the initial offer is:

A. Accepted C. Negotiated

B. Off the table D. A stalemate

23. An insurance contract is created by an insurer. Any ambiguities that appear in the future will be decided by a court in

favor of the insured since the policy is:

A. A contract of adhesion C. A unilateral contract

B. An aleatory contract D. A personal contract

24. The statements made by an applicant in an insurance application are considered to be:

A. Warranties C. Misrepresentations

B. Representations D. Countersignatures

25. A statement made by an applicant for insurance that is guaranteed to be absolutely true and is made part of the contract

is called a:

A. Representation C. Consideration

B. Conditional promise D. Warranty

26. Which of the following is considered to be a voluntary relinquishment of a known right?

A. Warranty C. Waiver

B. Estoppel D. Parole evidence

27. Deliberate or intentional deceit in making false statements is known as:

A. Larceny C. Fraud

B. Rebating D. Misrepresentation

28. Each of the following is a characteristic of an insurance contract, EXCEPT?

A. Aleatory C. PersonalB. Adhesion D. Trilateral

NILA ©

L/H Licensing

Chapter 12—20

Page 179: FORWARD - WordPress.com · Return of premium p 6-8 6.0 Annuities p 7-1 Annuity principles and concepts p 7-1 Accumulation period versus annuity period p 7-1 Owner, annuitant and beneficiary

29. Which of the following is the purpose of an accident and health policy's probationary period?

A. Broadens policy benefitsB. Allows for open enrollmentC. Allows an insurer to deny small pre-existing claimsD. Determines the lost ratio for an insurer

30. Several elements must be present in order for an insurance contract to be enforceable. One required element is that allparties be competent. Each of the following is competent to contract, EXCEPT:

A. Minor contracting for necessities C. An officer of an insurerB. An insured suffering a serious illness D. An individual under duress

31. An insured who is covered by a Blue Cross/Blue Shield plan is known as which of the following?

A. A beneficiary C. A subscriberB. A policyholder D. A provider

32. Each of the following is excluded by Blue Cross/Blue Shield coverage, EXCEPT:

A. Injury as a result of committing a felony C. Intentionally self-inflicted injuriesB. Dental care resulting from an accident D. Routine examination costs

33. If a person covered under a PPO seeks treatment outside the plan, what action might the organization take?

A. Increase the premium for coverage C. May reduce benefitsB. Expel the insured from the group D. Increase the number of exceptions

34. What is issued to an employer when it desires to seek coverage for employees through a MET or MEWA?

A. Satisfactory agreement C. Joint ventureB. Joinder agreement D. Summary judgment

35. In which of the following HMOs is a physician considered to be an employee?

A. A close corporation C. An open panel planB. A managed care plan D. A closed panel plan

36. Under which of the following organizations are the medical providers paid on a fee-for-service basis?

A. A health maintenance organization C. Blue Cross organizationB. A custodial care organization D. A preferred provider organization

NILA ©

L/H Licensing

Chapter 12—21

Page 180: FORWARD - WordPress.com · Return of premium p 6-8 6.0 Annuities p 7-1 Annuity principles and concepts p 7-1 Accumulation period versus annuity period p 7-1 Owner, annuitant and beneficiary

ANSWERS

1. B 2. D 3. B 4. C 5. D 6. B 7. A 8. B 9. D10. C11. A12. B

13. B14. B15. D16. A17. D18. B19. C20. B21. C22. B23. A24. B

25. D26. C27. C28. D29. C30. D31. C32. B33. C34. B35. D36. D

CHAPTER 12 KEY CONCEPTS

AdhesionAleatoryBlue Cross / Blue ShieldCHAMPUSConditionalCompetent PartiesConsiderationContract LawElimination periodEmployer plansErrors and omissionsEstoppelExclusionsHealth benefitsHealth lossesHMOLegal purpose

Legal capacityMET/MEWANo loss/no gainOffer & AcceptanceOpen panel plansPPOPoint of Service plansPreventive care Probationary periodReplacementRepresentationTricareUnderwritingUnilateralWaiting periodWaiverWarranty

NILA ©

L/H Licensing

Chapter 12—22

Page 181: FORWARD - WordPress.com · Return of premium p 6-8 6.0 Annuities p 7-1 Annuity principles and concepts p 7-1 Accumulation period versus annuity period p 7-1 Owner, annuitant and beneficiary

MARKETING,UNDERWRITING andPOLICY DELIVERY

Page 182: FORWARD - WordPress.com · Return of premium p 6-8 6.0 Annuities p 7-1 Annuity principles and concepts p 7-1 Accumulation period versus annuity period p 7-1 Owner, annuitant and beneficiary

INTRODUCTION

The underwriting policy carried out by an insurer directs the decisions made by the individualunderwriter. Underwriters not only determine whether to accept or reject a risk but they also classifyrisks, determine premiums and attach impairment riders or other limitations.

MARKETING, UNDERWRITING,AND POLICY DELIVERY

MARKETING OF HEALTH INSURANCE

Health insurance, like life, property and liability insurance is marketed to individuals, families and groups throughthe independent agent system, the direct writer system or direct response system (i.e., direct mail, junk mail). Allinsurers must comply with State laws with regard to the marketing and advertising of its health insurance products.Specific regulations govern the sales presentations utilized by producers. In most States, prepared sales presentationsare considered to be a form of advertising and thus fall under State advertising law and guidelines. In addition, mostStates require that an outline of coverage be provided to insureds / policy owners which summarizes the principalbenefits, limitations and exclusions that are contained in the health insurance policy.

UNDERWRITING CRITERIA

The underwriting department must acquire as much information as possible in order to make a sound decisionwhether to accept or reject an individual application for health insurance. The information available to anunderwriter that is principally used to determine eligibility for coverage comes from the health insurance application.The function and purpose of this department is to review applications for insurance coverage to determine if theproposed insured is insurable. If an application is acceptable it is safe to conclude that the underwriter feels that theinsurer will derive a profit from this business. If it is rejected, it is generally due to the fact that the risk is excessiveas compared with the premium that would be charged. In other words, the business written would not be profitable. Underwriting procedures for life and health insurance are very similar. The underwriting process begins with fieldunderwriting performed by the producer. The purpose of this action involves, primarily, the producer recordinganswers to questions asked of an applicant that are recorded on the application. The initial contact with a potentialpolicyholder begins in this phase. The insurer hopes that the producer will submit good and profitable risks tounderwriting. It is extremely important that the producer disclose all information regarding the applicant and / orthe proposed insured to the insurer in order to properly underwrite the risk. Each insurer must also conform with stateand federal laws regarding the dissemination of an applicant’s or insured’s private information according toconsumer privacy regulations (i.e. Privacy Act).

SOURCES OF UNDERWRITING INFORMATION — The underwriting department of a health insurancecompany receives information with which to make its decision whether to accept or reject a particular applicant. Noinsurer is permitted to engage in any unfair discrimination regarding applicants for health insurance. All of the

NILA ©

L/H Licensing

Chapter 13—1

Page 183: FORWARD - WordPress.com · Return of premium p 6-8 6.0 Annuities p 7-1 Annuity principles and concepts p 7-1 Accumulation period versus annuity period p 7-1 Owner, annuitant and beneficiary

following are sources of information available to an underwriter including: (1) insurance application; (2) agent'sreport; (3) physical or medical examination; (4) attending physician statement (APS); (5) consumer reports; or (6)medical information bureau reports.

! INSURANCE APPLICATION — The basic or principal type of underwriting information comes fromthe application. The application is made up of fact finding (i.e., name, address, etc.) questions and healthinsurance questions. The basic information provided on the application will enable the underwriter to decideif he or she will accept or reject the risk or require additional (i.e., health) information. When the applicationis being completed, the producer must inform the applicant of the notice of information practices such asthe Fair Credit Reporting Act. The application should be filled out completely and accurately. It should besigned by both the producer and the applicant. If the applicant is different from the person to be insured, bothmust sign. Any changes made to the application by the producer before it is submitted to underwritingmust be initialed by the applicant as well. An initial premium should be collected and submitted along withthe application.

! PRODUCER OR AGENT'S REPORT — The underwriter will also be provided with information fromthe agent's report. This report is generally located in the application. The producer or agent will provideany additional information at his or her disposal with regard to the applicant in this section of the application. This report will include any other information that does not appear on a previous part of the application.

! MEDICAL OR PHYSICAL EXAMINATION — The main focus of the application is the health of theapplicant. In some cases, the underwriter will determine that because of the (questionable) health status ofthe applicant, additional medical information is needed in order to make a final decision. Therefore, theunderwriter requests that the applicant submit to a physical or medical examination. Medical question-naires, lab tests, blood tests (i.e. HIV and AIDS) and genetic testing may also be used to acquire moreinformation. However, in most States insurers are prohibited from using genetic information with regardto underwriting.

! ATTENDING PHYSICIAN STATEMENT — When an underwriter desires additional healthinformation regarding an applicant, he or she may request an attending physician statement (APS). Thephysician of the applicant will complete a form that describes the current health status of the applicant.

! CONSUMER REPORT — A consumer or investigative inspection report may be requested by anunderwriter which will provide additional information regarding the applicant. These reports may berequested when an individual applies for any type of insurance. If a consumer report is completed, theapplicant is permitted to request a personal interview to review the findings of the report. An agent mustinform an applicant at the time of application that the insurer has the option of conducting a consumer reporton the applicant if it wishes.

! MEDICAL INFORMATION BUREAU (MIB) — The MIB is a clearinghouse of medical information. Most insurers subscribe to this provider of information. The MIB maintains computer files describing thephysical condition of applicants who have previously applied for health insurance with subscribing insurers. Therefore, an insurer can compare the health insurance application information of an applicant with thatsame applicant's health history appearing in previous applications submitted to any other insurer whosubscribes to the service. The MIB information does not include information from the applicant's personalphysician. It only includes information from previous health insurance applications. All MIBinformation is confidential and only available to member subscribers. A producer must inform an applicantat the time of application that a MIB may be requested by the insurer if it so desires. The primary purposeof the MIB is to allow an insurer to avoid high risk applicants.

NILA ©

L/H Licensing

Chapter 13—2

Page 184: FORWARD - WordPress.com · Return of premium p 6-8 6.0 Annuities p 7-1 Annuity principles and concepts p 7-1 Accumulation period versus annuity period p 7-1 Owner, annuitant and beneficiary

! MEDICAL QUESTIONNAIRES — Other medical questionnaires and examination information may beutilized by an underwriter to determine suitability for coverage. In addition, laboratory test results may alsobe used when required by an insurer (i.e. HIV and AIDS testing).

SIGNATURES REQUIRED ON THE APPLICATION — All insurance applications require at least twosignatures before the application is submitted to the home office for underwriting. The producer must always signthe application as well as the future policy owner (i.e., the applicant). If third party ownership is present, theproposed person to be insured must also sign. If the required signatures do not appear on the application, theunderwriting department will return the application to the producer for completion.

COMPLETING THE HEALTH INSURANCE APPLICATION — It is important for the agent to make surethat all applications include the required signatures. In addition, the agent must also see to it that the application iscompleted in its entirety. Incomplete applications will be returned to the agent. This will result in a delay in theunderwriting of the policy. Such a delay may result in a loss of business or at least a delay in the payment of commis-sions to the agent.

! CONDITIONAL RECEIPT — When an agent takes an application and collects an initial premium fromthe applicant, a conditional receipt must be provided. Coverage will generally begin as of the date appearingon the receipt as long as all "conditions" are satisfied and the application is approved by underwriting.Therefore, in many cases, an applicant is "conditionally insured" as of the date of the receipt as long as theapplication is approved by underwriting. The date on a conditional receipt is always a date prior to the issuedate of the policy.

ADDITIONAL UNDERWRITING CRITERIA — Underwriters also review other factors when determiningwhether or not to issue coverage and what premium to charge. These factors include: (1) the age of the insured;(2) the sex of the insured; (3) the insured's height, weight and general build; (4) the insured's health history and thehealth history of his/her family; (5) hobbies and personal habits of the insured; (6) the insured's occupation; (7) theinsured's financial condition; (8) the insured's personal activities; and (9) blood tests or lab fees.

The age and sex of the prospective insured are considerations since the older an applicant, the greater the premium. In addition, females are charged higher health insurance rates than males since, statistically, they seek medicaltreatment more often. An insured's build is a consideration since an individual who is overweight may be chargeda higher premium. Obviously the applicant's health status is an important underwriting consideration. However, sois the health history of the applicant's family. If a family is afflicted with a genetic condition which causes poorhealth, an insurer will want to take a closer look at the risk and may reject coverage.

The hobbies and personal activities of an applicant may affect policy issuance as well. An insurer may not wish towrite coverage on an applicant who sky-dives, bungy-jumps or engages in any other type of high risk activity. Hobbies are also known as avocations. An insured's occupation is also relevant to an underwriter since an individualengaged in a more hazardous occupation (i.e., private investigator) is more susceptible to an injury than a clericalworker. Finally, an applicant's financial status will be reviewed by an insurer since a person who is in financialtrouble is more likely to file false claims or fake an injury to collect benefits from a disability income policy.

POLICY ISSUANCE AND DELIVERY — Once the underwriters decide to accept a particular risk, a healthinsurance policy will be issued. The policy is then sent to the agent in order for it to be delivered to theinsured/policy owner. When the policy is delivered to the insured, the free-look period begins.

! PRODUCER RESPONSIBILITIES — When an agent delivers a policy he or she should take the timeto review with the policy owner the entire contract. In addition, the important parts of the policy should be

NILA ©

L/H Licensing

Chapter 13—3

Page 185: FORWARD - WordPress.com · Return of premium p 6-8 6.0 Annuities p 7-1 Annuity principles and concepts p 7-1 Accumulation period versus annuity period p 7-1 Owner, annuitant and beneficiary

explained to make sure the policy owner understands at least the basics of the contract. If an impairmentrider is attached to a policy, its restrictions or limitations must be explained to the policy owner at the timeof delivery. Generally, the policy owner must sign a statement that he or she is aware of and understandsthe effects and limitations of this type of rider.

! STATEMENT OF CONTINUED GOOD HEALTH — If an initial premium is not paid when theapplication is completed by the producing agent, this statement is generally required. If the agent senses thatcoverage may not be provided because of a potential health problem, no premium may be collected. If theapplication is later approved, the policy will be issued and sent to the agent for delivery. When the agenttravels to the policy owner's residence he/she must have a statement of continued good health signed by theinsured before the policy will actually be delivered. At this point, an initial premium will also be collected so that coverage is effective and the policy owner's consideration is complete.

INSURABLE INTEREST — The doctrine of insurable interest applies to anyone who is purchasing healthinsurance on the life of another person. The policy owner must possess a direct economic or financial interest in theperson being insured. All individuals possess an insurable interest in themselves. Insurable interest must exist when at the time of application.

RISK CLASSIFICATIONS — Underwriters will divide applications into different classes or categories of risk. The premiums charged will be affected by the risk classification in which each application is placed. There are threemain risk classifications utilized today including:

O STANDARD — A standard risk is generally one who is in good health and will be charged a standard ornormal premium rate. Standard rates are assessed to those who pose no additional risk factors. However,many insurers divide their standard classifications among smokers and nonsmokers. A smoker may bestandard in every other way except for the fact that he or she smokes cigarettes, cigars or even chewstobacco. Due to this increased risk an extra premium is charged. Some insurers classify a smoker as asubstandard risk.

O SUBSTANDARD — Substandard risks are those who are not in the best health. The applications of somesubstandard applicants may be approved with some modifications or an extra premium charge may beassessed if the insurer feels that the business written will still be profitable. Some applicants who engage inhobbies or personal activities that are dangerous are also classified as substandard such as scuba divers,skydivers, hang gliders, pilots and bungy jumpers. Other substandard risks who are in poor health or whoare obese, for example, may be declined or rejected. A substandard risk that is approved will be issued apolicy with an attached health condition rider of some sort. This means that the substandard risk is beingprovided with insurance coverage and will be assessed an extra premium because of the adverse orincreased exposure placed upon an insurer. The additional premium allows the insurer to provide coveragefor a substandard risk.

O PREFERRED — Those in perfect health and who are non-smokers receive the lowest premium rate. Thisis the risk that the insurer "prefers" since it will pay a lower ratio of claims on an insured who is in excellenthealth and is associated with low risk factors (i.e., non-smoker).

NOTE: Some insurers identify additional risk classifications to include declined risks and smoker vs.nonsmoker risks. Again, preferred risks generally involve nonsmokers while standard risks may includesmokers. Substandard risks are sometimes referred to as extra risks if they are written with an additionalpremium charge.

NILA ©

L/H Licensing

Chapter 13—4

Page 186: FORWARD - WordPress.com · Return of premium p 6-8 6.0 Annuities p 7-1 Annuity principles and concepts p 7-1 Accumulation period versus annuity period p 7-1 Owner, annuitant and beneficiary

GROUP MEDICAL EXPENSE INSURANCE

Group medical expense insurance may be purchased by an employer for his or her employees. Like group life andgroup disability, group medical expense insurance is a two party contract between the employer and the insurer. Theemployee is the "certificate holder." Group conversion (i.e., 31 days) functions the same as it does in a groupdisability policy where the covered employee may convert to an individual plan when leaving his or her job withoutproving insurability. Additional group insurance information is reviewed in Chapter 15.

CONSOLIDATED OMNIBUS BUDGET RECONCILIATION ACT (COBRA) — This Act mandated thatemployers must provide an employee and any “qualified beneficiary” with group health coverage following a“qualified event” (i.e., if employment is terminated). This federal law provides for the continuation of coverageunder specific circumstances including job termination, death, divorce or a child ceasing to be eligible for groupcoverage. COBRA states that the terminated employee may continue coverage under the employer's plan for up to18 months as long as he or she pays the premium. The employer will collect the premium from the terminatedemployee at a rate of no more than 102% of the group premium. Employers are mandated to offer COBRA if theyemploy twenty or more employees.

In addition, other qualifying events allow the beneficiary or dependents continuous coverage under the employer'splan if the employee dies, is terminated for a reason (i.e., being laid-off, resigning or retiring) other than grossmisconduct (i.e., stealing from an employer) or as a result of a divorce or legal separation. In some cases anemployer may not know of a qualifying event if it involves divorce, legal separation or the cessation of childcoverage. Therefore, the employee or family member must notify the employer within 60 days of the event or theright to elect COBRA coverage is lost. The continuation period for beneficiaries and dependents under thesecircumstances is 36 months. Employers who do not comply with COBRA may be stripped of their federal incometax deduction. An individual receiving coverage under COBRA may also apply it toward his or her creditablecoverage (according to HIPAA).

Before the Omnibus Budget Reconciliation Act of 1993 was passed, provisions regarding eligibility for groupcoverage were determined by an employer, underwriting practices of an insurer or State law. This Act containedregulations designed to better guarantee that group medical benefits will be made available to children. For instance,if a group medical plan provides coverage for dependent children of participants, it must also provide benefits forchildren who are adopted (or placed for adoption) with the same terms, conditions and benefits that apply to naturalchildren.

MARKETING OF GROUP PLANS — Group insurance is generally marketed through employer sponsoredplans or association sponsored plans (i.e., association of professional insurance agents).

UNDERWRITING CRITERIA — Several criteria must be reviewed when group health insurance underwritingis being done. Premiums are determined by the age, sex and occupation of the group to be insured. Group policiesare generally issued with less restrictive underwriting than an individual risk. Other criteria utilized when writinggroup coverage includes but is not limited to:

! The purchase of health insurance must be incidental to the formation of the group.

! 75% participation requirement for contributory plans.

! 100% participating requirements for non-contributory plans to prevent discriminatory practices by anemployer.

NILA ©

L/H Licensing

Chapter 13—5

Page 187: FORWARD - WordPress.com · Return of premium p 6-8 6.0 Annuities p 7-1 Annuity principles and concepts p 7-1 Accumulation period versus annuity period p 7-1 Owner, annuitant and beneficiary

The purpose of sound or competent underwriting, whether on the individual or group level, is to reduce or minimizeadverse selection. For instance, the participation requirements for a contributory plan, if lower than 75%, mayincrease adverse selection. Therefore, the insurer will avoid the risk. Experience rating may also be used todetermine what premium to charge for health insurance on larger groups of individuals. This means that past claimexperience may be used to influence the premium. Employees may join a group plan by enrolling at an allotted openenrollment period. An enrollment period is a thirty day period during the year when new employees can electcoverage under a group plan. In some cases, those who do not enroll during this period may still receive coverageas a late enrollee. A late enrollee is one who does not sign up when he or she is eligible but does so at a later time.

DELIVERY OF GROUP PLANS — Delivery criteria for group plans is similar to that regarding individual plans. Once a group health insurance policy is issued it must be delivered to the policyholder. The producer will generallydeliver a group health insurance policy to the location of the employer since the employer is the policyholder.

QUIZ

1. An underwriter derives information from all of the following sources, EXCEPT:

A. The application C. MIB report

B. The beneficiary D. Consumer report

2. Which of the following are required signatures that must appear on an insurance application?

A. Policyholder and agent C. Beneficiary and agent

B. Claimant and agent D. Agent and applicant

3. If an employee covered by a group medical expense plan dies, his family may continue coverage under the group plan

for:

A. 18 months C. 60 months

B. 36 months D. 120 months

4. In order for a contributory group plan to be written, an insurer requires what percentage of participation from all eligible

employees?

A. 25% C. 75%

B. 50% D. 90%

5. If a group plan is to be written on a non-contributory basis, what percentage of eligible employees must be covered?

A. 25% C. 75%

B. 50% D. 100%

ANSWERS

1. B

2. D

3. B

4. C

5. D

CHAPTER 13 KEY CONCEPTS

Application

COBRA

Delivery

Group Medical Expense

Insurable Interest

Issuance

Marketing

Risk Classifications

Underwriting

NILA ©

L/H Licensing

Chapter 13—6

Page 188: FORWARD - WordPress.com · Return of premium p 6-8 6.0 Annuities p 7-1 Annuity principles and concepts p 7-1 Accumulation period versus annuity period p 7-1 Owner, annuitant and beneficiary

DISABILITYINCOME

INSURANCE

Page 189: FORWARD - WordPress.com · Return of premium p 6-8 6.0 Annuities p 7-1 Annuity principles and concepts p 7-1 Accumulation period versus annuity period p 7-1 Owner, annuitant and beneficiary

INTRODUCTION

Disability income insurance is a type of health insurance that protects an insured's earned income if heor she cannot work due to total disability. In other words, it protects an insured's loss of earnings if hepossesses the inability to engage in his occupation and earn an income as a result of a covered accidentor sickness.

DISABILITY INCOME INSURANCE

TOTAL DISABILITY

Insurers define total disability in a variety of ways. Most insurers today cover an insured in "his or her ownoccupation." This is the occupation in which the insured was engaged when they applied for the policy. Additionaldefinitions include "inability to engage in any occupation for which the insured is qualified by reason of education,training or experience" or "the inability to engage in any gainful occupation,” which is basically a non occupationaldefinition of disability. The more liberal the definition of disability, the greater the premium. Some policies providebenefits for loss of time versus pure loss of income or income replacement. Most disability income insurers requirethat the disabled insured be under the care of a physician to verify the disability. Other concepts associated withdisability insurance include but are not limited to:

PRESUMPTIVE DISABILITY — This is a type of disability that is generally assumed to be total or of apermanent kind. It may involve the loss of use of limbs (i.e., paralysis) or the loss of sight, hearing or speech.

LOSS OF EARNINGS TEST — This test is utilized to determine if an income loss has actually occurred. Income or earnings include wages, salary, commissions, fees or compensation for services performed (i.e., work). Earned income does not include: (1) rental income from real estate; (2) interest on savings plans; or (3) dividendspaid on stocks, bonds or other like investments.

PARTIAL DISABILITY — Someone who is unable to perform one or some but not all of his or her duties ispartially disabled. If a person is partially disabled and has added a partial disability benefit rider to his or her plan,he will collect 50% of the monthly income benefit while partially disabled. Partial disability benefits are payablefor a maximum of six months.

RESIDUAL DISABILITY — Like partial disability, this involves a person who is able to perform some but notall of his or her duties. The benefit provided by a residual disability rider is paid if the loss of income is a residualeffect of the disability. This type of disability rider pays a percentage of lost income which usually exceeds the flat50% paid by partial disability. Partial and residual disability riders may not be added to the same policy. Again,residual disability will reimburse an insured based on his or her percentage of lost income due to a covered disability. If the insured is residually disabled and is losing 65% of his income per month, the rider will pay him 65% of hismonthly total disability benefit. Sometimes these benefits are referred to as "at-work" benefits. Both of these ridersprovide an "at-work" type of disability benefit since the insured is able to perform some but not all his or her duties. This means that the insured is still able to continue working in some capacity but is also able to receive partial orresidual benefits from the disability policy.

NILA ©

L/H Licensing

Chapter 14—1

Page 190: FORWARD - WordPress.com · Return of premium p 6-8 6.0 Annuities p 7-1 Annuity principles and concepts p 7-1 Accumulation period versus annuity period p 7-1 Owner, annuitant and beneficiary

PERMANENT AND TEMPORARY DISABILITY — Permanent disability involves the inability of theinsured to perform work ever again. This means that the insured is unable to perform the duties of his or heroccupation for an entire period (i.e., life). The loss of a leg is considered to be a permanent disability. Mostpresumptive forms are permanent. Temporary disability is a form of disability which reduces the ability to work ona temporary or limited period basis. A minor back injury may keep an individual out of work for two weeks. Therefore, he or she was temporarily disabled. Both permanent and temporary labels may be associated with totalor partial disability (i.e., permanent total vs. temporary partial). Whenever an insured is eligible to receive monthlybenefits from a disability income policy, a general requirement exists which states that the insured must be underthe care of a doctor. If benefits continue, periodic confirmation by the doctor must be provided which verifies thecontinuation of disability.

INJURY OR ACCIDENT VS. SICKNESS — Disability policies provide benefits if the insured has suffereda disability caused by an accident or an illness. An accident is a fortuitous event that is unexpected and unintendedand results in an injury. This is generally referred to as accidental bodily injury. This may be contrasted withaccidental means, a more restrictive definition of an accident, which indicates that the cause and result must beaccidental or no coverage is provided. For example, if a person in anger hurls a tool across the room and it bouncesoff a wall and strikes that person and he is injured, there would be no coverage according to the definition ofaccidental means. The result (i.e., injury) is accidental but the cause (i.e., throwing the tool) is not. If an individualjumps off a bus, the act of jumping is intentional even if an actual injury was accidental.

Sickness or illness also is covered under disability income policies. If an insured contracts a disease or other formof illness, there will be coverage unless the illness is restricted by a pre-existing condition or other policy limitations.

RECURRENT DISABILITY — This clause appears in disability income insurance. It is utilized when an insuredbecomes totally disabled for a time but appears to recover. However, in the next couple of months, the insured suffersa relapse (recurrence) of the same injury and becomes totally disabled again. The later disability would be considereda recurrence of the initial disability if it occurred within six months after the insured returned to work. Therefore,no new elimination period needs to be satisfied.

! The recurrent disability provision aids an insurer in determining how and when benefits are payable. If therecurrence occurs within the six-month period and the insured goes back out on total disability, benefits willcontinue as if the disability had not been interrupted. This is advantageous to the insured since he or she willnot have to satisfy another elimination period.

For example, assume Bob owns a disability income policy paying $1,000 per month with a 30 day waitingperiod. He becomes totally disabled and is out of work for five months. He then returns to work for threemonths but suffers a relapse of the same disability and is out of work for another four months. How manymonths of benefits will be paid? The answer is eight months. If Bob had gone back to work for sevenmonths and then suffered a relapse he would have collected seven months of benefits because Bob wouldhave had to satisfy a waiting period under each separate disability.

HOSPITAL CONFINEMENT BENEFIT — When this benefit is included in a disability income policy, anadditional benefit is paid over and above the regular monthly total disability benefit while the insured is confinedto a hospital. Generally benefits are payable from the first day of confinement for periods of up to six months. Thisis an indemnity type rider and not one of reimbursement.

NILA ©

L/H Licensing

Chapter 14—2

Page 191: FORWARD - WordPress.com · Return of premium p 6-8 6.0 Annuities p 7-1 Annuity principles and concepts p 7-1 Accumulation period versus annuity period p 7-1 Owner, annuitant and beneficiary

ADDITIONAL DISABILITY INCOME PRINCIPLES

INDIVIDUAL DISABILITY INCOME POLICY — An individual disability income policy is issued to aperson based on the information recorded on the application. In other words, the insurer will decide whether toaccept or reject the risk based on the underwriting information regarding the individual applicant. In addition, theapplicant is also responsible for paying the policy premium. This form of health insurance protects an individual (andfamily) against loss of earnings or income as a result of total disability. Disability may be described as "livingdeath." Once someone is unable to work and earn an income, it generally will be a living death since expenses suchas a mortgage, clothing, food and other necessities continue, but the income to provide them may not. Again,disability income policies are designed to protect one's earned income. These policies provide income or monthlyindemnity benefits to insureds who become totally disabled. Disability policies require that the insured be underthe care and treatment of a doctor or benefits will not be forthcoming. In other words, a physician must verify thatthe insured is unable to engage in or perform the duties of his or her occupation. This type of insurance willgenerally provide coverage until an insured retires and no longer earns an income. For example, if a person reachesage 65 and retires but continues to pay his/her disability income premiums and subsequently becomes disabled thenext year, the policy will usually not pay any benefits. In this case, the premiums paid following the person'sretirement will be refunded. However, some plans provide continuing coverage after age 65, or as long as theinsured continues to work. Other characteristics of a disability income contract which involve unique aspects inrelation to its underwriting include but are not limited to:

O PREMIUMS — Manual premium rates are determined based on the monthly income benefit selected. Additional criteria that will affect or influence the amount of the annual premium includes: the waitingperiod desired; the monthly income benefit or coverage amount desired; the age, sex, income, occupationand health of the applicant; the period of time for which the benefit is selected (i.e., for life); and whetheror not the insured owns other disability income insurance. A waiver of premium feature is generallyincluded in a disability income policy which waives premiums if the insured is totally disabled for 90consecutive days ( 6 months for life insurance).

O MONTHLY BENEFIT LIMIT — Most disability income insurers who issue individual policies permitan applicant to purchase or limit a monthly benefit that is 50 to 60% of their gross monthly earned income.Some insurers allow 80% of the net monthly income which actually accomplishes almost the same thing. Therefore, if the applicant's gross earned income is $3,000 per month, the insurer would allow him/her topurchase up to $1,500 to $1,800 of monthly benefit. Other forms of disability coverage, such as groupcoverage, permit greater percentages of monthly income (i.e., 60-70% and sometimes more).

O WAITING PERIOD — A waiting period, also known as the elimination period, is common to most typesof disability income insurance. It functions like a deductible. On each occasion that an insured becomestotally disabled, a waiting period must be satisfied during which no monthly benefit is payable. The longerthe waiting period selected, the lower the policy premium. The inclusion of an elimination period allowsan insurer to reduce the cost of coverage. Common waiting periods available include 30, 60, 90, 180, and365 days (although some group plans include seven or ten day waiting periods). The length of an eliminationperiod chosen by the insured should be based upon his or her earned income and the ability to pay premiums. For instance, if cash flow is the insured's primary concern, he or she may choose a longer waiting periodsince this would reduce the annual premium.

O BENEFIT PERIOD — The maximum length of time for which an insured collects monthly income willalso affect the premium charged. The most common benefit periods are 12, 24, and 30 months, 5 years, 10years, 20 years, to age 65, or for life.

NILA ©

L/H Licensing

Chapter 14—3

Page 192: FORWARD - WordPress.com · Return of premium p 6-8 6.0 Annuities p 7-1 Annuity principles and concepts p 7-1 Accumulation period versus annuity period p 7-1 Owner, annuitant and beneficiary

O OTHER CRITERIA — An insured's age will influence premiums since the older one is, the greater thetendency to become disabled. Therefore, the greater the risk to the insurer. An insured's sex will influencepremiums because a female statistically seeks medical treatment more often than a male and therefore willpay a higher premium. An insured's occupation will impact rates since more hazardous occupations carrymore risk. Therefore, a higher premium is required. Occupational considerations are very important withregard to classifying and assessing premium rates for disability risks. Again, the more risky the occupation,the greater the cost. This is why insurers have developed numerous occupational classes. This allows themto determine which exposures or risks are greater and to arrive at a premium rate for each. A proposedinsured's occupation is much more important to a disability income underwriter than it is to a life insuranceunderwriter. Individuals whose occupation is one of the professional type (i.e., physician, attorney, CPA,dentist, etc.) are engaged in a less hazardous or less risky type. Even if these individuals become ill or areinjured in an accident, they will attempt to do everything in their power to return to work since they will earna great deal more money if they are able to provide their professional services to the public as opposed toreceiving monthly disability income benefits. Occupations that are not of the professional type are lookedupon by the insurer as being more risky especially when the incentive to return to work is not as great. Therefore "risk" to the insurer involves much more criteria than simply the duties or functions of a particularjob. It also takes into consideration the severity or frequency of possible claims as well. These will beaffected by the risk that is insured. Morbidity will also influence the cost of coverage as well. This is thedisability equivalent of the mortality concept used in life insurance. Morbidity is the probability of anindividual becoming disabled at any particular age.

In most cases, insurers will place applicants into insurable and uninsurable groups of occupations basedupon the risk or hazard involved. Professionals will be in the former, among others, while test pilots, sometypes of police officers and deep sea divers are examples of those from which an insurer prefers to stay away.Seasonal employees also pose a challenge for disability insurers since the opportunity for malingering isgreater. Therefore, insurers generally prefer to avoid these risks. Some insurers utilize policy issuancealternatives in their underwriting which allows them to include someone who may normally be uninsurableif they are part of a larger group of proposed insureds. Depending upon the health history of an applicant,an insurer may issue a policy to an individual with a health condition but at a much greater premium rate,which in its mind will offset the higher risk. However, underwriting departments are continually evaluatingand re-evaluating disability exposures to determine acceptability and whether or not an underwriting profitcan be made. A hazardous risk twenty years ago may be insurable today. For example, many disabilityinsurers refused to underwrite window washers years ago, especially those who worked on high rise officebuildings located in a city. As time went on, however, some underwriters, based on their experience,realized that if a window washer on the 80th floor of the Empire State Building did not fasten his or herprotective belt securely and were blown off a scaffolding by a sudden gust of wind, the resultant claimwould not be of the disability variety. It would be a death claim and the disability policy owned by thewindow washer does not cover loss as a result of death. Therefore, some of this business could be profitableto the insurer.

LOSS SETTLEMENT — It is helpful to be able to calculate a disability claim. For example, assume that Georgeowns a policy paying $800 per month if he is totally disabled and the policy includes a thirty-day waiting period. Assume further that George is totally disabled for 90 days. Following the 30 day waiting period, he will collectmonthly income for 60 days or 2 months for a total of $1,600.

TAX CONSIDERATIONS — Premiums paid on an individual disability income policy are not tax deductible. Benefits received from this policy, however, are tax free without limitations. For instance, assume Joe has adisability income policy paying him $1,000 per month if he is disabled. The policy has a 30-day waiting period. Joeis then disabled for twelve (12) months. His policy would pay him $11,000 during that period tax free. Non-

NILA ©

L/H Licensing

Chapter 14—4

Page 193: FORWARD - WordPress.com · Return of premium p 6-8 6.0 Annuities p 7-1 Annuity principles and concepts p 7-1 Accumulation period versus annuity period p 7-1 Owner, annuitant and beneficiary

contributory group disability premiums are tax deductible to the employer (i.e., business expense) and monthlybenefits paid to the disabled employee are considered to be ordinary income. Lump sum or single sums may be paidout for presumptive types of disability such as loss of sight.

LIMITATIONS ON BENEFITS PROVIDED — Limitations are placed upon benefits provided by disabilityincome policies in order to reduce or prevent adverse selection. For example, if Bob makes $5,000 per month, hewill not be allowed to purchase a disability income policy paying him $5,000 per month if he becomes disabled. Thisincreases adverse selection since Bob can become "disabled", collect $5,000 per month and there is no reason whyhe should return to work. His policy is paying him what he would earn at his job and he doesn't have to work! Theremust be a relation of earnings to the disability income amount desired.

POLICY LIMITS — These are the stated income limits that will be paid to the insured if he or she is disabled. These limits are subject to limitations, reductions or other provisions listed in the policy.

OCCUPATIONAL VS.NON-OCCUPATIONAL POLICIES

Some disability income policies do not cover losses arising out of the occupation of the insured. In other words, suchpolicies do not provide coverage if the insured will also collect from Workers' Compensation or other social insuranceplans. This type of disability income contract is called non-occupational. Group disability insurance is an exampleof non-occupational coverage. If the policy pays a monthly income in addition to any benefits received by Workers'Compensation or other social insurance plans, it is referred to as an occupational policy. An example of anoccupational plan is an individual disability income policy.

EXCLUSIONS

EXCLUSIONS — Exclusions appearing in a disability income policy allow the insurer to reduce the cost of theproduct and protect it from adverse selection and catastrophic type losses. The more common exclusions appearingin a disability income policy include but are not limited to: (1) war, declared or undeclared; (2) intentionally self-inflicted injuries; (3) aviation related claims of a pilot or crew member unless a commercial aircraft or pilot isinvolved which is covered; (4) military service since the insured is covered by the government while in its service;and (5) as a result of engaging in any illegal occupation such as robbing a bank (i.e., felony). Some insurers alsoinclude a military suspension provision which limits or suspends disability income protection when the insuredserves in the armed services.

BENEFIT RIDERS

Some additional benefit riders are available to be added to a disability income policy for an additional premiumincluding but not limited to:

ADDITIONAL MONTHLY BENEFIT (AMB) / GUARANTEED INSURABILITY — This rider functionsjust like its counterpart in life insurance. It "guarantees" that the insured, at various specified option dates in the fu-ture, can purchase additional amounts of disability income insurance without taking a medical exam. In other words,

NILA ©

L/H Licensing

Chapter 14—5

Page 194: FORWARD - WordPress.com · Return of premium p 6-8 6.0 Annuities p 7-1 Annuity principles and concepts p 7-1 Accumulation period versus annuity period p 7-1 Owner, annuitant and beneficiary

it allows an insured to purchase additional amounts of disability income in the future — guaranteed issue. An extrapremium is required before this rider can be added. As long as the insured's income has increased over the years,the additional amounts may be purchased without proving insurability. It may also be referred to as the AdditionalPurchase Option (APO) or the Future Increase Option (FIO). ACCIDENTAL DEATH AND DISMEMBERMENT (AD&D) — Coverage for these losses may be addedfor an additional premium. The death benefit paid under accidental death coverage is called the principal sum. Theseverance benefit paid under accidental dismemberment is called the capital sum and is usually one-half of theprincipal sum. AD&D coverage is sometimes referred to as multiple indemnity coverage. The cost of this coverage,because it is limited to accidental causes, is the least expensive type of death protection when comparing it toindividual policies such as whole life and term life insurance.

! A dismemberment involves the loss of a limb as a result of an accident. The loss of use of a limb (i.e.,paralysis) may also qualify as a dismemberment loss under some policies. The loss of a toe or finger doesnot qualify. Some policies also pay dismemberment benefits for presumptive types of disability such as lossof sight, hearing or speech. Therefore, if an accidental death benefit of $20,000 was purchased and theinsured suffers a loss of hearing, the policy would pay (a capital) sum of $10,000 (i.e., 50% of the principalsum). The principal sum may be paid when an insured loses any combination of limbs, such as a hand anda foot, an arm and a foot, or both eyes.

! An accidental death benefit may be available up to a limiting age (i.e., age 70). In addition, death mustgenerally occur within 90 days of the accident in order to qualify.

! When accidental death coverage is added to a disability income insurance policy, a primary beneficiaryshould be designated. Premiums paid for this type of coverage are not tax deductible. Benefits payable tothe insured or beneficiary are not taxable.

COST OF LIVING (COL) — This rider is designed to offset the decline in the purchasing power of monthlydisability income benefits as a result of inflation. A $1,000 per month benefit with a COL rider will be increasedeach year by the lesser of the inflation rate as determined by the Consumer Price Index (CPI) or a specifiedpercentage stated in the policy (generally 5%).

SOCIAL INSURANCE SUPPLEMENT (SIS) — This rider helps low or middle income persons who mightpossess a gap in disability income coverage. It integrates benefits between a disability income policy with a SocialSecurity rider with other forms of social insurance such as Workers' Compensation or Social Security disabilitybenefits. This rider is referred to as the Social Insurance Supplement. It will pay a monthly benefit to an insured ifhe or she is not receiving benefits under Social Security. This rider may be coordinated with Worker's Compensationas well. Generally, if social insurance payments cease or are decreased, the monthly indemnity of the policy will bepayable immediately, or increased according to a predetermined additional monthly benefit if such benefits arereduced, if the maximum benefit period of the contract has not terminated.

REHABILITATION RIDER — This rider was developed based on the insurer’s desire to take an insured offdisability gradually and to assist him or her to become productive again. The rider will pay for reasonable expensesincurred for rehabilitation, training programs, tuition, books or other appropriate expenses.

MEDICAL REIMBURSEMENT BENEFIT — Also known as the nondisabling injury benefit, this pays upto a specified amount if the insured suffers an injury and no other claim is filed under the policy. In other words, theinsurer will pay for medical expenses up to a stated limit as long as the insured has suffered an injury.

NILA ©

L/H Licensing

Chapter 14—6

Page 195: FORWARD - WordPress.com · Return of premium p 6-8 6.0 Annuities p 7-1 Annuity principles and concepts p 7-1 Accumulation period versus annuity period p 7-1 Owner, annuitant and beneficiary

RETURN OF PREMIUM —This rider provides a refund provision for an insured. It actually offers a refund ofa percentage of all premiums paid during a specific period as long as, according to the insurer, there has beenfavorable claim experience. These riders may provide refunds at age 65 or every five or ten years after the policyis issued (depending upon the insurer). A cash surrender value rider is also available from some insurers. Thisrider returns all premiums to the policy owner at age 65, less any benefits received over the life of the contract.

OTHER RIDERS

IMPAIRMENT RIDER — This type of rider is also known as an exclusion rider. When this type of rider hasbeen attached to a disability income contract, it indicates that the insurer feels that it is accepting a greater risk. Inother words, since it may be unable to adequately assess the total risk the insurer decides to provide coverage for theindividual but limit it with regard to the impairment involved (i.e., illness, injury, etc.). For example, assume thata person is issued a disability income policy. Since he has experienced several (left) knee operations the insurer mayattach an impairment rider stating that it will not provide coverage if total disability results from any injury to the(left) knee. If such a rider is added, the producing agent should explain the limitations provided by the rider. Thistype of rider can be issued with or without an extra premium charge. In some cases, this type of rider is referred toas a coverage "waiver" or a "waiver of disability."

BUSINESS DISABILITY INSURANCE

USES OF BUSINESS DISABILITY INSURANCE — Disability income insurance may be used in businessscenarios to protect the owner of a business, the business itself or a key employee. Some business forms of disabilityincome insurance include:

BUSINESS OVERHEAD EXPENSE (BOE) — Businesses may use a disability income policy to ensurecontinuation if the owner becomes totally disabled. This type of plan covers the overhead expenses which are usualand necessary in the operation of the business if the owner is disabled. Additional business overhead expenseinformation is as follows:

! Overhead expenses covered include monthly rental cost or mortgage payment, expenses for heat, electricity,telephone, advertising, employee salary and other costs associated with the operation of the business. Theydo not include the salary or profit of the business owner. The salary of the owner should be protected byan individual disability income policy.

! These policies, like other forms of health insurance, reimburse the owner for overhead expense loss for themaximum benefit period selected. The business is reimbursed by the policy for exact overhead expensesincurred. Professional overhead expense plans are available to owners in a "profession" (i.e., physician,attorney, CPA, etc.).

! Premiums paid on BOE policies are tax deductible as a business expense. Benefits received are taxable tothe business as ordinary income, but are deductible when they are paid out by the business to meetcontinuing expenses which they are designed to cover.

KEY EMPLOYEE POLICY — A business owner would purchase this type of policy to protect and indemnifythe business in the event that a key-employee becomes disabled. Partners in a business may purchase this type ofprotection on one another as well. If the business owner buys a policy covering the key employee it will ease a major

NILA ©

L/H Licensing

Chapter 14—7

Page 196: FORWARD - WordPress.com · Return of premium p 6-8 6.0 Annuities p 7-1 Annuity principles and concepts p 7-1 Accumulation period versus annuity period p 7-1 Owner, annuitant and beneficiary

portion of the financial challenge placed upon the business when the key-employee becomes disabled. The employeris the policy owner and retains all ownership rights (i.e., naming or changing the beneficiary) of the policy. Thebenefits are paid to the business. Thus, premiums paid for this policy are not tax deductible and benefits receivedare tax free. Some key employee policies will pay a benefit directly to the disabled employee. In this case, if theemployer pays the premium, this amount is tax deductible as a business expense. The benefits received will generallybe taxable as income to the disabled employee.

DISABILITY BUY-SELL / BUY-OUT — The health insurance equivalent of the life insurance buy-sellagreement, a buy-out agreement funded by disability income insurance is written to protect the firm and a partner'smonetary interest (i.e., value) in the business if he or she becomes totally disabled. If a partner becomes disabled,the monthly income benefit is generally paid to the corporation (i.e. firm) and then later paid to the disabled partner(or family) according to the terms of the buy-out agreement. Some agreements provide for a lump-sum distributionof benefits. Whether the purchaser, policy owner, beneficiary and premium payor is the business entity (i.e., entityplan) or the business owners (i.e., cross purchase plan), the premiums are not deductible and the proceeds are exemptfrom income taxation. However, a shareholder may be taxed on a gain, if any is realized, over and above his or heroriginal basis in the stock. In this case "basis" refers to the amount the shareholder paid for the stock originally.

GROUP DISABILITY INSURANCE

Group insurance involves a two party contract between insurer and employer that provides coverage for coveredemployees. Group insurance underwriting is not as stringent as individual underwriting due to the larger number ofcovered persons. However, in the last several years insurers have become much more stringent when it comes tosmaller groups. Insurers may offer short-term or long-term disability programs. Short-term (STD) plans may beprovided 13 weeks, 26 or 52 weeks but generally for no longer than two years. Benefits are generally "co-terminus"which means that benefits are paid for the same length of time whether the disability is due to an accident or anillness. They are characterized by waiting periods (i.e., 14 days, 30 days, etc.) and may be contributory or noncontributory. Long-term (LTD) plans provide benefits for two years or longer and are usually issued for 5 years, 10years to age 65 and in some cases for life. Since the potential benefit periods are longer than STD plans, the cost isgreater. In addition, any benefits payable by a STD or LTD plan will be offset by any benefits payable by other formsof social insurance such as Workers' Compensation or Social Security. Group disability plans are not portable as areindividual disability policies.

TAX CONSIDERATIONS — Disability income premiums paid by an employer for LTD or STD plans aredeductible to the employer. The premiums paid are not considered income to the employee but the benefits payableto the employee are considered to be ordinary income and are taxable to the employee. If benefits are received undera plan to which the employee has contributed, the portion of the disability income benefit attributable to the employeecontributions is tax-free. For example, if the employee has 50% of the group disability premium payroll deducted,only half of the monthly benefit received will be taxable. If 25% of the premium is payroll deducted (i.e., paid bythe employee), then 75% of the benefit received will be taxable. When an employer pays the premium, the insurermust deduct Social Security (FICA) for the employee for the first six months of disability. If there is a waiting periodpresent, the FICA would be deducted for the remaining portion of the six month period after the waiting period wassatisfied. For example, if there was a 60 day waiting period, FICA would be deducted by the insurer for four months.If the employee pays the premium, there will be no FICA deduction. If the premium were shared by the employer andemployee, only that percentage of benefits attributable to the employer would be subject to the FICA deduction. Employer paid disability coverage will have tax consequences for a covered employee.

NILA ©

L/H Licensing

Chapter 14—8

Page 197: FORWARD - WordPress.com · Return of premium p 6-8 6.0 Annuities p 7-1 Annuity principles and concepts p 7-1 Accumulation period versus annuity period p 7-1 Owner, annuitant and beneficiary

COORDINATION WITH OR OFFSETS FROM OTHER PROVIDERS — Group disability benefitsare generally coordinated with or decreased by disability income received from government plans, pensions or otherforms of social insurance such as Social Security disability or Workers Compensation. Individual disability incomepolicies generally pay the monthly income benefit in addition to any income derived from other sources.

GOVERNMENT DISABILITY

Disability benefits may be paid by Social Security (OASDHI) under the federal government or through stategovernments providing state-sponsored disability benefits or Workers' Compensation. Disability benefits underSocial Security will be reviewed in a later chapter. Depending upon the State involved, Workers' Compensationgenerally pays anywhere from 60 to 90% of an employee’s salary if he or she is injured on the job.

WORKERS COMPENSATION — This casualty or liability type of insurance is purchased by an employer whoprovides benefits to employees who are injured arising out of the scope of their employment. In other words, itprotects employees who are injured "on the job." It include benefits for unlimited medical expenses, rehabilitation,funeral expenses if incurred, dependent benefits and disability income or loss of wages. In other words, among otherbenefits, Workers’ Compensation pays for medical expenses and weekly income for an employee injured within thescope of his or her employment. Most types of employees are eligible by this type of coverage except for seasonalor casual employees (i.e., part-time). Disability benefit amounts vary by State and are paid for total permanentdisability, total partial disability, temporary total disability and temporary partial disability. Benefits paid byWorkers’ Compensation are coordinated with benefits provided by medical plans, group disability and Medicare.

Employers are required by law to provide Workers’ Compensation benefits for employees. This means that it iscompulsory to provide such benefits. Employers may comply with the law in several ways. The most commonmethod is to purchase Workers’ Compensation insurance coverage in the voluntary (i.e., normal or regular) marketfrom an insurer. If such coverage cannot be secured when an employer’s policy has been cancelled or nonrenewed,coverage may be purchased in the involuntary or residual market through a State assigned risk plan. Employers mayalso comply with the law through self-insuring by demonstrating to State officials that they have the ability (i.e.,funds) to provide benefits on their own or by posting a surety bond. In a few monopolistic States, Workers’Compensation can only be purchased from a State operated Fund (i.e., North Dakota, Ohio, Washington, WestVirginia and Wyoming).

REFER TO THE TAX CONSIDERATIONCHART ON THE NEXT PAGE

NILA ©

L/H Licensing

Chapter 14—9

Page 198: FORWARD - WordPress.com · Return of premium p 6-8 6.0 Annuities p 7-1 Annuity principles and concepts p 7-1 Accumulation period versus annuity period p 7-1 Owner, annuitant and beneficiary

Tax Considerations of Health InsurancePremiums and Benefits

General Rule of Thumb:

If the premium is Not Deductible — The benefits are generally tax-free (i.e., not taxable).If the premium is Deductible — The benefits are usually taxable as ordinary income.

Health Insurance Policy Premiums Paid Benefits Received

Individual (Premium Paid by Policy owner)

Individual DisabilityIncome

Not tax deductible Not income taxable (tax free)

Individual Medical ExpenseIncluding Dental Expenses

May be included in unreimbursed medicalexpenses (i.e., and be tax deductible ifthose expenses exceed 7 1/2% of the in-sured's adjusted gross income).

Not income taxable (tax fee)

Individual Long-Term Care Not tax deductible (unless a qualifiedLTC plan)

Not income taxable (tax free)

Group Policy

Non-Contributory GroupDisability Income(Employer Paid)

Tax deductible to employer Income taxable to the employee

Contributory Group Dis-ability Income (SharedPremiums)

Tax deductible to employer Partially taxable to employee (what-ever proportion of the premium is paid bythe employer, that same proportion of thebenefit received by an employee is tax-able).

Group Medical Expenses Tax deductible to employer (i.e., businessexpense)

Not income taxable

Group Long-Term Care Tax deductible to employer (i.e., businessexpense)

Not income taxable

Group A D & D Tax deductible to employer (i.e., businessexpense)

Not income taxable

Business Health Insurance

Key Employee Disability(Owner/Payor of Premium)

Not tax deductible Not taxable as ordinary income

Business OverheadExpense

Tax deductible to the business Taxable as income to the business. NOTE: Taxable benefits are then used to pay businessexpenses which are then tax deductible.

Disability Buy-Out Policy(Owner Payor of Premium)

Not tax deductible Not income taxable

Sole Proprietors (i.e., ma-jor medical

Fully deductible as of year 2004 Not income taxable

Partnerships and LLCs Tax deductible to partnerships (paid

by partnership) and LLC

Not income taxable

NILA ©

L/H Licensing

Chapter 14—10

Page 199: FORWARD - WordPress.com · Return of premium p 6-8 6.0 Annuities p 7-1 Annuity principles and concepts p 7-1 Accumulation period versus annuity period p 7-1 Owner, annuitant and beneficiary

QUIZ

1. John owns an individual disability income policy paying $800 per month with a 30 day waiting period. If he is disabled for

90 days what amount paid by his policy is taxable as income?

A. $800 C. $2,400

B. $1,600 D. $0

2. Premium determination of disability income policies are based on several criteria. Each of the following will influence

disability income premiums, EXCEPT:

A. Insured's age C. Waiting period

B. Mortality D. Length of benefit period

3. A business overhead expense policy provides disability income protection for the owner of the business. All of the following

are covered expenses under a BOE policy, EXCEPT:

A. Employee salary C. Utilities

B. Employer salary D. Rent

4. Which of the following types of disability income policies excludes losses arising out of the occupation of the insured for

which the insured is entitled to received Workers' Compensation benefits?

A. Nonoccupational C. Presumptive

B. Occupational D. Residual

5. Disability income is one type of accident and health insurance contract. Which of the following is covered by this type of

policy?

A. Incurred medical expenses C. Injury suffered off the job

B. Loss of income due to illness D. Cost associated with surgery

6. All of the following are types of presumptive disability, EXCEPT:

A. Loss of sight C. Loss of speech

B. Loss of income D. Loss of use

7. Disability income policies protect an insured's loss of income. Which of the following is not considered earned income that

may be covered under a disability income policy?

A. Income from the job performed C. Commissions from sales

B. Income from rents D. Wages paid by employer

8. Residual disability covers:

A. Loss of income C. Presumptive disability

B. Loss of use D. A percentage of lost income

9. Under the definition of accidental means, the result and which of the following must be accidental?

A. The injury C. The cause

B. The income loss D. The illness

NILA ©

L/H Licensing

Chapter 14—11

Page 200: FORWARD - WordPress.com · Return of premium p 6-8 6.0 Annuities p 7-1 Annuity principles and concepts p 7-1 Accumulation period versus annuity period p 7-1 Owner, annuitant and beneficiary

10. What type of rider may be added to a disability income policy which allows the insured to purchase additional amounts of

disability insurance without a medical exam?

A. Cost of living C. Social Security

B. Accidental death D. Guaranteed insurability

11. An employee is covered by a disability income policy paid by the employer. However, 10% of the premium is payroll

deducted. If the employee is later disabled and collects a benefit for one month after the waiting period, what percentage

amount of the monthly benefit will be taxable income?

A. 10% C. 90%

B. 25% D. 95%

12. If a business pays the premium on a short term disability policy covering a key employee, receives the monthly benefit and

pays the disabled employee, the benefit to the employee is:

A. Nontaxable C. Taxable as recaptured income

B. Taxable as ordinary income D. Tax exempt to the employee

13. What type of disability income policy may have its premium increased for an entire class of policies?

A. Non cancelable C. Conditionally renewable

B. Guaranteed renewable D. Guaranteed retractable

14. Joan is covered by a disability income policy as a clerical worker. One of her hobbies is hang-gliding. She later leaves her job

to take a new position as an instructor for a hang-gliding school. How will this change affect her previously purchased

disability policy?

A. It will be canceled

B. The policy premium will be increased

C. The monthly benefit will be reduced

D. The monthly benefit will be increased

15. Which of the following classes of disability policy will provide higher benefits if an insured is injured off the job?

A. Occupational policy C. Limited policy

B. Non occupational policy D. Restricted policy

ANSWERS

1. D2. B3. B4. A5. B

6. B 7. B 8. D 9. C10. D

11. C 12. B13. B14. C15. B

NILA ©

L/H Licensing

Chapter 14—12

Page 201: FORWARD - WordPress.com · Return of premium p 6-8 6.0 Annuities p 7-1 Annuity principles and concepts p 7-1 Accumulation period versus annuity period p 7-1 Owner, annuitant and beneficiary

CHAPTER 14 KEY CONCEPTS

AD&DBenefit RidersBOEBuy-sell/Buy-outCOLKey employeeLimitationsMorbidityOffsetsPartial disabilityPresumptive disabilityRecurrent disabilityResidual disabilitySTD and LTDTotal disability

NILA ©

L/H Licensing

Chapter 14—13

Page 202: FORWARD - WordPress.com · Return of premium p 6-8 6.0 Annuities p 7-1 Annuity principles and concepts p 7-1 Accumulation period versus annuity period p 7-1 Owner, annuitant and beneficiary

INDIVIDUALand GROUP

MEDICAL EXPENSEPOLICIES

Page 203: FORWARD - WordPress.com · Return of premium p 6-8 6.0 Annuities p 7-1 Annuity principles and concepts p 7-1 Accumulation period versus annuity period p 7-1 Owner, annuitant and beneficiary

INTRODUCTION

Medical expense insurance covers the cost of medical care resulting from an accident or illness. In theprevious chapter we reviewed disability income which replaces lost income as a result of illness orinjury. Medical expense insurance provides for the payment or reimbursement of medical expenses in-curred. Medical expense insurance may be purchased by individuals or by groups. This latter form ofcoverage is generally purchased by employers for their employees. Medical expense coverage generallyincludes hospital expense coverage, surgical expense coverage and physicians expense. Theseintroductory plans evolved later to become major medical and comprehensive major medical plans. Thischapter will review the various types of medical expense coverages available to individuals and throughgroups.

INDIVIDUAL AND GROUPMEDICAL EXPENSE POLICIES

MEDICAL EXPENSE INSURANCE

Health insurance may be classified into three categories including medical expense insurance, disability incomeinsurance and long-term care insurance. Medical expense insurance, which is generally referred to as “health”insurance, involves a wide range of benefits that can cover almost every expense connected with hospital and medicalcare. This type of coverage pays for medical services provided to individuals and their families. Some plans providebasic benefits while others offer comprehensive coverage for all medical services provided as a result of an illnessor accident. Benefits paid by the policy are sent directly to the provider (i.e., physician) or institution (i.e., hospital)or as a reimbursement of actual expenses incurred. Disability income insurance (reviewed in Chapter 14) protectsa portion of an insured’s income that is lost as a result of disability due to an accident or illness. This type ofinsurance is also referred to as loss of time coverage or loss of income insurance. It pays a monthly income benefitto an insured when he or she cannot work due to a disability. Long-term care insurance pays a benefit to an insuredwhen he or she is unable to perform activities of daily living due to a physical or cognitive impairment (reviewed inChapter 16).

Medical expense insurance or “medical plans” include but do not limit coverage for hospital costs, physician’sexpenses, surgical expenses, prescription drugs, nursing care, diagnostic treatment, laboratory fees, rehabilitationexpenses, physical therapy and other miscellaneous or ancillary medical related expenses. Medical expense insurancemay be purchased on an individual or group (i.e., employer-sponsored) basis. An individual or family membercovered by these plans is referred to as an insured, whereas an HMO or PPO refers to covered persons as membersor subscribers. Policies may be written on a reimbursement or indemnity basis as well. For instance, assume thatJohn is covered by a reimbursement type of plan with a maximum hospital benefit of $500,000. He is hospitalizeddue to an illness for twenty days. The bill is $25,000. The policy will pay or reimburse John or the hospital theamount equal to the expenses incurred (i.e., $25,000). If John had owned an indemnity plan that paid $200 per dayfor a hospital stay, he would receive $4,000 (twenty days X $200 per day). Therefore, the indemnity plan does notpay specifically for the medical expenses. Rather, it pays an insured a stated benefit identified in the contract. In thisexample, the policy pays or “indemnifies” John an amount per day for hospitalization.

NILA ©

L/H Licensing

Chapter 15-1

Page 204: FORWARD - WordPress.com · Return of premium p 6-8 6.0 Annuities p 7-1 Annuity principles and concepts p 7-1 Accumulation period versus annuity period p 7-1 Owner, annuitant and beneficiary

BASIC HOSPITAL EXPENSE INSURANCE — This type of individual plan, sometimes referred to as a basicmedical expense plan, is designed to pay or reimburse all or some of the cost of room and board for a specifiednumber of days (i.e., 30, 60. 90, 120, 180, etc.). For instance, the policy may pay $100, $200, or $300 per day whilethe insured is confined to a hospital. Other covered medical expenses (while hospital confined) include nursingcare, x-rays, all in-patient testing, ambulance charges, lab testing fees, medication administered in the hospital,anesthesia, fees for the use of the operating room, prosthetic devices, hospital extras, other medicines, andancillary (miscellaneous) expenses.

Limits of Coverage — Hospital expense plans are generally written on a reimbursement basis. If written on thisbasis the insured pays the hospital and submits a claim to the insurer. This means that a basic hospital expense policywill pay "up to its policy limits" or "up to its coverage limit" as long as the covered accident or illness occurs duringthe policy period. An accident or illness that occurs prior to the policy period will not be covered by the healthinsurance policy. Some of these plans are also written on an indemnity basis where an amount per day ($200 per dayfor room and board) is paid by the insurer directly to the hospital "up to the stated limits" of the policy.

BASIC SURGICAL EXPENSE COVERAGE — Once hospital expense plans became popular, insurers beganto market surgical expense plans to cover the cost of surgery. Many health plans combined hospital and surgicalexpense coverage. Surgical expense plans cover the cost of the surgeon’s fee, an assistant surgeon, the fees of ananesthesiologist and any postoperative expenses. Policies with surgical expense coverage may function in any ofthree ways. Some plans cover this benefit on a scheduled basis (i.e., $2,000 for a knee operation) with an itemizedlist of all surgical procedures covered and their assigned dollar limit. This benefit schedule means that a maximumdollar amount of coverage is provided in an "itemized list" for each surgical procedure. Some surgical expense plansprovide coverage on a usual, customary and reasonable (UCR) basis. This means that the policy pays a benefitdeemed to be the usual, customary and reasonable charge for the geographical area where the procedure wasperformed. Another type of surgical expense coverage pays on a relative value basis. This type pays benefits basedon a point system assigned to each procedure. The policy identifies a dollar per point value (i.e., a conversion factor). For example, if knee surgery were 50 points and the policy paid $50 per point, the benefit paid would be $2,500.

PHYSICIANS EXPENSE — This type of insurance provides benefits to cover a physician's fees for nonsurgicalcare in the hospital, doctor's office, or in the home (i.e., house call). It is sometimes referred to as "outpatientservices" and is generally included with either the hospital or surgical expense plan. It is usually not written alone. An amount per visit (i.e., $50) is usually paid for treatment of injury or illness.

COMPARISON WITH OTHER PROVIDER TYPES — Basic medical expense plans provide benefits invarious ways. Some provide coverage on a fee for service or reimbursement basis. A provider (i.e., doctor) treatsa patient, charges a fee and is reimbursed for services provided by the insurer. HMOs do not function on areimbursement basis and offer coverage on a prepaid basis emphasizing preventive care. Some medical plans coverspecified or limited coverage while others provide comprehensive coverage. As mentioned, surgical expense planssometimes provide benefits on a scheduled or relative value basis while some insurers will only cover such servicesthat are usual, customary and reasonable (UCR). In managed care or HMOs scenarios, a member or subscriber mayhave a limited choice of providers versus medical plans which allow for a choice of any provider. Lastly,commercial insurers refer to their clients as "insureds," while HMOs, Blue Cross/Blue Shield and others refer tosuch individuals as participants, members or subscribers. These policies include common limitations with regardto benefits that are similar to limitations found in major medical and other forms of health insurance. The exclusionsappearing in these plans are also similar and in many cases identical to those found in a major medical plan. Inaddition, these policies also include provisions that permit an insured to control his or her cost of coverage as wellincluding a stop-loss limit, coinsurance, deductibles and other cost containment or cost savings method. With regardto claims, if an insurer fails to provide an insured with claim forms the insured may file an unfair claim grievance

NILA ©

L/H Licensing

Chapter 15-2

Page 205: FORWARD - WordPress.com · Return of premium p 6-8 6.0 Annuities p 7-1 Annuity principles and concepts p 7-1 Accumulation period versus annuity period p 7-1 Owner, annuitant and beneficiary

with the Department of Insurance and send to the insurer a provider certified explanation of the medical proceduresand treatment received.

MAJOR MEDICAL INSURANCE

MAJOR MEDICAL EXPENSE POLICY — Major medical health insurance evolved as a result of theincreased expenses associated with medical procedures, treatment and care. These policies were designed to protectan individual and family against catastrophic type losses. The hospital expense, surgical expense and physician'sexpense coverages previously described were the precursor of major medical insurance. This coverage is availableon an individual (non-group) or group basis.

OVERVIEW AND CHARACTERISTICS — Major medical insurance plans provide a broad range of benefitsunder one policy since they cover hospital and surgical expenses but with a greater ceiling of coverage. In otherwords, major medical plans possess higher coverage limits than did the earlier plans. Generally, these plans pickup where basic medical expense coverage leaves off whether providing a supplement to the basic plan or as a standalone plan.

Most major medical plans also include an initial deductible. Once this is satisfied, the insured and insurer pay acoinsurance amount. This is generally split on an 80/20 basis where the insurer pays the larger of the two. Majormedical provides "blanket" of coverage as well. Some types of major medical policies, cover first dollar expensesup to a specific dollar limit. Benefits provided by major medical plans are usually expressed as a percentage ofeligible expenses (i.e., 80%). Like most health insurance policies, a major medical policy will pay "up to its statedlimits." Premiums paid for major medical policies are influenced by provisions including the deductible selected,the coinsurance amount carried by the policyholder, the maximum benefits or ceiling of protection selected and thestop-loss limit selected by the policy owner.

CEILING OF PROTECTION — Most major medical policies carry a high coverage limit or ceiling ofprotection in amounts of $250,000, $500,000, $1 million or more. The policy will stipulate whether the policy limitapplies to an accident or an illness. For example, the contract may state that the $500,000 limit applies to eachaccident or each illness or it may apply as a lifetime maximum amount for all illnesses or accidents.

ADDITIONAL COVERAGES — Major medical policies provide coverage for medical services included in basichospital expense, surgical expense, and physician's expense contracts. These policies generally include coverageswith common limitations including but not limited to: hospital room and board charges, extended care, home healthcare, hospice benefits for terminally ill insureds, dental care, vision and hearing care, care for mental and nervousdisorders, care for alcohol and substance abuse and pregnancy and maternity coverage including costs associated withabortion. In addition, they may also provide further protection, with specific limitations, for prescription drugs, bloodtests and transfusions, artificial eyes, artificial limbs, crutches, casts, splints, nurse midwife expenses and wheelchairrentals.

EXCLUSIONS — Exclusions found in medical expense and major medical plans are similar to those appearing inother health insurance policies including loss caused by war; elective cosmetic surgery; routine dental care exceptif necessitated by an accident; routine physical examinations; occupational injuries or disease covered by Workers'Compensation; intentionally self-inflicted injuries; care provided in a government sponsored medical facility; long-term, custodial, convalescent or rest care; vision or hearing care; or private duty nursing. Some medical expenseand major medical plans exclude coverage for mental illness, alcoholism or drug addiction unless required to coversuch benefits by State mandate. Until the passage of the Pregnancy Discrimination Act, many major medical and

NILA ©

L/H Licensing

Chapter 15-3

Page 206: FORWARD - WordPress.com · Return of premium p 6-8 6.0 Annuities p 7-1 Annuity principles and concepts p 7-1 Accumulation period versus annuity period p 7-1 Owner, annuitant and beneficiary

medical expense plans excluded maternity benefits as well. This Act requires that benefit plans sponsored byemployers, with 15 employees or more, treat childbirth, pregnancy and other related conditions the same as anyother illness.

PROVISIONS AFFECTING COST

There are four elements that affect the cost of medical expense insurance including: (1) the maximum benefitselected; (2) deductibles; (3) coinsurance; and (4) stop loss limit. Obviously, the greater the benefit purchased byan insured, the more the cost. The other three elements affecting cost are described as follows:

AVAILABLE DEDUCTIBLES — Most major medical policies include an initial deductible. This is the amountof the covered loss that must first be absorbed by the insured before other policy benefits are paid. A deductible helpsto reduce medical costs since the insured retains a portion of the loss. Therefore, smaller losses will be absorbed bythe insured. Most deductibles are fixed dollar amounts. Policies will possess varying types of deductibles such asinitial deductibles, calendar year deductibles, individual or family deductibles or a corridor deductible. The inclusionof the deductible allows the insurer to provide a high limit of protection at a lower premium rate since the insuredis retaining a portion of the risk. Again, the primary purpose of a deductible is to eliminate smaller claims.

COINSURANCE AMOUNTS — Major medical polices also include a coinsurance clause which requires theinsured to share in a portion of the loss after a deductible is satisfied. Most coinsurance amounts are 80%. Thismeans that once a deductible is satisfied by an insured, the insurer will pay 80% of the remaining medical expenses. The insured's coinsurance amount is 20%. Policies normally include a stop loss limit. The insurer will pay itscoinsurance amount and the insured pays hers. Once the insured has incurred out-of-pocket expenses up to $2,500,for instance, the insurer pays 100% of any further expenses. Therefore, the stop loss limit in this example is $2,500. The coinsurance clause provides an incentive for insureds and physicians to keep expenses within manageable limits. The purpose of the coinsurance clause is to permit insurers to force insureds to share the cost of medical care.

STOP LOSS LIMIT (SLL) — These are the total "out-of-pocket" expenses that an insured incurs as a result ofdeductibles and coinsurance amounts. For example, an insurer may pay 80% of covered medical expenses after theinsured satisfies an initial deductible. Once the insured's out of pocket expenses reach a specified dollar amount(i.e., the stop loss limit), the insurer pays 100% of any further medical expenses for the policy period. Therefore,the SLL is the amount of out-of-pocket expenses the insured incurs during the policy period. Premium can be savedby increasing one's SLL. If the SLL is lowered, the cost increases.

CLAIM SETTLEMENT

COMPUTATION OF MAJOR MEDICAL LOSS — Assume that an insured was covered by a policy witha $500 deductible with a coinsurance clause of 80/20. He/she incurred expenses of $2,000. How much is theinsured responsible for? How much is the insurer responsible for?

! First of all, the insured is responsible for the initial $500 deductible. This leaves $1,500 of remaining ex-penses. With an 80/20 coinsurance clause the insurer is responsible for 80% of $1,500 or $1,200. Theinsured must pay 20% of $1,500 or $300. Therefore, the insured is responsible for $800 ($500 plus $300)and the insurer pays $1,200. In addition, the insured's "coinsurance amount" is $300. The insurer'scoinsurance amount is $1,200.

NILA ©

L/H Licensing

Chapter 15-4

Page 207: FORWARD - WordPress.com · Return of premium p 6-8 6.0 Annuities p 7-1 Annuity principles and concepts p 7-1 Accumulation period versus annuity period p 7-1 Owner, annuitant and beneficiary

! Some major medical or comprehensive major medical plans are written with basic (first dollar) expenses. These plans generally includes a corridor deductible. The corridor deductible must be satisfied after theinsurer pays the basic expenses. Therefore, this deductible falls between basic expenses and major medicalcoverage. This corridor deductible must be satisfied by the insured once the base plan pays up to itsspecified limit. Then the 80/20 coinsurance amount may be applied. For example, assume John is coveredby a major medical plan with basic coverage up to $1,000. The policy then includes a $500 corridordeductible and an 80/20 coinsurance amount. What will each party be responsible for if John incurs $4,000of covered expenses?

The insurer pays the initial $1,000 under the base plan. This leaves $3,000 of remaining expenses. John thensatisfies the $500 corridor deductible. Now there are $2,500 in remaining expenses. John pays for 20% oran additional $500 and the insurer pays $2,000. Therefore, of the entire $4,000 bill, John is responsible for$1,000 and the insurer $3,000. In addition, John's coinsurance amount is $500 (20% of $2,500). Theinsurer's coinsurance amount is $2,000 (80% of $2,500).

OTHER BENEFITS AVAILABLE

SUPPLEMENTAL ACCIDENT BENEFIT — This type of benefit is generally included in major medical plansalthough some insurers may require an extra premium for it. It provides that an insured will receive first dollarcoverage up to a stated amount (i.e., $500) if he or she suffers an accidental injury. Coverage is generally providedup to the stated amount without the requirement of a deductible.

COVERAGE FOR DEPENDENT CHILDREN — Major medical plans, like most health insurance plans, maycover an entire family including the insured, the spouse and the children. As children are born into a marriage theyare covered immediately, although an appropriate premium must be paid for them. Dependent children are generallycovered by their parents health insurance plan until age 19; or if they are a full-time student until age 23 (in someStates it is beyond age 23). Once they have graduated from college, have dropped out of college, get married or areno longer full-time students, they must buy coverage of their own since they will no longer be covered by theirparent’s policy. Coverage may also continue for children who are incapable of earning their own living due to anemotional, mental or physical infirmity. In fact, many States require that these plans cover physically challengedor handicapped dependents. These children are considered (covered) dependents as long as the condition exists andthey continue to be dependent upon their parents. In other words, such children are covered by their parents planuntil they are self-reliant. However, periodic proof of dependency must be provided to the insurer by the parents.

COMPREHENSIVE MAJOR MEDICAL

A major medical plan may sometimes be written with a base plan which pays up to a stated limit initially. Then theinsured satisfies the deductible before the coinsurance amounts are calculated. The comprehensive major medicalplan combines basic (first dollar) expenses and a major medical plan. It may be characterized by an initial orcorridor deductible and provides the most comprehensive (i.e., broadest) health insurance coverage. This means thatit covers virtually all types of medical services and supplies. A comprehensive major medical plan is very similarto a traditional major medical plan except that it provides first dollar coverage up to a specified limit (i.e., basicexpenses), before a deductible needs to be satisfied by the insured. Most policies provide similar coinsurancepercentages as those appearing in major medical plans but others may include varying benefits such as 90/10 ratherthan 80/20.

NILA ©

L/H Licensing

Chapter 15-5

Page 208: FORWARD - WordPress.com · Return of premium p 6-8 6.0 Annuities p 7-1 Annuity principles and concepts p 7-1 Accumulation period versus annuity period p 7-1 Owner, annuitant and beneficiary

EXCLUSIONS — Exclusions found in medical expense and major medical plans are similar to those appearing inother health insurance policies including loss caused by war; elective cosmetic surgery; routine dental care exceptif necessitated by an accident; routine physical examinations; occupational injuries or disease covered by Workers'Compensation; intentionally self-inflicted injuries; care provided in a government sponsored medical facility; long-term, custodial, convalescent or rest care; vision or hearing care; or private duty nursing. Some medical expense andmajor medical plans exclude coverage for mental illness, alcoholism or drug addiction unless required to cover suchbenefits by State mandate. Until the passage of the Pregnancy Discrimination Act, many major medical and medicalexpense plans excluded maternity benefits as well. This Act requires that benefit plans of employers with 15employees or more treat childbirth, pregnancy and other related conditions the same as any other covered illness.

TAX CONSIDERATIONS

Amounts or benefits received by an insured from a major medical or medical expense plan are not taxable asincome. This rule applies whether the policy is a group or individual plan. However, if any reimbursements exceedactual expenses, the excess received by the insured must be included as ordinary income. Premiums paid onindividual policies of medical expense insurance are not deductible. These rules also apply to dental insurancewhich is often offered separately (i.e., stand alone plan) or as part of a major medical expense plan. With regard topartners and sole proprietors, since these entities are considered to be self-employed, premiums paid are currentlydeductible. In addition, premiums paid for individual medical expense policies are tax deductible if they and othernon-reimbursed medical expenses, including premiums paid for certain qualified health insurance policies (i.e., long-term care), exceed 7 ½ % of the insured's adjusted gross income.

DENTAL INSURANCE

This is a specialized form of health expense coverage focusing on diagnostic and preventive treatment that isdesigned to pay for regular dental care and care necessitated by accidents. It is generally written on a group basisand is subject to a deductible. However, many dental plans provide preventive care which is indicated by the factthat there is no deductible for routine dental examinations. Coverage limitations for certain types of treatment arealso included. Coinsurance requirements are generally included in dental coverage as well as exclusions.

Categories of dental treatment — Dental insurance may be covered under a medical expense plan or as a "stand-alone" plan. If included in a medical expense plan, coverage may be provided on a scheduled or nonscheduled basis.If the plan is an "integrated" one, the deductible may be satisfied by either dental or medical expenses incurred. Anintegrated plan is one that includes both dental and medical expense coverage. When dental coverage is offered asan optional benefit, the plan is considered non-integrated. Whether integrated or non-integrated, dental plans maybe either scheduled or nonscheduled. The latter pays benefits on a usual, customary and reasonable basis (UCR). However, a scheduled plan possesses individual categories of treatment with stated limits of coverage for each. Mostdental plans provide coverage for diagnostic and preventive treatment which is not provided under a medical expenseplan. Examples of diagnostic and preventive treatment include oral exams, x-rays and teeth cleaning. Othercategories of treatment are as follows:

Restorative or restoration benefits — These include fillings, crowns and other procedures that restore the naturalfunction of teeth.

Oral surgery — This includes coverage for tooth extraction and additional surgical treatment of injuries, diseasesor jaw defects.

NILA ©

L/H Licensing

Chapter 15-6

Page 209: FORWARD - WordPress.com · Return of premium p 6-8 6.0 Annuities p 7-1 Annuity principles and concepts p 7-1 Accumulation period versus annuity period p 7-1 Owner, annuitant and beneficiary

Endodontics — Root canals or treatment for diseases of the dental pulp within teeth.

Periodontics — This involves care due to gum disease of the surrounding and supporting tissues of the teeth.

Prosthodontics — This benefit provides for the replacement of missing teeth or artificial devices such as bridgeworkor dentures.

Orthodontics — This pays benefits for corrective teeth devices such as braces and retainers.

TYPES OF PLANS — Dental plans are generally written on an indemnity basis (i.e., reimbursement) and providea choice of providers (i.e., dentists, specialists, etc.). Scheduled plans are also known as basic plans. They functionlike a surgical expense plan. They generally contain no deductibles or coinsurance amounts and provide first-dollarcoverage. Nonscheduled plans are also known as comprehensive plans and are the most common type of dental plan.These plans include deductibles and coinsurance unlike a scheduled plan. They are generally divided into threecategories including: (1) routine or basic diagnostic and preventive services such as x-rays, cleaning and flouride treatment; (2) restorative services (that are not routine) such as fillings, oral surgery and periodontics; and (3) majorservices such as crowns, orthodontics, facial reconstructions or bridgework. Many standard plans include coveragefor routine care and other benefits may be added for an additional premium. Generally the coinsurance amounts are80% for basic services and 50% for major services and the policy usually includes deductibles. Another type of planavailable that provides dental care is known as a combination plan. These plans combine the features of both of theaforementioned plans. For example, this plan may cover diagnostic and preventive services on a UCR basis butcontain a schedule of benefits for other covered services. All dental plans include limitations, exclusions andpredetermination of benefits. Common exclusions involve cosmetic procedures unless due to an accident, servicesfurnished by governmental agencies, treatment provided that would be covered by Workers' Compensation, duplicatedentures or the replacement of lost or stolen dentures, and oral hygiene instructions or like training. Dental plans mayalso include limitations on benefits such as a calendar year maximum (i.e., $2,500) or a lifetime maximum coveragelimit (i.e., $2,000 for orthodontics). Most dental plans also require a review of suggested treatment to ensure thatit is reasonable and necessary. This is referred to as the predetermination of benefits requirement. Some plans alsorefer to it as pre-certification or prior authorization. If services are not pre-certified, coverage is still provided. However, the dentist and the insured will not know the exact benefits to be paid.

EMPLOYER GROUP DENTAL EXPENSE — As mentioned previously, dental coverage may be written aspart of an integrated plan or a "stand-alone" plan. No matter what type of plan is involved, the insurer continuesto be wary of adverse selection. In some cases, insurers impose more stringent underwriting to minimize, avoid ordecrease adverse selection. For example, some insurers limit or reduce benefits by 50% or exclude them for aspecific period following late enrollments. Other insurers include longer than usual probationary periods which helpsto reduce adverse selection or minimize claims. In addition, dental plans generally include no conversion privilege although such coverage is subject to the continuation rules of COBRA. Rules regarding open enrollment periodsand late enrollees are also applicable to dental plans.

LIMITED POLICIES

Various types of health insurance are provided by limited contracts which generally cover one or a few types oflosses. These policies must stipulate in large print and provide notice to an insured that they only provide limitedbenefits. Limited policies include but are not limited to:

NILA ©

L/H Licensing

Chapter 15-7

Page 210: FORWARD - WordPress.com · Return of premium p 6-8 6.0 Annuities p 7-1 Annuity principles and concepts p 7-1 Accumulation period versus annuity period p 7-1 Owner, annuitant and beneficiary

1. HOSPITAL INDEMNITY / INCOME — Pays a daily benefit to an insured if he or she is confined to ahospital.

2. ACCIDENT ONLY — Pays a benefit if an insured suffers an injury as a result of any type of accident, onor off the job.

3. ACCIDENTAL DEATH AND DISMEMBERMENT (AD+D) — This policy pays a tax free lumpsum death benefit (i.e., principal sum) when an insured dies but also pays lump sum benefits (i.e., capitalsum) when an insured suffers the severance of a limb in an accident or suffers a presumptive disability asa result of the accident (i.e., loss of sight).

4. CANCER AND SPECIFIED DREAD DISEASE — Sometimes referred to as critical illness plans,these limited policies pay a specified amount for treatment of cancer or some other specified disease (i.e.,heart disease) only.

5. TRAVEL ACCIDENT — Pays a specific amount for injuries suffered on common carriers (i.e.,commercial airline, train, bus, etc.).

6. BLANKET POLICIES — Pays a benefit for members of a covered group who are formed for a reasonother than employment. Student members of a team sport, bus excursion passengers, or summer campsmay purchase this coverage.

7. CREDIT DISABILITY — This type of coverage is available to banks, finance companies, retailers orother creditors with regard to loans or time payment purchases. Any payments made by this type of planmust be used to relieve the debtor (i.e., borrower) of making monthly payments during the time he or sheis disabled. Premiums may be paid in full by the creditor, in full by the debtor, or shared by both (i.e.,contributory). The maximum amount of coverage may not exceed the amount of the indebtedness. Inaddition, coverage will decrease as the loan is being paid.

8. HOSPITAL INDEMNITY — Sometimes referred to as a hospital income policy, this type of limitedhealth insurance contract pays a specified benefit when the insured is confined to a hospital due to anaccident or illness. The contract generally pays an amount per day when the insured is in the hospital. Hospital indemnity plans may pay a benefit only if the insured is confined to a hospital due to an accidentand some only pay the stipulated daily benefit if the confinement is due to illness.

9. PRESCRIPTION DRUGS — Separate plans that pay for prescription drugs have been available for manyyears. Even though most individuals receive this coverage through an individual or group health insuranceplan, these limited plans continue to be available. This type of plan covers the cost of drugs (except thosedispensed in a hospital or extended care facility) required to be dispensed by a physician’s prescription. Normally, drugs for which a prescription is not required are not covered (even if a physician writes it on aprescription form for some reason) with the exception of injectable insulin for diabetics.

In addition, this plan does not cover devices or appliances such as bandages, hypodermic needles or chargesto administer drugs. Drugs used for the treatment of sexual dysfunction or infertility are generally notcovered unless required by state law. Since the cost of drugs is increasingly expensive, many plans requirepre-certification before they will be covered. Most of these plans have a copayment requirement when aprescription is being filled. Two methods are used to provide prescription drug coverage including areimbursement approach and a service approach. In the former, an insured pays the cost of the drugs,completes a claim form which is sent to the insurer and he or she is “reimbursed.” In the latter, drugs are

NILA ©

L/H Licensing

Chapter 15-8

Page 211: FORWARD - WordPress.com · Return of premium p 6-8 6.0 Annuities p 7-1 Annuity principles and concepts p 7-1 Accumulation period versus annuity period p 7-1 Owner, annuitant and beneficiary

provided to covered persons by the pharmacy upon receipt of a prescription, proper identification such asa prescription card and a copayment (i.e., $10).

Pharmacy benefit managers (PBMs) administer prescription drug plans on behalf of insurers, HMOs, PPOs,self-funded employers, Blue Cross-Blue Shield plans and third party administrators (TPAs). Theseorganizations even design prescription drug programs for employers or a benefit plan provider. PBMsdevelop networks for the dispensing of prescription drugs and engage in other related activities such as: (1)drug utilization review; (2) physician profiling and education; (3) pharmacy profiling and education and; (4)patient profiling and education. PBMs also integrate formularies into prescription drug plans. A formularyis a list of preferred medications for a specific medical condition developed by a committee of pharmacistsand physicians. A formulary informs doctors of approved uses of new panel drugs on the market andappropriate uses for existing drugs. They also inform physicians of the appropriate times for generic andtherapeutic substitutions.

10. VISION CARE — This coverage is available as an individual or carve-out plan, or through a group healthinsurance plan. It may provide coverage for: (1) routine eye exams by an optometrist or opthamologist: (2)vision correction: and (3) basic frames (upgrades of frames must be paid by the insured). Routine eye examsare generally covered since other health problems such as high blood pressure, diabetes or kidney problemsmay be discovered during a routine eye examination. Benefits are generally provided on a reasonable andcustomary basis. These plans usually do not cover safety lenses, prescription sunglasses, plastic lenses,upgrades of frames, benefits for necessary eye surgery or treatment for eye diseases. The latter twoexclusions are covered by a medical expense plan. In addition, some plans do cover elective procedures toimprove vision such as LASIK surgery.

COST CONTAINMENT

During the past decade numerous cost-saving services have been created by the health care industry. Insurers orhealth care organizations are utilizing a combination of cost containment techniques in order to "manage care" moreefficiently. Several cost-saving methods are available including preventive care, such as wellness programs, physicalexaminations, mammograms or pap smears. Characteristics of preventive care include providing care on an outpatient basis, identifying medical problems early and encouraging early treatment. Preventive care would not includechemotherapy, hospice care or admitting a patient into the hospital. These programs encourage participants to engagein a healthier lifestyle, involving exercise and diet, to prevent or reduce the chances of an adverse medical conditionin the future. Hospital outpatient benefits, such as preadmission testing, help to assess if hospital treatment isnecessary which will reduce health care expenses. Alternatives to hospital services, such as skilled nursing facilities,birthing centers, hospice centers and ambulatory surgical centers have helped to reduce the cost of health care. Casemanagement procedures such as pre-admission certification, prospective review, concurrent review, ambulatoryservices and utilization review have also aided in lowering the cost of care. Pre-admission certification requires aninsured to be examined in an outpatient facility prior to admission for surgery. Diagnostic and other forms of testingwill be performed during pre-admission certification. A walk-in or ambulatory facility may be used to avoidadmitting an ill individual to a hospital.

A new trend with regard to cost savings involves utilization review. This is sometimes referred to as casemanagement. Utilization review, most of which is performed by a physician with expertise in the area beingreviewed, may be performed on a prospective basis, a concurrent basis, a retrospective basis or a combination of allthree. A prospective review involves analyzing a case to determine what type of treatment is necessary. Prospectivereview is used by a physician if he or she wants to find out prior to treatment being provided if such treatment is

NILA ©

L/H Licensing

Chapter 15-9

Page 212: FORWARD - WordPress.com · Return of premium p 6-8 6.0 Annuities p 7-1 Annuity principles and concepts p 7-1 Accumulation period versus annuity period p 7-1 Owner, annuitant and beneficiary

covered by an insured’s medical plan as evidenced by preadmission authorization. Another examples of prospectivereview is second surgical opinions. Medical plans generally require that an insured receive a second opinion whensurgery is initially recommended. The cost of this service is generally paid by the insurer. If the opinions do notconcur, the insured will decide whether or not to have the surgery. Concurrent review involves the monitoring ofa hospital stay by a nurse of a patient while he or she is in the hospital to determine when they will be released, ifthey require home health care or if a transfer to another facility such as a hospice center is warranted. Retrospectivereview involves an analysis of care, after the fact, to determine if it was necessary and appropriate. The purpose ofthis review is not to deny claims but to monitor trends regarding treatment so that future actions may be taken toreduce or eliminate unnecessary health care costs, especially in high cost areas.

GROUP HEALTH INSURANCE

Group health insurance refers to arrangements where coverage is provided for groups of individuals (i.e., employees)under a single master contract. Group plans providing medical expense benefits are not as standardized as group lifeplans or even group disability contracts. Many group medical expense plans are limited to specific types of benefits,although there are some that are broad enough to provide coverage for virtually any type of medical expense. Likeany type of group plan, individuals cannot form a group with the exclusive intent to purchase insurance coverage.The group must form for an employment related purpose. In other words, there must generally be an employer-employee relationship in order to purchase group coverage. The group contract is purchased by the employer.Therefore, the employer is the policy owner of the master contract. The covered employees receive a booklet orcertificate of coverage which describes and verifies benefits provided by the group plan. It also identifies how longcoverage will last and how to file or report a claim. The covered employee is also known as the certificate holder. Another distinguishing feature of group insurance is the use of experience rating. If the group is large enough, theactual claim experience of the group will be a factor in determining the future premium to be charged. The"experience" is determined either at the issue date or at the end of a policy period. If applied at the end of the policyperiod, it is used "prospectively" or "for the future" policy period. Experience rating may also be used to determinethe amount of a refund to which an employer is entitled if he or she possesses a better claim experience thananticipated. Many experts feel that experience rating provides premium equity among all policy owners. Some groupplans utilize a community rating method. This type of method is generally used for smaller groups or individualsand utilizes the identical premium rate structure for all subscribers or groups in a community regardless of their pastor potential loss experience. In other words, the premiums in community rating are generally based upon the overallclaim experience of the insurer. Variations in premium rates generally result from variations in coverage. BlueCross/Blue Shield plans generally function on this basis.

ELIGIBLE GROUPS — As mentioned, employment related groups or individual employer groups are eligiblefor this type of coverage. There are several other "groups" which are eligible for such coverage as well including:(1) negotiated trusteeships; (2) trade associations; (3) customer groups such as creditor-debtor groups; and (4) laborunion groups. Another type of group permitted to provide group medical expense coverage is a MET or MEWAas reviewed in Chapter 12, although these groups are not allowed to function in several States. According to GroupInsurance by Burton Beam, "negotiated trusteeships", sometimes referred to as Taft-Hartley Trusts, are formed asa result of collective bargaining of benefits between a union of employees and their employer. The union membersare in the same or a related industry, such as the United Auto Workers. The Taft-Hartley Act provided for thecreation of Taft-Hartley Trusts. The Act prohibits employers from paying funds directly to a labor union for thepurpose of providing group health insurance to its members. Therefore, payments by an employer may be made toa trust fund created to pay for the member's group health insurance. The group contract is issued to the trustees ofthe fund (i.e., trust)."

NILA ©

L/H Licensing

Chapter 15-10

Page 213: FORWARD - WordPress.com · Return of premium p 6-8 6.0 Annuities p 7-1 Annuity principles and concepts p 7-1 Accumulation period versus annuity period p 7-1 Owner, annuitant and beneficiary

Trade associations are associations that have been formed for reasons other than purchasing insurance. Theseassociations are generally made up of many employers in the same industry, line of business or other common interest(i.e., alumni or professional group). Since adverse selection and administrative costs are greater in a tradeassociation plan, most States require a minimum percentage of participation on the part of the employers who belongto the association. Customer groups, including creditor-debtor relationships, give rise to groups that are eligible for group term life and disability insurance. These types of coverage are made available to creditor-debtor organizationssuch as banks, retailers and finance companies to offer credit insurance on loans or debts. These types of policiesfunction a bit differently than other forms of life or health insurance in that the policyowner (i.e., the bank) must alsobe the beneficiary of the contract as well. Labor unions, again, may establish group plans for members subject tothe Taft-Hartley Act. In addition, State laws generally provide that members should not be forced to pay entirepremiums. The cost of coverage must be paid by either the union alone or split between the union and the members. Multiple Employer Trusts (i.e., METs), again, "are legal entities in the form of trusts which may be sponsored by aninsurer, an independent administrator or some other organization. They are designed to make available group healthinsurance benefits to participants." METs may be: "fully insured when sponsored by an insurer; insured whenmanaged by a third-party administrator (TPA) other than an insurer; or self-funded which means they are establishedand marketed by the persons or organizations who will manage or administer them. Self-funded indicates thatpremiums are paid by subscribing employers.

Other groups may be eligible for group insurance under State regulation. Groups such as alumni associations,veterans associations, professional groups, credit card holders and savings account depositors may be eligible as well. Blanket customer groups may secure coverage through a group plan to cover sports teams, passenger on commoncarriers and others.

MARKETING CONSIDERATIONS — Insurers must take into consideration State regulatory requirements whenadvertising and marketing group and any other type of health insurance plan. Regulations provide for very stringentpenalties in the case of unfair marketing or sales practices in any State locale. In addition, according to GroupInsurance by Burton Beam, a group contract will generally insure individuals residing in more than one State. Thisraises a question as to which State possesses regulatory jurisdiction over the contract. Under the doctrine of comity,by which States recognize within their own territory the laws of other States, it is generally accepted that the Statein which the group insurance is delivered to the policy owner is the State of jurisdiction. This means that the groupcontract must only conform to the laws and regulations of the acceptable place of delivery (i.e., State of delivery).

EMPLOYER GROUP HEALTH INSURANCE — Employers may offer group health insurance coverage toemployees. Group coverage is not underwritten in the same manner as an individual policy. In other words, grouphealth insurance underwriting criteria is far different than individual health insurance underwriting. Insurers reviewthe characteristics of a particular group as well as plan design factors, persistency factors and administrativecapabilities. An insurer views persistency in a couple of different ways. This concept is important to the insurer asfar as keeping business “on its books,” so to speak. It is likewise important to an insurer if an employer or businesscontinually changes insurers. Groups that change insurers frequently are not looked upon as sound risks to insure. Generally, medical or physical examinations are not required as part of the underwriting process since the numberof participants is greater; sometimes in the thousands. This larger number of covered persons will reduce the chancefor adverse selection as mentioned previously in Chapter 1. In addition the administrative cost of group underwritingis less than that for individual underwriting.

The size of the group and its other characteristics, such as what entity is responsible for the payment of the premium,will help determine cost. If the plan is contributory where premiums are shared by employer and employee, at least75% of eligible employees must participate. If the plan is noncontributory, 100% of eligible employees mustparticipate. With regard to eligibility for group coverage, according to Group Insurance, "an eligible employee mustbe in a covered classification, must satisfy any probationary period required and be actively working full-time."

NILA ©

L/H Licensing

Chapter 15-11

Page 214: FORWARD - WordPress.com · Return of premium p 6-8 6.0 Annuities p 7-1 Annuity principles and concepts p 7-1 Accumulation period versus annuity period p 7-1 Owner, annuitant and beneficiary

Rarely is coverage provided for part-time employees. The element that limits a part-time worker’s eligibility is thenumber of hours he or she works; not one’s age, income or total length of time that the employee has worked forthe organization. Further, in Group Insurance, Burton Beam states that "the same medical expense benefits that areprovided for an eligible employee will also be available for the employee's dependents." Again, rarely are benefitsprovided for a dependent spouse and children unless the employee is covered by the group plan. Coverage isgenerally provided as of the date that the employee's coverage commences. If the group plan is contributory andcoverage for a dependent is not elected within 31 days after becoming eligible, future coverage will only be availableafter an annual open enrollment period or when insurability is demonstrated to the satisfaction of the insurer. Again,the open enrollment period is available once a year for those employees who do not enroll when initially eligible.

Group plans, like individual plans, generally include a coordination of benefits provision. This provision protectsan insurer against any duplications of coverage. The provision prevents the covered employee from receiving morein benefits than in expenses incurred. In other words, it prevents the covered employee from profiting as a result ofbeing covered by more than one policy. This provision establishes the primacy of insurance policies. In other words,priorities are established for the payment of benefits under each policy. As stated in Group Insurance, "coverageas an employee is primary to coverage as a dependent." The length of time that coverage has been in force willsometimes determine primacy. If primacy cannot be established for some reason, the plan having provided coveragefor the longest period of time will be deemed primary.

When coverage is changed, whether benefits are modified or reduced or an employer changes insurance companies,covered employees and dependents may be affected. Coinsurance amounts and any deductible carryover must beconsidered. In some cases, when a plan is changed, an employee is allowed to apply any deductible incurred duringthe final quarter of the year to the following year's deductible under the subsequent plan. The purpose of the carry-over provision is to credit expenses incurred so as not to penalize the insured. No loss-no gain provisions are alsoapplicable which prevent a covered employee from profiting from an accident or illness.

Laws vary by State with regard to the termination of group coverage. Unless State law intervenes, coverage for anemployee and his or her dependents terminates on the date employment is terminated. However, other termination possibilities exist depending upon the scenario involved including but not limited to: (1) the date on which anemployee ceases to be eligible for whatever the reason: (2) the date when a master policy is surrendered or canceled;(3) the date when benefits are exhausted; or (4) at the end of a period for which an employee has made any requiredcontribution. Many medical expense plans provide extended benefits for a covered employee or dependent who istotally disabled at the time of termination. However, the disability must have resulted from an illness or injury thatoccurred while the person was covered under the employer’s policy. Usually, the same level of benefits is availableto the employee or dependent. The extended benefits will continue until the disability ceases but generally not formore than twelve months.

Coverage under group plans may be continued for certain employees and dependents even after employment has beenterminated. COBRA, for example, provides for the continuation of group coverage after an employee has beenterminated or dies. COBRA was briefly described in Chapter 13. Many group plans provide an extension of benefitsonce employment is terminated. For instance, any covered employee or dependent who is totally disabled at the timeof termination may continue to receive benefits, as long as the injury or illness occurred during the period that theemployee or dependent was covered by the plan. The level of benefits will vary as will the length for which coverageis provided. Group plans also provide a conversion privilege as well. This allows an employee whose employmentis terminated to simply purchase an individual health insurance plan at a higher individual rate. Proving insurabilityis generally not required if an employee wishes to convert to an individual plan, although the new individual planmay not include the same benefits as in the group plan. Therefore, there are many ways in which to secure coverageafter benefits are terminated under a group plan.

NILA ©

L/H Licensing

Chapter 15-12

Page 215: FORWARD - WordPress.com · Return of premium p 6-8 6.0 Annuities p 7-1 Annuity principles and concepts p 7-1 Accumulation period versus annuity period p 7-1 Owner, annuitant and beneficiary

GROUP VS INDIVIDUAL INSURANCE

GROUP INDIVIDUAL

One master policy is issued to the group Each person covered possesses his or her own policy

Covered members have the same benefits / coverages Each person selects his or her benefits /coverages

Only eligible group members can apply Any individual can apply

Group underwriting (lower administrative costs) Individual underwriting (greater administrative costs)

Coverage ceases when the member leaves the group Coverage continues as long as premium is paid

Less expensive premium with few restrictions More expensive with more restrictions

The essentials of group versus individual forms of insurance apply to life and health insurance

SMALL EMPLOYER MEDICAL EXPENSE INSURANCE — Specific regulations must be followed byinsurers who market group health insurance plans to employers of small businesses. Generally, this is defined as agroup which has 25 or fewer employees working on at least 50% of the days during the previous calendar quarter(some States allow up to 50). With regard to participation requirements, eligibility and availability of coverage,benefit plans marketed to such groups must offer coverage to all eligible employees and dependents. In addition,an employer may not be discriminated against due to the nature or category of the business. Coverage under the planmust be renewed unless the employer fails to pay the premium, has engaged in fraud or terminates the business. Further, for the initial year, the policy cannot usually include a pre-existing condition provision of more than one yearthat limits benefits. Plans must also offer standard and essential types of benefits. Unfair limitations may not beincluded since coverage is being offered to a small group. State regulatory authorities stipulate that prohibitedmarketing practices with regard to individual forms of health insurance also apply to group plans. The types of plansavailable for small groups include health care center plans, HMO plans and small employer carrier plans. Withregard to the latter, percentage participation by the employees is an important underwriting consideration. Withregard to renewability, rates can be changed annually due to changes in the mix of employees and the group claimexperience. All policies must be renewed by the insurer except for nonpayment of premium or the failure to meetthe minimum participation requirements. In addition, carriers marketing small employer health plans must offer atleast two health plans to the employer (i.e., basic health plan and standard plan).

Small employer medical plans include a preexisting condition waiting period provision but with limitations. Amedical condition can be treated as preexisting if it was treated, or if a prudent person could have sought medicaladvice, within a specified period which cannot exceed six (6) months. Coverage for preexisting conditions cannotbe excluded for more than twelve (12) months following the effective date of coverage. Some plans are exemptedfrom providing certain types of coverage including dental insurance, vision care, Medicare Supplements, long-termcare and disability income coverage. With regard to small group continuation, coverage can be continued throughCOBRA when an employee has his or her employment terminated.

NILA ©

L/H Licensing

Chapter 15-13

Page 216: FORWARD - WordPress.com · Return of premium p 6-8 6.0 Annuities p 7-1 Annuity principles and concepts p 7-1 Accumulation period versus annuity period p 7-1 Owner, annuitant and beneficiary

REGULATION OF EMPLOYER GROUP PLANS

Federal regulation has affected the establishment and character of group insurance. The following regulations haveinfluenced group insurance plans tremendously during the past two decades.

EMPLOYEE RETIREMENT INCOME SECURITY ACT — The purpose or applicability of ERISA is toprotect the interests of plan participants and beneficiaries in employee benefit plans. In addition, this federal lawwas also intended to protect the interests of beneficiaries of participants as well. Sometimes referred to as thePension Reform Act, ERISA usually applies to pension plans, group insurance plans and other employee welfarebenefit plans. The most important sections of this federal law pertain to fiduciary responsibility, and reporting anddisclosure.

1. FIDUCIARY RESPONSIBILITY — ERISA identifies very specific and detailed standards forfiduciaries and other interested parties who oversee employee benefit plans. A fiduciary is a person or entitywho exercises discretionary authority or control over benefit plan management. It is an entity that must showa high degree of trust and confidence when overseeing assets. ERISA requires that a fiduciary must act inthe sole interest of the plan participants and their beneficiaries. A fiduciary should, according to ERISA: actfor the exclusive purpose of providing benefits to participants and their beneficiaries; defray reasonableexpenses of administering the plan; diversify the investments of the plan to minimize risk of large losses;and govern the plan consistent with the rules and provisions of ERISA. Any fiduciary who breaches theseduties is personally liable for the full amount of any loss. A breach of duty may also result in civil penaltiesas well. ERISA also stipulates rules for interested parties or “parties-at-interest” who may be assisting afiduciary. A party-at-interest includes any counsel or employee of the plan, any person providing servicesto the plan or an employer of a plan.

2. REPORTING AND DISCLOSURE — ERISA requires that certain information concerning anyemployee benefit be made available by the plan administrator to the following: (1) plan participants orbeneficiaries; (2) the Department of Labor; or (3) the IRS. The information provided must include a summaryplan description, a summary of material modifications, an annual return, an annual report, any terminal reportand certain underlying documents. ERISA imposes severe monetary penalties for failure to comply with thesereporting and disclosure requirements.

AGE DISCRIMINATION IN EMPLOYMENT ACT (ADEA) — The ADEA applies to employers withtwenty or more employees and affects employees age 40 and older. This Act stipulates that, with some exceptions,compulsory retirement is not allowed. Therefore, employee benefits must continue for older workers, althoughreductions in benefits are allowed. The Act permits a reduction in the level of some benefits for older employeesso that the cost of providing benefits for such employees is no greater than the cost of providing them to youngeremployees. The exception to this rule is the cost of medical expense coverage. This means that if an employee age65 or older continues to work, he or she must still be offered coverage under the group plan even though the costmay be greater (and assuming he or she is still part of the group). Therefore, primary coverage is provided by thegroup plan. If participation in an employee benefit plan is voluntary, an employer can generally require an increasein or larger employee contributions rather than reducing benefits. For instance, if an employer pays half the costof benefits for younger workers, it must pay half the cost for the older workers. However, if the workers pay theentire cost of a benefit, older workers may be required to pay the total cost of their coverage to the extent that thisis a condition of participation in the plan. The ADEA also applies to group disability and group medical expensecoverage. Again, with regard to medical expense coverage, all employers must offer workers over age 65 the samemedical expense coverage that they offer to younger employees. The benefits cannot be reduced for older employees

NILA ©

L/H Licensing

Chapter 15-14

Page 217: FORWARD - WordPress.com · Return of premium p 6-8 6.0 Annuities p 7-1 Annuity principles and concepts p 7-1 Accumulation period versus annuity period p 7-1 Owner, annuitant and beneficiary

due to the increased cost to the employer. There is no provision in the Act that stipulates that benefits must becontinued for retired workers.

CIVIL RIGHTS ACT / PREGNANCY DISCRIMINATION ACT — The Pregnancy Discrimination Act of1978 was an amendment to the Civil Rights Act of 1964. This law requires that women affected by pregnancy,childbirth or related medical conditions, such as abortion, be treated the same for employment-related purposesas other persons who are not affected by the same conditions. In other words, if a woman is pregnant and is unableto perform her employment duties, she will be entitled to benefits as if she was suffering from any illness coveredby the plan. With regard to disability insurance, pregnancy will be considered a disability like any other inabilityto perform one’s duties. Therefore, if an employer provides any type of disability insurance, sick-leave coverage ormedical expense coverage for employees, the employer must provide coverage for pregnancy and its related medicalconditions on the same basis as other disabilities. This Act applies to disability insurance and medical expensecoverage. This Act applies to benefit plans, both insured and self-insured, covering employers with fifteen (15) ormore employees.

NONDISCRIMINATION RULES — These rules apply to employee benefit plans that provide retirement benefits.Their purpose is to deny favorable tax treatment to plans that do not provide equitable benefits to a large cross sectionof employees. For example, non-qualified deferred compensation plans provided for highly paid executives are legalplans. However, since the plans are not provided for a large number of employees, contributions to the plan will notbe tax deductible. This means that the owners and officers of a firm cannot receive tax favored benefits if a plan isdesigned primarily for them and not for other employees.

RELATIONSHIP WITH MEDICARE — Since most employees and their dependents are eligible for Medicarewhen they reach age 65, group plans must include a provision to prevent any duplication of benefits. In years past,group plans included a provision stipulating that anyone eligible for Medicare benefits would not be eligible forcoverage under a group plan. This type of provision conflicts with the ADEA which prevents discrimination inemployee benefit plans. According to Medicare secondary rules, Medicare is often the secondary payer toemployer-sponsored medical expense coverage. Generally, the group plan covering the employee is primary andMedicare, if applicable, is secondary. Medicare is also secondary in other instances as well. For example, Medicareis secondary when a person is being treated for end-stage renal disease (i.e., kidney failure) with dialysis or kidneytransplant. In this case, the group plan providing coverage is primary for the initial thirty months of treatment. Then Medicare becomes primary and the group plan is secondary. If for some reason the group plan did not providecoverage, Medicare would become primary.

An employer’s plan may also cover certain individuals age 65 or older who are not covered by the provisions of theADEA. This means retirees and active employees of firms with less than twenty employees. Many employers alsoprovide for their employees a Medicare carve-out or supplement plan. Group plan benefits are reduced to the extentthat benefits are payable under Medicare for the same expenses in a Medicare carve-out. This means that, ifapplicable, Medicare pays first and then the group plan pays a portion of the medical expenses. In a Medicaresupplement, benefits are provided that are not covered by Medicare Part A and B such as deductibles, coinsurance,co-payments and other excluded expenses such as prescription drugs.

THE HEALTH INSURANCE PORTABILITY AND ACCOUNTABILITY ACT (HIPAA) — Thislegislation has expanded health insurance coverage regulation. The specific regulation is called the Health InsurancePortability and Accountability Act. The primary purpose of the Act was to help ensure that individuals would notlose medical coverage or be subject to a new preexisting condition period if they changed or lost their job. In otherwords, the Act hopes to increase access and portability of health care insurance and to reduce or eliminate pre-existing condition exclusions or waiting periods when coverage is sought under a new plan. Many of the importantprovisions of HIPAA involve but are not limited to: (1) increased availability of medical expense coverage by placing

NILA ©

L/H Licensing

Chapter 15-15

Page 218: FORWARD - WordPress.com · Return of premium p 6-8 6.0 Annuities p 7-1 Annuity principles and concepts p 7-1 Accumulation period versus annuity period p 7-1 Owner, annuitant and beneficiary

limitations on what can be included in preexisting conditions; (2) increased portability of medical expense coverage;(3) expansion of eligibility for COBRA benefits; and (4) favorable tax treatment with regard to certain healthinsurance plans (i.e., LTC insurance). HIPAA applies to groups of two or more. It does not govern disabilityinsurance. Key provisions included in this recent law include but are not limited to:

1. PORTABILITY — Provisions in HIPAA regarding portability do not allow employees to take specificinsurance from one job to another. It places limitations on preexisting condition exclusions and allowsemployees to use evidence of prior insurance coverage to reduce or eliminate the length of any preexistingcondition exclusion when employees become covered by another medical expense plan. The portabilityprovision applies to almost all group health plans that have at least two participants (i.e., employees) onthe first day of the plan year. This regulation does not apply to disability insurance. A creditable coveragecertificate must be issued to qualified employees by employers upon request.

If an individual (and/or family) was covered by another group plan previously, with no break or gap incoverage of 63 days or more (i.e., cannot be more than 62 days), he or she is qualified to receive a certificate(assuming they are attempting to receive coverage under a new plan). In other words, if an employee hasbeen without coverage for at least 63 days between jobs, the full preexisting condition period applies. Thepurpose of this certificate is to prove creditable coverage and also to reduce a pre-existing waiting period. In other words, an employee who has one month of creditable coverage may apply this toward thesatisfaction of one month (i.e., month to month) of "pre-existing waiting period."

2. ESTABLISHING ELIGIBILITY — State law prohibits a group health policy from establishing eligibility

for enrollment or coverage based upon a person's individual health status, physical or mental medicalcondition or history, genetic background, claim history (i.e., experiences), evidence of insurability, disabilityor whether the person has received health care in the past.

3. PENALTY EXEMPTION — An exemption is now provided from the 10% penalty tax on premature IRAwithdrawals taken after 1996 if they are utilized to pay medical expenses in excess of the 7 ½% of a person'sadjusted gross income. A further exemption of this penalty tax (i.e., excise tax) is provided if a withdrawalis made to pay for medical insurance if the individual has received unemployment compensation for at leasttwelve weeks.

4. TAX CONSIDERATIONS — The tax treatment of long-term care insurance was made more favorableby this Act with regard to qualified contracts. If the plan is qualified, the premium paid for it (up to aspecific cap that varies each year) may be combined with other unreimbursed medical expenses and may bedeductible if this amount exceeds 7 ½ of the individual’s adjusted gross income. Benefit amounts receivedare generally not taxable unless they exceed certain per diem levels.

! The permitted tax deduction for the health insurance premiums of self-employed individuals (andfamily) is now the entire premium paid. It was formerly a partial deduction starting in 1996.

5. PRE-EXISTING CONDITIONS — According to HIPAA, the limitations found in a preexisting conditionexclusion appearing in a medical plan are allowed only if treatment, advice, care or diagnosis for thecondition is received by the insured within six months prior to an employee’s attempt to enter a new groupmedical plan. If an insured is suffering from an undiagnosed condition that is later uncovered, the preexistingcondition limitation does not apply. Maternity, births and adoptions will not be subject to preexistingcondition language, even if genetic information is available suggesting that members of a person’s familymay develop a particular condition. However, the latter two must be added to the policy within thirty daysafter birth or adoption.

NILA ©

L/H Licensing

Chapter 15-16

Page 219: FORWARD - WordPress.com · Return of premium p 6-8 6.0 Annuities p 7-1 Annuity principles and concepts p 7-1 Accumulation period versus annuity period p 7-1 Owner, annuitant and beneficiary

HIPAA has limited the period of time for which a "group health" policy may deny coverage for pre-existingconditions to twelve (12) months. In the case of a late enrollee, the pre-existing condition limitation is noweighteen (18) months. The length of the preexisting condition period must be reduced by the length ofcreditable coverage earned. An individual will receive creditable coverage if he or she receives medical careprotection under: (1) a group health plan; (2) an individual medical plan; (3) Medicare; (4) Medicaid; (5) amilitary-sponsored medical plan; (6) a State medical services risk pool; (7) the Peace Corps; or (8) a medicalcare program of the Indian Health Service.

If an employee has twelve (12) months of creditable coverage from a previous plan, he or she may use thatto reduce the preexisting condition exclusion waiting period under a new plan. For example, Dan workedfor an employer and had coverage under the employer’s health plan from March 1, 2002 until April 5, 2003. He then changed jobs. Since he was covered by the previous plan for more than twelve months (March 1,2003 is called his enrollment date), no preexisting condition exclusion can be applied. In addition, the twelvemonths of creditable coverage does not have to come from a single employer.

6. GUARANTEED ISSUE — New statutes passed recently also provide important guarantees with regard

to health insurance coverage. All small employers are guaranteed to be able to have health insurance madeavailable to them. Insurers cannot refuse to cover small groups (i.e., 2 to 50 employees) based on the healthhistory of group members. Larger groups are not guaranteed the availability of coverage, but once they havecoverage they are guaranteed renewability. Renewability of any group may be denied for nonpayment ofpremium, fraud, termination of coverage in the market or failure to meet participation or contributionrequirements.

7. RENEWABILITY — State law provides for the guaranteeing of health insurance coverage for all grouphealth plans unless the plan has: (1) engaged in fraud; (2) terminated coverage; (3) failed to pay the premium;(4) moved beyond the service area; or (5) terminated association membership. With regard to individualplans, a similar guarantee of renewability is also provided unless an individual has engaged in theaforementioned acts (i.e., fraud, nonpayment, etc.).

TYPES OF FUNDING AND ADMINISTRATION

During the past decade employers have considered benefit funding methods as an alternative to the conventional ortraditional fully insured group health insurance policy. The traditional form of group insurance is characterized byan employer paying premiums and an insurer paying claims. The reason why employers have been looking foralternatives to the traditional group plan is to reduce the cost of health insurance coverage and improve cash flow.Since the cost of medical care has increased during the past twenty years, the cost to provide adequate coverage foremployees has increased as well.

METHODS OF ALTERNATIVE FUNDING — The opposite of a conventional or traditional fully insuredgroup plan is a self-funded plan. In a totally self-funded plan, an employer is responsible for paying claims,administering the plan and bearing the risk that actual claims will exceed those expected. Most employers do notoperate totally self-funded plans since they do not wish to bear the entire risk. The methods used by employers fallsomewhere between conventional fully funded plans and totally self-funded plans. There are three general methodsof alternative funding used today including: (1) modified fully insured plans; (2) partially self-funded plans; and(3) fully self-funded (self-administered) plans.

NILA ©

L/H Licensing

Chapter 15-17

Page 220: FORWARD - WordPress.com · Return of premium p 6-8 6.0 Annuities p 7-1 Annuity principles and concepts p 7-1 Accumulation period versus annuity period p 7-1 Owner, annuitant and beneficiary

MODIFIED FULLY INSURED PLANS — There are several types of alternative funding methods that areregarded as modifications of traditionally fully insured plans (i.e., buying coverage from an insurer) since an insurerhas the ultimate responsibility for paying all benefits promised under the contract. Generally, most insurers permitonly medium and large employers to use these modifications. Some of the more common types of modified fullyinsured plans include but are not limited to:

1. PREMIUM-DELAY ARRANGEMENTS — This arrangement allows the employer to defer the paymentof premiums beyond the normal thirty day grace period. Most of these arrangements lengthen this graceperiod to 60 or 90 days. This allows the employer use of the funds for other purposes. In addition, itprovides a financial advantage to the employer in that he or she can earn more interest on the amount of thedelayed premiums. Once an insurance contract is terminated, the employer is responsible for any deferredpremiums.

2. RESERVE REDUCTION ARRANGEMENTS — This is similar to a premium-delay arrangement. Inthis case, the employer is permitted to retain an amount of the annual premium that is equal to the claimreserve. This arrangement allows the employer the use of the retained funds. Again, if the insurance contractis terminated, any deferred premiums must be paid by the employer.

3. RETROSPECTIVE RATING ARRANGEMENTS — According to this arrangement, an insurercharges the employer an initial premium that is less than what would be justified by expected claims for theyear. This discount may be up to 10% of the traditional group insurance premium. Then, if claims exceedthe premium paid, the employer must pay an additional fee at the end of the policy period (i.e., retrospec-tively). Again, the savings during the year allows the employer use of such funds.

PARTIALLY SELF-FUNDED PLANS — Partially self-funded plans are those where an employer bears aportion of the risk with regard to medical expense plans. The major problem with a fully self-funded medicalexpense plan is that it is difficult for an employer to estimate the average severity of claims. Most employers areunwilling to assume the entire risk but do have at their disposal some partially-self-funded arrangements includingbut not limited to:

1. STOP-LOSS COVERAGE — This allows an employer to utilize elements of self-funding to protect itselfagainst a catastrophic claim. Stop-loss coverage is one form of protection for employers against anunexpectedly high level of claims. If total claims during the year exceed some specified dollar limit, aninsurer will assume the responsibility for the claims beyond this stop-loss limit (up to its maximum limitspecified in the contract). This partially self-funded plan improves the employer’s cash flow since it limitsits financial risk. The employer is also subject to a deductible which is the amount the employer mustassume before the stop-loss insurer begins to pay.

2. ASO CONTRACTS — Known as an Administration Services Only (ASO) contract, this plan stipulatesthat the employer purchases specific administrative services from an insurer (or from an independent TPA). These services usually include the administration of claims but could also include prescription drug cards,COBRA administration and employee communications. This means that an employer has the option topurchase services for those administrative functions that can be handled more cost effectively by anotherparty (i.e., insurer or TPA). If an administrator handles claims, it pays a claimant or provider from a bankaccount belonging to the employer. The administrator is not responsible for paying claims from its ownassets if the employer’s account is insufficient. The fee paid to the administrator may be a percentage of theamount of the paid claim, a flat amount per processed claim or a flat charge per employee or for theemployer.

NILA ©

L/H Licensing

Chapter 15-18

Page 221: FORWARD - WordPress.com · Return of premium p 6-8 6.0 Annuities p 7-1 Annuity principles and concepts p 7-1 Accumulation period versus annuity period p 7-1 Owner, annuitant and beneficiary

3. 501 (c) (9) TRUST — The Internal Revenue Code provides for the establishment of voluntary employee’sbeneficiary associations (VEBAs) through Section 501-c-9. This is a funding vehicle for the employeebenefits that are offered to members. The funds accumulating in the trust are used for employee benefitplans. Contributions made to the trust are tax deductible just as if they were paid to an insurer as a premium. Any appreciation earned on the assets of the trust are free of taxation as well. If a trust is not established tohandle these funds, contributions are not tax deductible unless they are used for other employee benefits andany earnings are subject to taxation. The cost to establish and maintain this trust is prohibitive (i.e.,expensive). Membership in the trust is restricted to those persons who share a common employment-relatedbond. In addition, membership must be voluntary on the part of employees. The trust must provide eligiblebenefits only (i.e., medical expense, death, disability, etc.) and its sole purpose must be to provide benefitsto its members or their beneficiaries. Finally, the trust must be controlled by: (1) its membership; (2) anindependent trustee such as a bank; or (3) some other fiduciary on behalf of the members.

FULLY SELF-FUNDED / SELF-ADMINISTERED — Totally self-funded (or self-insured) employee benefitplans are the opposite of traditionally fully insured group insurance plans. Characteristically, in a fully self-insuredplan, the employer is responsible for paying claims, administering the plan and bearing the risk that actual claims willexceed those expected. The advantages of self-funded plans are that State premium taxes are eliminated, state-mandated benefits are avoided and cash flow is improved. In addition, an employer may also be able to reduce hisor her operating expenses to the extent that the self-funded plan can be administered at a lower cost than purchasingcoverage from an insurer. An employer is best served to consider a fully self-funded plan if conditions are suitableincluding: (1) the severity of claims are predictable; (2) the plan is noncontributory; (3) employees are not membersof a union, (4) the employer possesses the ability to effectively and efficiently handle claims; (5) the employerpossesses the ability to provide other administrative services (i.e., plan design, communication to employees,compliance with government regulations, etc.); and (6) the employer has the ability to negotiate discounts with healthcare providers (i.e., physicians, hospitals).

The benefits offered by employers that are most suitable for self-funding include short-term disability, dental care,vision care, prescription drugs and legal expense coverage. The major problem with self-funding medical expensecoverage, again, is the severity of claims. Small and medium size employers will shy away from self-fundingalthough many small employers will self-fund basic medical expense benefits and insure major medical benefits withan insurer. Larger employers with thousands of employees are more apt to engage in self-funding. Other types ofinsurance that present an employer with a problem when he or she considers self-funding includes long-termdisability, life insurance and AD&D coverage, since large claims involving these areas are difficult to predict.

MEDICAL SAVINGS ACCOUNTS

A medical savings account is a personal savings account that may be used to pay for unreimbursed medical expensesincluding deductibles and coinsurance amounts. Coinsurance amounts are also known as co-payments. Employersmay even sponsor an MSA but they are for the benefit of an employee and are portable. If the employee leaves theemployment of his or her employer, the MSA may be taken along. The MSA may also be described as a tax-deferredbank or savings account combined with a low premium/high deductible insurance policy designed for individuals orfamilies in order to fund health care expenses or medical insurance. Participants are encouraged to invest thepremium savings in an MSA by making tax-deductible contributions to the MSA. Unused monies in the accountmay be rolled over tax-free and when a threshold is reached, may be invested to pay for future medical expenses andmore general uses after retirement.

NILA ©

L/H Licensing

Chapter 15-19

Page 222: FORWARD - WordPress.com · Return of premium p 6-8 6.0 Annuities p 7-1 Annuity principles and concepts p 7-1 Accumulation period versus annuity period p 7-1 Owner, annuitant and beneficiary

ELIGIBILITY – Anyone who is self-employed or an employer with 50 or fewer employees is eligible to choose anMSA as an alternative to current ways of paying for medical care. For the self-employed and others, an MSA canbe established by purchasing the high-deductible insurance policy and depositing the amount of the deductible in theMSA account. Health expenditures from the MSA account are tax-free and accumulated balances earn interest whichis also tax-free if used to pay for medical expenses incurred.

CASH CONTRIBUTIONS – The maximum annual MSA contribution allowable is generally 65% of the medicalplan deductible for an individual and 75% if a family plan. Contributions can be made by the individual (i.e., accountholder) or the employer but not both in the same year. If the employer is the contributor to the MSA, there must becomparable contributions for all participating for each plan year.

TAX TREATMENT – Contributions made by an individual are deductible from income thus reducing one’sadjusted gross income. Contributions made by an employer are deductible as a business expense and excluded fromthe income of an employee, therefore not subject to FICA taxes.

DISTRIBUTIONS – Distributions for qualified medical expenses are not subject to income taxes provided that theexpenses were not covered by a health insurance policy. If the account holder (i.e., employee) becomes disabled ordies, the funds in the account will be distributed subject to income taxation but no penalty. Distributions for otherreasons prior to age 65 will be subject to income taxation and a 15% penalty tax. Distributions for other reasons afterage 65 will be subject to income taxation but no penalty. Amounts not used by year end are not forfeited and canaccumulate to a substantial amount on a tax-deferred basis. With regard to estate tax considerations, if the ownerof an MSA dies, the balance of the account is included in his or her gross estate for estate tax purposes.

ESTABLISHING AN MSA – An individual high deductible health plan may include a deductible range of $2,000(single) to $4,050 (family), with a stop-loss maximum of $4,050 and $7,400 respectively.

QUALIFIED MEDICAL EXPENSES – The following are some examples of qualified medical expenses thatcan be paid with tax-free funds from an MSA:

! Acupuncture

! Alcohol treatment

! Ambulance charges

! Artificial limbs or teeth

! Birth control pills

! Braces

! Chiropractic care

! Christian Science treatment

! COBRA premiums

! Contact lenses

! Cosmetic surgery

! Dental treatment

! Eyeglasses

! Health insurance

! Hearing aids

! Insurance premiums

! Lab fees

! Legal abortions

! Long-term care premiums

! Meals

! Nurse midwife costs (in home or birthing

center)

! Oxygen

! Physician services

! Psychiatric care

! Psychologist fees

! Seeing eye dog

! Special education

! Sterilization

! Surgeon’s fees

! Therapy

! Vasectomy

! Wheelchair

NILA ©

L/H Licensing

Chapter 15-20

Page 223: FORWARD - WordPress.com · Return of premium p 6-8 6.0 Annuities p 7-1 Annuity principles and concepts p 7-1 Accumulation period versus annuity period p 7-1 Owner, annuitant and beneficiary

HEALTH SAVINGS ACCOUNTS

This is a tax-sheltered savings account similar to the IRA, but earmarked for medical expenses. It is referred by someas the next generation of MSA. Deposits to an HSA are 100% tax deductible for the self-employed and can bewithdrawn to pay for routine medical bills with tax-free dollars. What is not used from the account each year staysin the account and continues to grow interest on a tax-favored basis. These funds can supplement retirement just likean IRA. When combined with a lower cost, high deductible medical insurance plan, the HSA provides moreflexibility and choices for the individual and/or family. For example, determine the amount of premium spent on ahigh cost traditional medical plan. Use a portion of that premium to purchase a lower cost, high deductible medicalplan. Deposit the amount left over into the tax deductible HSA. This account can be used to pay for smaller coveredmedical expenses until the deductible is met. Up to 100% of the deductible amount can be contributed to the HSA,subject to maximum limits.

O ELIGIBILITY — Any individuals under the age of 65 is eligible to contribute to an HSA if they are coveredby a government approved (i.e., qualified) high deductible health plan (HDHP). In addition, any individualis eligible who: (1) is not covered by any other health plan that is not an approved HDHP, except permittedinsurance such as Workers’ Compensation, insurance for a specified disease or illness, and insurance thatpays a fixed amount per day for hospitalization; (2) is not entitled to benefits under Medicare; and (3) maynot be claimed as a dependent on the tax return of another person.

O QUALIFIED WITHDRAWALS — These are tax-free (and penalty free) at any age if used to pay for medical expenses including but not limited to: (1) physician’s visits; (2) prescription drugs; (3) chiropractic;(4) dental; (5) vision; (6) many alternative therapies such as acupuncture; and (7) all other traditional in-patient and out-patient medically related expenses (as those covered under an MSA). Withdrawals may alsobe used to pay for COBRA premiums, health insurance premiums while an individual is receivingunemployment compensation or long-term care insurance premiums. Eligible expenses are the same asthose deemed eligible under an MSA.

O PENALTIES FOR NON-QUALIFIED WITHDRAWALS — Non-qualified withdrawals prior to age 65 will be subject to a 20% penalty in 2011 and must be reported as ordinary income. Non-qualifiedwithdrawals over age 65 must be reported as ordinary income in the year withdrawn (like an IRA).

O DEDUCTIBLES AND CONTRIBUTIONS — In order to qualify for an HSA, the law requires that anindividual carry a high deductible health plan. The size of the deductible depends upon the number of peopleactually being insured. If there is one person to be issued, it is classified as a single or “self-only” plan. Ifthe policy covers two or more people, it is classified as a family plan. In 2011, the minimum deductibleallowed for self-only coverage is $1,200 ($5,950 for a stop loss / out of pocket limit) and the maximumallowable contribution limit is $3,050. For a family plan, the amounts are $2,400 and $6,150 ($11,900 outof pocket) respectively.

O FUNDING — HSAs can be funded by a combination of employer, employee and family membercontributions. However, all contributions made by or on behalf of an eligible individual are aggregated forthe purpose of applying the maximum limit. Rollovers from MSAs and other HSAs are not subject to annualcontribution limits. Rollovers from an IRA or Flexible Spending Account (FSA) are not permitted.

O TAX TREATMENT — Contributions made by a family member on behalf of an eligible individual aretax deductible. Employer contributions on behalf of an eligible individual are tax deductible to the employerand excluded from employee income tax and not subject to FICA payroll taxes. Distributions from an HSA

NILA ©

L/H Licensing

Chapter 15-21

Page 224: FORWARD - WordPress.com · Return of premium p 6-8 6.0 Annuities p 7-1 Annuity principles and concepts p 7-1 Accumulation period versus annuity period p 7-1 Owner, annuitant and beneficiary

used exclusively to pay for qualified medical expenses of the account beneficiary, his or her spouse ordependents are income tax free. Distributions not used exclusively to pay for qualified medical expensesof the account beneficiary (spouse or dependents) will be included in the recipient’s gross income and issubject to the 20 % federal tax penalty (in 2011). If a distribution is made due to a disability of the accountholder, the amount is subject to taxation but not the tax penalty.

FLEXIBLE SPENDING ACCOUNTS

A flexible spending account is an IRS Section 125 cafeteria plans that allows employees the option of pre-tax payrolldeduction for some insurance premiums, unreimbursed medical expenses and child / dependent care expenses. Employers use this to retain quality employees and help hire new employees. Employers save when employees electpre-tax payroll deduction because lower adjusted gross income also reduces matching FICA and FederalUnemployment Tax.

QUIZ

1. Bill is covered by a comprehensive major medical plan providing basic first dollar coverage of $1,000. The policy also

includes a $500 corridor deductible and coinsurance of 80/20. If Bill incurs covered expenses of $3,000, what will be

his coinsurance amount?

A. $300 C. $800

B. $500 D. $1,200

2. Tom is covered by the same plan as Bill. He incurs covered expenses of $4,000. How much of this total bill will be paid

by the insurer?

A. $500 C. $2,000

B. $1,000 D. $3,000

3. The maximum amount of expenses that an insured is responsible for before a major medical plan pays 100% of remaining

expenses is known as the:

A. Coinsurance limit C. Percentage participation

B. Limit of liability D. Stop loss limit

4. Health insurance policies include an exclusion provision. All of the following would be excluded under an accident and

health policy, EXCEPT:

A. Self-inflicted injury C. Cosmetic medical expenses

B. Hearing examination D. Eye examination

5. Which of the following provides for the availability of health insurance benefits for small companies with common

employments which band together?

A. Multiple funded plans C. Multiple employment associations

B. Multiple employer trusts D. Long term care associations

NILA ©

L/H Licensing

Chapter 15-22

Page 225: FORWARD - WordPress.com · Return of premium p 6-8 6.0 Annuities p 7-1 Annuity principles and concepts p 7-1 Accumulation period versus annuity period p 7-1 Owner, annuitant and beneficiary

6. Surgical expense benefits are paid according to a:

A. Reimbursement basis C. Schedule

B. Prepaid limitation D. Stated offset

7. Which of the following best describes a comprehensive major medical policy?

A. It is a combination of basic expenses and surgical expenses

B. It combines major medical and limited policy protection

C. It combines basic expenses and major medical protection

D. It combines blanket protection and physician’s expenses

8. What is the purpose of the coinsurance clause?

A. To lower claim amounts

B. To aid an insurer in better servicing the insured

C. To encourage an insured to share in the cost of care

D. To provide long-term care to an insured

9. Physician’s expense coverage is also referred to as:

A. Basic hospital expense C. Direct response expense

B. Surgical expense D. Out-patient expense

10. Once an insured satisfies a stop-loss limit, an insurer generally pays what amount of any excess expenses?

A. 75% C. 90%

B. 80% D. 100%

11. Coverage for newborn infants is provided by a medical expense policy:

A. When a child reaches age two C. From the moment of birth

B. When a premium is paid D. 31 days following his or her birth

12. Which of the following policies pays a benefit if an insured suffers an illness due to cancer only?

A. Accident only policy C. Dread disease policy

B. Sickness income policy D. AD & D rider

13. What is the primary purpose of a medical expense insurance deductible?

A. To increase the total premium C. To reduce policy limitations

B. To broaden policy benefits D. To eliminate smaller claims

14. A policy paying benefits on a relative value basis is most probably a:

A. Hospital expense plan C. Dread disease plan

B. Surgical expense plan D. Physician’s expense plan

NILA ©

L/H Licensing

Chapter 15-23

Page 226: FORWARD - WordPress.com · Return of premium p 6-8 6.0 Annuities p 7-1 Annuity principles and concepts p 7-1 Accumulation period versus annuity period p 7-1 Owner, annuitant and beneficiary

15. If an insured's 20-year old son drops out of college, will he continue to be covered under his parent's major medical

policy?

A. No, since coverage for dependent children terminates at age 19

B. No, since he is no longer a full-time student

C. Yes, as long as he continues to live in his parent's household

D. Yes, as long as the proper premium is paid

16. Which of the following is an eligible dependent who may be covered by a group or individual medical expense plan?

A. A legally separated spouse of the insured

B. A married child living in the insured’s household

C. A step child of the insured

D. The insured’s twenty-six year old son living in the household

17. Which of the following is an example of an endodontic procedure?

A. Tooth extraction C. Retainers and braces

B. Root canals D. Dentures and bridgework

18. Which of the following is an illustration of a prosthodontics procedure?

A. Tooth extraction C. Retainers and braces

B. Root canals D. Dentures and bridgework

19. Which of the following can a doctor use if he or she wants to find out prior to treatment being provided if such treatment

is covered by an insured’s medical plan?

A. Concurrent review C. Comprehensive review

B. Retrospective review D. Prospective review

20. According to HIPAA, employers must make full health care coverage available immediately to new employees who were

previously covered by group health insurance at their old job for at least how many months?

A. Six C. Nine

B. Eight D. Twelve

21. An employee actively at work on the date coverage is transferred to another insurance carrier is:

A. Covered following a thirty-day waiting period

B. Covered automatically and exempt from any probationary period

C. Covered automatically but subject to a probationary period

D. Covered subject to the preexisting condition limitation

22. When employees are actively at work on the date coverage can be transferred to another insurance carrier, which of the

following occurs?

A. The risk must be reevaluated

B. Deductibles and coinsurance are carried over to the new plan

C. The deductible is carried over to the new plan

D. The coinsurance amount is carried over to the new plan

NILA ©

L/H Licensing

Chapter 15-24

Page 227: FORWARD - WordPress.com · Return of premium p 6-8 6.0 Annuities p 7-1 Annuity principles and concepts p 7-1 Accumulation period versus annuity period p 7-1 Owner, annuitant and beneficiary

23. Which of the following provides comprehensive insurance coverage ?

A. Vision care C. Medical expense

B. Dental care D. Prescription drug benefits

24. Which of the following is based upon a fee for service approach?

A. Blue Cross and Blue Shield C. Individual disability plan

B. Health Maintenance Organization D. Major Medical plan

25. Services provided by a nurse midwife are covered when they occur:

A. In a birthing center C. Prior to the policy period

B. In a skilled nursing facility D. Following the policy period

ANSWERS

1. A2. D3. D4. A5. B

6. C7. C8. C9. D10. D

11. C12. C13. D14. B15. B

16. C17. B18. D19. D20. D

21. B22. B23. C24. D25. A

CHAPTER 15 KEY CONCEPTS

InsuranceCoinsuranceComprehensive major medicalCost containmentDeductibleDental insuranceDependent coverageExclusionsGroup plansHIPAAHealth Savings Accounts

Hospital expensesLimitationsLimited policiesMajor MedicalMedical savings accountsPhysician’s expensePreexisting conditionsPregnancy Discrimination ActSurgical expensesTax considerations

NILA ©

L/H Licensing

Chapter 15-25

Page 228: FORWARD - WordPress.com · Return of premium p 6-8 6.0 Annuities p 7-1 Annuity principles and concepts p 7-1 Accumulation period versus annuity period p 7-1 Owner, annuitant and beneficiary

INDIVIDUAL GROUP MEDICAL EXPENSE COVERAGES

Individual and group medical expense insurance generally provides coverage for hospital expenses, surgicalexpenses and out-patient (physician) services including but not limited to:

COVERED MEDICAL EXPENSES

Room and boardNursing care (RN, midwife, nurse practitioner)Special dietsX-raysLab FeesMedication/Prescription drugsMedical supplies (casts, splints, crutches, braces)Operating room chargesAnesthesiaRadiation therapyHospital bedsWheelchair rentalsAmbulance services

Physical and speech therapyEmergency careMaternity benefitsHome health careVision careBlood and blood plasmaPacemakersCharges for out-patient service centersPhysician services (M.D., osteopath, podiatrists,speech pathologists, chiropractors, dentists, therapists,psychologists, social workers, etc.)Surgical expensesPhysician expense

DEPENDENT COVERAGE ELIGIBILITY

Dependents of an insured are eligible for coverage under an individual or group plan. In a group health insurancepolicy, the same benefits provided for an employee are available to his or her dependents. In addition, if a coveredemployee was previously without dependents and therefore had no dependent coverage, any new dependents due tomarriage, birth or adoption are eligible for coverage as of the date they became a dependent.

The following are considered eligible dependents:

1. An employee’s spouse who is not legally separated from the employee.2. Unmarried dependent children.3. Natural, adopted and step-children under the age of 19; up to age 23 (some States its age 25), if a

full-time student.

Therefore, a spouse who is legally separated or a married child living in the employee/insured’s home are not eligibledependents. Adopted or step-children would be considered eligible.

NILA ©

L/H Licensing

Chapter 15-26

Page 229: FORWARD - WordPress.com · Return of premium p 6-8 6.0 Annuities p 7-1 Annuity principles and concepts p 7-1 Accumulation period versus annuity period p 7-1 Owner, annuitant and beneficiary

SECTION VI

Social Securityand Other

Accident & HealthConcepts

Page 230: FORWARD - WordPress.com · Return of premium p 6-8 6.0 Annuities p 7-1 Annuity principles and concepts p 7-1 Accumulation period versus annuity period p 7-1 Owner, annuitant and beneficiary

INSURANCE forSENIOR CITIZENS

and SPECIALNEEDS INDIVIDUALS

Page 231: FORWARD - WordPress.com · Return of premium p 6-8 6.0 Annuities p 7-1 Annuity principles and concepts p 7-1 Accumulation period versus annuity period p 7-1 Owner, annuitant and beneficiary

INTRODUCTION

Social insurance differs from commercial or private insurance in that it is operated by the governmentin some capacity. Social insurance is provided for those in society who may not be able to affordburdensome health costs on their own. Therefore, social insurance attempts to assure the members ofsociety a minimum standard of living. Social insurance is provided through Social Security (OASDHI),Medicare, Medicaid, Workers Compensation, and other mediums These mediums provide protectionfor senior citizens and others who are in special situations.

INSURANCE FOR SENIOR CITIZENSAND SPECIAL NEEDS INDIVIDUALS

MEDICARE

Medicare is a federal health insurance program comprised of two parts. This government sponsored programprovides health care and other medical benefits for those age 65 and older and those eligible who are disabled. Medicare is administered by OASDHI and funded or financed through payroll taxes paid through Social Securityby employers, employees and the self-employed (i.e., sole proprietor). Part A of Medicare covers Hospital Insurance(HI). Part B covers Supplementary Medical Insurance (SMI). Medicare generally does not pay for treatment outsidethe U.S. (some limited exceptions include Canada and Mexico).

HOSPITAL INSURANCE — With regard to enrollment and eligibility, all individuals who are 65 years of ageand older who are entitled to Social Security benefits are automatically eligible for Medicare benefits Part A and PartB. In addition, some people under age 65 are eligible if they are suffering from certain disabilities or are in end-stagerenal disease (i.e., kidney failure or on dialysis). If a person is not receiving Social Security or Railroad Retirementbenefits three months prior to turning age 65 or if a person requires regular dialysis or a kidney transplant, one mustapply for Medicare benefits. There is a general enrollment period each year (i.e., January 1 through March 31 ). st st

According to the Social Security manual published by the National Underwriter, 2011, Part A of Medicare providescoverage for hospital insurance. Coverages provided under Part A includes: (1) in-patient services includingprescription drugs; (2) skilled nursing care; (3) home health care following a hospital stay; and (4) hospice carefor terminally ill seniors. Plans providing hospice care may not be required to provide home health care benefits butcan do so if desired. In addition, hospice benefits include prescription drugs (i.e., painkillers) and other palliativeor pain management benefits, bereavement counseling, daily needs and inpatient respite care. Chemotherapy andother curative measures costs would not be covered by hospice care. If a patient decides to cancel the hospice careand pursue chemotherapy, he or she may return to regular Medicare coverage. With regard to cost-sharing, a perbenefit period deductible (i.e., $1,132 in 2011) must also be satisfied by the eligible senior under hospital insurance. Part A medical benefits will be provided following the hospitalization period if warranted by a physician.

Hospital insurance covers hospital care after a deductible is satisfied during the patient's initial 60 days in thehospital. From the 61st to the 90th day, the patient pays a specified (i.e., coinsurance amount) charge per day whileMedicare pays the remainder. The benefit period or "spell of illness" commences on the first day the insured isadmitted to a hospital (or extended care facility). This period ends when the insured has been discharged for 60

NILA ©

L&H Licensing

Chapter 16—1

Page 232: FORWARD - WordPress.com · Return of premium p 6-8 6.0 Annuities p 7-1 Annuity principles and concepts p 7-1 Accumulation period versus annuity period p 7-1 Owner, annuitant and beneficiary

consecutive days. Covered in-patient expenses under Part A include room and board costs, x-rays, lab tests, andordinary nursing services. Hospital costs are paid directly to the hospital or health care provider by Part A / HospitalInsurance. Each person is also provided with a lifetime reserve of 60 days as well. The lifetime reserve days are notrenewable. In other words, once all sixty are used there are none left.

According to the Social Security Handbook, published by the National Underwriter on behalf of the federalgovernment, Part A also provides benefits of up to 190 days during a senior's lifetime for psychiatric care providedin a hospital. In addition, if a person is insured under a private insurance contract issued by a commercial carrier oran alternative health care provider such as a Health Maintenance Organization, and he or she is also covered byMedicare, the former coverage is primary and Medicare is the secondary payor.

ELIGIBILITY — The Social Security Manual, 2011, identifies who is eligible for Part A Medicare benefits. Itstates that "anyone age 65 and older who is eligible for Social Security benefits is covered under Part A at no monthlycost." Medicare pays for hospital care if the patient meets the following four conditions: (1) a physician prescribesin-patient hospital care for treatment of the illness or injury; (2) the patient requires the type of care that can only beprovided in a hospital; (3) the hospital is participating in Medicare; and (4) any review committee does not disapproveof the stay. Cost-sharing of medical expenses is provided which means that the patient must absorb deductibles (forPart A and B) and coinsurance amounts (Part B). There are other cost-sharing options available for individuals withMedicare. Some people are covered by employer sponsored health plans even following retirement age (i.e.,providing medical expense or disability protection).

Medicare covers the following in-patient services: (1) bed and board in a semi-private room; (2) the cost of a privateroom only if it is required for medical reasons; (3) the services of the hospital's medical social workers; (4) the useof regular hospital equipment, supplies and appliances such as oxygen tents, wheel chairs, crutches, casts, surgicaldressings and splints; (5) drugs and biologicals ordinarily furnished by the hospital; (6) operating room costs; (7)blood transfusions after the first three pints; (8) x-rays and other radiology services; (9) lab tests and rehabilitationservices such as physical, occupational and speech therapy; and (10) diagnostic or therapeutic items and appliances(i.e., pacemakers).

Medicare does not pay for: (1) the services of physicians and surgeons; (2) the services of a private duty nurse orattendant (unless the patient's condition requires such services); (3) the first three pints of blood; (4) personalconveniences (i.e., telephones or television rentals); (5) supplies and appliances for use outside the hospital unlesscontinued use is required (i.e., pacemaker); (6) private rooms; or (7) private duty nurses. The only type of nursinghome care Medicare pays for is skilled nursing facility care for rehabilitation, such as recovery time following ahospital discharge if approved by the attending physician. Medicare will not pay for custodial services such asactivities of daily living (ADLs) like bathing, eating, dressing or going to the toilet. These latter costs may be coveredby some long-term care (LTC) policies. Generally, if a person is covered by an individual medical expense plan andis also eligible for health care under Medicare, the individual or private policy is primary. Medicare is secondarywhen it comes to kidney disease (i.e., renal disease) or kidney failure. The individual’s group medical expense planis primary for a specific length of time.

HMOs providing benefits must meet geographical requirements (i.e., be within seven miles of the member). Coverage is available through Medicare plus choice plans as well. This is sometimes referred to as Part C. Federallaw now allows Medicare participants to opt out of the traditional program in order to enroll in other coordinatedplans of their choice such as HMOs, PPOs or Provider Sponsored Organizations (i.e., PSOs). "Private fee for serviceplans" are also available which are similar to PSOs, except for the fact that they may pay providers more thanMedicare and they may charge a participant a high premium. Seniors may also create a Medical Savings Account(MSA) or Health Savings Account (HSA) and use contributions for future health care expenses.

NILA ©

L&H Licensing

Chapter 16—2

Page 233: FORWARD - WordPress.com · Return of premium p 6-8 6.0 Annuities p 7-1 Annuity principles and concepts p 7-1 Accumulation period versus annuity period p 7-1 Owner, annuitant and beneficiary

SUPPLEMENTARY MEDICAL INSURANCE (SMI) / PART B — Part B of Medicare is an optional orvoluntary coverage. It functions similar to a major medical contract. Benefits are provided after a $162 calendar yeardeductible is satisfied. After the deductible is satisfied, 80% of the approved Medicare charges will be paid. Remember, 80% of the approved charges will be paid, not 80% of the actual charges. The insured enrolls in PartB by paying a monthly premium (i.e., $96.40 per month in 2011 for most beneficiaries) for SMI which is usuallydeducted from his/her monthly Social Security retirement benefit. Again, with regard to cost-sharing, the insuredis responsible for the deductible and the 20% coinsurance amount. The deductible and the applicable coinsuranceamounts in addition to any medical expenses that are excluded are referred to as "gaps" in coverage. These gaps canbe filled in later if the insured possesses a Medicare Supplement or "Medigap" policy. The approved amount isdetermined by the program. When a doctor accepts assignment on a claim, he or she agrees to receive the approvedamount. Eligibility for Part B follows the same requirements as Part A. In other words, a senior is eligible forbenefits under Part B when he or she is eligible for benefits under Part A. A person may enroll in Part B at any timeonce he or she is no longer covered by other individual or group health insurance.

SMI is funded by premiums paid by insureds and tax funds. Coverages provided include: (1) physician's andsurgeon's services; (2) home health services even if the insured has not been in the hospital; (3) diagnostic lab tests,surgical dressings, splints and medical equipment (rental or purchase); and (4) outpatient services and office visits. Flu shots and other types of immunizations are now covered.

According to the Social Security Manual, medical and other services excluded by Part B include routine physicalexaminations, eye exams, hearing exams, routine foot care, most prescription drugs that can be self-administered,dental care or dentures, cosmetic surgery, eyeglasses or hearing aids.

An individual may enroll in Part B at the time of enrollment in Part A or at a later date. An initial open enrollmentperiod is available according to the "Medicare Manual", which is a seven month period beginning on the first dayof the third month before the month age 65 is attained. If Part B is requested at a later date, the individual may enrollduring a general enrollment period which is January 1st to March 31st each year. Coverage will then be providedas of July 1st that year.

Key Terms — Other covered claim costs and services include lab tests, diagnostic tests, surgical supplies anddressings (i.e., casts, splints), out-patient surgery in a doctor's office, ambulance services, artificial limbs or braces,physical and occupational therapy, home dialysis supplies and services, nurse mid-wife services, artificial devicessuch as pacemakers, medical equipment such as a wheel chair and many others. Please refer to the MEDICAREBENEFIT SUMMARY at the end of this chapter.

In the mid 1980s, Congress amended the Medicare program to allow a beneficiary to elect coverage under a HMOas an alternative to the traditional program. According to federal regulations, the HMO is provided 95% of whatMedicare would expect to pay to provide benefits if a beneficiary electing HMO coverage had stayed in thetraditional program. The HMO is then expected to provide at least the same benefits as those that are available underMedicare. In some cases, the HMO has actually provided broader benefits (i.e. prescription drugs) than that availableunder the traditional program. In 1999, Part C of Medicare became effective. This is referred to as Medicare +Choice. This part of the program expands the choices available to most beneficiaries by permitting them to choosebenefits through one of several alternatives to the traditional Part A and B. This new part includes HMOs, PPOs,PSOs, private contracts with doctors, medical savings accounts or private fee-for-service plans. The government expanded this program through the Medicare Modernization Act of 2003. It is now called MedicareAdvantage. Part C now offers expanded benefits, generally for an additional premium, through private insuranceprograms that have contracts with Medicare, and Medicare pays a predetermined amount of money to the privateprovider for an individual’s health care (i.e., HMOs and PPOs). To be eligible for benefits under Part C, one must

NILA ©

L&H Licensing

Chapter 16—3

Page 234: FORWARD - WordPress.com · Return of premium p 6-8 6.0 Annuities p 7-1 Annuity principles and concepts p 7-1 Accumulation period versus annuity period p 7-1 Owner, annuitant and beneficiary

be enrolled in Parts A and B (which means that Part B premiums must still be paid). In addition, the individual mustalso live in the plan’s service area and must not be suffering from end-stage renal disease (i.e., kidney failure). Plansavailable include Medicare Managed Care Plans, PPOs, Private Fee-For-Service Plans and Specialty Plans. Noevidence of insurability is required if enrollment occurs during a six month period following the individual’s 65th

birthday. After this six month period the individual must prove that he or she is insurable. The additional Part Cchoices and the availability of greater benefits means that there may not be the need for Medicare Supplement plans. An individual may only participate in one Medicare Advantage plan at any particular time. If the individual iscovered by a group medical plan at or after age 65, coordination of benefits between Part C and the group plan mayoccur.

The Act of 2003 has also provided for a prescription drug plan (PDP) effective on 1 / 1 / 06. This is known as PartD. It provides benefits for those age 65 and older and other disabled individuals. PDPs will be offered andadministered by private companies providing conventional type prescription drug coverage including monthlypremiums and co-payments. These companies must provide two prescription options for the different prescriptioncategories. To be eligible an individual must be age 65 or older and entitled to Social Security benefits. In addition,the following are also eligible: (1) anyone under age 65 who is permanently disabled and has been receiving SocialSecurity disability benefits for at least two years; (2) anyone receiving kidney dialysis for permanent kidney failureor someone in need of a kidney transplant; or (3) and individual suffering from ALS.

A standard drug plan is offered in areas where there are no private plans available. In 2011 the plan also includesa $ 310 annual deductible and a 25 % co-payment on the next $ 2,840 of prescription drugs. Once the first $ 2,840in benefits has been provided, no further benefit is payable for the next $ 4,550 of prescription drug charges. Thisgap is referred to as the “donut hole.” Once this out-of-pocket expense has been incurred catastrophic drug coveragebegins. The insured then pays a co-payment for each prescription through the end of the year. Medicare alsoprohibits the selling or renewing of Medicare Supplement policies H, I or J if Part D is carried by an individual sincethere are some duplicate drug coverages in those Medicare Supplement plans.

MEDICAID

Medicaid is a government sponsored program which provides aid to families with dependent children. Temporaryaid for needy families (TANF) provides such assistance. Tax funds from the federal government and each Statefinance this program which provides medical assistance for certain individuals and families with low incomes orlimited assets. Medical benefits however, are administered by each State. It is not funded by and is a separateprogram from Medicare.

PUBLIC ASSISTANCE PROGRAM — Each State receives federal funds and combined with its own funds(derived from tax revenues) has been able to expand public assistance programs. Medicaid provides medical carefor: low income persons, persons with limited assets, those with insufficient income, and those receiving welfarebenefits; and those in need who are blind, aged, disabled, or under twenty-one years of age. The determination ofbenefits is based on need regardless of age (up to age 65). To qualify, applicants must have both income and assetsbelow certain limits, which vary from State to State.

Covered medical care costs will vary between each State since each State administers its own program, determinesthe scope of services provided and determines its own eligibility requirements. Medicaid covers in-patient hospitalservices, out-patient hospital services, physician services, lab and x-ray services, home health care, rural health clinicservices, family planning services, nurse mid-wife services, prenatal care, and necessary transportation. In some

NILA ©

L&H Licensing

Chapter 16—4

Page 235: FORWARD - WordPress.com · Return of premium p 6-8 6.0 Annuities p 7-1 Annuity principles and concepts p 7-1 Accumulation period versus annuity period p 7-1 Owner, annuitant and beneficiary

States, a person may be eligible if his or her high medical bills force them to "spend down" much of their incomeand assets.

MEDICARE SUPPLEMENTS / MEDIGAP

Medigap or Medicare Supplement policies are sold by private or commercial insurers and other health care providers,the purpose of which is to help fill in any gaps in coverage under Medicare. Medigap policies provide accident andsickness coverage. Since insureds are subject to deductibles and coinsurance amounts (i.e., SMI), the purpose ofMedigap policies is to cover medical expenses not covered by Medicare (Part A or Part B). In other words, Medigapis designed to help pay one's Medicare cost-sharing amounts (i.e., deductibles and coinsurance). As mentionedpreviously, deductibles, coinsurance amounts and excluded medical expenses are considered to be "gaps" in coverage. The National Association of Insurance Commissioners (NAIC) has established twelve regulated levels of coverageincluding plans A through L. The best time to buy a policy may be during one's Medigap open enrollment period. For a period of 6 months from the date you are first enrolled in Medicare Part B and are age 65 or older, one has theright to purchase a Medigap policy. This is referred to as the open enrollment period.

STANDARDIZED PLANS — The foundation of all Medicare Supplement plans is the core benefits included. All Medicare Supplement plans must include the core benefits. Additional benefits or levels of coverage may alsobe included but the core benefits are always present. The core benefits include: (1) Part A copayments for days 61-90of hospitalization; (2) Part A copayments for days 91 through 150 which is the 60 lifetime reserve days; (3) 100 %of charges for 365 additional days of hospital confinement once Part A and lifetime reserve are used; (4) coveragefor the three pints of blood not covered (i.e., blood deductible) under Part A of Medicare; and (5) Part B copaymentsfor approved physician and other medical expenses.

! Medigap premium rates are approved by the Insurance Commissioner of each State.

! Medigap plans possess common exclusions like those found in most accident and sickness policies includingbut not limited to: routine exams (i.e., physical, hearing, eye); dental care; elective or cosmetic surgeries;intentionally self-inflicted injuries, dentures, eyeglasses or hearing aids.

! Plans D, G, I and J cover short-term personal care services when Medicare is paying for home health careservices after an illness, injury or surgery. Personal care is assistance with activities of daily living (ADLs)such as bathing, eating, dressing, using the toilet and moving about in the home. Plans E and J coverpreventive care or “preventive screening.” For every home health visit that is paid for by Medicare, thisbenefit will pay for one personal care visit. Please refer to the "Medigap Benefits by Plan" chart at the endof this chapter.

REQUIREMENTS — The NAIC Model (policy) Act states that Medigap policies must meet several requirements. For example, at the time of application, all applicants must be provided with a Medicare Supplement Buyer's Guide. All Medigap policies must also include a 30 day free-look or right to examine provision. Additional requirementsinclude:

! All Medicare Supplement policies must generally be at least guaranteed renewable for life.

! Medigap policies may not contain any exclusions, limitations, or reductions which are not consistent withnor more restrictive than Medicare.

NILA ©

L&H Licensing

Chapter 16—5

Page 236: FORWARD - WordPress.com · Return of premium p 6-8 6.0 Annuities p 7-1 Annuity principles and concepts p 7-1 Accumulation period versus annuity period p 7-1 Owner, annuitant and beneficiary

! Pre-existing conditions may not be excluded under these policies. In addition, any waiting period connectedwith pre-existing conditions may generally not exceed 6 months.

! Benefits must increase automatically to cover deductibles and coinsurance not covered by Medicare. Premiums may increase on a class basis (i.e., geographic). Some plans may provide coverage forprescriptions.

! Agents must receive a signed statement from an insured that duplicate coverage is not being solicited. Soliciting duplicate coverage is prohibited.

! During the initial six months after qualifying for Part B of Medicare, insurers may not charge higher ratesbased upon adverse health, number of claims submitted or other medical conditions.

! These policies must include a six month open enrollment when an insured (i.e., Medicare beneficiary)initially purchases Part B of Medicare.

! A new probationary period may not apply to a policy that replaces a similar Medicare Supplement policy.

! Whenever a replacement of a Medigap plan occurs, a "Notice of Replacement" must be provided to theapplicant which must also be signed by the agent and the applicant.

! Insurers must achieve specific loss ratios. In other words, they must pay out in claims a minimum percentageof what is collected in premiums. The minimum loss ratio requirement for individual Medigap policies is65% while the requirement for group Medigap policies is 75%.

ADVERTISING AND MARKETING — Medigap policies are marketed to those who are covered by Medicare. In other words, these policies are generally sold to individuals who are age 65 and older. All states strictly regulatethese policies since abuses in the sale of them to the elderly are much too common.

ADDITIONAL MEDIGAP INFORMATION — As with any health insurance policy, replacement regulationsapply to Medicare Supplement policies as well. When a Medigap plan is being replaced with another a noticeregarding replacement must be completed and signed by both the applicant and the sales representative (i.e., agent).In addition, an outline of coverage must be provided to the applicant/insured. The outline provides an overview ofthe benefits provided by the actual insurance contract. A buyer's guide which provides the Medigap purchaser withinformation so a sound purchasing decision may be made, must also be provided, generally, at the time of application.Most States require that the insurer provide a notice of change to the policyowner of any change in benefits at least30 days prior to the effective date of any Medicare benefit or Medigap policy change.

! Medicare Supplements cover the gaps in coverage under the Medicare program including deductibles,coinsurance or co-payment amounts, excluded medical expenses, out-patient prescription drugs, custodialcare in a nursing home, and the blood deductible.

! The core benefits provided by a Medigap policy must include the 3 pint blood deductible, co-insurance/co-payments under Part B of the Medicare approved expense amount and all charges for 365 days ofhospitalization after all Part A in-patient hospital and lifetime reserve days are exhausted.

! The sale of duplicate Medigap coverage is prohibited. No producer or sales representative is permitted tosolicit additional Medigap coverage to an insured who already owns such a contract.

NILA ©

L&H Licensing

Chapter 16—6

Page 237: FORWARD - WordPress.com · Return of premium p 6-8 6.0 Annuities p 7-1 Annuity principles and concepts p 7-1 Accumulation period versus annuity period p 7-1 Owner, annuitant and beneficiary

! Plan A (core benefits) must be offered to purchasers. Insurers cannot pick and choose the benefits to beincluded in Plan A. All of the aforementioned coverages must be included. The coverages provided by thecore benefits are standardized.

! Three methods are used to calculate Medigap premiums including: (1) issue age; (2) attained age; or (3) noage rating (whoever purchases it pays the same rates).

LONG-TERM CARE INSURANCE

Long-term care insurance provides a broad range of medical and personal services for individuals who needassistance with daily activities for a lengthy period of time. The reason such assistance is needed is due to a mentalor physical impairment as a result of the aging challenge. The assistance needed may be provided in a nursing home,an adult day care center or at the senior’s home. The cost of care provided in a nursing home today is approachingastronomical proportions. Medicare and Medigap plans provide minimal protection for this risk exposure. Thus,the need for long-term care insurance. Long-term care coverage can be purchased as an individual policy, as partof a group plan or through a rider added to a life insurance policy. Such coverage is designed to cover at leasttwelve consecutive months for one or more necessary types of care in a setting other than a hospital. This coveragedoes not include coverage for hospital confinement. The National Association of Insurance Commissioners (NAIC)created and authored the Long-Term Care Insurance Model Act which specifies the minimum standards thatproducts must satisfy in order to be considered long-term care insurance. This model included the following: (1) Insurers must provide an outline of coverage to the consumer which summarizes the benefits and features of thepolicy; (2) The policy owner must be provided with a free-look period (usually thirty days) during which the policymay be returned to the insurer and a full refund of premium will be paid to the owner; (3) The policy may notinclude substantially greater benefits for skilled nursing care than that provided for intermediate or custodial care; (4) Policies must include a renewability provision of at least guaranteed renewable; and (5) Impairment riderslimiting or denying coverage for specific health conditions are not permitted. Producers selling these policies areprincipally responsible for determining the suitability of such plans for the applicant. Insurers are not generallypermitted to offer producers additional incentives to sell such plans since they are primarily purchased by seniorcitizens.

There are four major levels of care including skilled nursing care, intermediate nursing care, custodial care and homehealth care. Skilled nursing care consists of daily nursing and rehabilitative care that may be performed by (or underthe supervision of) skilled medical personnel and based on the orders of an attending physician. This involvestwenty-four hour nursing service subject to periodic review by a physician. Intermediate (nursing) care involvesoccasional nursing and rehabilitative care that must be based on a physician’s orders and provided by skilled medicalpersonnel. Custodial care involves the handling of personal needs such as bathing, walking, eating, dressing ortaking medication. Medically unskilled persons providing such non-medical care are generally not medical personnel,although the care must be approved and ordered by a physician (and supervised by a registered nurse). Home healthcare may involve all of the aforementioned types of care, or visits by an RN, LPN or a licensed vocational nurse (plushousework, meal preparation and certain types of therapy such as physical therapy), except that it is provided in thepatient’s home rather than in a nursing facility. It is not performed in an institutional (nursing care facility) setting. Other benefits that are available include but are not limited to: assisted living care, adult day care (that must beprovided in a legally operated and licensed facility that provides care at least five days a week), care coordinationand respite care.

NILA ©

L&H Licensing

Chapter 16—7

Page 238: FORWARD - WordPress.com · Return of premium p 6-8 6.0 Annuities p 7-1 Annuity principles and concepts p 7-1 Accumulation period versus annuity period p 7-1 Owner, annuitant and beneficiary

! Assisted Living Care (ALCFs) — This form of care is provided in a facility that cares for seniors whoare too frail to care for themselves but do not yet require the level of care provided by a nursing home. Thistype of care supplements nursing care and is not intended to be a substitute for it.

! Respite Care — Individuals who provide living assistance at home are called care givers. Respite careprovides relief or time off for the major care giver (i.e., gives the care giver a break) and may even includean overnight stay by a respite care giver. Respite care is also provided at adult day care centers. Adult daycare is either medically-based (i.e., physical therapy) or recreational-based (i.e., arts, crafts, entertainment).

! Adult Day Care — This type of care is designed for seniors who live at home but whose family membersare not able to stay at home with them during the day since the primary care giver is absent or at work. Thelevel of care provided at adult day care centers is similar to home health care. These centers usually providetransportation to and from the center.

Custodial care, home health care, respite care and adult day care are all benefits that are used to reduce the necessityof admission into a senior care facility. The need for LTC has been necessitated due to the increase in our lifeexpectancy. Medicare or Medigap do not pay for treatment in a nursing home for anything more than a limitedbasis. Generally eligibility with regard to purchasing coverage is similar to other types of health insurance plans suchas individual or group disability coverage, the latter of which requires that the employee be full-time, actively at workand so on. The policy is designed to pay a daily benefit for at least twelve consecutive months once a reasonablewaiting period is satisfied. These daily benefit amounts generally are available from $50 to $250 per day but willvary depending upon the level of care provided. For example, the benefit for skilled nursing care is greater than homehealth care.

Some policies are available that may pay a little more or less than the norm. Benefits are continually revised andupgraded. Benefit periods available, like disability income policies, may include twelve months, twenty-four months,five years, ten years or for life. LTC policies generally include an elimination period (available from 0 to 365 daysthough this will vary by State) that functions just like it does in a disability income policy. Again, the purpose of anelimination period is to lower the cost of coverage for the benefit of everyone.

Long-term care insurance promises to pay expenses incurred if the insured is unable to engage safely in variousactivities of daily living. LTC policies promise to pay benefits if the insured is unable to perform basic dailyactivities without assistance. Most policies require that a functional assessment of the insured be made and thathe or she be unable to perform two activities of daily living for at least ninety days before benefits are triggered. Depending upon the policy, coverage is triggered when an insured is unable to perform such activities. Theseactivities of daily living include: bathing, dressing, eating, transferring or moving about the home, continence,toileting and taking medication. It is also conceivable that a senior may be able to perform these activities butcannot live alone safely. Therefore, LTC plans include a provision describing that the policy will pay benefits if theinsured is suffering from a cognitive impairment. This impairment includes a senior suffering from Alzheimersdisease, strokes or other types of brain damage or senile dementia.

OPTIONAL BENEFITS AND QUALIFIED PLANS — Some LTC policies include a guarantee ofinsurability option as well as a return of premium option. The guaranteed insurability option functions like it doesin a disability or life insurance policy and, if elected, provides an increase in benefits (generally limited to a 5%increase) without the insured proving insurability. In addition, a qualified plan according to HIPAA is one that: (1)cannot pay expenses that are reimbursable under Medicare; (2) is at least guaranteed renewable; (3) does not includea cash surrender value; (4) only provides long-term care services; (5) complies with various consumer protectionprovisions; and (6) states that all refunds of premium and policy owner dividends, if any, must be applied as futurereductions in premiums or to increase future benefits. HIPAA further states that qualified long-term care services

NILA ©

L&H Licensing

Chapter 16—8

Page 239: FORWARD - WordPress.com · Return of premium p 6-8 6.0 Annuities p 7-1 Annuity principles and concepts p 7-1 Accumulation period versus annuity period p 7-1 Owner, annuitant and beneficiary

include necessary diagnostic, preventive, therapeutic, caring, treating, and rehabilitative services. A qualified planmust include at least five of the ADLs described in the HIPAA legislation.

Whenever a LTC policy is replaced, any preexisting condition limitation in the new policy will be waived. Toprevent against an unintentional lapse, the insurer issuing this type of policy must receive from the applicant awritten designation of at least one person, in addition to the applicant, who is to receive notice of lapse or terminationof the policy due to nonpayment of premium (or a written waiver stating that the applicant does not desire todesignate another to receive notice). UNDERWRITING — Underwriting practices will vary by insurers. Generally, applicants are subject to individualunderwriting whether applying for group or individual coverage. If a LTC plan is employer paid, coverage may be“guaranteed issue” as long as benefits are available to all employees. These plans usually contain no pre-existingcondition provision or limitation.

EXCLUSIONS — There are several exclusions found in a long-term care policy including war, treatment for drugor alcohol abuse, intentionally self-inflicted injury, attempted suicide, institutional care received outside the U.S.,or care for which benefits are payable under Workers Compensation.

TAX CONSIDERATIONS — Premiums paid on an individual long-term care policy are not tax deductible. Thebenefits received, therefore, will not be taxable. However, according to HIPAA, LTC plans, whether group orindividual, are considered to be accident and health insurance. This means that premiums paid by an employer undera group plan are tax deductible. The benefits received by an employee are not taxable (unless the benefit is paid ona per diem basis and exceeds certain amounts). In most cases, expenses paid for long-term care services, includingpremiums paid for qualified plans, are treated like any other medical expense. This means that if such expensesexceed 7 ½ % of an individual’s adjusted gross income, they are tax deductible. Please refer to the chart on page14-10 for tax considerations information.

NONFORFEITURE OPTIONS — Some States require that LTC insurers offer nonforfeiture benefits as partof the policy. This results in a policy with a higher premium than a plan that does not include such benefits. Thesebenefits will allow the insured to receive some value from the plan if the policy lapses for nonpayment of premium. This option is normally available in one of three different forms. First, after paying premiums for a specified numberof years, if the insured ceases payment, a reduced paid-up policy would result. Second, after paying premiums fora specified number of years, if the insured stops paying premiums, the same benefit would apply for a shortenedperiod of time. Third, following the payment of premiums for a specified minimum number of years, if premiumscease, a specified dollar amount will be refunded to the insured (this is the most expensive form). LTC policies maynot be issued unless nonforfeiture benefits have been offered. The offer is usually made at the time of applicationor prior to the issuance of the policy. There is no age requirement or limit that such an offer must be made beforeor at a specific age.

INFLATION PROTECTION — Many States require that an insurer offer some sort of inflation protection thepolicy owner of a long-term care plan. The cost of such protection is usually built into the initial premium. Ifincluded, the benefit usually involves a 5% increase each year.

NILA ©

L&H Licensing

Chapter 16—9

Page 240: FORWARD - WordPress.com · Return of premium p 6-8 6.0 Annuities p 7-1 Annuity principles and concepts p 7-1 Accumulation period versus annuity period p 7-1 Owner, annuitant and beneficiary

SOCIAL SECURITY

SOCIAL SECURITY — The Old Age Survivors Disability Health Insurance (OASDHI) is the formal name forSocial Security. Benefits under this social insurance program are payable to covered workers (and their dependents)when a worker dies, retires, or becomes disabled.

BENEFITS UNDER OASDHI — Social security provides for several forms of benefits, a few of which will bereviewed:

! RETIREMENT BENEFITS — Workers who are age 65 or older and fully insured are entitled to monthlyretirement income for the remainder of their lives. The retirement income paid to the person who retires atage 65 is based upon primary insurance amount (PIA). The retirement income paid by Social Security beginson the first day of the month in which an eligible individual reaches age 65 (or other qualifying age). However, retirement income is not paid automatically when a retiree reaches age 65. The recipient mustcomplete an application and submit it if he or she wishes to receive monthly income. All other OASDHI/ Social Security benefits are based on this amount. Recipients may receive retirement income at age 62 ifdesired, but at a reduced amount.

! LUMP SUM DEATH BENEFIT — When a fully or currently insured worker dies, a lump sum deathbenefit of $255 is paid to an eligible surviving spouse or dependent.

! DISABILITY BENEFITS — An individual is eligible for disability benefits under Social Security basedupon his or her length of employment. These benefits will be paid to a worker (and eligible dependents) ifthat worker meets the restrictive total disability definition under Social Security.

This definition is “the inability to engage in any gainful activity by reason of any medically determinablephysical or mental impairment which has lasted or could be expected to last for a continuous period of 12months or result in death. The impairment must be so severe that the individual is unable to engage insubstantial gainful work that exists in the national economy regardless of whether or not such work existsin the immediate area in which the worker lives.”

Once a worker is eligible, he or she must satisfy a five month waiting period before benefits are payable. Therefore, benefit payments would begin in the sixth month. These benefits are also based on a worker'sprimary insurance amount (PIA).

! DEPENDENT BENEFITS — As mentioned previously, benefits are paid to the children of a deceasedworker during the blackout period until the youngest child reaches age eighteen (18).

INSURED STATUS — A worker is considered to be fully insured if he has earned the required number of quartersof coverage. A worker with 40 quarters of coverage (i.e., 10 years of employment) is considered to be fully insured. A fully insured worker is entitled to retirement benefits and survivors are eligible for retirement benefits when theworker dies. It does not provide for disability benefits. This benefit is provided if a worker is fully and disabilityinsured. In this case, a worker must have 40 quarters of coverage and 20 of the 40 quarters must be earnedimmediately preceding the disabling event. Therefore, under this classification, the worker is entitled to retirementbenefits, disability benefits and survivors are eligible for retirement benefits when the worker dies. A worker iscurrently insured if he or she has earned at least 6 quarters of coverage in the 13 quarters immediately precedingdeath or disability. This status provides for survivor benefits only. It does not provide for disability or retirementbenefits.

NILA ©

L&H Licensing

Chapter 16—10

Page 241: FORWARD - WordPress.com · Return of premium p 6-8 6.0 Annuities p 7-1 Annuity principles and concepts p 7-1 Accumulation period versus annuity period p 7-1 Owner, annuitant and beneficiary

FUNDING — Funding for Social Security and all its social insurance counterparts (i.e., Medicare, Medicaid) is ac-complished through tax funds collected from employers (7.65%), employees (7.65%), and the self-employed (15.3%). Therefore, employers, employees and self-employeds (i.e., sole proprietor) fund social insurance programs.

QUIZ

1. Social Security is also known as:

A. CPI C. Part A

B. OAS D. OASDHI

2. Disability benefits paid by Social Security are based on the disabled workers:

A. Attained age C. Social Security benefit

B. Primary insurance amount D. Unearned income

3. Medicare Supplement policies must include a right to examine provision of:

A. Twenty days C. Forty-five days

B. Thirty days D. Sixty days

4. Medicare is financed through tax revenue. All of the following fund this program in some manner, EXCEPT:

A. An employer C. A beneficiary

B. A sole proprietor D. An employee

5. Disability benefits will be paid to a covered worker if he meets the definition of total disability utilized by Social

Security. A worker is eligible if he is unable to engage in:

A. His own occupation C. His trade or job

B. Any occupation D. A fully insured status

6. If a covered worker qualifies for disability benefits under Social Security, monthly income benefits would begin after

a waiting period of:

A. Three months C. Six months

B. Five months D. Eight months

7. A covered worker is considered to be "fully insured" if he or she possesses:

A. 20 quarters of coverage C. 40 quarters of coverage

B. 30 quarters of coverage D. 60 quarters of coverage

8. Medicare is a federally sponsored two part health insurance program. Each of the following medical services is covered

under Part A of Medicare, EXCEPT:

A. Home health care C. Physicians care

B. Skilled nursing care D. Inpatient care

9. Part B of Medicare is an optional coverage. Which of the following would not be covered by this portion of Medicare.

A. Eye examination C. House call

B. Office visit D. Outpatient charge

NILA ©

L&H Licensing

Chapter 16—11

Page 242: FORWARD - WordPress.com · Return of premium p 6-8 6.0 Annuities p 7-1 Annuity principles and concepts p 7-1 Accumulation period versus annuity period p 7-1 Owner, annuitant and beneficiary

10. Medicaid is a federally and state sponsored health program for needy and low income individuals. All of the following

are true regarding Medicaid, EXCEPT:

A. Medicaid provides aid to families with dependent children

B. Medicaid provides health care for needy persons who are blind

C. Medicaid is financed by Medicare

D. The extent of covered costs varies by state

11. All of the following are acts of daily living, EXCEPT:

A. Moving about the home C. Breathing

B. Dressing D. Bathing

12. When will an individual be entitled to retirement income under Social Security?

A. On his or her 65th birthday

B. On the last day of the month in which he/she turns age 65.

C. On the first day of the month in which he/she turns age 65.

D. Age 62

13. Relief to a primary care giver is provided by:

A. Respite care C. Rehabilitation care

B. Home health care D. Intermediate care

14. Medigap policies must generally be written on which of the following bases?

A. Non cancelable C. Conditionally renewable

B. Guaranteed renewable D. Guaranteed renewable for life

15. Medicare Supplement policies may not include a pre-existing condition limitation that exceeds:

A. Three months C. Nine months

B. Six months D. Twelve months

16. The purpose of Medicaid is to:

A. Provide health care to those who have unlimited assets

B. Provide health care to those who have a pre-existing disability

C. Provide health care to those who have a need for coverage

D. Provide health care to those who have insufficient income

17. What is the purpose of an elimination period which appears in a Long-Term Care policy?

A. To reduce an insured's loss ratio

B. To guarantee that an insurer will provide benefits for everyone

C. To enable an insurer to deny small pre-existing claims

D. To allow an insurer to lower costs for everyone's benefit

18. Social Security is also known as:

A. OSHA C. OASHI

B. OASDHI D.. PIA

NILA ©

L&H Licensing

Chapter 16—12

Page 243: FORWARD - WordPress.com · Return of premium p 6-8 6.0 Annuities p 7-1 Annuity principles and concepts p 7-1 Accumulation period versus annuity period p 7-1 Owner, annuitant and beneficiary

19. Policies providing hospice care benefits are not required to provide:

A. Care for terminally ill patients C. Psychological treatment benefits

B. Ambulance expenses D. Home health care

20. Medicare does not pay for treatment:

A. In southwestern states C. In states that have domestic private carriers

B. Outside the U.S. D. All of the above

21. Which of the following would not be eligible for enrollment in the Medicare Prescription Drug plan ?

A. A person age 63 who has been permanently disabled for three years

B. A person age 66 who is not eligible for Social Security benefits

C. A person in need of a kidney transplant

D. Someone suffering from ALS

22. Once a specific amount of benefit is initially covered by Part D of Medicare, a gap exists before more costs are covered.

This gap in coverage that exists under the prescription drug plan is known as:

A. The donut maker C. A donut hole

B. Catastrophic coverage D. Whole life coverage

23. Medicare Advantage offers expanded benefits for an additional premium. In order to be eligible for this coverage, also

known as Part C, which of the following is true ?

A. An individual must be enrolled in Part A of Medicare

B. An individual must be enrolled in Part A and Part B of Medicare

C. An individual must be enrolled in Part A , B and D of Medicare

D. An individual must be suffering from end-stage renal failure

24. To be eligible for coverage under Part C of Medicare, an individual does not have to prove that he or she is insurable

if enrollment occurs during what period of time following the attainment of age 65 ?

A. One month C. Six months

B. Three months D. Twelve months

25. Which of the following Medicare coverage sections provides managed health care?

A. Part A C. Part C

B. Part B D. Part D

26. Which of the following long-term care benefits provides twenty-four hour care?

A. Intermediate nursing care C. Home health care

B. Skilled nursing care D. Custodial care

27. Which provider is considered the secondary payor if an individual is covered by both Medicare and a group health

insurance plan?

A. Group health plan C. Medicaid

B. Long-term care D. Medicare

NILA ©

L&H Licensing

Chapter 16—13

Page 244: FORWARD - WordPress.com · Return of premium p 6-8 6.0 Annuities p 7-1 Annuity principles and concepts p 7-1 Accumulation period versus annuity period p 7-1 Owner, annuitant and beneficiary

28. A long-term care policy provides coverage for which of the following?

A. Hospital confinement

B. Alzheimers disease

C. Long-term disability

D. Drug abuse treatment

ANSWERS

1. D

2. B

3. B

4. C

5. B

6. B

7. C

8. C

9. A

10. C

11. C

12. C

13. A

14. D

15. B

16. D

17. D

18. B

19. D

20. B

21. B

22. C

23. B

24. C

25. C

26. B

27. D

28. B

CHAPTER 16 KEY CONCEPTS

Currently insured

Fully insured

Definition of disability

Insured status

Donut hole

Long-term care

Medicaid

Medicare Parts A through D

Medigap

Social Security

NILA ©

L&H Licensing

Chapter 16—14

Page 245: FORWARD - WordPress.com · Return of premium p 6-8 6.0 Annuities p 7-1 Annuity principles and concepts p 7-1 Accumulation period versus annuity period p 7-1 Owner, annuitant and beneficiary

MEDIGAP BENEFITS BY PLAN

MEDIGAPBENEFITS

A B C D E F G H I J K L

Basic Benefits / / / / / / / / / / / /

Part A: HospitalDeductible($1,132)

/ / / / / / / / / 50%

Part A

Deduct-

ible

50%

Part A

Deduct-

ible

Part A: SkilledNursing HomeCo-Insurance

/ / / / / / / / 50%

Skilled

Nursing

Coinsur-

ance

50%

Skilled

Nursing

Coinsur-

ance

Part B: Deduct-ible ($162)

/ / /

Foreign TravelEmergency

/ / / / / / / /

At-HomeRecovery

/ / / /

Part B: ExcessDoctor Charges

100% 80% 100% 100%

Preventive (Care)Screening

/ /

OutpatientPrescriptionDrugs

Basic

$1,250

Basic

$1,250

Extend-

ed

$3,000

NOTES: States may allow only some of the twelve benefits plans to be sold.States may approve innovative additional benefit plans under certain circumstances.These standard benefit plans do not apply to policies sold in Massachusetts, Minnesota and Wisconsin.Availability of plans will vary from company to company.

NILA ©

L&H Licensing

Chapter 16—15

Page 246: FORWARD - WordPress.com · Return of premium p 6-8 6.0 Annuities p 7-1 Annuity principles and concepts p 7-1 Accumulation period versus annuity period p 7-1 Owner, annuitant and beneficiary

MEDICARE BENEFIT SUMMARY(year 2011)

SERVICES MEDICARE PAYS YOU OR YOUR MEDIGAPINSURANCE PAYS

Hospital Inpatient

(Days 1-60)

(Days 61-90)

(Days 91-150)

(Days 151 ...)

Full costs after $1,132 / stay deductible

Full costs after co-insurance

Full costs after co-insurance

$0

$1,132/per benefit period*

$283/per day per benefit period*

$566/day (lifetime reserve 60 days if

used) **

Full costs

Skilled N(Duarsyisn g1 - 2H0o)me Care

(Days 21-100)

(Days 101 ...)

(if patient meets Medicare's condition)

Full costs

Full costs after co-insurance

$0

$0

$141.50/per day per ben. period*

Full costs

Home Health Care

Skilled Visits 100% if patient meets conditions $0

Hospice

210+ days Full costs

Most costs for drugs & respite care

$0

Small co-payments

Medical Services

(Part B)

80% of approved amount after $162 deduct-

ible

20% plus deductible and charges

above approved amount

Clinical Lab Services 100% of approved amount $ 0

Outpatient Hospital Treat-

ment

80% of approved amount 20% of billed charges

Prescription Drugs $0 ( most outpatient drugs) Full costs (unless Part D incl)

* A benefit period begins on the first day you receive service as an inpatient in a hospital and ends after you have been out of the hospital orskilled nursing facility for 60 days in a row or remain in a skilled nursing facility but do not receive care there for 60 days in a row.

* * Lifetime Reserve — Medicare helps pay for inpatient service for up to 90 days in each benefit period. Medicare also includes a 60 daylifetime reserve. After 90 days the patient pays the full bill unless the lifetime reserve of 60 days is drawn upon. There is no limit to thenumber of 90 day benefit periods a person can have in a lifetime (except for hospitalization due to mental illness). However, the lifetimereserve of 60 days is not renewable. Lifetime reserve days include an extra 60 hospital days a patient can use if the patient has a longillness and needs to stay in the hospital for more than 90 days. But the patient only has 60 reserve days in a lifetime. For instance, if apatient uses 5 reserve days in his first hospital stay this year, he will only have 55 reserve days remaining, whether or not he has a newbenefit period. The patient determines when he wants to use reserve days. In 2011 there is a $566 coinsurance amount per day for reservedays used.

NILA ©

L&H Licensing

Chapter 16—16

Page 247: FORWARD - WordPress.com · Return of premium p 6-8 6.0 Annuities p 7-1 Annuity principles and concepts p 7-1 Accumulation period versus annuity period p 7-1 Owner, annuitant and beneficiary

ACCIDENT & HEALTHPOLICY

PROVISIONS

Page 248: FORWARD - WordPress.com · Return of premium p 6-8 6.0 Annuities p 7-1 Annuity principles and concepts p 7-1 Accumulation period versus annuity period p 7-1 Owner, annuitant and beneficiary

INTRODUCTION

All accident and health insurance policies must contain twelve uniform provisions. State governmentshave attempted to induce uniformity in the conditions contained in health insurance contracts. Therefore, states have enacted laws requiring that certain provisions appear in all health insurancecontracts issued in a particular state. There are actually twenty three uniform provisions. Twelve aremandatory and must be included in all health insurance policies. Eleven are optional and may beincluded by an insurer if desired. Additional provisions available for inclusion in a health insurancepolicy will also be reviewed.

ACCIDENT & HEALTH POLICY PROVISIONS

REQUIRED UNIFORM POLICY PROVISIONS

Twelve uniform provisions are mandatory and must be included in all health policies issued and delivered. Theserequired uniform provisions include:

GRACE PERIOD — A grace period is included in all policies of health insurance. This period shall be allowedfor payment of a premium in default. In other words, a policy will not lapse if the premium is not paid by its due date. Health insurance protection remains in force during the grace period just as life insurance protection remains in forceduring the grace period. If a covered claim arises during this period, it will be paid but will be reduced by theamount of the premium owed. Most grace periods are 31 days.

REINSTATEMENT — A policyholder may reinstate health insurance coverage following a policy lapse as longas such action is requested of the insurer within three years of the date of the lapse. The insurer will effect areinstatement if the policyholder pays all premiums in default and demonstrates insurability. If a reinstatementapplication is required, the reinstatement will take effect when approved by the insurer. However, once the reinstate-ment application is provided to the insurer, it has 45 days to inform the applicant whether or not the policy has beenreinstated. If such notice is not provided within the required period, the policy will be reinstated automatically.

! The reinstated policy will immediately cover accidents which occur after the reinstatement date. Sicknessor illness will not be covered until 10 days after the policy is reinstated, which protects the insurer againstany pre-existing conditions.

! Most reinstatement periods are three years. This means that an insurer may permit reinstatement of a policyfor up to three years after the policy lapse. Some States have reinstatement periods of five years.

NOTICE OF CLAIM — According to this required provision, the policyowner must furnish the insurer with anotice of claim within 20 days after an illness or injury occurs (or as soon as reasonably possible). The policyownermay notify the insurer or the producing agent of the loss.

NILA ©

L&H Licensing

Chapter 17—1

Page 249: FORWARD - WordPress.com · Return of premium p 6-8 6.0 Annuities p 7-1 Annuity principles and concepts p 7-1 Accumulation period versus annuity period p 7-1 Owner, annuitant and beneficiary

CLAIM FORMS — Once the insurer receives notice of claim from the policyowner, it must send claim forms tothe policy owner within 15 days. If the insurer does not send out claim forms to an insured in a timely fashion, thepolicy owner may file an unfair claim settlement practice. In addition, the policyowner is permitted to send in proofof loss in any reasonable form. A claim form is also referred to as a "proof of loss" form.

PROOF OF LOSS — This provision states that once the policyowner suffers a loss, he or she has 90 days inwhich to submit a written proof of loss to the insurer which is verified by a physician or hospital. A failure to sendin written proof of loss within the 90 days will not invalidate the claim as long as it is determined that the policyowner was unable to do so (i.e., coma). A completed claim form is generally the written proof of loss supplied tothe insurer. Do not confuse this "proof of loss" provision with the previous one listed — the "proof of loss form"provision.

TIME LIMIT ON CERTAIN DEFENSES — This provision stipulates that the insurer has two years fromthe date of policy delivery to contest any information recorded on the application. In other words, an insurer maynot deny a claim due to misstatements made by the applicant on the original application after the policy has been ineffect for two years or more (unless those misstatements are fraudulent). Therefore, this provision allows aninsurance company to deny a claim because of the concealment of material health information if the insurerdiscovers it during the first two (2) years of the policy's existence. *

ENTIRE CONTRACT — This provision states that the policy, and any other attached forms such as riders,amendments or endorsements, plus a copy of the original application constitutes the entire contract.

LEGAL ACTION — This provision states that no legal action can be brought against an insurer (by the insured)until 60 days after written proof of loss has been furnished to the insurer. In addition, any suit brought against theinsurer by the policy owner must be done within 3 years from the date the proof of loss was furnished. Therefore,this provision is advantageous for an insurer and prevents legal action on the part of the insured during the initial 60days following the furnishing of written proof of loss.

PHYSICAL EXAMINATION AND AUTOPSY — This provision allows the insurer, at its own expense, theright to require a physical examination or autopsy of an insured prior to the issuance of a policy or the payment ofbenefits.

PAYMENT OF CLAIMS — This provision states that health insurance benefits must be payable to the insured.It also allows payment of benefits directly to a hospital or provider of medical services. If a death benefit is providedunder a health insurance policy (i.e., accidental death benefit), this provision provides for payment of benefits to anamed beneficiary. No payments will be made to a creditor of the insured.

CHANGE OF BENEFICIARY — The policyowner may change a revocable beneficiary at any time at his or herdiscretion. However, if the beneficiary is irrevocable, the policyowner cannot change it without the permission ofsuch beneficiary.

TIME OF PAYMENT OF CLAIMS — This provision requires that a claim be paid by the insurer once itreceives the completed claim form. In the case of a disability income claim, policy benefits must be paid on amonthly basis within thirty days of receipt of the written proof.

NILA ©

L&H Licensing

Chapter 17—2

Page 250: FORWARD - WordPress.com · Return of premium p 6-8 6.0 Annuities p 7-1 Annuity principles and concepts p 7-1 Accumulation period versus annuity period p 7-1 Owner, annuitant and beneficiary

OPTIONAL UNIFORM HEALTHINSURANCE PROVISIONS

An insurer may select any of the following provisions to include in a health insurance contract that it issues. Theinclusion of these provisions generally depends upon the type of policy being written. For example, certain optionalprovisions are only applicable to disability income policies and not applicable to major medical plans (i.e., changeof occupation). The optional provisions are as follows:

CHANGE OF OCCUPATION — This provision allows for some type of reduction when an insured changeshis job. For example, if the insured changes to a more hazardous occupation (i.e., clerical worker to a hand-glidinginstructor), the policy premium will remain the same but the monthly income benefit will be reduced. If the insuredchanges to a less risky occupation (i.e., police officer becomes a librarian), the benefit provided will remain constantand the premium will be reduced.

MISSTATEMENT OF AGE — If an insured misstates his or her age (i.e., whether overstated or understated)on the health insurance application, the insurer will adjust benefits accordingly under the policy to the amount thatshould have been provided had the correct age been stated. In other words, the insurer will pro-rate the claim andpay what the premium would have purchased at the correct age.

! For example, assume that an insured owns a disability income policy with a monthly benefit of $500. Theannual premium is $100. However, at the time of application the insured stated his age as 25 rather than hisactual age of 30. When the insurer uncovers the misstatement it will keep the premium at $100 but reducethe monthly benefit to $400. This is the benefit that the $100 premium would have purchased at the correctage (i.e., age 30).

OTHER INSURANCE IN THIS INSURER — This provision states that if the insured is already coveredby a policy with one insurer but is also covered by another plan with the same company, only the maximum benefitis payable. The purpose of this provision is to prevent an insured from profiting if he or she is covered by twopolicies with the same insurer.

INSURANCE WITH OTHER INSURER — If the policy owner possesses duplicate coverage with anotherinsurer on "an expense incurred basis" and does not notify the primary insurer of the other policy, the liability of theinsurer is limited to its proportionate share of the expenses incurred. For example, assume that this optionalprovision appears in a medical expense plan. If an insurer returns premium to an insured on a pro-rated basis andonly pays a portion of medical expenses incurred, it is generally because the insurer has discovered that the insuredhas duplicate coverage of some sort.

INSURANCE WITH OTHER INSURER — This provision is rarely included in policies today. It is identicalto the previous provision except that it applies to benefits provided on an "other than expense incurred basis".

RELATION OF EARNINGS TO INSURANCE — This provision appears in disability income policies. Itprevents over-insurance since it protects an insurer against an insured purchasing more monthly benefit in relationto his or her monthly income. Normally it is only included in noncancelable or guaranteed renewable policies. Thisprovision protects against moral hazards (i.e., filing a false claim). An individual whose disability income benefitsare greater than his income does not possess much incentive to return to work.

NILA ©

L&H Licensing

Chapter 17—3

Page 251: FORWARD - WordPress.com · Return of premium p 6-8 6.0 Annuities p 7-1 Annuity principles and concepts p 7-1 Accumulation period versus annuity period p 7-1 Owner, annuitant and beneficiary

UNPAID PREMIUMS — This provision states that the insurer may deduct any unpaid or owed premium from thebenefits payable by the policy. This clause is generally used if a premium has yet to be paid by its due date and theinsured suffers a covered loss during the grace period.

CANCELLATION — This provision provides the insurer the right to cancel the contract at any time with at least5 days written notice to the insured. All claims originating prior to the cancellation will be paid. This clausegenerally appears in cancelable contracts. Other renewability provisions included in a policy may supersede thisclause.

CONFORMITY WITH STATE STATUTES — As the policy states, this clause "modifies the policy tocomply or conform with minimum state requirements." Its title provides the impression that it is a required uniformprovision but it is not.

ILLEGAL OCCUPATION — This clause allows an insurer to deny a claim if the insured is injured during thecommission of an illegal act or while engaged in any other felonious occupation. In other words, an insurer maydeny a claim, void the contract and return any unearned premium to the insured if the latter suffers an injury or illnessduring the commission of an illegal occupation (i.e., robbing a bank). If a claim payment has already been made andthe insurer later finds that the insured was injured while committing a crime, it will attempt to recover the claimamount previously paid and deny the claim.

INTOXICANTS AND NARCOTICS — This clause relieves the insurance company of liability for losses ifthe insured was under the influence of non-prescribed drugs or alcohol when the loss occurred. Physician prescribeddrugs would not fall under the realm of this clause.

OTHER PROVISIONS

Some additional provisions that may appear in certain health insurance contracts, but may not be uniform, includebut are not limited to the following:

FREE-LOOK — This provision appears in most health insurance policies. It provides the insured with 10 days toexamine the policy from the date of policy delivery. If the insured decides during the free-look period to return thepolicy to the insurer, he or she may do so and receive a full premium refund. Some types of policies include twentyor thirty day free-look periods (i.e., Medigap). The free-look provision is also known as the "right to examine"provision.

! The free-look period begins when the policy is delivered. The notice of the right to examine generallyappears on the face page of the policy. If a ten day free-look is included, this period ends ten days later. Medical expense plans, disability income, and other forms of health insurance include this provision. However, a policy providing flight insurance would have no need to include a free-look since coverage isprovided for mere hours at a time.

! For example, if a policy is delivered on January 27th when will the 10 day free-look end? To determinewhen it ends, begin counting the next day — January 28th as day one. Therefore, the free-look period wouldend on February 6th.

NILA ©

L&H Licensing

Chapter 17—4

Page 252: FORWARD - WordPress.com · Return of premium p 6-8 6.0 Annuities p 7-1 Annuity principles and concepts p 7-1 Accumulation period versus annuity period p 7-1 Owner, annuitant and beneficiary

INSURING CLAUSE — This provision is actually a paragraph which provides a summary of the coverageprovided by the policy. It is usually located on the face page of the policy and identifies the parties to the contract(i.e., insurer and policy owner), the benefit to be provided, the annual premium that must be paid and the frequencywith which they are paid (i.e., monthly), the policy period (i.e., 1/1/10 — 1/1/11), and a beneficiary designation ifone is needed. This clause allows an insurer to deny a claim if the policy was purchased after an accident or illnessoccurred. In other words, the insuring clause stipulates that coverage is provided only during the policy period. Thisprovision also prevents a claim from being paid by an insurer for any recurring injury which occurs after the policyfirst goes into effect. This is the provision that states what benefits the insurer will pay when a covered claim occurs.

CONSIDERATION CLAUSE — In order for the policy to be legal and enforceable both parties must exchangesomething of value. The insurer's consideration is its promise to pay a valid claim. The policy owner's considerationis two-fold: (1) the premium paid and (2) statements (i.e., representations) made on the health insurance application.

RENEWABILITY PROVISIONS — Health insurance policies also contain a renewability provision whichstipulate conditions for renewal. There are several renewability provisions including:

! NONCANCELABLE — A noncancelable policy is one that may never be canceled as long as the policyowner pays the premium in a timely fashion. In addition, none of the policy's benefits or conditions maybe modified nor can there be any increase in cost. This allows the policy to be renewable up to a specifiedage which is generally the standard retirement age of 65. It is the most advantageous renewability provisionfor a policy owner but is also the most expensive.

! GUARANTEED RENEWABLE — This provision also states that the policy may not be canceled as longas the premium is paid in a timely fashion. Policy benefits, conditions and terms may not be changed in anyway. However, premiums may be increased but only on an occupational or other type of class basis (i.e.,all truck drivers). This type of provision also allows coverage to be renewed up to a specified age, such as65. Therefore, guaranteed renewable policies may have their premiums increased for an entire class ofpolicies. Individual disability income policies written on this basis are based upon occupational class whileMedigap plans are based upon geographical class.

! CANCELABLE / NONRENEWABLE — A cancelable provision allows the policy to be canceled by theinsurer at its discretion. This is the least advantageous of all renewability provisions. The premium for thistype of plan is lower than others. An insurer which issues a cancelable policy may cancel if an insured filestoo many claims in a policy period.

! CONDITIONALLY RENEWABLE — According to this provision the insurer may terminate the contractby not renewing it under certain conditions stated in the policy. There is a guarantee of continuance but notto the extent of the noncancelable or guaranteed renewable policies (i.e., age 65).

! OPTIONALLY RENEWABLE / RENEWABLE AT COMPANY'S OPTION — This provisionstates that the insured has no guarantee of continuance. As long as no or few claims are filed, the insurer willprobably renew. If a claim is filed the insurer may choose to non-renew.

! PERIOD OF TIME — Health insurance policies such as "short-term major medical" plans are onlyrenewable for a stipulated term (i.e., 6 months).

PRE-EXISTING CONDITIONS — This clause or limitation states that coverage under a health insurance policywill not be provided for any health condition of the insured that existed prior to coverage becoming effective. Thisprovision helps to protect an insurer if an applicant knows he or she is ill. Therefore, its purpose is to protect an

NILA ©

L&H Licensing

Chapter 17—5

Page 253: FORWARD - WordPress.com · Return of premium p 6-8 6.0 Annuities p 7-1 Annuity principles and concepts p 7-1 Accumulation period versus annuity period p 7-1 Owner, annuitant and beneficiary

insurer against adverse selection. Most policies include a pre-existing condition limitation (i.e., waiting period) ofseveral months. This means that if an insured is already being treated for a medical condition and then changesmedical plans, he or she must wait a specific period of time before the new plan provides coverage for the preexistingcondition. Insurers are generally not permitted to include this provision with a waiting period of more than one year. If the pre-existing condition is severe enough, the insurer may issue an impairment rider which will restrict coveragefor the illness or injury in question for a period of years or for the life of the policy. This provision allows an insurerto deny a claim for a health condition existing prior to coverage being provided under a new policy.

COMMON ACCIDENT PROVISIONS — This clause when included in a health insurance policy, ischaracterized by one deductible when two or more individuals in the same family are injured in the same accident. Usually each individual would have to satisfy a deductible if this provision is not included in the contract.

OWNER'S RIGHTS — This provision states that the policyholder is entitled to all ownership rights under thehealth insurance contract including the right to assign benefits to another or to name or change the beneficiary.

BENEFICIARY PROVISION — In cases where an accidental death benefit is added to the health insurancepolicy, a beneficiary may be designated. The primary beneficiary receives accidental death proceeds first. Acontingent beneficiary receives the death benefit if the primary beneficiary predeceases the insured.

MODES OF PREMIUM PAYMENT — The policy owner may select the method by which to pay annualpremiums including monthly, quarterly, semi-annual or annual. The annual mode is the least expensive mode ofpremium.

COORDINATION OF BENEFITS / NON-DUPLICATION OF BENEFITS — Over-insurance is achallenge that may exist in group and individual health insurance policies. Insurance companies attempt to controlover-insurance by including a coordination of benefits (COB) provision in such policies. This provision states thatbenefits will not be paid for medical expense amounts reimbursed by other policies. An insured is not permitted tocollect in claim benefits more than 100% of total covered medical expenses, no matter how many policies he owns. In addition, it also designates the order by which the policies will pay (i.e., which policy pays first or is primary). Forexample, assume John is covered by two major medical plans. The primary policy has a limit of $4,000 for a coveredloss. His secondary policy has a limit of $3,000 for the same covered loss. John suffers a covered loss and incursexpenses of $6,000. His primary carrier will pay up to its limit (i.e., $4,000) and the secondary carrier pays $2,000.

REDUCTIONS IN COVERAGE — Health insurance policies do not reduce coverage amounts following a lossaccording to this provision. Although most policies have maximum lifetime stated coverage limits.

QUIZ

1. All of the following are required uniform health insurance provisions, EXCEPT:

A. Time of payment of claims C. Change of occupation

B. Notice of claim D. Physical examination and autopsy

2. The entire contract provision is a required uniform policy provision. Which of the following statements is true regarding

this provision?

A. The entire contract consists of the health insurance policy

B. The entire contract stipulates to whom a death benefit will be paid

C. The entire contract provides for the payment of benefits within 90 days of the loss

D. The entire contract consists of the health insurance policy and a copy of the original application

NILA ©

L&H Licensing

Chapter 17—6

Page 254: FORWARD - WordPress.com · Return of premium p 6-8 6.0 Annuities p 7-1 Annuity principles and concepts p 7-1 Accumulation period versus annuity period p 7-1 Owner, annuitant and beneficiary

3. What health insurance provision allows an insurer to deny a claim if it finds that the insured concealed information about

his health?

A. Reinstatement C. Legal action

B. Grace period D. Time limit on certain defenses

4. Statements made by an applicant on a health insurance application are contestable by an insurer for an initial period of:

A. One year C. Three years

B. Two years D. Five years

5. An insured suffers a covered illness. He or she must notify the insurer of the loss within:

A. 5 days C. 15 days

B. 10 days D. 20 days

6. Which of the following provisions is a required uniform health insurance provision:

A. Misstatement of age C. Legal actionB. Recurrent disability D. Conformity with state statutes

7. A policy owner may have his policy reinstated after a lapse if a reinstatement application is completed and approved. Once this application is submitted, an insurer must notify the insured of its approval or declination within:

A. 30 days C. 45 daysB. 31 days D. 60 days

8. Written proof of loss must be supplied to an insurer within what period of time?

A. 30 days C. 90 days B. 60 days D. 120 days

9. When an insured suffers a covered illness, she must supply notice of loss to the insurer. Once this notice is received bythe insurer, it must send a proof of loss form to the insured within:

A. 10 days C. 20 daysB. 15 days D. 90 days

10. According to the time of payment of claims provision, insurers must provide payment of disability income benefits notless frequently than:

A. Daily C. MonthlyB. Weekly D. Quarterly

11. All health policies include a renewability provision. Which of the following provides an insured with the most advan-tageous provision?

A. Noncancelable C. CancelableB. Guaranteed renewable D. Conditionally renewable

12. The waiting period that must be satisfied before premiums will be waived under an accident and health contract is:

A. Two months

B. Three months

C. Six months

D. Nine months

NILA ©

L&H Licensing

Chapter 17—7

Page 255: FORWARD - WordPress.com · Return of premium p 6-8 6.0 Annuities p 7-1 Annuity principles and concepts p 7-1 Accumulation period versus annuity period p 7-1 Owner, annuitant and beneficiary

13. Jack owns a disability income policy with a 60 day waiting period. He is disabled for three months, returns to work for

three months, but has to go out on disability for three more months with the same injury. How many months benefits

will his policy pay?

A. Two months C. Six months

B. Four months D. Nine months

14. Paul applies for a health policy on February 5th. Underwriting approves the application on February 12th. The policy

is delivered by the agent on February 15th. When does the ten-day free-look period end?

A. February 24 C. February 26thth

B. February 25 D. February 22ndth

15. In a health insurance policy, the policyholder's consideration is the:

A. Premium paid

B. Statements made on the application

C. The original offer

D. The premium paid and representations on the application

16. The fact that an A&H premium payment of $500 must be paid is stipulated in which of the following parts of the policy?

A. Insuring clause C. Premium payment

B. Beneficiary provision D. Consideration clause

17. Once an insured is injured and notifies the insurer, the latter must send out a proof of loss form within:

A. 15 days C. 30 days

B. 20 days D. 90 days

18. If an insurer pays an A&H claim but later finds that the insured was injured while engaging in an illegal activity, what

action will it take?

A. Increase the premium

B. Deny the claim

C. Attempt to recover the claim payment

D. Deny the claim and return the premium

19. Joan buys a disability income policy. It insured her as a librarian. She later changes jobs and becomes a scuba diving

instructor. What action will the insurer take with regard to her policy?

A. It will be canceled

B. The premium will be reduced

C. The covered will be reduced

D. The coverage will be increased

20. What type of A&H policy can have its premiums increased for an entire class of policies?

A. Noncancelable

B. Guaranteed renewable

C. Conditionally renewable

D. Cancelable

NILA ©

L&H Licensing

Chapter 17—8

Page 256: FORWARD - WordPress.com · Return of premium p 6-8 6.0 Annuities p 7-1 Annuity principles and concepts p 7-1 Accumulation period versus annuity period p 7-1 Owner, annuitant and beneficiary

ANSWERS

1. C2. D3. D4. B5. D

6. C 7. C 8. C 9. B10. C

11. A12. B13. B14. B15. D

16. A17. A18. C19. C20. B

CHAPTER 17 KEY CONCEPTS

Beneficiary provisionCommon accidentConsiderationCoordination of benefitsFree-lookInsuring clause

Mandatory provisionsOptional provisionsOwners rightsPre-existing conditionsReductions in coverageRenewability provisions

NILA ©

L&H Licensing

Chapter 17—9

Page 257: FORWARD - WordPress.com · Return of premium p 6-8 6.0 Annuities p 7-1 Annuity principles and concepts p 7-1 Accumulation period versus annuity period p 7-1 Owner, annuitant and beneficiary

ACCIDENT & HEALTHSAMPLE

EXAM

Page 258: FORWARD - WordPress.com · Return of premium p 6-8 6.0 Annuities p 7-1 Annuity principles and concepts p 7-1 Accumulation period versus annuity period p 7-1 Owner, annuitant and beneficiary

ACCIDENT AND HEALTHINSURANCE SAMPLE EXAM

1. Which of the following hospice expenses is not covered in a cost containment setting?

A. Antibiotic drugs C. Painkillers

B. Palliative measures D. A specially designed hospital bed

2. An optional policy provision which permits the insurer to adjust benefit payments under certain situations or allows the

insured to receive a reduced premium under certain circumstances, best describes:

A. Coordination of benefits provision

B. Relation of earnings to insurance provision

C. Excess earnings provision

D. Change of occupation provision

3. Which of the following is a primary objective or goal of a Preferred Provider Organization?

A. To channel patients to providers who discount medical services

B. To reduce the cost of health care through preventive services

C. To increase the availability of fee-for-services plans

D. To relieve the provider of the responsibility for indemnification

4. Blue Cross and Blue Shield is an example of a:

A. Service insurer C. Medicare insurer

B. Mutual Assessment insurer D. Health Maintenance Organization

5. Which of the following medical plans provides comprehensive health care?

A. Dental care plan C. Basic medical expense coverage

B. Health maintenance organization D. Surgical expense plan

6. Which of the following providers of medical care provides a choice of physicians and hospitals to the member as well

as medical services at a reduced cost?

A. Group dental plan C. Preferred provider organization

B. Qualified plan D. Multiple employer trust

7. Each of the following is a cost savings method, EXCEPT?

A. Prospective review C. Elective cosmetic surgery

B. Second surgical opinion D. Preventive care

8. All of the following elements must be included in a contract in order for it to be legal, EXCEPT:

A. Offer and acceptance C. Consideration

B. Conditional receipts D. Competent parties to the contract

N IL A ©

L/H Licensing A & H Sample Exam 18—1

Page 259: FORWARD - WordPress.com · Return of premium p 6-8 6.0 Annuities p 7-1 Annuity principles and concepts p 7-1 Accumulation period versus annuity period p 7-1 Owner, annuitant and beneficiary

9. An individual has applied for health insurance and given the producer the initial premium. The producer provides the

applicant with a receipt which specifies that the individual is covered for 60 days from the date of application. Which

type of receipt was issued?

A. Approval receipt C. Tentative

B. Unconditional D. Insurability receipt

10. Which element of an insurance contract requires that the proposed insured be competent and have the capacity to enter

into a contract?

A. Competent parties C. Legal intent

B. Competent consideration D. Legal purpose

11. Premiums may not be increased on which type of health insurance policy?

A. Noncancelable C. Optionally renewable

B. Cancelable D. Guaranteed renewable

12. All of the following statements are true regarding a representation, EXCEPT:

A. A representation of a fact is the same as an expression of opinion or attitude

B. Representations appear on an application

C. A policy is usually voidable if a material fact is misrepresented

D. Representations may be written or oral

13. Individual health insurance administered as group insurance for groups which cannot be covered under a master policy

is:

A. HMO coverage C. Franchise coverage

B. Self insured coverage D. Service coverage

14. All of the following are true regarding comprehensive major medical insurance, EXCEPT:

A. It consists of basic and major medical coverage

B. It may use a corridor deductible

C. There are no first dollar benefits

D. It has coinsurance features

15. Hospice care is provided under Medicare and by medical plans. Which of the following would not be covered by such

care?

A. Pain relief C. Prescription drugs

B. Special hospital bed D. A surgical procedure to remove small tumors

16. Which of the following best defines a warranty?

A. A failure to disclose a material fact

B. Any statement made by the applicant as part of the insurance contract

C. A statement or condition that the applicant represents to be absolutely true

D. A statement that the contract is in accordance with state law

N IL A ©

L/H Licensing A & H Sample Exam 18—2

Page 260: FORWARD - WordPress.com · Return of premium p 6-8 6.0 Annuities p 7-1 Annuity principles and concepts p 7-1 Accumulation period versus annuity period p 7-1 Owner, annuitant and beneficiary

17. An insurer who fails to enforce a policy provision had engaged in a:

A. Warranty C. Evidentiary hearing

B. Waiver D. Estoppel

18. An applicant makes an offer to an insurer and the latter makes a counteroffer. Which of the following is correct with

regard to the applicant’s offer?

A. It is conditional C. It is a proposal

B. It is a promise D. It is no longer on the table

19. All of the following are true regarding Health Maintenance Organizations (HMOs), EXCEPT:

A. If an employer with at least 25 employees offers health care benefits, they must be enrolled in an HMO

B. HMOs stress preventive health care

C. HMOs are considered prepaid health care plans

D. Members may select a primary care physician

20. All of the following statements are true regarding surgical expense benefits in a basic medical expense policy,

EXCEPT:

A. Only those operations listed in the schedule are covered

B. Benefits are usually limited to a specific dollar amount

C. A relative value may be used to determine benefits

D. Benefits may be expressed as multiples of the hospital benefit maximum

21. Which of the following is not an optional policy provision found in a basic medical expense policy?

A. Notice of claim C. Insurance with other insurers

B. Misstatement of age or sex D. Intoxicants and narcotics provisions

22. A major medical policy would cover expenses for all of the following, EXCEPT:

A. In-patient physician's fee C. Charges for emergency surgery due to an accident

B. Elective cosmetic surgery D. Room and board expenses

23. Which of the following medical plans is a combination of an HMO and a PPO?

A. A point of service plan C. Multiple employer trusts

B. Group medical plan D. COBRA

24. Assume that a Major Medical policy has a $100 initial deductible and 80/20 coinsurance. The insured's covered

expenses are $1,000. How much does the insured have to pay?

A. $100 C. $280

B. $180 D. $720

25. An individual is a member of a Health Maintenance Organization. How is this member able to make an appointment

with a specialist?

A. By receiving a referral from the primary care physician

B. By completing a claim form within the required time

C. By selecting an open panel plan

D. By selecting a closed panel plan

N IL A ©

L/H Licensing A & H Sample Exam 18—3

Page 261: FORWARD - WordPress.com · Return of premium p 6-8 6.0 Annuities p 7-1 Annuity principles and concepts p 7-1 Accumulation period versus annuity period p 7-1 Owner, annuitant and beneficiary

26. Which of the following factors is not of primary importance when underwriting health insurance?

A. Physical condition and medical history of the applicant

B. The occupation of the applicant

C. The age and sex of the applicant

D. The marital status of the applicant

27. Social security benefits are provided for all of the following situations, EXCEPT:

A. Disability C. Workers' Compensation

B. Retirement D. Survivor's benefits

28. The two basic parts of the Medicare program are:

A. Hospital insurance and medical expenses

B. Inpatient hospital services and outpatient services

C. Disability benefits and home health care services

D. Hospital services and home health services

29. According to the Health Insurance Portability and Accountability Act of 1996, if a new employee satisfies all

requirements, the new employer must offer health insurance coverage on which of the following bases?

A. Conditional C. Qualitative

B. Probationary D. Guaranteed issue

30. Which of the required uniform provisions in a health insurance policy prevents the producer from changing any policy

provision?

A. Entire contract provision C. Conformity with state statutes

B. Time limit of certain defenses D. Legal action provision

31. According to HIPAA, an individual seeking health insurance coverage under a new plan must satisfy a preexisting

condition exclusion period if he or she has a gap in coverage of:

A. 31 days C. 63 days or more

B. 62 days or more D. 69 days

32. All of the following statements are true regarding reinstatement of a health insurance policy, EXCEPT:

A. Reinstatement may occur without an application for reinstatement

B. Sickness is covered after a 10 days probationary period

C. The reinstatement premium can be applied retroactively for a period of up to 90 days prior to reinstatement

D. Accidents are covered immediately upon reinstatement

33. All of the following are true regarding the insuring clause of a health insurance policy, EXCEPT:

A. The application information is part of the consideration

B. There is always a probationary period present

C. It identifies the illness not covered

D. It states that both accidents and sickness can be covered

N IL A ©

L/H Licensing A & H Sample Exam 18—4

Page 262: FORWARD - WordPress.com · Return of premium p 6-8 6.0 Annuities p 7-1 Annuity principles and concepts p 7-1 Accumulation period versus annuity period p 7-1 Owner, annuitant and beneficiary

34. Which of the following is usually not excluded in a disability income policy?

A. Aviation losses C. Occupational injuries

B. War related injuries D. Intentionally self-inflicted injuries

35. All of the following are important health insurance underwriting factors, EXCEPT:

A. Medical information C. The initial premium

B. The producer's statement D. The application

36. The Fair Credit Reporting Act:

A. Provides guidelines on premium financing

B. Is not applicable to the health insurance industry

C. Establishes procedures for consumer reporting agencies

D. Prohibits the use of information on an individual's character and general reputation

37. Which of the following best describes a situation where a medical care giver contracts with a health organization to

provide services to members but maintains the right to treat patients who are not subscribers?

A. Tabular plan C. Closed panel plan

B. Open panel plan D. Multiple employer trust

38. Which health insurance provision states that in consideration of the payment of premiums, the insurer agrees to pay the

policy benefits to the insured upon receipt of proof of loss?

A. The insuring clause C. The consideration provision

B. The entire contract D. The proof of loss provision

39. According to the proof of loss provision of a health insurance policy, how long does the insured have to file a

completed proof of loss?

A. 15 days C. 60 days

B. 20 days D. 90 days

40. The law of large numbers provides that:

A. Predictions become more accurate as the number of exposure units increases

B. Large losses are easier to predict than small losses

C. Most losses are not predictable

D. All losses are predictable

41. All of the following entities are permitted to be the policy owner of a group medical plan, EXCEPT?

A. A risk retention group C. A trust

B. Employer D. An association

42. Under reimbursement plans, benefits are paid to the:

A. X-ray technician C. Beneficiary

B. Insured D. Hospital administrator

N IL A ©

L/H Licensing A & H Sample Exam 18—5

Page 263: FORWARD - WordPress.com · Return of premium p 6-8 6.0 Annuities p 7-1 Annuity principles and concepts p 7-1 Accumulation period versus annuity period p 7-1 Owner, annuitant and beneficiary

43. Disability income benefits received by an insured are taxable if:

A. The insured is under 59 ½

B. The employer paid the premium

C. The insured paid the premium

D. The insured is not eligible for the income tax exclusion

44. A Medical Savings Account is available to groups with how many employees?

A. 50 or fewer C. 25 or fewer

B. 25 or more D. 100 or less

45. All of the following are true regarding Blue Cross/Blue Shield, EXCEPT:

A. Blue Cross/Blue Shield is a type of mutual organization

B. Blue Cross/Blue Shield is not regulated by the Insurance Department since they are not considered to be

insurers

C. Blue Cross pays claims on a service basis to participating medical facilities

D. Blue Cross/Blue Shield plans are identified as prepaid since subscribers pay a set monthly fee for medical

services

46. Which of the following is a purpose of managed care?

A. Managed care coordinates medical expenses

B. Managed care provides for the continuation of coverage

C. Managed care controls health insurance claims expenses

D. Managed care supplements COBRA

47. Which of the following is true regarding the Consolidated Omnibus Budget Reconciliation Act (COBRA)?

A. COBRA is a federal law which applies to all employers with 15 or more employees

B. Any terminated employee may convert his/her health insurance for a period not to exceed 36 months

C. COBRA is a federal law which provides terminated employees and survivors of deceased employees with

continuation of coverage options under certain circumstances

D. Beneficiaries of a deceased employee may continue coverage for up to 18 months

48. All of the following is information that is included in an employee’s certificate of insurance, EXCEPT?

A. Information concerning the filing of a claim

B. The benefits provided by the group plan

C. The premium that is paid by the policy holder

D. The policy period

49. An insured has a comprehensive major medical expense plan which provides 100% coverage for the first $5,000 of

covered expenses followed by a corridor deductible of $250, 80/20 coinsurance on the next $5,000 of expenses,

followed by 100% coverage thereafter. The insured incurs $12,500 of covered expenses. How much will the insured

pay?

A. $11,250 C. $1,650

B. $9,000 D. $1,250

N IL A ©

L/H Licensing A & H Sample Exam 18—6

Page 264: FORWARD - WordPress.com · Return of premium p 6-8 6.0 Annuities p 7-1 Annuity principles and concepts p 7-1 Accumulation period versus annuity period p 7-1 Owner, annuitant and beneficiary

50. If a deductible must be satisfied each time a new claim is submitted in a calendar year, it is known as:

A. A calendar year deductible C. A stop-loss deductible

B. An all cause deductible D. A per occurrence deductible

51. Considerations or factors in underwriting group health insurance include all of the following, EXCEPT:

A. Composition of the group C. The individual health histories of each patient

B. The possibility of adverse selection D. Participation requirements

52. Which of the following are examples of service plans providing prepaid medical services?

A. Blue Cross/Blue Shield and HMOs C. Blue Cross/Blue Shield and self insured plans

B. HMOs and self insured plans D. HMOs, Blue Cross/Blue Shield and METs

53. Which of the following best describes when a medical care giver contracts with a health organization to provide

services to its members but retains the right to treat patients who are not members?

A. An open panel plan C. A comprehensive plan

B. A closed panel plan D. A reimbursement plan

54. Which of the following fee for service plans allows an insured to select health care from any qualified provide?

A. Commercial insurance plan C. Blue Cross / Blue Shield

B. HMO D. Managed care

55. All of the following are covered benefits on Part A of Medicare, EXCEPT:

A. Home health care C. In-patient hospital care

B. Out-patient diagnostic services D. Hospice care

56. Which of the following is not considered to be preventive care as defined by a group or individual medical plan?

A. Chemotherapy treatment C. Annual physical examination

B. Mammograms D. Pap smears

57. The one-time period following the effective date of a health insurance policy before certain sickness coverage becomes

available or effective is the:

A. Elimination period C. Grace period

B. Probationary period D. Incontestable period

58. The loss of sight or the loss of any two limbs is an example of:

A. Presumptive disability C. Partial disability

B. Residual disability D. Occupational disability

59. All of the following are true regarding Workers' Compensation, EXCEPT:

A. Benefits are provided for disability, death, medical and rehabilitation

B. The waiting period for benefits under the loss of wages provision is 30 days

C. Disability benefits are provided for permanent total and permanent partial disability

D. Workers' Compensation is a state program as opposed to a federal program

N IL A ©

L/H Licensing A & H Sample Exam 18—7

Page 265: FORWARD - WordPress.com · Return of premium p 6-8 6.0 Annuities p 7-1 Annuity principles and concepts p 7-1 Accumulation period versus annuity period p 7-1 Owner, annuitant and beneficiary

60. A Comprehensive Major Medical policy provides 100% coverage for the first $10,000 of medical expenses. The plan

has a corridor deductible of $250 and 80/20 coinsurance. The insured incurs $16,750 in medical expenses. How much

of this bill will be paid by the insurer?

A. $1,550 C. $13,400

B. $15,200 D. $15,400

61. What type of rating helps an employer with low claims experience receive lower premiums?

A. Community rating C. Manual rating

B. Experience rating D. Retrospective rating

62. Health insurance used to cover students, passengers of a common carrier or sports team is:

A. Specified disease coverage C. Blanket health insurance

B. Accident only insurance D. Group specified disease coverage

63. Which of the following is provided by a Business Overhead Expense policy?

A. Accidental death benefits for the beneficiary

B. Disability income benefits paid for utility costs

C. Disability income benefits paid to the surviving stockholders

D. Disability income benefits to purchase the business interest of a disabled owner

64. Which of the following is not true regarding conversion of group insurance?

A. The insured has 31 days following termination of employment to convert to an individual plan

B. Dependent children of an insured also have a conversion right

C. Evidence of insurability is usually required

D. The conversion privilege must be provided in group health policies

65. Which of the following is not true regarding continuation of coverage under COBRA?

A. Coverage may be continued for up to 18 months if a covered employee dies

B. The coverage that is continued must be paid for by the terminated employee or beneficiary of a deceased

employee

C. A divorced spouse is eligible for continuation of coverage

D. Terminated employees are eligible for continuation of coverage

66. The purpose of the coordination of benefits provision in group health insurance is to:

A. Assure that the insured collects all benefits due

B. Protect against over-insurance

C. Eliminate benefits for unnecessary surgery

D. Reduce outpatient benefit payments

67. In a contributory plan of group health insurance, the participation requirement is:

A. 50%

B. 75%

C. 100%

D. There is no participation requirement for a contributory plan

N IL A ©

L/H Licensing A & H Sample Exam 18—8

Page 266: FORWARD - WordPress.com · Return of premium p 6-8 6.0 Annuities p 7-1 Annuity principles and concepts p 7-1 Accumulation period versus annuity period p 7-1 Owner, annuitant and beneficiary

68. Which of the following is true regarding taxation of disability income benefits?

A. Benefits are not taxable if they begin after the insured is 59 ½

B. Benefits are not taxable if the premium is paid by the employer

C. Disability income benefits are always taxable to the insured

D. Benefits are not taxable if the premium is paid by the insured

69. A group insurance provision which limits the insured's liability for covered claims is the:

A. Corridor deductible C Stop loss provision

B. Restoration of benefits provision D. Medical expense limitations provision

70. All of the following are true regarding Accidental Death and Dismemberment coverage, EXCEPT:

A. Benefits for loss to certain members may be provided according to the capital sum

B. Generally, accidental death and dismemberment contracts are high cost policies

C. Benefits for the accidental death are provided according to the policy's principal sum

D. This coverage may be sold as part of a life insurance policy

71. Which of the following is not a cost saving service?

A. Cosmetic surgery resulting from an accident C. Pre-admission testing

B. Second surgical opinion D. Concurrent review

72. A health care insurer which stresses preventive health care is:

A. Blue Cross/Blue Shield C. A Health Maintenance Organization

B. Any commercial insurer D. A primary health provider

73. All of the following are true regarding Blue Cross/Blue Shield, EXCEPT:

A. Blue Cross/Blue Shield is a non-profit organization in many states

B. Blue Shield covers physicians services

C. Blue Cross is a hospital service plan

D. Blue Cross pays claims on a reimbursement basis

74. Which of the following best describes a health care provider that provides comprehensive coverage?

A. Health maintenance organization C. Point of service plan

B. Supplemental medical expense plan D. Long-term care plan

75. Which of the following is used to monitor the insured’s hospital stay to make sure that everything is proceeding

according to schedule?

A. Prospective review C. Concurrent review

B. Retrospective review D. Utilization review

76. Which of the following is correct regarding the claim process?

A. The insured has 15 days to notify the insurer of a claim and the insurer has 20 days to provide claim forms

B. The insured has 60 days to notify the insurer of a claim and the insurer has 20 days to provide claim form

C. The insured has 60 days to notify the insurer of a claim and the insurer has 90 days to provide claim forms

D. The insured has 20 days to notify the insurer of a claim and the insurer has 15 days to provide claim forms

N IL A ©

L/H Licensing A & H Sample Exam 18—9

Page 267: FORWARD - WordPress.com · Return of premium p 6-8 6.0 Annuities p 7-1 Annuity principles and concepts p 7-1 Accumulation period versus annuity period p 7-1 Owner, annuitant and beneficiary

77. Which of the following is not considered one of the Uniform Mandatory policy provisions of a health insurance

policy?

A. Proof of loss C. Conformity with state statutes

B. Change of beneficiary D. Legal actions

78. Which of the following is true regarding Relation of Earnings to insurance provision?

A. It only applies to disability income coverage

B. It protects the insured from Over-insurance

C. It permits the insured to increase policy benefits for disability income coverage

D. It only applies to medical expense insurance

79. Insurance can best be defined as:

A. A systematic form of wagering

B. Protection against living too long

C. The spreading of financial risk over a large group of persons

D. The spreading of financial risk over a large group of individuals in order to minimize the potential loss to any

one person

80. The administrative cost for group health insurance is less than that for:

A. Limited coverage C. Blanket coverage

B. Individual insurance D. Insurance issued on an expense incurred basis

81. Refusing to invest in the stock market best describes which of the following methods of risk management?

A. Prevention C. Retention

B. Reduction D. Avoidance

82. The type of reinsurance which is negotiated on an individual risk basis is:

A. Treaty reinsurance C. Excess loss reinsurance

B. Facultative reinsurance D. Quota share reinsurance

83. An insurer who permits the agent to collect past due premiums in order to reinstate a lapsed policy even though the

agent's contract does not permit it, is an indication of:

A. Implied authority C. Apparent authority

B. A contract violation subject to penalty D. Non-contractual authority

84. All of the following are optional coverages that may be covered by an HMO, EXCEPT?

A. Nursing home care C. Home health care

B. Pharmacy fees D. Emergency services

85. Which of the following best describes the purpose of a gatekeeper in a Health Maintenance Organization?

A. Cost control with regard to specialists C. Collecting prepaid premiums

B. Providing support services D. Reviewing services provided to members

N IL A ©

L/H Licensing A & H Sample Exam 18—10

Page 268: FORWARD - WordPress.com · Return of premium p 6-8 6.0 Annuities p 7-1 Annuity principles and concepts p 7-1 Accumulation period versus annuity period p 7-1 Owner, annuitant and beneficiary

86. Regarding the concept of agency, an agent is considered to always be representing:

A. The insurer C. Either the insurer or the insured

B. The insured D. Anyone

87. Which of the following provisions guarantees payment of premiums in the event of the total disability of the insured?

A. Automatic premium loan C. Extended term provision

B. Disability income rider D. Waiver of premium benefit

88. An insured wishes to guarantee his or her future insurability. Which provision or option should be included in the

policy?

A. Payor provision C. Return of premium provision

B. Guaranteed insurability option D. Automatic premium loan provision

89. What provision in a health insurance policy allows the insurer to void coverage if the insured has engaged in material

misrepresentation?

A. Entire contract provision C. Insuring provision

B. Time limit on certain defenses provision D. Incontestable clause

90. All of the following are true regarding the reinstatement provision, EXCEPT:

A. Reinstatement does not require proof of insurability

B. It must take place within three years of the due date of the defaulted premium

C. Unpaid premiums plus interest must be paid

D. It is designed primarily to protect the insured

91. Which of the following best describes assignment?

A. The legal transfer of the insured's rights and interest under a policy

B. The legal transfer of the insurer's rights and interest under a policy

C. The legal transfer of the policyowner's rights and interest under a policy

D. The legal transfer of the beneficiary's rights and interest under a policy

92. Medical plans charges that establish the amount of benefit paid based upon prevailing charges which fall within a

standard range of fees usually charged for a specific procedure by a physician in a particular geographic area are

known as:

A. Usual, customary and reasonable C. Warranties

B. Fee for service charges D. Managed care

93. All of the following are true regarding the misstatement of age provision, EXCEPT:

A. This provision prohibits the insurer from voiding the policy due to an error in age

B. The insurer must discover the misstatement of age within the 2 year contestable period

C. If the age is misstated, the amount payable under the policy will be adjusted to reflect the proper amount for

the correct age

D. Misstatement of sex may also be included as part of this provision

94. Premiums paid for a qualified individual long-term care policy may be:

A. Tax exempt C. Not tax deductible

B. Tax deductible D. Tax free without limitation

N IL A ©

L/H Licensing A & H Sample Exam 18—11

Page 269: FORWARD - WordPress.com · Return of premium p 6-8 6.0 Annuities p 7-1 Annuity principles and concepts p 7-1 Accumulation period versus annuity period p 7-1 Owner, annuitant and beneficiary

95. Ted is eligible for Medicare Part A and is hospitalized for 110 days. Prior to his hospital stay, he had not used any of

his lifetime reserve days. Select the following correct statement.

A. Ted must pay only a deductible amount for the hospitalization portion of his stay

B. Ted must pay a deductible amount and a daily coinsurance charge for up to 90 days. Medicare pays the entire

remaining hospital bill for days 91 through 110

C. Ted pays nothing, Medicare pays the entire hospital bill

D. Ted pays the deductible amount for the first 60 days in the hospital, a daily coinsurance charge for days 61

through 90, and an increased daily coinsurance charge for days 91 through 110, if the lifetime reserve days

are used

96. All of the following statements pertaining to Medicare and Medicare Supplement insurance are correct, EXCEPT:

A. Dale is covered under Medicare Part B insurance. He has a total of $1,100 of approved medical charges

submitted to Medicare, after paying the required deductible. Of that total, Dale must pay $880.

B. The general enrollment period for Medicare extends from January 1st to March 31st

C. Emily became 65 on June 10th last year. Her initial enrollment period for Medicare extends from March 1st

to (but not including) October 1st

D. Douglas is a Medicare participant. His Medicare Supplement policy normally will pay Medicare deductible

and coinsurance amounts

97. Which of the following is not provided by the Home Health Care Benefit of Part A of Medicare?

A. Physical therapy

B. Medical supplies furnished by the home health agency

C. General household services

D. Prescribed medical equipment

98. All of the following must be provided by Medicare Supplement policies, EXCEPT:

A. The policy must pay the eligible hospital expenses that Part A doesn't cover during the 61st through 90th days

of the hospital stay during a benefit period

B. The policy must pay what Part A doesn't pay for eligible expenses during the lifetime reserve days

C. Once Medicare hospitalization benefits expire, the policy must pay 100% of Medicare-eligible expenses for

an additional 365 days

D. The policy must pay eligible hospital expenses that Part A doesn't cover including the deductible

99. Benefits for some illnesses are often excluded under Long-Term Care policies. Which of the following would be

covered by this policy?

A. Alcoholism C. Alzheimer's disease

B. Mental or nervous conditions D. Mental retardation

100. Which of the following is not a minimum standard requirement for Medicare Supplement policies?

A. Medicare Supplement policies must have a 10 day free look

B. Medicare Supplement policies must be guaranteed renewable for life

C. Medicare Supplement policies must cover preexisting conditions after six months

D. There must be annual automatic adjustments to reflect changes in Medicare deductibles and co-payment

101. When an unskilled person provides care for a long-term care patient, which of the following is being provided ?

A. Respite care C. Intermediate care

B. Custodial care D. Adult day care

N IL A ©

L/H Licensing A & H Sample Exam 18—12

Page 270: FORWARD - WordPress.com · Return of premium p 6-8 6.0 Annuities p 7-1 Annuity principles and concepts p 7-1 Accumulation period versus annuity period p 7-1 Owner, annuitant and beneficiary

102. Which of the following may be added to a long-term care policy to keep pace with an increase in the cost of such care?

A. Nonforfeiture benefit C. Respite care benefit

B. Home health care benefit D. Inflation protection benefit

103. Which of the following is used to fund Part A of Medicare ?

A. Self-employment tax C. Payroll taxes

B. Exclusion ratio D. No loss no gain statutes

104. Some accident and health insurance policies provide benefits as a result of accidental means. Which of the following

best describes the concept of accidental means ?

A. The cause of a loss must be due to an accident

B. It is an expected or intended cause of loss

C. The result of an injury must be caused by an accident

D. The cause and the result of an injury must be accidental

105. A disability policy definition which states that an individual cannot perform all the duties of his occupation but can

perform some duties, best describes:

A. Partial disability C. Total disability

B. Residual disability D. Temporary disability

106. Which of the following allows for the portability of health insurance coverage ?

A. ADEA C. HIPAA

B. ERISA D. OBRA of 1993

107. For what length of time may a mentally or physically handicapped dependent remain covered under a health insurance

plan?

A. Up to age 19 C. For life

B. Up to age 23 if a full-time student D. For as long as they are handicapped

108. When an individual misstates his or her age on an application for health insurance, what will the insurer do when a

claim is later filed?

A. Pay the claim

B. Deny the claim

C. Pro rate the claim to pay for what the premium would have purchased at the correct age

D. Short rate the claim to pay for what the premium would have purchased at the correct age

109. Benefits paid by accidental death and dismemberment coverage are:

A. Taxable as ordinary income C. Not taxable

B. Taxable as a capital gain D. Not taxable unless they exceed $5,200

110. Which of the following is paid by Workers’ Compensation coverage?

A. Medical expenses

B. Payments for the pain and suffering of an employee

C. Weekly income

D. Weekly income and unlimited medical expenses

N IL A ©

L/H Licensing A & H Sample Exam 18—13

Page 271: FORWARD - WordPress.com · Return of premium p 6-8 6.0 Annuities p 7-1 Annuity principles and concepts p 7-1 Accumulation period versus annuity period p 7-1 Owner, annuitant and beneficiary

111. Which of the following is excluded by a major medical expense plan?

A. Necessary cosmetic surgery

B. Self-inflicted injury as a result of an accident

C. Benefits paid to an employee injured on the job

D. Dental care resulting from a fall

112. How are benefits taxed when paid by an AD&D policy for dismemberment or loss of sight?

A. Tax free C. Partially taxable

B. Taxable D. Tax deductible

113. Which of the following will occur when an individual gives an insurer an incorrect age at the time of application for

insurance and later notifies the insurer of a covered loss?

A. The insurer will reduce the claim and pay what the premium would have purchased at the attained age.

B. The insurer will pay a benefit equal to what the premium would have purchased at the correct age.

C. The insurer will deny the claim due to the material misrepresentation of the policy owner.

D. The insurer will pay the claim as long as the proper premium is eventually paid.

114. Physicians providing services for subscribers of open-panel plans may also treat patients who are not HMO

subscribers. In this case, on what basis are such patients treated?

A. Indemnity basis C. Reimbursement basis

B. Fee for service basis D. Relative value basis

115. Which of the following is a disadvantage of a closed-panel plan?

A. It manages care geographically

B. It controls behavior of patients by controlling costs

C. It offers preventive care on a usual, customer and reasonable basis

D. It limits choice of physicians

116. A primary advantage of field underwriting is:

A. Sharing commissions C. Seeing the prospect in person

B. Providing fiduciary services D. Disclosing information to the consumer

117. Once an individual is laid-off, he or she has what length of time to elect medical coverage under COBRA?

A. Thirty days C. Ninety days

B. Sixty days D. One hundred eighty days

118. Which of the following best describes the standard range of fees charged by a physician?

A. Fee for service C. Capitation

B. Usual, customary and reasonable D. Co-payment

119. Medical plans, disability income policies and Medicare may all be offset by or coordinate with which of the following?

A. Long-term care insurance C. Occupational policies

B. Non-contributory plans D. Workers’ Compensation coverage

N IL A ©

L/H Licensing A & H Sample Exam 18—14

Page 272: FORWARD - WordPress.com · Return of premium p 6-8 6.0 Annuities p 7-1 Annuity principles and concepts p 7-1 Accumulation period versus annuity period p 7-1 Owner, annuitant and beneficiary

120. Physically challenged and handicapped children are covered by their parents individual or group medical plan on what

basis?

A. Up to age 19 unless a full-time student

B. Up to age 23

C. Up to age 25

D. For as long as they are dependent on their parents

121. The purpose of a carry-over provision in an individual or group medical expense plan is:

A. To discourage overinsurance

B. To prevent profiting by owning a medical plan

C. To credit expenses incurred so as not to penalize the insured

D. To satisfying participation requirements and increase persistency

122. Which of the following protects the interests of participants in employee benefit plans?

A. ERISA C. ADEA

B. OSHA D. Pregnancy Discrimination Act

123. If an employer has fifty employees covered by a group medical plan and one is eligible for Medicare, where is the

latter employee covered?

A. Medicare is primary

B. The group plan is primary

C. Medicare and the group plan will pay a proportionate share of any medical expenses incurred

D. The group plan is secondary

124. The best type of employer risk to select self-funding is one that has:

A. Administrative services C. Effective cost controls

B. Predictable claims D. Negotiated discounts

125. Medicare provides some nursing home coverage if it is part of the treatment ordered by:

A. The federal government C. The attending physician

B. A skilled nursing facility D. The retiree

126. Which of the following is responsible for the funding of Medicaid?

A. The federal government C. Medicare

B. Social Security D. The state and federal government

127. According to HIPAA, a qualified long-term care plan is one that satisfies each of the following, EXCEPT:

A. Does not pay expenses reimbursable by Medicare

B. Must include a non-cancellable renewability provision

C. Does not include a cash surrender value

D. Only provides long-term care services

N IL A ©

L/H Licensing A & H Sample Exam 18—15

Page 273: FORWARD - WordPress.com · Return of premium p 6-8 6.0 Annuities p 7-1 Annuity principles and concepts p 7-1 Accumulation period versus annuity period p 7-1 Owner, annuitant and beneficiary

128. Which of the following is an example of preventive care included as part of a dental expense plan?

A. Minimum coinsurance amount for periodontics

B. Basic coverages for treatment for bridgework

C. No deductible for routine dental care

D. Blanket protection with regard to crowns and fillings

129. Which of the following provisions is intended to prevent an individual from profiting if he or she is covered by two

insurance policies issued by the same insurer?

A. Conformity with state statutes C. Other insurance with this insurer

B. Insurance with other insurers D. Time of payment of claims

130. Which part of an accident and health policy identifies the benefits the insurer will pay?

A. Insuring clause C. Coordination of benefits provision

B. Consideration clause D. Renewability provision

131. In order to be insurable a risk must be:

A. Non-measurable C. Unpredictable

B. Calculable D. Transferable

132. The tendency of poorer risks to seek protection under an insurance policy best describes:

A. Capital indifference C. Adverse selection

B. Outstanding selection D. Reverse discrimination

133. When two parties enter into an insurance contract, which party assumes risk for a premium?

A. Applicant C. Insurer

B. Policyowner D. Insured

134. All of the following are responsibilities of an insurance company’s underwriters, EXCEPT:

A. Assess risks C. Collect premiums

B. Determine premiums D. Decline or approve applicants

135. Field underwriting helps to guard against which of the following?

A. Moral hazards C. Substandard selection

B. Risk assessment D. Unfair premium determination

136. Which of the following types of medical plans pay providers in advance for medical services provided?

A. Comprehensive plans C. Fee for service plans

B. Prepaid plans D. Scheduled plans

137. The employer who secures group health insurance for his or her employees is known as the:

A. Certificate holder C. Benefit plan holder

B. Master policy holder D. Employee benefits manager

N IL A ©

L/H Licensing A & H Sample Exam 18—16

Page 274: FORWARD - WordPress.com · Return of premium p 6-8 6.0 Annuities p 7-1 Annuity principles and concepts p 7-1 Accumulation period versus annuity period p 7-1 Owner, annuitant and beneficiary

138. A Health Maintenance Organization offers medical services to subscribers living in specific geographic boundaries or

territories. Generally, this means that the territories are divided by?

A. State C. Voting districts

B. County D. Zip code

139. Physicians are considered to be employees of an HMO under which of the following?

A. Open panel plan C. Comprehensive plan

B. Closed panel plan D. Major medical plan

140. In a Point of Service (POS) plan, which of the following will occur if a subscriber utilizes the services of a non-

member physician?

A. The physician will be paid on a fee for service basis and the subscriber’s premium will be increased

B. The physician will be paid on a fee for service basis and the subscriber must pay a higher coinsurance amount

C. The plan will not pay if treatment is provided by a non-member physician

D. The plan will pay a portion of the charges

141. Which of the following employer paid accident and health benefits would create a taxable event to an employee?

A. Accidental death benefit C. Disability benefit

B. Medical plan benefit D. AD&D rider

142. If an individual qualifies for Medicare and also possesses coverage under a group medical plan, which plan is primary?

A. Medicare Part A C. Group medical plan

B. Medicare Part B D. Managed care plan

143. When is an individual enrolled in Part B of Medicare?

A. When he or she turns age 65

B. Once an individual is FICA qualified

C. Once you meet the Social Security definition of disability

D. Once an individual enrolls and pays a premium

144. A 72 year old man with limited assets may secure nursing home benefits from which of the following?

A. Medicare Part A C. Long-term care

B. Medicare Part B D. Medicaid

145. Which of the following has an open enrollment period?

A. Individual disability income C. AD&D coverage

B. Medicare D. Limited policies

146. Employer paid premiums for long-term care insurance are:

A. Taxable as ordinary income to the employee

B. Tax free

C. Tax deferred to the employee

D. Tax deductible for the employer and tax free to the employee

N IL A ©

L/H Licensing A & H Sample Exam 18—17

Page 275: FORWARD - WordPress.com · Return of premium p 6-8 6.0 Annuities p 7-1 Annuity principles and concepts p 7-1 Accumulation period versus annuity period p 7-1 Owner, annuitant and beneficiary

147. Which of the following individuals or risks would an insurer be most likely to avoid when deciding to conduct

insurance business?

A. Preferred risk C. Substandard risk

B. Standard risk D. Pure risk

148. Which of the following is covered by a long-term policy, whether an individual or group plan?

A. Hospital confinement C. In-patient care

B. Prescription drugs D. Alzheimer’s disease and dementia

149. Which of the following long-term care benefits provides twenty-four hour care and treatment?

A. Custodial care C. Hospice care

B. Skilled nursing care D. Respite care

150. Which of the following long-term care benefits provides for assistance with activities of daily living?

A. Respite care C. Skilled nursing care

B. Custodial care D. Home health care

151. What Part of Medicare provides for the enhanced possibility for managed health care?

A. Part A C. Part C

B. Part B D. Part D

152. When does an employer seek Workers’ Compensation coverage from an assigned risk plan?

A. When premiums under his or her current plan increase

B. When the number of employees increase

C. When the probationary period ceases

D. When coverage cannot be secured in the voluntary market

N IL A ©

L/H Licensing A & H Sample Exam 18—18

Page 276: FORWARD - WordPress.com · Return of premium p 6-8 6.0 Annuities p 7-1 Annuity principles and concepts p 7-1 Accumulation period versus annuity period p 7-1 Owner, annuitant and beneficiary

ACCIDENT AND HEALTH SAMPLE EXAM ANSWERS

1. A 2. D 3. A 4. A 5. B

6. C 7. C 8. B 9. B10. A

11. A12. A13. C14. C15. D

16. C17. B18. D19. A20. A

21. A22. B23. A24. C25. A

26. D27. C28. A29. D30. A

31. C32. C33. C34. C35. C

36. C37. B38. A39. D40. A

41. A42. B43. B44. A

45. B46. C47. C48. C49. D

50. D51. C52. A53. A54. A

55. B56. A57. B58. A59. B

60. B61. B62. C63. B64. C

65. A66. B67. B68. D69. C

70. B71. A72. C73. D74. A

75. C 76. D 77. C 78. A 79. D 80. B

81. D 82. B 83. A

84. D 85. A 86. A 87. D 88. B

89. B 90. A 91. C 92. A 93. B

94. B 95. D 96. A 97. C 98. D

99. C100. A101. B102. D103. C

104. D105. A106. C107. D108. C

109. C110. D111. C112. A113. B

114. B115. D116. C117. B118. B119. D120. D

121. C122. A

123. B124. B125. C126. D127. B

128. C129. C130. A131. B132. C

133. C134. C135. A136. B137. B

138. B139. B140. B141. C142. C

143. D144. D145. B146. D147. C

148. D149. B150. B151. C152. D

N IL A ©

L/H Licensing A & H Sample Exam 18—19

Page 277: FORWARD - WordPress.com · Return of premium p 6-8 6.0 Annuities p 7-1 Annuity principles and concepts p 7-1 Accumulation period versus annuity period p 7-1 Owner, annuitant and beneficiary

GLOSSARY OFLIFE and HEALTH

TERMS

Page 278: FORWARD - WordPress.com · Return of premium p 6-8 6.0 Annuities p 7-1 Annuity principles and concepts p 7-1 Accumulation period versus annuity period p 7-1 Owner, annuitant and beneficiary

GLOSSARY OFLIFE INSURANCE TERMS

ACCELERATED BENEFITS RIDER — This rider may be added to a life insurance policy that permits the policy owner to

“accelerate” or receive a certain percentage of the death benefit while the insured is still alive when he or she has been diagnosed

with a terminal illness. Whatever amount is paid to the owner under the rider is subtracted from the face amount when death occurs

and the remainder is paid to the designated beneficiary.

ACCIDENT — An unforeseen, unintended, unexpected event, or fortuitous event that causes death, injury or damage.

ACCIDENTAL DEATH AND DISMEMBERMENT INSURANCE — A form of insurance affording benefits in the event

of accidental death; the accidental loss of sight, speech or hearing; loss of use of limbs (i.e., paralysis); or loss of (a) member(s),

such as an arm or a leg.

ACCIDENTAL DEATH BENEFIT — A lump sum payment for loss of life due to an accident that was the direct cause of death.

The cause of the death must be accidental for a benefit to be payable under the policy.

ACCUMULATION PERIOD — This is the pay in period of an annuity during which the contract owner receives tax deferral.

During the accumulation period the contract owner is referred to as the policyowner. Once this period ends, the annuity phase or

the income phase commences.

ACTUARY — A person who calculates policy rates, reserves and dividends and makes other applicable statistical studies and

reports.

ADDITIONAL PREMIUM — Used in Universal Life Policies. Additional premiums can be paid into the policy account in an

amount above the target premium. Current tax laws limit the amount of excess cash value that can be accumulated in a life insurance

policy. The insurance company may not accept the additional premium if it nears this limit without increasing the limit of life

insurance (subject to underwriting).

ADDRESS CHANGE — Most States require that a producer notify the applicable regulatory department of any residence or

business address change. In addition, if a producer legally changes his or her name, notice of the change must also be provided.

ADVERSE SELECTION — The tendency of a disproportionate number of poor risks to seek or buy insurance or maintain

existing insurance in force (i.e., the selection against the insurance company). Sound underwriting reduces adverse selection.

AGE BASED PENALTY — When an individual prematurely withdraws funds prior to age 59 ½ from an annuity, a modified

endowment contract or a qualified plan, he or she will be assessed a 10 % penalty. No such penalty will be assessed if the

distribution is made after this stipulated age.

AGE CHANGE — The date halfway between birthdays when the age of the applicant changes to the next higher age. With some

insurers, the age is based upon the applicant's age at his nearest birthday. In others, it is based upon the age of his last birthday.

AGENT — An authorized representative of an insurer who is licensed to sell life insurance and annuity contracts. An agent

represents the insurer who sponsors him or her.

NOTE

A few of these terms are not included in your text but are here for your personal knowledge.

NILA ©

L/H Licensing

Glossary 19 -1

Page 279: FORWARD - WordPress.com · Return of premium p 6-8 6.0 Annuities p 7-1 Annuity principles and concepts p 7-1 Accumulation period versus annuity period p 7-1 Owner, annuitant and beneficiary

ALIEN INSURER — This is an insurer organized under the laws of a country other than the United States (i.e., Sun Financial

Services of Canada).

ANNUITANT — The individual whose life the annuity is based. If the annuity is paid out for a lifetime, the annuitant’s age is used

to determine the payments. The annuitant is the person who normally receives the annuity payment, however, that may or may not

be the case.

ANNUITIZATION — The period for which annuity benefits are paid.

ANNUITY — A contract between an insurer and owner affording periodic income payments for a fixed period of time or during

the lifetime of an annuitant. It may be defined as the systematic reimbursement or liquidation of an estate.

ANNUITY CERTAIN — An annuity that provides a benefit payment payable for a specified length of time regardless of whether

the annuitant lives or dies.

ANNUITY SURRENDER CHARGE — A fee charged by the insurance company for a partial or full withdrawal of funds made

during a specified period after the annuity is funded. Contract language determines the size of the fee and the length of surrender

charge period. Annuity surrender charges are intended to discourage the movement of funds for the defined period, allowing the

insurance company to recover their setup and administration costs.

APPLICATION — The statement of information given when a person applies for life, health, or disability insurance. The

insurance company’s underwriter uses this information as a basis in determining whether the applicant qualifies for acceptance under

the company’s guidelines. Applications are attached to and made a part of all individual contracts.

ASSIGNEE — An individual to whom the rights under a life insurance contract are transferred.

ASSIGNMENT — The act of transferring ownership rights of a life insurance policy by the owner to a third party (i.e., the

assignee). If all rights and the entire contract are assigned to another party an absolute assignment has occurred. If one or some

of the rights in the policy are transferred to another party but not all, a collateral or conditional assignment has taken place.

ATTAINED AGE — The age that a person or an insured has attained on a given date. For life insurance purposes, the age is based

on either the nearest birthday or the last birthday, depending upon the practices of the insurance company involved. This is also

referred to as “current age.”

ATTENDING PHYSICIAN STATEMENTS — Known as APS, these are used when the application or medical examiner’s report

reveals conditions or situations, past or present, about which more information is desired. Because of Physician/Patient

confidentiality, the applicant must sign an authorization, which allows the physician to release information to the insurance company

underwriter.

AUTHORIZED INSURER — This is an insurer that possesses a certificate of authority in a particular State. This certificate

“authorizes the insurer to transact insurance business in a particular State. An authorized insurer is also called an admitted or a

licensed carrier.

AUTOMATIC PREMIUM LOAN — A provision in a life insurance policy that states that if the policy owner fails to pay a

premium by the end of the grace period, the amount of the premium due will be loaned to the insured automatically from the policy’s

cash value. However, the cash value of the policy must be sufficient to cover the automatic loan. Generally, the policy owner must

request that this provision be made a part of the policy at the time of application (they may also request it once the contract takes

effect).

BACKDATING — Making the effective date of a policy earlier than the application date. Backdating is used to make the issue

age lower than at applicant’s real age in order to get a lower premium. State laws normally limit the time to which policies can be

backdated to six months. Backdating is not allowed to variable contracts due to the nature of the investment.

NILA ©

L/H Licensing

Glossary 19 -2

Page 280: FORWARD - WordPress.com · Return of premium p 6-8 6.0 Annuities p 7-1 Annuity principles and concepts p 7-1 Accumulation period versus annuity period p 7-1 Owner, annuitant and beneficiary

BENEFICIARY — The person(s) designated to receive policy proceeds in the event of the insured's death. Policies may include

a primary, contingent (secondary) or tertiary beneficiary.

BINDING RECEIPT (Unconditional Receipt) — Insurance becomes effective on the date of the receipt and continues for a

specified period of time or until the insurer declines the application.

BUYER’S GUIDE — This is a pamphlet that describes and compares various forms of life or health insurance. This guide must

be provided to a consumer by the producer when the latter is attempting to solicit insurance. This guide makes information available

to the consumer that assists them in making an informed decision when purchasing insurance coverage.

BUY - SELL AGREEMENT — An agreement between partners in a business or between an owner and a key employee. The

agreement provides for the continuation of the business when, for example, one of the partners dies. The remaining partners “buy-

out” the interest of the deceased partner by paying an agreed upon amount of funds to the deceased’s survivors. Life insurance is

generally used to fund or support the agreement.

CASH REFUND ANNUITY — A life annuity contract which provides that upon the death of the annuitant, a beneficiary will

receive a lump sum payment that represents the difference between the amount the annuitant paid to the insurer and the total income

payments received by the annuitant.

CASH VALUE — The equity portion of a whole life policy that increases with each subsequent premium payment. Interest is

paid by the insurer on the cash value which is tax deferred. The cash value in a whole life policy is designed to equal the policy’s

death benefit at age 100. This is when a traditional whole life policy is designed to mature.

CASH SURRENDER VALUE — The amount that is available in cash upon the surrender of a policy by the owner before or after

the policy matures (as a death claim or otherwise). Also referred to as surrender value.

CEDING COMPANY — In a reinsurance transaction, this is the insurer that transfers risk to another insurer called the

reinsurance company.

CERTIFICATEHOLDER — This is a covered employee in a group life or health plan. The employer receives the master policy

and each employee receives a certificate or a booklet describing their group benefits. A certificateholder in a group life plan

possesses an incident of ownership in the contract since they are allowed to designate the beneficiary.

CERTIFICATE OF AUTHORITY — This is the name of the license to transact insurance business that is issued to an

insurance company.

CERTIFICATE OF INSURANCE — A document containing information regarding the master policy of a group indicating that

an employee has coverage.

CLAIM — The demand to an insurer for the payment of benefits under a policy.

COMMINGLING — This is a term that means that a producer is combining premiums collected with personal funds. This is not

a legal activity and may be construed as larceny.

COMMISSION — Compensation or payments made by an insurer to a producer for the sale of an insurance policy.

COMMON DISASTER CLAUSE — This provision defines the method of payment of the proceeds of the policy by the insurer

if the insured and the named beneficiary die simultaneously in the same accident.

CONCEALMENT — Failure by an applicant to disclose in his or her application a material fact that is relevant to the acceptance

or the declination of an application for insurance coverage.

CONDITIONAL RECEIPT — A form, normally required to be signed by the agent and given to the prospective owner at the

time a new application is completed. The issuing of a receipt is subject to individual company rules. Most require that the agent

NILA ©

L/H Licensing

Glossary 19 -3

Page 281: FORWARD - WordPress.com · Return of premium p 6-8 6.0 Annuities p 7-1 Annuity principles and concepts p 7-1 Accumulation period versus annuity period p 7-1 Owner, annuitant and beneficiary

collect an initial premium and most usually grant some level of limited coverage, under special conditions, before issuance of the

policy. Without a valid Conditional Receipts, no coverage is in force until the policy is issued, delivered, and accepted (initial

premium paid).

CONSIDERATION — One of the requirements of a valid contract. The representations on the application and the premium is the

policy owner’s consideration. The insurance company's “promise to pay” is its consideration.

CONSULTANT — This is an individual who provides advice and consultation for a fee. In most States there are no regulations

governing the fee charged by a consultant. In order to operate as a consultant, an individual must secure a consultant’s license.

Consultants are also referred to as an insurance advisers counselor, specialist or analyst.

CONTINGENT BENEFICIARY (SECONDARY) — A person who is entitled to a death benefit only if there is no primary

beneficiary alive when the insured dies. A minor may be listed as a beneficiary but an insurer is not permitted to pay policy

proceeds to a minor beneficiary.

CONTINUING EDUCATION — States now have continuing education regulations in place that require producers and other

insurance professionals to maintain competency by satisfying specific requirements on a regular basis in order to hold an insurance

license. If a licensee fails to comply with such laws, he or she will have the license suspended or revoked until compliance is

completed.

CONTRIBUTORY PLAN — A term applied to a group insurance plan under which both the employee and the employer

contribute to premium payment. This may also be referred to as a shared premium plan.

CONVERSION PERIOD — Group life insurance includes a conversion privilege that permits the policy owner the right to

change coverage from term life to whole life without demonstrating insurability if effected during a 31 dat conversion period. This

privilege is also available to an employee if he or she wishes to change group term coverage to permanent whole life insurance.

CONVERTIBILITY — The right of a policy owner to exchange an existing policy for other policies offered by the insurance

company. This may be the conversion of individual term insurance to an individual permanent plan that company sells or the

conversion of group disability, life, or health to an individual plan.

CORRIDOR — In Universal Life Insurance, a level of pure insurance must be maintained more than the accumulated cash value.

Tax law regulates the amount.

CREDIT LIFE INSURANCE — Life insurance designed to pay the balance of a loan if the insured dies before the loan has been

repaid in full. Generally, credit life insurance is sold by a lender or finance company.

CROSS - PURCHASE PLAN — This is a type of buy-sell plan where each partner buys a policy covering each of the other

partners in order to have sufficient funds available to buy out the deceased partner’s interest upon his or her death.

DEATH BENEFIT — The amount that is paid upon the death of the insured. This is also called the face amount, coverage amount

or policy proceeds.

DEATH CLAIM — A completed form that proves the insured’s death. This must be filed with the insurer along with a copy of

the death certificate in order to receive policy proceeds.

DECREASING TERM INSURANCE — A type of temporary or pure protection characterized by a reducing face amount each

year. The cost for this coverage remains constant. Sometimes called mortgage redemption or mortgage protection insurance since

it is primarily used in conjunction with a debt or loan.

DEFAMATION — This is an unfair practice where a producer or insurer makes false and maliciously critical statements regarding

the financial condition of a competing insurer.

NILA ©

L/H Licensing

Glossary 19 -4

Page 282: FORWARD - WordPress.com · Return of premium p 6-8 6.0 Annuities p 7-1 Annuity principles and concepts p 7-1 Accumulation period versus annuity period p 7-1 Owner, annuitant and beneficiary

DEFERRED ANNUITY — A classification of an annuity where income payments commence more than one year after the

payment of the first (or single) premium to the insurer, usually at retirement.

DEFERRED COMPENSATION — A plan that allows selected individuals to defer the receipt of income in accordance with a

written agreement with their employer. The plan may be qualified or non-qualified.

DEFINED BENEFIT PLAN — A employee benefit retirement plan that uses a definite formula to determine the exact benefit

amount. Employer contributions to the plan are actuarially determined. The benefit to be paid in the future will determine the

amount of the contributions.

DEFINED CONTRIBUTION PLAN — This is an employee benefit plan under which each participant’s benefits are based

solely upon the contributions made to the participant’s account. The amount of the contributions will determine the future benefit

at retirement.

DEPENDENT RIDER — This is a rider that may be added to a life insurance policy specifically to provide coverage for

dependents of the primary insured. Dependent riders may cover a spouse, children or dependent parents. A family rider, for

instance, may cover the primary insured’s spouse and children (natural and adopted).

DISCLOSURE STATEMENT — A comparison form required by various state regulatory agencies to be given every policy owner

when replacing an existing policy with another.

DISCOUNT VALUE — The difference between an amount due in the future and its present value computed at a specific rate of

interest.

DIVIDEND — A refund of part of the premium under a participating policy or a share of surplus funds. They are derived from

savings in mortality and expenses.

DIVIDEND OPTIONS — The owner of a participating life insurance policy is granted several choices in which to receive or use

dividends earned:

• Receive the dividend in cash

• Use it to reduce the premium

• Apply it to purchase paid-up additional amounts of life insurance

• Leave it to accumulate at interest

• Use it to purchase one year term insurance

DOMESTIC INSURER — This is an insurer conducting business in the State in which it was formed, organized or chartered.

A company that has its home or principal office in the State in which it is conducting business.

ENDOWMENT — A whole life policy that, following an endowment period, pays a stated amount to the insured. If the insured

dies during the endowment period, the face amount of the policy is paid to the primary beneficiary.

EQUITY INDEXED ANNUITY — This is a type if fixed or non-variable annuity. The contract pays a guaranteed minimum

interest rate and account assets are tied to an index such as Standard and Poors. If the index is higher than the guaranteed rate each

year, the contract owner receives the greater return. If the index is lower than the guaranteed rate, the contract owner receives the

minimum guaranteed rate.

ESTATE TAX — A tax payable to the Federal Government and in some cases State Governments on the death of an individual.

There are Federal and State rules for calculating the tax. Life insurance can go to the beneficiary, free of income tax liability, but

without proper planning the amount of the death benefit may be included as part of the deceased’s estate and subject to estate tax.

ESTOPPEL — This is a legal principle that protects an insured if the insurer or its producers make an error and later the insurer

attempts to deny a claim. For instance, a producer makes a false representation to an applicant who relies on the statement. Later,

harm is caused to the policy owner when a claim is denied because of this reliance. The insurer will then be “estopped” or prevented

from denying the claim.

NILA ©

L/H Licensing

Glossary 19 -5

Page 283: FORWARD - WordPress.com · Return of premium p 6-8 6.0 Annuities p 7-1 Annuity principles and concepts p 7-1 Accumulation period versus annuity period p 7-1 Owner, annuitant and beneficiary

EVIDENCE OF INSURABILITY — Any statement or proof of a person's health history and current health status that qualifies

that person for coverage.

EXCEPTIONS — Provisions in a policy that eliminate coverage for specified causes of death.

EXCLUSION RATIO — This is a formula used to determine what portion of a periodic annuity payment is taxable.

EXCLUSIONS — Another name for exceptions.

EXECUTOR — An individual named in a will and approved by a Probate Court to carry out the provisions of the will. If the

named individual (or contingents) can not or will not accept the appointment, then the Probate Court will name an Administrator.

EXPERIENCE RATING IN GROUP INSURANCE — Premium is computed on the basis of past losses and expenses incurred

by the insurer in the settlement of claims and other expenses involving a particular group.

EXPRESS AUTHORITY — The authority granted to a producer by an insurer as identified in the agency agreement to which

both parties agreed.

EXTENDED TERM INSURANCE — A non-forfeiture option available when a policy is surrendered in which the same face

amount of the policy is continued in force for a specified additional period of time. However, the coverage has changed from

permanent to level term protection. This is the nonforfeiture option that provides the policy owner with the greatest face amount

of coverage.

FACE AMOUNT — Another name for the death benefit of a policy.

FACILITY OF PAYMENT CLAUSE — A provision in a policy that permits the insurer to pay insurance proceeds to persons

other than the insured, the designated beneficiary or the estate of the insured in order to pay burial expenses.

FAIR CREDIT REPORTING ACT — A federal law passed in 1970 that provides an insurer with the right to receive additional

information with regard to applicants for insurance coverage. This law permits an insurer to conduct a consumer report on

applicants and proposed insureds. An applicant for insurance must be informed of the purpose of the report. If coverage is declined

due to information in the report, the insurer must provide the name and address of the reporting agency so that the applicant may

secure a copy of the information in the report.

FAMILY INCOME POLICY — A policy that combines a whole life policy with a decreasing term rider in order to provide a

death benefit together with monthly income payments to the beneficiary. Monthly income payments are made only from the date

of death until the maturity date of the contract. Then the lump sum part of the whole life coverage is paid.

FAMILY MAINTENANCE POLICY — This type of policy combines whole life insurance and a level term rider. It provides

for the payment of a monthly income during a stated period of years once the insured dies. The monthly income is payable from

the date of death to the end of the pre-selected period. The payment of the face amount of the policy is payable at the end of the

pre-selected period.

FAMILY POLICY — A policy covering the entire family. Whole life insurance covers the primary insured (i.e., breadwinner)

with varying amounts of level term on the rest of the family.

FIDUCIARY RESPONSIBILITY — Producers possess this responsibility since they handle the monies of the public. A

producer must account for all premiums collected or they have failed to uphold this duty. Anyone who possesses this duty must

act with a high degree of trust.

FIXED ANNUITY — An annuity contract that pays a predetermined amount of income every month to an annuitant. The insurer

bears the investment risk in a fixed annuity.

NILA ©

L/H Licensing

Glossary 19 -6

Page 284: FORWARD - WordPress.com · Return of premium p 6-8 6.0 Annuities p 7-1 Annuity principles and concepts p 7-1 Accumulation period versus annuity period p 7-1 Owner, annuitant and beneficiary

FIXED PERIOD — This is a settlement option under a life insurance policy that pays the beneficiary an amount of monthly

income for a specified period of time rather than a lump sum settlement.

FOREIGN INSURER — This is an insurer conducting business in a State other than the State in which it was organized or

chartered. This classification of insurer is licensed to conduct business in a particular State(s) but its principal office is situated in

another State. For instance, Aetna Life and Casualty of Hartford, Connecticut is a domestic carrier in the State of Connecticut. Aetna

is also authorized in the State of New York. Therefore, Aetna operates in New York as a foreign insurer.

FRATERNAL ASSOCIATION — This is an organization that markets life and health insurance product solely to its members.

FREE - LOOK PERIOD — All life insurance policies must include at least a ten day free-look period in a life insurance contract.

This period begins when the policy is delivered by the producer. If during this period the policy owner decides to return the contract

to the insurer, he or she will receive a full premium refund. A mail order or direct response insurer must include a free-look period

of at least thirty days.

GENERAL AGENT — A General Agent is given supervisory authority over the agents under his or her jurisdiction. Usually he

or she performs the following functions: selecting, writing and servicing the insurance business in force in his or her territory.

Generally, the General Agent signs contracts with agents to sell and service the insurer’s policies and contracts. Usually, a General

Agent is not an employee of the Life Insurance Company but self employed like an independent agent.

GRACE PERIOD — A specified period of time after a premium payment is due, during which the protection of the policy

continues even though the payment for the renewal premium has not as yet been received.

GUARANTY FUND — Each State has a life and health insurance guaranty fund that is designed to protect policy owners against

the insolvency of an insurer. In order for the Fund to be operated, all insurers licensed to sell life and health insurance must

participate (provide funds for the Fund).

GUIDELINE PREMIUM — The maximum premium that can be paid into Universal Life Policies and still have the benefit

qualify as life insurance under federal tax laws. If a guideline premium is paid on a regular basis, there may be little margin for any

additional premium payments into a Universal Life Insurance policy account.

HAZARD — This is a condition present that increases the chance of a possible loss. There are three primary types of hazards

including physical hazards (i.e., faulty wiring in a building), moral hazards (i.e., dishonest or criminal activities and morale hazards

(i.e., leaving keys in the ignition, carelessness).

IMMEDIATE ANNUITY — This is a type of annuity contract that pays a monthly income commencing one, three, six, or twelve

months after purchase. This type of contract must be funded with a lump sum or single premium.

IMPLIED AUTHORITY — This is authority granted to a producer by an insurer that is not specifically identified in the agency

agreement.

INCONTESTABLE CLAUSE — A provision of life insurance policies that states once the policy has been in force for two years,

during the lifetime of the insured, the insurance company may not challenge the validity of the policy. Over the years, case law has

established precedence that the Incontestable Clause applies to cases of fraud.

INCREASING TERM INSURANCE RIDER — This type of rider is characterized by an increasing amount of term life

coverage each year. There are two primary types including a return of premium rider and a return of cash value rider.

INDEXED CONTRACTS — Contracts where the policyholder can share in a percentage of the growth of an indexed investment

(a Mutual Fund tied to the Standard & Poors Index is an example). The principle or benefit is guaranteed and in many cases, a

minimum interest is guaranteed. These products are not considered securities.

INDIVIDUAL INSURANCE — A policy which affords protection to a policyholder. Sometimes it is called personal insurance.

NILA ©

L/H Licensing

Glossary 19 -7

Page 285: FORWARD - WordPress.com · Return of premium p 6-8 6.0 Annuities p 7-1 Annuity principles and concepts p 7-1 Accumulation period versus annuity period p 7-1 Owner, annuitant and beneficiary

INDIVIDUAL RETIREMENT ACCOUNT — A retirement type plan that allows individuals to receive tax deferral of interest

while providing some with tax deductibility of contributions. An age based penalty may apply to those who receive premature

distributions prior to age 59 ½. In addition, the amount withdrawn will must be reported as ordinary income in the year received.

INSPECTION REPORT — A report that contains general information regarding the health, habits, finances and reputation of

an applicant. This report is developed by a firm that specializes in rendering this type of service.

INSTALLMENT REFUND ANNUITY — The same as a cash refund annuity, except that money is refunded in installment

payments and the insurer makes payments to the designated beneficiary until the total of the payments made to the annuitant and

the beneficiary equals the consideration paid.

INSURABLE INTEREST — The relationship that exists between parties that justify one owning life insurance on the other.

People are said to have an unlimited insurable interest in their own life. An insurable interest may exist in the life of another person

if there is a chance of a financial or emotional loss at that person’s death. The insurable interest could be between two or more

individuals or an entity one or more individuals. The insurable interest must exist at the time of policy issue.

INSURANCE — The transfer of risk from one party to another through a legal contract.

INSURED — The individual covered by a life insurance policy.

INSURING CLAUSE — The policy provision that describes the scope and limits of the coverage afforded. It also identifies the

parties to the contract and the annual premium.

INVESTMENT PERFORMANCE — The earnings in variable products are based upon how the market performs since funds

are invested in securities. When investment performance is good, the contract owner experiences an increase in funds. When

performance is poor they experience a decrease.

IRREVOCABLE BENEFICIARY — The policy owner may not change the designated beneficiary without the beneficiary’s

consent. An irrevocable beneficiary is allowed to receive a copy of the policy to understand what ownership right are present.

JOINT AND LAST SURVIVOR ANNUITY — An annuity issued on the lives of two or more persons that is payable as long

as the survivor lives.

JOINT LIFE INSURANCE — A life insurance policy covering the lives of two or more persons that pays a death benefit and

ends when the first insured dies. This is also referred to as “first to die” insurance.

JUVENILE INSURANCE — Life insurance policies owned by an adult and written on the lives of children.

KEY EMPLOYEE INSURANCE — An individual policy designed to reimburse an employer for the loss of a key employee's

service and contributions due to death. The employer generally pays the premium and is the owner of the contract.

LAPSE — This term means that the policyholder failed to pay the premium within the grace period. Therefore, no coverage exists

once the policy lapses.

LARCENY — An unfair trade practice involving the commingling or misappropriation of premiums by a producer.

LAW OF LARGE NUMBERS — This is a mathematical rule that states that the more the number of events present, the more

accurately can we determine the possible outcomes. The law of large numbers allows an insurer to more accurately predict future

losses.

LEGAL CAPACITY — One of the four required elements that must be part of a legal contract. This element states that the

parties to the contract must be legally competent to enter into the contract. Most individual possess legal capacity except for minors

(i.e., those contracting for food, clothing and other necessities ARE competent), those under the influence or those who are insane.

Insurers are competent except for those experiencing financial difficulties such as insolvency or impairment.

NILA ©

L/H Licensing

Glossary 19 -8

Page 286: FORWARD - WordPress.com · Return of premium p 6-8 6.0 Annuities p 7-1 Annuity principles and concepts p 7-1 Accumulation period versus annuity period p 7-1 Owner, annuitant and beneficiary

LEVEL PREMIUM — A premium that remains constant, fixed or predetermined throughout the life of a policy.

LIFE EXPECTANCY — The average number of anticipated years of life remaining for individuals who are the same age in

accordance with the mortality table indicated in the policy.

LIFE INSURANCE — Insurance upon the lives of human beings that creates an immediate and guaranteed estate upon the death

of an insured or at the end of a predetermined period (in whole life insurance this is age 100).

LIMITED PAYMENT LIFE INSURANCE — A plan of life insurance under which the premiums are payable for a specified

number of years after which the policy remains in effect for life without any additional payments. However, the policy still does

not mature until age 100.

LIVING BENEFITS RIDER — Attached to a life insurance policy and allows for the payment of a percentage of the death

benefit for terminally ill insureds. Normally there is no cost for this rider.

LOAN VALUE — The amount specified in a whole life policy that the insurer will lend to a policy owner from the cash value

at the rate of interest the insurer charges as identified in the contract.

LONG-TERM CARE RIDER — Attached to a life insurance policy and allows for the payment of a percentage of the death

benefit if an individual is not terminally ill but requires long-term care.

MARKET VALUE ADJUSTED ANNUITY — This type of annuity shifts some but not all of the investment risk from the

insurer to the policy owner since the annuity account value will fluctuate as market interest rates fluctuate. It is a single premium

deferred annuity that allows the contract owner to lock in a guaranteed interest rate over a specified maturity period. These annuity

contracts pass along more risk to the policy owner.

MATURITY DATE — The date on which a policy becomes payable due to the death of the insured or as a result of an insured's

living to the end of a specified period (i.e. age 100). In whole life insurance, the cash value is designed to equal the face amount

at maturity.

MATURITY VALUE — The amount paid under a whole life insurance contract if the insured reaches the age of the mortality

table on which the contract was based. If an endowment contract, it is the cash value amount at the end of the endowment period.

MINIMUM PREMIUM — Also known as the Minimum Continuation Premium, is the premium lower than the target premium.

Used in Universal Life Policies to skip premiums or pay lower premiums. Two constraints are placed on the minimum premium

amount: that it is above the insurance company’s cost to process the payment and that there is enough cash value in the policy

account to pay mortality and expense charges.

MISREPRESENTATION — A false statement that an applicant makes on an application for an insurance policy. An omission

(i.e., concealment) of a material fact can also be construed as a misrepresentation. A misrepresentation is material if the insurer,

having known the true facts, would have declined the application as applied for by the applicant. Statements made by an applicant

or the owner are considered representations and not warranties.

MISSTATEMENT OF AGE — This is a provision in a life insurance policy that states that if the age of the insured is

understated on an application and the insurer later learns of the error, it will adjust the death benefit lower. The premium will remain

the same. If the age has been overstated the insurer will adjust the benefit higher. If the insurer discovers the error at the time of

death it will adjust the proceeds payable.

MODIFIED ENDOWMENT CONTRACT — This is a whole life policy that fails to satisfy a seven pay test. When a policy

owner over funds the contract attempting to use the policy as a short term investment vehicle, the policy will be designated for tax

purposes as a MEC. This means that any cash distributions from the contract will be subject to taxation. In addition, if the

withdrawal is made prior to age 59 ½ , the policy owner will be subject to an age based penalty of 10 %.

NILA ©

L/H Licensing

Glossary 19 -9

Page 287: FORWARD - WordPress.com · Return of premium p 6-8 6.0 Annuities p 7-1 Annuity principles and concepts p 7-1 Accumulation period versus annuity period p 7-1 Owner, annuitant and beneficiary

MODIFIED LIFE POLICY — This is a whole life plan that is characterized by a lower premium during the initial years of the

contract to make it more affordable for the policy owner. The premium then increases after this introductory period and remains

level for the life of the contract.

MORAL HAZARDS — Habits, morals or financial practices of an insured that increase the possibility or extent of a loss.

MORTALITY — The measure of the probability of surviving or dying at each age. Mortality rates usually reflect the actual

experience of an insurance company with its insureds, adjusted for expected future changes. The rates may differ from company

to company depending on the risks each company accepts.

MORTALITY TABLE — A statistical table that indicates the probability of death and survival at each age up to age 100. This

table informs us of the “death rate” at any particular age from 0 through 100.

MORTGAGE REDEMPTION PLAN — This is another name for a decreasing term life insurance policy. This type of plan

is used to provide funds to a survivor in order to pay off a debt. Also called mortgage protection coverage or reducing term

insurance.

MUTUAL INSURANCE COMPANY — A life insurance company owned and controlled by its policy holders. Mutual

insurance companies issue participating policies which may pay dividends to policy holders.

NATURAL PERSON — This is a human being. When a corporation owns an annuity it must list a natural person as the annuitant.

If it lists a non-living entity, it will lose the benefit of tax deferral.

NET AMOUNT AT RISK — The difference between the face amount of a policy and the reserve.

NON - CONTRIBUTORY PLAN — A group employee benefit plan under which the employer pays for the full cost of the

benefits for his employees. This is an “employer pay all” plan.

NON - FORFEITURE FACTOR — An actuarial figure, indicated in the policy's table of loan and surrender value that is used

to calculate the non-forfeiture values of the policy.

NON - FORFEITURE VALUES — Benefits required by law to be made available to the policy owner in the event that he

surrenders the policy by discontinuing premium payments. These values state that the owner will not forfeit or lose all that he has

invested in the policy. Also referred to as nonforfeiture options, they include surrender for cash, extended term insurance and

reduced paid-up insurance.

NON-MEDICAL LIFE INSURANCE — Insurance that is issued without requiring the applicant to submit to a medical

examination. The insurer relies on the applicant's answers to the questions regarding his physical condition, personal references

and inspection reports. However, the insurance company retains the right to require a medical examination if an investigation

indicates a need for one.

NON - PARTICIPATING INSURANCE — A type of life insurance policy issued by a stock insurer. It is an insurance contract

that does not pay dividends to the policyholders.

NON-QUALIFIED ANNUITY — This is a classification of annuity whose description is based upon the fact that interest or

earnings paid into the contract are tax deferred and the contributions to the contract are not tax deductible.

NON-QUALIFIED DISTRIBUTION — This is the same as a premature distribution.

NON-QUALIFIED PLAN — This is a type of retirement plan that permits tax deferral of interest but contributions are not tax

deductible. The reason for the latter is due to the fact that the employer sponsoring the plan may legally discriminate and provide

the plan for specific employees only such as highly paid executives.

NILA ©

L/H Licensing

Glossary 19 -10

Page 288: FORWARD - WordPress.com · Return of premium p 6-8 6.0 Annuities p 7-1 Annuity principles and concepts p 7-1 Accumulation period versus annuity period p 7-1 Owner, annuitant and beneficiary

ORDINARY INCOME — When gains in a life insurance policy are realized upon surrender, they are taxed as ordinary income

and not capital gains. Whenever interest or earnings are realized in any policy, they taxed as ordinary income.

ORDINARY LIFE INSURANCE — Insurance policies of $1,000 or multiples thereof that provide coverage for the entire life

of the policyholder and for which the premiums are payable until death. It is also referred to as whole life insurance, straight life

insurance, or continuous premium life, and is different from term insurance in that it includes a cash value build-up.

OWNERSHIP PROVISION — This is a provision in a life insurance policy that identifies the owner rights in the contract. The

policy owner possesses the right to name or change the beneficiary, select the premium payment mode, or assign one or more rights

to another party. In a whole life policy the owner also has the right to borrow against the cash value and receive dividends.

PAID-UP ADDITIONS — An additional amount of insurance purchased through dividends (single premium insurance) that

increases the amount of protection provided.

PAID-UP INSURANCE — Life insurance on which future premium payments are not required. Frequently, the term is used to

identify a ten, twenty or thirty payment life insurance policy on which 10, 20 or 30 annual premium payments have been paid. Even

though the policy is “paid-up” at the end of the payment period, the contract does not mature until age 100.

PARTICIPATING POLICY — A life insurance policy that entitles the policy holder to share in the divisible surplus of the

insurer through dividends.

PAYOR RIDER — A rider that provides for the waiver of premiums on a policy covering a child following the death or the total

disability of the adult owner (i.e. premium payor).

PENSION TRUST — A plan under which an employer provides retirement benefits for his employees with favorable income-tax

treatment for the employer and employee. Generally, policies are issued individually as special pension plan life insurance policies

and/or annuity contracts.

PER CAPITA — A type of beneficiary designation where proceeds of the policy are to be paid “per head” or “per person”.

Assume that an insured has three primary beneficiaries who are to receive per capita distributions of proceeds upon the death of the

insured. One of these individuals predeceases the insured. Once the insured dies, the proceeds with be divided equally “per person”

between the remaining beneficiaries.

PER STIRPES — This type of beneficiary designation means that proceeds of a life insurance policy will be paid according to

a family line, branch or root. Per stirpes actually means “by the root”. Assume that an insured has three primary beneficiaries who

are to receive per stirpes distributions of proceeds upon the death of the insured. One of these individuals predeceases the insured.

Three equal shares will be paid, one of which goes to the family line or root of the deceased beneficiary (i.e., to the heirs of the

deceased beneficiary).

PERMANENT DISABILITY — A disability that is expected to continue for the lifetime of the insured. Usually, the disability

is presumed to be permanent after it has continued for a specified period (e.g. one year).

PERMANENT LIFE INSURANCE — Any plan of life insurance that is designed to last throughout the life of the insured.

Level premium, cash value, and non-forfeiture options characterize permanent life insurance.

PHYSICAL HAZARD — That type of hazard that arises from the physical characteristics of an individual. It may exist because

of a physical condition present at birth, past medical history or a current condition.

POLICY — The printed document or contract issued by the insurer to the policy owner providing insurance coverage on the life

of an insured.

POLICY FEE — A small transaction fee charged by some insurers for the first or subsequent years of the life of an insurance

policy, in addition to the regular premium.

NILA ©

L/H Licensing

Glossary 19 -11

Page 289: FORWARD - WordPress.com · Return of premium p 6-8 6.0 Annuities p 7-1 Annuity principles and concepts p 7-1 Accumulation period versus annuity period p 7-1 Owner, annuitant and beneficiary

POLICY LOAN — A loan made by an insurer to a policy owner under his or her policy permanent insurance policy. A policy

loan is not taxable. However, if the whole life plan has been designated as a modified endowment contract, such a loan will be

subject to taxation.

POLICY PROCEEDS — The amount actually paid as a death, surrender, or maturity benefit. In the case of death, it includes

the face value plus any earned dividends less any outstanding loans and interest. If surrender benefit, the amount includes any cash

value less surrender charges and outstanding loans and interest. If maturity benefit the amount includes the cash value less any

outstanding loans and interest.

POLICY TERM — The period of time for which a policy remains in existence

PREDETERMINED PREMIUM — This is the type of premium paid for whole life insurance coverage. Also referred to a

fixed, level or constant premium, this is predetermined for life or until death or policy surrender.

PREMATURE DISTRIBUTION — Whenever funds are withdrawn from a qualified plan, annuity, IRA or a modified

endowment contract prior to age 59 ½, the withdrawal is considered a premature distribution. Penalties will be assessed and amounts

withdrawn will be subject to taxation with the exception of death, disability and other qualified events.

PREMIUM — The initial payment and subsequent periodic payments required to keep a policy in force.

PRIMARY BENEFICIARY — The first person entitled to policy proceeds upon the death of the insured. Proceeds are paid

income tax free.

PRINCIPAL — One of the parties in an agency relationship. The other is the agent who acts on behalf of the principal. This

relationship may be applied to the insurance business where a producer (i.e., agent) represents the insurer (i.e., the principal).

PROCEEDS — The net amount of money that is payable by the insurer upon the death of an insured or when the policy matures.

This is also called the policy’s face amount, coverage amount, coverage limit or death benefit.

PRODUCER — This is a licensed insurance agent. A producer may also be referred to as an agent or field underwriter.

PUBLIC ADJUSTER — An individual who determines or settles losses to commercial or personal property on behalf of a client

/ insured.

QUALIFIED ANNUITY — A qualified annuity is characterized by tax deferral of interest and contributions that are tax

deductible.

QUALIFIED DISTRIBUTION — This is a distribution of funds from a qualified plan, MEC or annuity that is not subject to

a penalty or taxation.

QUALIFIED PLAN — This is a retirement type vehicle that receives favorable tax treatment. Contributions to the plan are tax

deductible while earnings credited are tax deferred. In order to be classified as a qualified plan according to IRS guidelines, the

plan must be in writing, must be communicated to all employees, cannot discriminate against any employees and must provide

survivor benefits.

QUALIFIED VERSUS NON-QUALIFIED — Deals with the income taxation of contributions to various plans. Qualified plans

cannot discriminate, must comply with specific IRS codes and regulations, and contributions are deductible from income taxes in

the year they are made. Non-qualified plans can discriminate, do not need to be filed with the IRS and contributions are not tax

deductible. Some plans will only be Qualified (401(k) and Medical Savings Accounts), some only Non-Qualified (deferred

compensation), and some can be either depending on how they are set up (Individual Retirement Accounts and annuities).

RATED — A method under which an insurer may issue a policy for a substandard risk by increasing the premium based on the

increased risk involved.

NILA ©

L/H Licensing

Glossary 19 -12

Page 290: FORWARD - WordPress.com · Return of premium p 6-8 6.0 Annuities p 7-1 Annuity principles and concepts p 7-1 Accumulation period versus annuity period p 7-1 Owner, annuitant and beneficiary

READJUSTMENT INCOME — In life insurance needs, the amount of income needed after the death or disability of a wage

earner to allow the family unit to adjust to the new standard of living (may be lower or higher).

READJUSTMENT PERIOD — In life insurance needs the period following the death or disability of a wage earner, which the

family needs to adjust to the new standard of living. Generally, this will be less than one year, however it can extend for several

years in the case of disability and/or emotional acceptance by family members.

REBATING — Paying, offering or giving anything of value not specified in the policy to any person as an inducement to apply

for or renew an insurance policy. Rebating is illegal in most States.

REDUCED PAID-UP INSURANCE — A non-forfeiture option in a policy used when the contract is surrendered that provides

for the continuation of the insurance at a reduced face amount.

REINSTATEMENT — The resumption of coverage under a policy that has lapsed. Before an insurer allows this, the insured must

prove insurability and all back premiums must be paid. In addition, all statements on the reinstatement application are contestable

for another two years.

REINSURANCE — The underwriting by one insurance company (called the reinsurer) of part or all of an individual risk written

by another insurer (called the ceding company). In other words, this is the transfer of risk from one insurer to another. Facultative

reinsurance occurs when the ceding company presents a risk to the reinsurer and the latter decides whether to accept or declined

the risk on a case by case basis. Automatic or treaty reinsurance involves an arrangement already in place where a reinsurer will

automatically assume its portion of the risk if it has been accepted by the ceding insurer.

RENEWABLE TERM LIFE INSURANCE — Insurance that may be renewed at the end of the policy term without evidence

of insurability. The premium increases at the end of each term and is based on the attained age of the insured at the time of renewal.

RENEWAL — The continuance of coverage under a policy beyond the initial period.

REPLACEMENT — This is a legal activity where a producer convinces a prospective client to lapse or surrender a life or health

policy and purchase a new one. Producers must provide a “Notice Regarding Replacement” to the consumer when this activity may

occur. The producer must also notify the insurer that a replacement is occurring as well.

RESERVE — The combined funds required by law that are set aside by an insurer to assure the payment of future claims.

REVOCABLE BENEFICIARY — The designated beneficiary may be changed at the owner’s request without the consent of

the beneficiary.

RIDER — This is an additional attachment to a policy. Riders may provide an additional benefit (i.e., benefit rider) or restrict

coverage (i.e., impairment rider). Benefit riders generally require an additional premium such as the waiver of premium rider,

accidental death benefit rider, payor rider, guaranteed insurability rider or the accelerated benefits rider.

RISK AVOIDANCE — A type of risk management where an exposure is eliminated. The exposure to loss produced by a specific

activity may be eliminated by avoiding the activity. For example, if an individual does not wish to lose money in the stock market,

he or she should not invest.

RISK FINANCING — This is a general description of techniques or methods used to provide funds to pay for losses due to

unexpected and fortuitous events.

RISK REDUCTION — This is a method of risk management that attempts to reduce the severity of a possible loss but does not

eliminate the possibility that loss.

RISK SHARING — This is a method of risk management, such as a partnership, where parties agree to share in uncertainty to

minimize possible losses.

NILA ©

L/H Licensing

Glossary 19 -13

Page 291: FORWARD - WordPress.com · Return of premium p 6-8 6.0 Annuities p 7-1 Annuity principles and concepts p 7-1 Accumulation period versus annuity period p 7-1 Owner, annuitant and beneficiary

RISK TRANSFER — This occurs when one party transfers risk to another. This may be accomplished by the purchase of an

insurance policy, through incorporation or by entering into a hold-harmless agreement.

ROLLOVER — This is the transfer of funds from one type of retirement or qualified plan to another. When the transfer of funds

is effected from plan to plan, the owner does not receive any funds. If the funds are paid to the individual, the rollover must be

accomplished within 60 days or the individual is subject to a penalty.

ROTH IRA — This is a fairly new IRA which allows an individual to make non-deductible contributions which are tax free when

withdrawn. To be eligible to contribute an individual or family must meet income restrictions. Contributions may continue after

age 70 ½ and penalty free withdrawals may be made prior to age 59 ½ for qualified education expenses.

SETTLEMENT OPTIONS — Methods by which a beneficiary may choose to receive life insurance policy proceeds.

SINGLE PREMIUM — The payment of one premium that is large enough to cover the cost of a life or annuity contract for life.

Also known as a lump sum premium.

SPECULATIVE RISK — This is a type of risk that presents the chance for gain or loss. It is not legal to insurer such risks (i.e.,

betting on a race).

SPENDTHRIFT PROVISION — A provision in a life insurance policy that states that when the proceeds of the policy are left

with the insurer, no entity may have access to or attach a lien to the funds. This provision protect a beneficiary from creditors when

proceeds are left with the insurer.

STANDARD PROVISIONS — Certain provisions that must be included in a life insurance policy such as the incontestable

clause, grace period, assignment, reinstatement and so forth.

STOCK LIFE INSURANCE COMPANY — A life insurance company owned and controlled by its stockholders who share

in its divisible surplus. Generally, stock insurance companies issue non-participating life insurance. However, some of them also

issue participating life insurance.

STRAIGHT LIFE ANNUITY — This is also referred to as a pure life annuity or life annuity. It provides income to an annuitant

for life with no refund to a beneficiary. A straight life annuity may be purchased with a “period certain.” This is a guaranteed period

of time for which benefits will be paid.

SUBSTANDARD RISK — An applicant whose physical condition does not meet the normal minimum standards. If the

substandard classification is due to adverse health, the application may be declined or written with a “rated-up” premium. An

applicant may be in excellent health but considered substandard due to his or her personal activities, hobbies or avocations (i.e.,

scuba diving, sky-diving, etc.).

SUICIDE CLAUSE — A provision specifying that in the event the insured commits suicide within two years from the date the

policy was issued, the insurer’s liability is limited to the payment of a single sum equal to the premiums actually paid.

SUPERANNUATION — This means outliving one’s income. The purpose of a straight life annuity is to protect an individual from

outliving his or her income since this contract pays a monthly income benefit until the annuitant dies.

SURPLUS — The amount by which the assets of an insurer exceed its liabilities.

SYSTEMATIC PREMIUM PLAN — A plan under which the policy owner authorizes a bank to deduct the necessary funds

from his or her account each month to pay a premium that is forwarded to the insurer by the lender.

TARGET PREMIUM — A suggested premium used in Universal Life policies. It does not guarantee there will be adequate funds

to maintain the policy to any time, especially to life. It may give an indication of what will be needed (under conservative estimates),

to maintain the policy. The validity of a target premium is based on an individual insurance company’s marketing stance, investment

performance, and cost control.

NILA ©

L/H Licensing

Glossary 19 -14

Page 292: FORWARD - WordPress.com · Return of premium p 6-8 6.0 Annuities p 7-1 Annuity principles and concepts p 7-1 Accumulation period versus annuity period p 7-1 Owner, annuitant and beneficiary

TAX DEFERRAL — This is a tax benefit one receives when interest is not taxable in the year credited. In other words, this

involves postponing taxes to a future tax year because of an anticipated lower tax bracket, or for current use of funds. Tax deferral

is legal while tax evasion is illegal.

TAX EXEMPT — This is the same as tax free or non-taxable. For instance, dividends paid on a life insurance policy are tax free

or tax exempt.

TAX SHELTERED ANNUITY — This is a type of qualified plan that receives tax deferral through a salary reduction. Those

eligible for this type of plan include, educational employees, clergymen and women, employees of a non-profit organization and

employees of charitable organizations. This may also be referred to as a tax deferred annuity.

TEMPORARY LICENSE — An insurance license that may be issued to a survivor of a deceased producer for a specified period

of time. This license allows the income stream of renewals to continue until the holder disposes of the business or decides to secure

a producer license. Generally these licenses are issued for a period of 180 days. They usually do not permit the holder to write new

coverage but to only receive renewal commissions based upon business already in existence.

TERM LIFE INSURANCE — Temporary life insurance that is generally designed to afford coverage for a limited number of

years. The policy includes no cash value and can be described as pure protection.

TESTIMONIALS — These are statements made by an individual or organization promoting the insurance products marketed by

an insurer. Such statements must be true and represent the current opinion of the person making them. These statements also

promote the insurer as well.

TOTAL DISABILITY — An illness or injury that prevents an insured from continuously performing the duties pertaining to his

or her occupation or from engaging in any other type of work for remuneration.

TRUST — A legal document, instrument, or agreement where ownership of property is transferred and management of property

is given to a named trustee for the intent expressed in the trust agreement.

TWISTING — An illegal sales practice involving a producer illegally or unfairly inducing to lapse his or her present insurance

policy and purchase another. This is not the same as replacement. Replacement is a legal activity.

UNFAIR TRADE OR SALES PRACTICES — All States have regulations prohibiting these activities. Also called unfair

marketing practices, such activities are illegal and subject the individual engaging in these activities to monetary fines and / or

imprisonment. These unfair acts include but are not limited to: rebating, twisting, misrepresentation, defamation, coercion and

boycott, larceny and perjury.

UNIFORM SIMULTANEOUS DEATH ACT — A state law which mandates that if the insured and the beneficiary die in

the same accident, and it cannot be determined which died first, it will be assumed that the beneficiary dies first, and the death

proceeds will then pass to the contingent beneficiary.

UNILATERAL — Insurance policies are unilateral in nature since they are one sided. This means that the insurer draws up the

language of the contract and the policy owner does not.

UNDERWRITING — The analysis of information pertaining to an applicant that was obtained from various sources and the

determination of whether or not the insurance should be issued as requested, offered at a higher premium or declined.

VARIABLE ANNUITY — An annuity that invests funds in the stock market. Once income commences, the monthly amount

will vary based on investment performance. In a variable annuity the contract owner bears the investment risk.

VESTING PROVISION — This is a pension plan participant’s right to receive benefits from employer contributions to a plan

even if the participant is no longer an employee of that employer. All employee contributions made to a plan are fully vested at the

time they are made. Employer contributions are vested according to a schedule contained in the plan.

NILA ©

L/H Licensing

Glossary 19 -15

Page 293: FORWARD - WordPress.com · Return of premium p 6-8 6.0 Annuities p 7-1 Annuity principles and concepts p 7-1 Accumulation period versus annuity period p 7-1 Owner, annuitant and beneficiary

VIATICAL SETTLEMENT — This is a transaction where the owner of a life insurance policy decides to sell the contract to a

viatical settlement company (i.e., a viatical settlement provider). This company buys the contract for a percentage of the face amount

from the policy owner. The provider or company then changes the name of the beneficiary to itself since it is the new owner of the

contract. When the insured dies, the provider receives the death benefit. The amount paid to the policy owner is a percentage of

the face amount in the contract (e.g., 85 % of the face amount). The owner is allowed to accelerate a portion of the death benefit

while he is still alive when they have been diagnosed with a terminal illness. A viatical settlement is one of three ways in which to

accelerate benefits. The other two include an accelerated benefit rider and a long-term care rider added to a life insurance policy.

VIATOR — This is another name for the policy owner of a life insurance contract in a viatical settlement.

WAIVER — To voluntarily relinquish or abandon a known right under an insurance contract. This is a legal principle that protects

the consumer if an insurer waives its rights under an insurance contract. A common type of waiver occurs when an insurer fails to

enforce a policy provision.

WAIVER OF PREMIUM — A benefit rider available to be included in a life insurance policy that waives the payment of

premiums after the policy owner has been totally disabled for a specific period of time (i.e., six months). Once the policy owner

has been disabled for six consecutive months, any premium paid during that time will be refunded by the insurer retroactively.

Future premiums will be waived on a month to month basis until the disabled party returns to work. Proof of disability from a

medical professional is required.

WAR CLAUSE — A clause in the exclusive provision of a life insurance policy that limits an insurer’s liability if death is caused

as a result of war.

WARRANTIES AND REPRESENTATIONS — Most State laws specify that all statements by the applicant on the application

are considered to be representations and not warranties. A warranty must be absolutely and literally true. A breach of warranty may

be sufficient to void the policy whether or not the warranty is material and whether or not such breach of warranty had contributed

to the loss. A representation need only be substantially true to the best of the applicant’s knowledge. Generally, a representation

is considered to be fraudulent if it relates to a situation that would be material to the risk and that the applicant made with fraudulent

intent.

NILA ©

L/H Licensing

Glossary 19 -16

Page 294: FORWARD - WordPress.com · Return of premium p 6-8 6.0 Annuities p 7-1 Annuity principles and concepts p 7-1 Accumulation period versus annuity period p 7-1 Owner, annuitant and beneficiary

GLOSSARY OFHEALTH INSURANCE TERMS

ACCIDENTAL BODILY INJURY — Bodily injury resulting from an accidental causes. Most health policies state that as long

as the result of an injury is unforseen and unexpected (i.e., an accident), the policy will pay the covered benefit.

ACCIDENTAL MEANS — Some health insurance policies have included this definition of covered accident in the past. This

means that to be covered, the cause and result of an injury must be unforeseen, unexpected and unintended. In other words, the cause

of the action and the result must not be intentional or there will be no coverage. For example, if a person jumps off a ladder and

breaks his ankle, there would be no coverage since the act of jumping (i.e., the cause) was intentional (or not unforseen).

ACTIVITIES OF DAILY LIVING — Long-term care policies include coverage for an insured who is unable to care for

himself or herself and cannot perform these activities including bathing, eating, dressing, continence, using the toilet and transferring

from bed to chair. Coverage under a long-term care plan can be triggered when an insured is unable to perform two or more of these

activities.

ADHESION — Insurance policies are contracts of adhesion since they are drafted by the insurer and offered to the insured without

the possibility of a material modification. If it is later determined that the language in the contract is ambiguous, courts usually rule

in favor of the insured.

ADMISSION CERTIFICATION — A utilization review procedure for assessing an individual’s physical and psychological

condition and whether that condition requires admission to a hospital or other inpatient facility.

ADULT DAY CARE — Daytime services provided which will allow an elderly adult to function in his or her own home rather

than an inpatient facility. This type of care may be covered in a long-term care policy.

ADVERSE SELECTION — Also referred to as anti-selection, this is the tendency of more bad risks than good risks to purchase

insurance coverage from an insurer. Poorer risks or less desirable insureds tend to seek or continue insurance to a greater extent

than better risks.

AGE LIMITS — This involves minimum or maximum age limits for insuring of new applicants or for renewing policies.

ALEATORY — A legal description for an insurance contract where the consideration and the subject of the agreement (i.e.

premium versus benefit) involve unequal values.

ALLOCATED BENEFITS — A provision that pays specific expenses according to rates set in a schedule in the policy.

AMBULATORY CARE FACILITY — A health care facility that is contained within or in close proximity to a hospital; and

provides diagnostic or medical services including minor surgery on an outpatient basis.

ANCILLARY EXPENSES — These are miscellaneous medical expenses that are not specifically listed but will be paid by a

medical plan.

BENEFIT PERIOD — The length of time during which a Medicare beneficiary is eligible for Part A benefits.

NOTE

Please review this glossary of health insurance terms in order to further prepare yourself for your state licensing exam.

NILA ©

L/H Licensing

Glossary 19 -17

Page 295: FORWARD - WordPress.com · Return of premium p 6-8 6.0 Annuities p 7-1 Annuity principles and concepts p 7-1 Accumulation period versus annuity period p 7-1 Owner, annuitant and beneficiary

BLACKOUT PERIOD — The time during which a surviving spouse stops receiving Social Security survivors benefits (when

the youngest child is no longer eligible for benefits) and begins receiving Social Security retirement benefits.

BLANKET POLICY — Covers a number of individuals who are exposed to the same hazards, such as members of an athletic

team.

BLUE CROSS — An independent, nonprofit (or in some States “for profit”) membership association providing protection against

the costs of hospital care in a limited geographical area. Provides coverage for hospital costs.

BLUE SHIELD — An independent, nonprofit (or in some States “for profit”) membership association providing protection against

the costs of surgery and other items of medical care in a limited geographical area. Provides coverage for physician’s fees.

BUSINESS OVERHEAD EXPENSE INSURANCE — Insurance that provides coverage for rent, utilities, the cost of labor

(i.e. employee salary) and other business overhead expenses when the primary business owner becomes disabled. Business overhead

expense plans do not cover the loss of income of the business owner nor any business profits.

CAFETERIA PLANS — A benefit plan under which employees of a firm may select the benefits programs that best fits their

needs. If the employee’s allocation is not used up, the employee may receive the difference in cash.

CANCELABLE POLICY — A policy that may be terminated either by the policy owner or the insurer by notification to the

other party in accordance with the terms of the policy.

CAPITAL SUM — This is the amount payable under accidental dismemberment coverage. It may be the amount payable for

accidental loss of two members or both eyes (or one of each). An indemnity for the loss of one member or the sight of one eye is

usually a percentage of the capital sum. In many policies the capital sum is 50% of the principal sum.

CARRY-OVER PROVISION — In health insurance policies, a provision allowing an insured who had no claims during a policy

year to apply any medical expenses incurred during the part of that policy year (normally last three months) towards the new policy

year’s deductible. Sometimes called a deductible carryover credit.

CATASTROPHE INSURANCE — Health policies that provide substantial benefits for serious, prolonged or expensive

disabilities that cause enormous financial problems for the insured. Usually, this is called Major Medical Insurance.

CLASSIFICATION — The underwriting occupational category into which a risk is placed depending upon his or her susceptibility

to injury or illness.

CLASSIFIED INSURANCE — A term applying to the writing of life or health insurance on substandard risks.

CLOSED PANEL PLAN — A requirement in many HMO plans that a subscriber must receive medical treatment only from

physicians associated with or approved by the HMO.

COBRA — The Consolidated Omnibus Budget Reconciliation Act of 1985 provided for the continuation of group health coverage

when an employee is terminated, laid-off or dies.

COINSURANCE (Percentage Participation) — A provision that specifies that the insurance company will pay only part of a loss

and that requires the policy holder to pay the balance. For example, in the case of Major Medical Insurance, the insurer may be

obliged to pay 80% of an insured's expenses in excess of the deductible amount, if any, and the insured is required to pay the other

20%.

COMMON CARRIER — An individual or concern engaged in the transportation of goods or persons in return for a fee.

COMPOSITE RATE — In group insurance, all members of the group have the same rate, regardless of their status as a single

or members of a family.

NILA ©

L/H Licensing

Glossary 19 -18

Page 296: FORWARD - WordPress.com · Return of premium p 6-8 6.0 Annuities p 7-1 Annuity principles and concepts p 7-1 Accumulation period versus annuity period p 7-1 Owner, annuitant and beneficiary

COMPREHENSIVE MAJOR MEDICAL INSURANCE — A policy designed to provide protection offered by both a basic

and a major medical policy. It is characterized by a low deductible amount, coinsurance clause and high maximum benefits.

COMPULSORY HEALTH INSURANCE — A plan of insurance under the supervision of a State or the Federal Government

that requires protection for medical, hospital, surgical and disability benefits. Statutory Disability Benefit Laws are in effect in

several States including but not limited to: New York, New Jersey, Rhode Island, California, and in Puerto Rico and Hawaii.

CONSUMER REPORT — Insurers are allowed to conduct a consumer report to obtain additional information as long there is

no invasion of privacy present. A common type of consumer report involves a credit report.

CONVERSION PRIVILEGE — The right granted the owner / insured to change his coverage from a group policy to individual

policy. If a member of a group resigns, he or she is provided an opportunity to secure an individual policy within a specified period

thereafter, regardless of whether or not he is in good health at that time (i.e. without insurability).

COORDINATION OF BENEFITS — A provision found in group health policies specifying how benefits will be paid when

other health insurers cover an insured.

COPAYMENT PROVISION — The insured must pay a small charge (deductible) each time they receive certain covered

benefits. Examples are $20 per doctor visit or $15 per prescription. Copayments normally count toward the policy deductible that

must be met before any coinsurance provision takes effect. Most plans have higher copayments for services received out-of-network.

COST CONTAINMENT — Also known as cost sharing or managed care, this involves any medical expense plan that attempts

to contain costs by controlling the behavior of participants. Case management plans such as utilization review is a common cost

containment vehicle.

COST OF LIVING RIDER — Optional coverage that may be purchased for disability income and long-term care policies.

The rider increases the benefit over time, based on some inflation rate (normally the Consumer Price Index) or a fixed annual

percentage.

COST SHARING — Where insureds pay part of health insurance costs through deductibles, copay, and coinsurance.

CREDIT DISABILITY — This is a type of disability insurance sold by a lender that will cover the payment of a debt or other

type of installment loan if the insured (i.e., borrower) becomes disabled.

CUSTODIAL CARE — One of the four levels of care in a long-term care insurance policy. This type of care is the only one of

the four that does not require medical training.

DECLINATION — An insurer’s rejection of an application for insurance.

DECLINED RISK — This is an individual whose application for coverage was rejected by an insurance company.

DEDUCTIBLE — If an initial deductible, the amount of loss that must be absorbed by an insured before policy benefits become

payable. The insurer pays benefits only for the loss in excess of the amount of the deductible. If a corridor deductible, the insurer

pays basic medical expenses up to a limit. The insured then satisfies a deductible. Whatever amount of expenses remain are split

between the insurer and the insured according to their coinsurance amounts (.e., most often 80 % paid by the insurer and 20 % paid

by the insured).

DENTAL EXPENSES — These types of expenses are covered by dental expense coverage. This type of coverage may be

purchased individually or as part of a group plan. Routine dental expenses are not covered by medical plans and other forms of

accident and health policies unless they are the result of an accident.

DEPENDENT COVERAGE — Group and individual health insurance plans provide coverage for dependents including a spouse

and children. Child dependents are covered generally up to age 19 but if a full-time student up to age 23 (some plans extend this

to age 25). Coverage may be extended beyond this age for children who are emotionally and / or physically challenged and require

NILA ©

L/H Licensing

Glossary 19 -19

Page 297: FORWARD - WordPress.com · Return of premium p 6-8 6.0 Annuities p 7-1 Annuity principles and concepts p 7-1 Accumulation period versus annuity period p 7-1 Owner, annuitant and beneficiary

full-time care. Step-children are also covered as dependents. In addition, eligibility for coverage for a dependent child ceases when

he or she marries, even if the primary residence is the home of the parents. Also, coverage for a dependent spouse ceases when a

legal separation occurs.

DISABILITY — A physical condition that makes an insured incapable of performing one or more duties of his occupation, or, in

the case of total disability, that prevents him from performing any other type of work for remuneration.

DISABILITY BUY - OUT — Sometimes referred to as a disability buy - sell, this involves an agreement between partners or

between an owner and a key employee. If, for instance, a partner becomes totally disabled, the remaining partners have agreed to

“buy - out” the disabled partner’s interest in the business. The agreement is supported or funded by a disability income plan. When

a partner becomes disabled, monthly income benefits from the disability policy are paid to the firm and later distributed to the

disabled partner according to the agreement.

DISMEMBERMENT — Loss of hand, leg, arm or foot by severance through or above the wrist or ankle joints. It also includes

the entire and irrevocable loss of sight of one or both eyes, loss of speech, loss of hearing or loss of use (i.e. paralysis). The losses

of sight, speech and hearing are also called presumptive forms of disability.

DOUBLE INDEMNITY — A clause for the payment of twice the regular benefit if an injury is sustained under certain specified

circumstances. This is a more limited benefit than a double indemnity benefit under life insurance policies.

DREAD DISEASE POLICY — A limited policy that only provides coverage for a specific illness. A cancer only plan is the

primary type of dread disease policy in existence.

DUPLICATE COVERAGE — A term usually applied to benefits where an injury is covered by several policies with one or more

insurers providing the same type of benefits. This may result in over insurance.

DUPLICATE COVERAGE INQUIRY (DCI) — In health insurance, used to determine if there is more than one health

insurance policy to cover the loss. This process is used when trying to determine the coordination of benefits provision.

EARNED INCOME — Gross salary, wages, commissions and fees derived from active employment. This does not include

unearned income, such as income from investments, rents, annuities or insurance policies.

ELECTIVE BENEFITS — An option in a disability policy that permits an insured to receive a lump sum benefit rather than

receiving a periodic (i.e. monthly) payment.

ELECTIVE COSMETIC SURGERY — A common exclusion in medical plans and disability policies. Cosmetic surgery as

a result of an accident will be covered.

ELIMINATION PERIOD — A period of time after the inception of a disability during which benefits are not payable. This

provision is also referred to as a waiting period. An elimination period must be satisfied for each separate disability. It functions

similar to a deductible.

EMERGENCY CARE — Treatment for emergency services is a usual and customary medical service covered by most forms

of individual and group medical expense insurance, HMOs, PPOs and other medical plans.

ENDODONTICS — A type of dental care that pays benefits for root canal or treatment for disease of the dental pulp within teeth.

EXCLUSION PROVISION — A Provisions in the policy that eliminates coverage for specified losses or causes of loss. Also

known as exceptions.

EXTENDED CARE FACILITY — A health care facility that is intended to offer care, including skilled nursing care,

rehabilitation and convalescent care over a long period of time. This type of facility does not provide acute care.

NILA ©

L/H Licensing

Glossary 19 -20

Page 298: FORWARD - WordPress.com · Return of premium p 6-8 6.0 Annuities p 7-1 Annuity principles and concepts p 7-1 Accumulation period versus annuity period p 7-1 Owner, annuitant and beneficiary

FEE FOR SERVICE — A billing method for health services where the provider charges separately for each service rendered.

Many doctors and health clinics bill in this manner.

FIDUCIARY — This is an individual occupying a position of special trust and confidence, usually one holding the funds or items

of value of another under personal care, custody or control.

FRANCHISE POLICY — An individual policy written to cover a group of persons that does not qualify for true group insurance.

The benefits may vary slightly within the group.

GATEKEEPER — In health insurance plans (HMO and PPO) where insureds must select a primary care physician (PCP). That

physician (the gatekeeper) is the only one that can refer the insured to other health care providers within the plan.

GRACE PERIOD — A specified period, generally 30 days, following the policy’s premium due date, in which a policy owner

may make payment. During this period, coverage remains in force. If the premium is not paid by the policy owner during the grace

period the policy will lapse

GROUP DISABILITY INSURANCE — This type of insurance is purchased by an employer. The premiums paid may be

contributory or non-contributory.

GROUP INSURANCE — A plan of insurance where the unit that is underwritten is the group as a whole, not the individual or

individuals. Many group plans will have a guarantee issue if a certain number of individuals are covered by the group. A group

must be formed for a purpose other than getting insurance. Examples are labor unions, employer-employee groups, and multiple

employer groups. A flow of lives (younger participants consistently entering the group while older participants leave the group)

and minimum participation requirements are major underwriting considerations of group insurance.

GUARANTEED RENEWABLE — A policy that is non-cancelable by the insurance company, provided the premiums are paid.

The insurance company must raise premiums by class. A point in time will be reached beyond which most renewable policies will

not be renewed, but it must be stated in the contract.

HEALTH INDEMNITY PLAN — A type of health insurance policy (usually group insurance) in which an insured is reimbursed

by the insurer after paying his or her own medical expenses, less any deductible or coinsurance amounts.

HEALTH INSURANCE — A broad term covering the various forms of insurance relating to the health of persons. It includes

such coverages as accident, sickness, disability, hospital and medical expenses. This type of insurance is also referred to as Accident

and Sickness Insurance.

HEALTH MAINTENANCE ORGANIZATION (HMO) — A prepaid health care plan, that provides services to its members

(insured individuals) through approved providers of health services. Members must use plan providers. HMO plans tend to stress

prevention by encouraging regular check-ups and visits for early diagnosis and treatment.

HEALTH SAVINGS ACCOUNTS (HSA) — Created by the Medicare Prescription Drug Improvement and Modernization

Act of 2003. Replaces Medical Savings Accounts. A trust created exclusively for paying qualified medical expenses of the account

holder. Allowable contributions are tax deductible and growth in tax-free if the account is used as intended. HSAs replaced Medical

Savings Accounts (MSAs) in 2004.

HOME HEALTH CARE — Part-time care received in the insured’s home covered by Part A of Medicare. It can include skilled

nursing care, therapy, services of health aides, medical social services, medical supplies, and 80% of certain durable medical

equipment. It does not include housekeeping, meal preparation or delivery, shopping, full-time nursing care, blood transfusions,

drugs, or custodial care.

HOSPICE CARE — A provider of services for the relief of pain and symptom management for terminally ill individuals and

support services for their family. No treatment of the patient takes place. Hospice care is covered under Part A - Medicare.

HOSPITAL BENEFITS — Benefits payable for charges incurred while the insured is confined to or treated in a hospital.

NILA ©

L/H Licensing

Glossary 19 -21

Page 299: FORWARD - WordPress.com · Return of premium p 6-8 6.0 Annuities p 7-1 Annuity principles and concepts p 7-1 Accumulation period versus annuity period p 7-1 Owner, annuitant and beneficiary

HOSPITAL EXPENSE INSURANCE — A policy covering benefits subject to a specified daily maximum for a specified period

of time while the insured is confined to a hospital. It also provides a limited allowance up to a specified amount for miscellaneous

hospital expenses such as operating rooms, anesthesia, diagnostic testing and other covered expenses.

HOSPITAL INSURANCE (HI) — Also referred to as Medicare Part A Coverage. It provides in-patient hospital care, skilled

nursing care, home health care, and hospice care subject to deductibles, copayment, and benefit period.

INCONTESTABLE CLAUSE (Time limit on certain defenses) — A provision of life insurance policies that states once the policy

has been in force for two years, during the lifetime of the insured, the insurance company may not challenge the validity of the

policy. Over the years, case law has established precedence that the Incontestable Clause applies to cases of fraud.

INDEMNITY — An indemnity plan states that the policy will restore an insured to the same financial position in which he or she

existed prior to a loss. Medical expense plans and other forms of health insurance may be written on an indemnity basis.

INDIVIDUAL DISABILITY INCOME — A disability policy that is personally owned by an individual. Premiums payable

on individual plans are not tax deductible. Benefits received are income tax free. This policy pay a portion on an insured’s monthly

income if he or she cannot work due to an accident or illness.

INJURY INDEPENDENT OF ALL OTHER MEANS — An injury resulting from an accident, provided that the accident

was not caused by an illness.

INPATIENT — A person who is confined to a hospital.

INTERNAL LIMITS — Maximums found in most hospital and major medical policies that insurers will pay for specified

expenses. An example is psychiatric care, room charges, drug costs, and maternity care.

JOINDER AGREEMENT — This is an agreement that an employer must sign when seeking and receiving health insurance

coverage through a Multiple Employer Trust. It specifies the responsibilities of the employer / insured.

KEY PERSON DISABILITY INSURANCE — Also known as key employee disability, this is an insurance plan purchased

by an employer to cover a valuable employee.

LAPSE — This means that insurance coverage terminates as a result of the policy owner failing to pay an owned premium by the

end of the grace period.

LEVEL TERM LIFE INSURANCE — A term life insurance policy characterized by a level death benefit every time coverage

is renewed. The premium will increase at each renewal as well. Coverage is usually renewable up to a specified age such as 70.

LIMITED POLICIES — Those that restrict benefits to specified accidents or diseases, such as travel policies, dread disease

policies (i.e. cancer only), ticket policies, accident only policies and others.

LONG-TERM CARE (TLC) — Care provided to individuals where there is some degree of disability and/or health problem.

This can include a wide range of services, many which may not be medically necessary and not covered by traditional health

insurance policies.

LONG-TERM DISABILITY — A type of group disability income policy. Benefits received from this type of plan are offset

by any amounts collectable from social insurance such as Workers’ Compensation. Group disability policies may also be referred

to as a non-occupational plan.

LOSS OF INCOME BENEFITS — Income benefits payable to the insured because he is unable to work due to an insured

disability.

LOSS OF INCOME INSURANCE — Policies that provide benefits to help replace an insured's earned income lost as a result

of a covered illness or accident.

NILA ©

L/H Licensing

Glossary 19 -22

Page 300: FORWARD - WordPress.com · Return of premium p 6-8 6.0 Annuities p 7-1 Annuity principles and concepts p 7-1 Accumulation period versus annuity period p 7-1 Owner, annuitant and beneficiary

MAJOR MEDICAL EXPENSE INSURANCE — Policies especially designed to help offset the medical expenses resulting

from catastrophic or prolonged illnesses or injuries. Generally, they provide benefit payments of 80% of all types of covered

medical expenses above a certain amount first paid by the insured and up to the maximum limit of liability provided by the policy.

MALINGERING — Intentionally prolonging a disability in order to collect greater insurance benefits.

MANAGED HEALTH CARE PLAN — A health insurance provider system where a group of providers share in the financial

risk or have a financial incentive to deliver required services in a cost-effective manner.

MEDICAID — A health insurance plan regulated by both the federal and state laws, funded by federal and state dollars, and

administered the state government in each state. Individuals qualify for benefits by having income and/or assets at or below poverty

level.

MEDICAL SAVINGS ACCOUNT — A trust created exclusively for paying qualified medical expenses of the account holder.

Allowable contributions are tax deductible and growth in tax-free if the account is used as intended. MSAs were replaced by Health

Savings Accounts (HSAs) in 2004.

MEDICARE — A federal program of health insurance and medical care for eligible persons who are 65 years of age or over. It

consists of Part A, Hospital expenses and Part B, Supplementary medical insurance (SMI).

MEDICARE + CHOICE — Known as Medicare Part C, it is an expansion of Medicare where HMOs, PPOs, or other managed

care providers offer benefits similar to Medicare Part A - Hospital Insurance and Medicare Part B - Supplemental Medical Insurance

(SMI), with the exception of hospice care. The participant pays a monthly premium set by the managed care organization and the

federal government makes monthly payments to the provider.

MEDICARE SUPPLEMENT INSURANCE — Individual health insurance designed to fill the gaps in Medicare Part A -

Hospital Insurance (HI) and Part B - Supplemental Medical Insurance (SMI). There are ten standardized plans (Plans A-J). These

are the only plans that can be offered. Medicare Supplement Insurance cannot duplicate any Medicare benefit, but they can provide

coverage for deductibles and copayment.

MISCELLANEOUS EXPENSES — Also referred to as ancillary charges, there are hospital charges other than room and board

and other hospital charges.

MORBIDITY TABLE — This table demonstrates the incidence and extent of disability that may be expected from a given group

of persons. This table is used in the computation of rates. It is comparable to a Mortality Table used in connection with life

insurance.

NON CANCELLABLE — A policy that an insurer is not permitted to terminate or amend during its term except for non-payment

of a premium. Generally, the renewal of the policy is guaranteed at the option of the insured up to a specified age and at a fixed

premium. Also referred to as non-cancellable and guaranteed renewable.

NON-CONFINING SICKNESS — An illness that prevents the insured from working, but that does not confine him to his home,

a hospital or sanitarium.

NON-CONTRIBUTORY PLAN — A type of group plan where the employer pays the entire premium.

NON-DISABLING INJURY — One that requires medical care but does not result in a loss of time from work.

NON-OCCUPATIONAL POLICY — A policy which insures a person against off-the-job accidents, injuries or sickness only.

Any accident and health plan that excludes Workers’ Compensation benefits may be classified as a non-occupational policy. A

group disability policy is an example of this policy classification.

NILA ©

L/H Licensing

Glossary 19 -23

Page 301: FORWARD - WordPress.com · Return of premium p 6-8 6.0 Annuities p 7-1 Annuity principles and concepts p 7-1 Accumulation period versus annuity period p 7-1 Owner, annuitant and beneficiary

OCCUPATIONAL POLICIES — This is a type of health insurance policy that pays a benefit to an insured when they are injured

or sick on or off the job. A personally owned disability income policy is an example of this type of policy. These policies will pay

a benefit in addition to any benefits paid by Workers’ Compensation or any other social insurance program.

OPEN ENROLLMENT PERIOD — A period during which eligible individuals can choose to enter a plan. In the case of an

alternate plan, they normally do not have to provide evidence of insurability.

OPTIONAL PROVISIONS — Accident and health policies may include any of the optional uniform policy provisions. Change

of occupation, misstatement of age or sex, conformity with State statutes, illegal activities and insurance with other insurers are

several of these common optional provisions.

ORTHODONTICS — A type of dental expense that pays benefits for corrective teeth devices such as braces and retainers.

OPTIONALLY RENEWABLE POLICIES — Policies that are renewable at the option of the insurer.

OUTPATIENT — One who receives care at a clinic or hospital without being confined to that institution as a patient.

OVER-INSURANCE — An excessive amount of insurance carried by an insured that might tempt him to prolong his period of

disability (i.e., malingering).

PALLIATIVE CARE — The primary medical service provided by hospice care. This also refers to pain management care for

the terminally ill individual.

PARAMEDICAL EXAMINATION — An abbreviated physical examination given by a nurse or technician recording only the

insured’s medical history, blood profile, and measurements or reading of certain vital tests. Company underwriters evaluate the data

and may request detailed testing. Each company has its own guidelines concerning which classes of applicants get a paramedical

and which tests they get. Paramedical examinations save expenses and in many cases shorten the time needed to issue policies.

PARTIAL DISABILITY — An illness or injury that prevents an insured from performing one or more of his or her occupational

duties. Usually this benefit pays 50% of the total disability benefit for a specified period (i.e. 6 months).

PERSONALLY OWNED DISABILITY INCOME POLICY — This is the same as an individual disability income policy.

PERIODONTICS — A type of dental care treating diseases of the surrounding and supporting tissues of the teeth such as gum

disease.

POINT OF SERVICE PLANS (POS) — A blend of Health Maintenance Organization and Preferred Provider Option where

the insured receives services from a Primary Care Physician (PCP) under the plan. The plan may not pay for any services the insured

receives outside the network without a referral from the PCP.

POLICY PERIOD — Also known as “policy term,” this is the period for which a premium is paid and coverage is provided.

PRE-EXISTING CONDITION — An injury occurring, sickness contracted or physical condition that existed prior to the issuance

or inception of a health policy. According to HIPAA, a preexisting condition is defined as a mental or physical condition for which

medical advice diagnosis, care or treatment was recommended or received within the six month period ending on the enrollment

period.

PREFERRED PROVIDER OPTION (PPO) — A cost containment method where health service providers, including

physicians, hospitals, and clinics, negotiate with the health insurance company, stable, predetermined and usually reduced charges

for each procedure. In return for this reduced charge, the health insurance company streamlines administration and provides referrals

to the provider while giving the policyholder additional benefits in the form of a higher Co-pay percentage or a low single visit

deductible.

NILA ©

L/H Licensing

Glossary 19 -24

Page 302: FORWARD - WordPress.com · Return of premium p 6-8 6.0 Annuities p 7-1 Annuity principles and concepts p 7-1 Accumulation period versus annuity period p 7-1 Owner, annuitant and beneficiary

PREPAID FINANCING — This is another term for premium payments. In other words, insurance coverage is financed through

a prepaid premium.

PRESUMPTIVE DISABILITY — In disability income policies a percentage of disability will be considered without a waiting

period if the insured suffers the total and complete loss of specific body parts, body functions, or senses. Examples: Speech, hearing

in both ears, sight in both eyes, use of one hand and one foot, use of both hands, sight in one eye and use of one hand or foot.

PREVENTIVE CARE — A primary type of medical care that is covered by HMOs and other alternative forms of health care.

Preventive care hopes to identify or detect medical problems before they manifest themselves in to an illness. Some examples

include routine physical examinations, well baby care, pap smears, mammographies or family planning services.

PRIMARY CARE NETWORK (PCN) — In health insurance a group of doctors that give the primary care for the participants

of a particular plan.

PRINCIPAL SUM — The label given for the death benefit paid by accidental death coverage.

PROBATIONARY PERIOD — A specified number of days after the date of the issuance of the policy, during which coverage

is not afforded for sickness. Sickness protection does not become effective until after the end of such probationary period. Sickness

contracted during the probationary period is not covered regardless of the duration of such disability. A probationary period is a

one-time event, whereas an elimination period may occur upon each separate disability.

PROOF OF LOSS FORM PROVISION — This is a required uniform accident and health policy provision that is also called

the claim form provision. It states that once an insured notifies the insurer about an accident or illness, the insurer is required to

send the insured a claim form within 15 days. If this does not occur, the insurer could be charged with an unfair claim settlement

practice.

PROSTHODONTICS — A type of dental care that provides for the replacement of missing teeth or artificial devices such as

bridgework or dentures.

PROVISIONS — Each state has enacted standard policy provision laws, which require life insurance companies to include certain

provisions in every life insurance policy. The company may select the actual language, but the state departments of insurance must

approve the wording. State insurance codes impose requirements that specific provisions must be included in all life insurance

policies sold and/or delivered in that state. Some policies are exempt from some provisions (group policies, term, premium, and

non-participating are examples).

RATED POLICY (RATING UP) — The basis for an additional charge to the standard premium because the person insured

is classified as a greater than normal risk. Above standard rates usual result from impaired health or hazardous occupations.

RECURRENT DISABILITY CLAUSE — A provision that specifies a period of time during which the recurrence of a

condition is considered a continuation of a prior period of disability or hospital confinement.

REIMBURSEMENT BASIS — Some medical plans function on this basis. This means that the medical provider will be

reimbursed after they provide health care services to an insured. In addition, if an insured pays the bill, they may complete a claim

form and send it to the insurer along with a copy of the charges in order to be reimbursed.

REINSTATEMENT — The act of restoring a lapsed life insurance policy to its original status. Normally, reinstatement involves

payment of back premiums with interest, payment of all policy loans and providing evidence of insurability within a specified time.

RELATION OF EARNINGS TO INSURANCE — A provision in the policy that permits the insurer to reduce the monthly

income disability benefits payable if the insured's total income from benefits exceeds either his current monthly earnings or his

average monthly earnings during the two-year period immediately preceding the disability.

NILA ©

L/H Licensing

Glossary 19 -25

Page 303: FORWARD - WordPress.com · Return of premium p 6-8 6.0 Annuities p 7-1 Annuity principles and concepts p 7-1 Accumulation period versus annuity period p 7-1 Owner, annuitant and beneficiary

RESIDUAL DISABILITY — A type of partial disability which may be added to a disability policy as a rider. This rider will

pay a percentage of the policy’s total disability benefit if the insured cannot perform all of his or her regular duties. Therefore, the

monthly benefit payable is based on the insured’s percentage of lost income.

RESPITE CARE — This type of care is generally included in a long-term care policy. When a primary care giver needs a break

from providing care for a spouse with Alzheimer’s Disease, for instance, the policy will pay for the expenses incurred for a medical

to provide occasional care at the home of the insured. Respite care allows family members to take a needed break.

RESTORATIVE BENEFITS — These are dental expenses that pay for filings, crowns and other procedures that restore the

natural function of teeth.

RIDER — A legal attachment amending a policy. Additional benefits or a reduction in benefits are often incorporated in policies

by the attachment of either a benefit or an exclusion rider.

SCHEDULED BASIS — In a surgical expense plan or in a medical plan that provides surgical expenses, a list of maximum

amounts payable for such procedures.

SECTION 125 PLAN — A qualified plan that allows employees contributions to be made for health care with pre-tax dollars.

SERVICE BENEFITS — Those benefits that are received in the form of specified hospital or medical care rather than in terms

of cash amounts. These benefits are provided as part of a medical plan issued by service providers such as Blue Cross and Blue

Shield.

SKILLED NURSING FACILITY (SNF) — Paid under Part A Medicare. Facility must be approved and requires a minimum

3-day hospital stay before being placed in the facility. Treatment must be medically necessary and prescribed by a doctor. There

is a coinsurance and benefit period limitation.

SM ALL EMPLOYER CARRIER — These are insurers that specialize in writing medical expense coverage for small

businesses. Most often, a business is eligible for small employer medical expense insurance if they employ less than 25 employees.

In some States this amount is less than 50 employees.

SPECIAL CLASS — An applicant who cannot qualify for a standard policy, but may secure one with a rider waiving the payment

for a loss involving certain existing health impairments. He or she may be required to pay a higher premium or to accept a policy

of a type other than the one for which he has applied.

STANDARD RISK — The classification of a person applying for life, health, and/or disability income insurance who fits the

physical, occupational, and life style standards (four DWI convictions is not a standard life style) on which normal premium rates

are based.

STOP LOSS LIMIT — This is a provision in a medical plan that states that once the insured’s out of pocket expenses for

medical treatment reach a specific limit during the policy period, the insurer pays 100 % of all remaining covered health care costs

until the end of the policy period. Out of pocket expenses refer to deductibles and coinsurance amounts.

SUBSCRIBER — This is another name for an insured. Blue Cross plans and HMOs refer to covered insureds as subscribers or

members.

SUBSTANDARD RISK — Also referred to as impaired risk. The classification of a person applying for an insurance policy who

does not meet the requirements set for the standard risk. An additional premium is charged on substandard risks to provide for the

probability that such persons will have higher than normal claims incidences in health and disability income insurance and will die

younger in life insurance.

SUPPLEMENTAL MEDICAL INSURANCE (SMI) — Also referred to as Medicare Part B Coverage. Unlike Part a

coverage, this coverage is optional. It can provide benefits for doctors’ services, home health care (if not covered by Part A), and

out-patient medical services and supplies.

NILA ©

L/H Licensing

Glossary 19 -26

Page 304: FORWARD - WordPress.com · Return of premium p 6-8 6.0 Annuities p 7-1 Annuity principles and concepts p 7-1 Accumulation period versus annuity period p 7-1 Owner, annuitant and beneficiary

SURGICAL EXPENSE INSURANCE — A policy that provides benefits to pay for the cost of surgical procedures.

SURGICAL SCHEDULE — A list of benefit limits that are payable for various types of surgical procedures with the respective

maximum payable based upon the severity of the procedures.

TESTIMONIALS — These are statements made by an individual or organization promoting the insurance products marketed by

an insurer. Such statements must be true and represent the current opinion of the person making them. These statements also

promote the insurer as well.

THIRD PARTY ADMINISTRATOR (TPA) — In group insurance, provides the administration for the employer or

association. They may be involved in certifying eligibility, collecting and remitting premiums, preparation and payment of claims,

and the preparation and submission of required state reports.

TIME LIMIT ON CERTAIN DEFENSES — A required Uniform Provision that prohibits an insurer from contesting

statements on an application for health insurance (i.e., two years). This provision is called the incontestable provision in life

insurance.

TRAVEL ACCIDENT INSURANCE — Provides benefits for accidental injury while traveling, usually on a common carrier.

This is a type of limited policy.

TRIGGER OF COVERAGE — A description of when benefits will be payable under a long-term care policy. For example,

coverage is “triggered” under a long-term care plan, whether an individual or group plan, when the insured is unable to perform two

or more activities of daily living such as eating, dressing or bathing.

UNREIMBURSED M EDICAL EXPENSES — Any unreimbursed medical expenses are tax deductible if they exceed 7 ½ of

the insured’s adjusted gross income. Unreimbursed medical expenses are derived from deductible amounts absorbed by the insured

as well as any coinsurance amounts. An individual who self insures may also deduct medical expenses if they exceed 7 ½ of his

or her AGI. Premiums for medical plans and qualified long-term care plans may also be added in with these expenses.

WAITING PERIOD — The duration of time between the beginning of insured's disability and the commencement of the period

for which benefits are payable. Another term to describe an elimination period.

WAIVER — A legal concept that waives the liability of the insurer for certain disabilities or injuries ordinarily covered in the

policy.

WAIVER OF PREMIUM — A provision included in some policies that exempts the insured from the payment of premiums

after he has been totally disabled for a period of at least 90 consecutive days. This varies from the life insurance waiver, which is

180 days.

NILA ©

L/H Licensing

Glossary 19 -27

Page 305: FORWARD - WordPress.com · Return of premium p 6-8 6.0 Annuities p 7-1 Annuity principles and concepts p 7-1 Accumulation period versus annuity period p 7-1 Owner, annuitant and beneficiary

SECTION VII

StateInsurance

Law

Page 306: FORWARD - WordPress.com · Return of premium p 6-8 6.0 Annuities p 7-1 Annuity principles and concepts p 7-1 Accumulation period versus annuity period p 7-1 Owner, annuitant and beneficiary

STATEINSURANCE

LAW

Page 307: FORWARD - WordPress.com · Return of premium p 6-8 6.0 Annuities p 7-1 Annuity principles and concepts p 7-1 Accumulation period versus annuity period p 7-1 Owner, annuitant and beneficiary

MASSACHUSETTSSTATE INSURANCE LAW

LAW OVERVIEW — The following information is based on the State examination content outlinepublished by Thomson Prometric. The life insurance exam will consist of 100 questions, ten of which willbe based on the information herein. The accident and health exam will also contain 100 questions. Ten ofthese questions will be based on the law and regulations as well. The passing score for each exam is 70 %.

MASSACHUSETTS LIFE, ACCIDENT ANDHEALTH COMMON LAW CONTENT

The following law and regulatory information is applicable to both life and health insurance producers. Stateexamination questions have been derived from this information and will appear on: (1) the life insurance exam andon (2) the accident and health insurance exam. The common content to be reviewed includes but is not limited to: Massachusetts producer licensing information; the powers and duties of the Insurance Commissioner; illegal salespractices; the fiduciary responsibilities of producers; advertising regulations and; types of insurers.

MASSACHUSETTS PRODUCER LICENSING (Chapter 175:162 G - X) — The following entities maysecure an insurance license in this State including:

1. PRODUCER (Chapter 175:162 H, L) — A resident individual may secure a producer license in this Stateif he or she satisfies all Massachusetts requirements. The Commonwealth is now issuing producer licensesin the following lines of authority: life insurance; accident and health or sickness insurance; propertyinsurance; casualty insurance; personal lines; limited lines credit insurance; and variable life and annuityproducts.

2. NON-RESIDENT INDIVIDUAL (Chapter 175:162 - N) — An individual may secure this type oflicense if they are licensed in their home State and satisfy all Massachusetts requirement for a non-residentlicense.

3. RESIDENT BUSINESS ENTITY (Chapter 175:162 - L) — A corporation may also secure an insurancelicense to conduct insurance transactions in this State.

4. PUBLIC INSURANCE (FIRE) ADJUSTER — Section 162 of Massachusetts law states that a fireadjuster is a person who represents a policyholder in the negotiation of a loss payable by a fire insurancepolicy. A fire loss adjuster settles or adjusts losses on behalf of the public. This State requires that the

adjuster be at least 21 years of age or older; pass a State exam; submit a letter of interest to the Division of

Insurance; and be competent and trustworthy.

NILA ©

L/H Licensing – Massachusetts Law

Chapter 20—1

Page 308: FORWARD - WordPress.com · Return of premium p 6-8 6.0 Annuities p 7-1 Annuity principles and concepts p 7-1 Accumulation period versus annuity period p 7-1 Owner, annuitant and beneficiary

5. MOTOR VEHICLE DAMAGE APPRAISER — Section 162 states that this is an individual, accordingto Massachusetts law, who attempts to determine the amount of a loss in order to assist in the settling of aclaim. Therefore, an appraiser provides estimates of damage to autos. An applicant must submit a workexperience letter from a licensed Massachusetts appraiser with whom he or she has worked for a period ofat least three months. In addition, a copy of certification received from a “board-approved” damage appraisercourse must be submitted or proof of two years working experience writing appraisals. A State exam mustalso be passed as well.

6. ADVISER — Chapter 175 and 177 A and B of Massachusetts insurance law identifies an adviser as anindividual who provides insurance advice for compensation. The compensation is in the form of aprearranged fee, whose amount must be disclosed to the client in writing prior to the consultation. Currentlythere is no regulation governing the fee that may be charged by an adviser. An adviser is also referred to asan insurance consultant, counselor, specialist, analyst or policy holder adviser. An individual using any ofthe aforementioned titles or descriptions may only do so if he or she possesses an adviser license. A licensedattorney or CPA acting within the scope of his or her profession may provide insurance related advicewithout having an adviser license. All of these descriptions indicate that the adviser provides insurancecounseling of some sort. To provide advice for a fee, a person must secure an adviser's license from theCommonwealth. An adviser may not be an officer or regular salaried employee of an insurer (or be aproducer acting for an insurer).

7. SPECIAL BROKER — A special broker is an individual who places insurance business with insurersthat are not authorized to transact business in this State. Also known as a surplus lines broker, this licensemay be secured by an individual of full age who has been deemed suitable by the Commissioner. This typeof broker may negotiate insurance for many types of insurance except accident and health, workerscompensation, compulsory motor vehicle liability or life insurance. Whenever a special broker procures anyinsurance with an unauthorized insurer, he or she shall file with the Commissioner an affidavit stating thatthe full amount of insurance secured was not procurable, after a diligent effort, with a standard insurer. Aspecial broker shall keep a separate account of the business transacted. In addition, he or she shall reportannually in January of each year of all transactions, premiums collected and returned and of any policiescancelled during the year ending the preceding December 31 . Anyone who fails to file such reports withst

the Commissioner shall be subject to a fine of not less than $100 nor more than $500, imprisonment for notmore than a year, or both. This type of license may be issued to an individual, partnership, corporation orfirm. The holder must also post a surety bond to guarantee performance as well.

8. REINSURANCE INTERMEDIARY — This license can be secured by an individual or firm thatnegotiates reinsurance contracts with reinsurers on behalf of an insurer. No person may act as a reinsuranceintermediary in the Commonwealth unless he or she is a licensed producer and maintains an office in thisState, or is licensed as a producer in another State and their home State has reciprocity with theCommonwealth of Massachusetts. The Commissioner may require that such an intermediary post a suretybond and maintain an errors and omissions policy. Every applicant for this license must have been licensedas an insurance producer for the lines for which he or she intends to transact business as an intermediary fora period of at least three years prior to applying for such license. The application shall state all businessaffiliations of the applicant for the preceding ten years. The applicant must also pass a written examinationand be deemed competent and trustworthy. Once issued to an individual, the license is good for three yearsfrom the date of issue (if issued to a partnership or firm it is good for one year and may be renewed). Anyoneacting as an insurance intermediary without a license shall be subject to a fine of not more than $1,000,imprisonment for not more than six months, or both. This license may be issued to a firm or corporation forperiods of one year and if all individuals doing business under the license are deemed suitable by theCommissioner. A nonresident licensee may secure this license as well if qualified, if licensed in good

NILA ©

L/H Licensing – Massachusetts Law

Chapter 20—2

Page 309: FORWARD - WordPress.com · Return of premium p 6-8 6.0 Annuities p 7-1 Annuity principles and concepts p 7-1 Accumulation period versus annuity period p 7-1 Owner, annuitant and beneficiary

standing in his or her home State and must name the Commissioner as agent for service of process. Eventhough this individual is negotiating a reinsurance agreement on behalf of ceding company, he or she has nopower to bind coverage on behalf of the ceding company.

9. VIATICAL SETTLEMENT BROKER — This is a licensed individual that, on behalf of a viator (i.e.,

policyowner) and for a fee, commission or other valuable consideration, offers or attempts to negotiate aviatical settlement between a viator and one or more viatical settlement providers. In other words, a viaticalsettlement broker brings a viator and a viatical settlement firm together. An individual must secure a licensewith the Commissioner in order to act as a viatical settlement loan broker. This license may be renewed fromyear to year, on the anniversary date, upon payment of a renewal fee. If this fee is not paid, the license willexpire. The Commissioner may also suspend or revoke the license for cause as well. The viatical settlementbroker will engage in these activities on behalf of the viator in return for a fee, commission or other valuableconsideration. Therefore, the viatical settlement broker owes a fiduciary duty to the viator to act accordingto the viator’s instructions and in his or her best interests. A viatical settlement broker does not include anattorney, accountant or financial planner retained to represent the viator.

10. TEMPORARY LICENSE (Chapter 175:162 - Q) — This type of license may be issued to a surviving

spouse, or to a court appointed designated person when a licensed individual dies, becomes mentally orphysically disabled, or enters the armed services. The period for which this type of license is issued is 180days and does not require a State examination. This license does not allow the holder to write new businessbut only to receive renewal commissions. This type of license may not continue once the owner disposes ofthe business. If an individual designated in a business entity license dies or becomes disabled, a temporarylicense may be issued to a member or employee of that business entity. A temporary license may be issuedby the Commissioner in any instance where he or she deems it necessary.

QUALIFICATIONS FOR LICENSURE

Producers and advisers must fulfill specific requirements set forth by the State of Massachusetts in order to obtaintheir respective insurance licenses:

PRODUCER LICENSE REQUIREMENTS — According to Chapter 175, Sections 162 of State law, anyindividual may secure a producer's license by fulfilling State requirements. The requirements are as follows:

! Must be a Massachusetts resident although there is no requirement that the candidate has to have residedhere for a period of time prior to applying for the license.

! Must be 18 years of age or older (i.e., must be of “full age”). However, there is no requirement that thecandidate possess a high school diploma or a college degree.

! Must pass the State examination in the line(s) of authority for which the individual applied .! Must be deemed competent, honest and trustworthy and not have committed any act that is a ground for

denial, suspension or revocation.! Must pay the appropriate license fee and submit any other information that may be required by the Division

of Insurance (i.e., background questions, DBA certificate if applicable, State exam scores, documentationrelative to exempt status, etc.). A producer candidate does not have to be sponsored or appointed by aninsurer to take the State examination, nor does he or she have to attend or complete a pre-licensing or othereducational course in order to sit for the examination.

NILA ©

L/H Licensing – Massachusetts Law

Chapter 20—3

Page 310: FORWARD - WordPress.com · Return of premium p 6-8 6.0 Annuities p 7-1 Annuity principles and concepts p 7-1 Accumulation period versus annuity period p 7-1 Owner, annuitant and beneficiary

NON-RESIDENT— Section 162 - N states an individual may secure a non-resident license in Massachusetts if thefollowing requirements are satisfied:

! Must be currently licensed in good standing in his or her home State.! Must not have committed any act for which the license could be denied, suspended or revoked.! Must pay the appropriate fee and provide any information that the Division of Insurance requires.

A non-resident business entity may also secure a license in this State as long as the entity is licensed in good standingin its home State, pays the appropriate fee, submits the proper application to the Division of Insurance, provides alist of members in the organization who will have the authority to solicit business and designate a licensed producerresponsible for the business entity’s compliance with the insurance laws of the Commonwealth. A non-residentproducer who moves to the Commonwealth must submit an application within 90 days of establishing legalresidence to become a resident licensee.

RESIDENT BUSINESS ENTITY — Section 162 - L states that a corporation may secure an insurance licenseas well as long as State requirements are satisfied. The entity :

! Must pay the appropriate fees and provide any other information required by the Division of Insurance.! Must submit their articles of incorporation or organization along with the requested business entity name

which includes the word “insurance”.! Must submit the appropriate application and a list of members who are covered by the license and possess

the authority to solicit business on behalf of and in the name of the business entity.! Must designate a licensed producer responsible for the business entity’s compliance with the insurance laws

of the Commonwealth.

ADVISER (Chapter 175:177 A and B) — According to State law, an adviser may also be an individual, partnership,firm or corporation. The requirements to become an adviser are:

! Must submit to the Division of Insurance: a letter of interest describing their background; a resumedescribing credentials and work history (for the preceding ten years); and a written contract describingadvising services

! Must be age 18 or older! Must pass a written exam! Must have never had an insurance license revoked or suspended! Must be honest, trustworthy and competent as indicated by three references of the Commonwealth.

MAINTENANCE, DURATION, TERMINATIONAND EXPIRATION OF LICENSES

A producer and adviser license may be secured when each candidate satisfies specific State requirements. As longas the licensee does not engage in unfair practices and pays the required license fee, the license may continue fromrenewal period to renewal period. Notice of license renewal will be sent by the Division 30 to 45 days prior to thelicense renewal date (i.e., producer birth date).

PRODUCER LICENSE —Chapter 175, Section 162 - H and L describe producer licensing. Once issued, aproducer's license is good for three years and renews on the holder’s birthday. The triennial fee is $225. Aproducer's license will expire if the fee is not paid. This will also cause all appointments with insurers to be canceledas well. According to Section 162 - M (b-d) of Massachusetts law, a producer who allows his or her license to lapse

NILA ©

L/H Licensing – Massachusetts Law

Chapter 20—4

Page 311: FORWARD - WordPress.com · Return of premium p 6-8 6.0 Annuities p 7-1 Annuity principles and concepts p 7-1 Accumulation period versus annuity period p 7-1 Owner, annuitant and beneficiary

may, within 12 months from the due date of the renewal fee, reinstate the license without the necessity of passingthe state examination, but a penalty in the amount of double the renewal fee is required. If the license is notreinstated within 12 months after its lapse, the candidate must pass the state examination once again. An individualmay not actually begin to engage in sales activity until his or her license has been issued by the Commonwealth.Simply passing the exam does not legally allow an individual to commence insurance activities. In addition, alicensed producer who is unable to comply with renewal requirements due to service in the military, or otherextenuating circumstances, may request a waiver of the requirements from the Commissioner.

! Exemptions — Certain applicants are exempt from paying the license fee and lead paint surcharge including:(1) a person who is a Massachusetts resident and a veteran; (2) a person who is blind: and (3) a widow /widower of a licensee. Exempt applicant must submit the following documentation to the Division ofInsurance: a veteran must submit a copy of the DD-214 discharge papers; a blind person must submit adocument verifying legal blindness; and a widow / widower must submit the name and Social Securitynumber of the deceased licensee. Nonresident producers may be exempt from the fee is they are blind or awidow / widower as long as written proof is provided to the Division of Insurance.

! Producer Appointments — Section 162-S states an insurance producer shall not act as an agent of aninsurer unless the insurance producer becomes an appointed agent of that insurer. An insurance producerwho is not acting as an agent of an insurer is not required to become appointed. To appoint a producer asits agent, the appointing insurer shall file, in a format approved by the Commissioner, a notice ofappointment within 15 days from the date the agency contract is executed or the first insurance applicationis submitted. An insurer may also elect to appoint a producer to all or some insurers within the insurer’sholding company system or group by the filing of a single appointment request. Upon receipt of the noticeof appointment, the Commissioner shall verify within a reasonable time, not to exceed 30 days, that theinsurance producer is eligible for appointment. If the insurance producer is determined to be ineligible forappointment, the Commissioner shall notify the insurer within 5 days of its determination. An insurer shallpay an appointment fee for each insurance producer appointed by the insurer. An insurer shall also remita renewal appointment fee in the amount prescribed by Massachusetts law.

Section 162-T states that an insurer or authorized representative of the insurer that terminates theappointment, employment, contract or other insurance business relationship with a producer shall notify theCommissioner within 30 days following the effective date of the termination. Upon the written request ofthe Commissioner, the insurer shall provide additional information, documents, records or other datapertaining to the termination or activity of the producer. The insurer or the authorized representative of theinsurer shall promptly notify the Commissioner if, upon further review or investigation, the insurer discoversadditional information that would have been reportable to the Commissioner. Within 15 days after makingthe notification required by state law, the insurer shall mail a copy of the notification to the producer athis or her known address. If the producer is terminated for cause, the insurer shall provide a copy of thenotification to the producer at his last known address by certified mail, return receipt requested, postageprepaid or by overnight delivery using a nationally recognized carrier (i.e., Federal Express).

Within 30 days after the producer has received the original or additional notification, the producer may filewritten comments concerning the substance of the notification with the Commissioner. The producer shall,by the same means, simultaneously send a copy of the comments to the reporting insurer, and the commentsshall become a part of the Commissioner’s file and accompany every copy of a report distributed or disclosedfor any reason about the producer. In the absence of actual malice, an insurer, the authorized representativeof the insurer, a producer, the Commissioner, or an organization of which the Commissioner is a member andwhich compiles information and makes it available to other Insurance Commissioners or regulatory or lawenforcement agencies, shall not be subject to civil liability, and a civil cause of action of any nature shall not

NILA ©

L/H Licensing – Massachusetts Law

Chapter 20—5

Page 312: FORWARD - WordPress.com · Return of premium p 6-8 6.0 Annuities p 7-1 Annuity principles and concepts p 7-1 Accumulation period versus annuity period p 7-1 Owner, annuitant and beneficiary

arise against these entities or their respective agents or employees, as a result of any statement or informationrequired by or provided under state law.

A producer shall report to the Commissioner any administrative action taken against the producer inanother jurisdiction or by another governmental agency in the Commonwealth within 30 days of the finaldisposition of the matter. This report shall include a copy of the order, consent to order or other relevantlegal documents. Within 30 days of the initial pretrial hearing date, a producer shall report to theCommissioner any criminal prosecution of the producer taken in any jurisdiction. The report shall includea copy of the initial complaint filed, the order resulting from the hearing and any other relevant legaldocuments.

BUSINESS ENTITY AND NON-RESIDENT BUSINESS ENTITY— Section 162 - L states that theselicenses will renew one year from the original date of licensure. The renewal fee is $75 per member or partnercovered by the license.

ADVISER — The non-refundable license fee of an adviser must be paid to the Commonwealth every three years andis $200. Anyone acting as an adviser without a license or during a license suspension may be fined not less than $50nor more than $500, receive a prison sentence of not more than six months, or both. An adviser is not subject to thesame continuing education requirement as a producer.

CONTINUING EDUCATION

As of April 4, 1983, all producers must complete State approved continuing education requirements in order tomaintain their respective licenses. An auto club agent, travel accident agent, baggage agent, fraternal agents orany other person holding a license for which no examination is required, are not subject to any C.E. requirements. Chapter 175, Section 177-E of Massachusetts law and Regulation 211, CMR 50 identifies the C.E. requirements.

! During the first 3 years (i.e., 36 months) of licensure, a resident producer must complete 60 credit hoursof approved study.

! During the next 3 years and each subsequent 36 month period, the licensee must complete 45 credit hoursof approved C.E.

! Surplus or excess credit hours may be carried over to the ensuing 3 year period only.! If a licensee fails to comply with this State law C.E. requirements, his or her license will be suspended until

they comply, although extensions may be granted by the Commissioner for cause shown.

The C.E. courses approved by the Division of Insurance include the following: (1) Life Underwriting Training Councilcourses (LUTC, 25 credits each); (2) Chartered Life Underwriter courses (CLU, 30 credits each); (3) InsuranceInstitute of America courses (IIA, 25 credits each); (4) Chartered Property Casualty Underwriter courses (CPCU, 30credits each); (5) Certified Insurance Counselor courses (CIC, 25 credits); (6) Accredited Associate in Insurancecourses (AAII, 25 credits each); (7) any other course, including correspondence courses (i.e., self study), that areapproved by the Commissioner. Any person teaching an approved C.E. course will receive the same number of creditsas an attendee. In addition, an individual who holds a nonresident license in this State is exempt from the C.E.requirement if they are subject to a C.E. requirement in their home State. Anyone with a limited or restricted licenseis also exempt from satisfying the C.E. requirement.

NILA ©

L/H Licensing – Massachusetts Law

Chapter 20—6

Page 313: FORWARD - WordPress.com · Return of premium p 6-8 6.0 Annuities p 7-1 Annuity principles and concepts p 7-1 Accumulation period versus annuity period p 7-1 Owner, annuitant and beneficiary

SUSPENSION OR REVOCATION OF LICENSES

Massachusetts law Chapter 175, Section 162 - R specifies conditions for disciplinary action involving licensesuspension or revocation of producers and advisers, or any insurance licensed person or organization. TheCommissioner of Insurance may suspend or revoke a license for violations of State law. However, a license may notbe suspended until after a hearing has been held in order to allow the licensee to defend himself or herself. Writtennotification must be provided to the licensee of any such action and in addition, the sponsoring insurer, if a produceris involved, must be notified as well. The length of the suspension or revocation will be determined by the Commis-sioner. State law dictates that no one may act as a producer without a license. Anyone engaging in such conductmay be subject to a monetary fine of not less than $20 nor more than $500.

The Commissioner may place on probation, suspend, revoke or refuse to issue or renew an insurance producer’slicense (or levy a monetary penalty) for any one of the following acts:

1. Providing incorrect, misleading, incomplete or materially untrue information in a license application;2. Violating any insurance laws, or violating any regulation, subpoena or order of the Commissioner or of

another State’s Commissioner;3. Obtaining or attempting to obtain a license through misrepresentation or fraud;4. Improperly withholding, misappropriating or converting any moneys or properties received in the course of

doing insurance business (i.e., larceny);5. Intentionally misrepresenting the terms of an actual or proposed insurance contract or application for

insurance;6. Having been convicted of a felony (a prohibited person who has been convicted of a felony may be issued

a license in certain situations if they have received written permission from an authorized representative ofthe Insurance Department);

7. Having committed or been found to have committed any insurance unfair trade practice;8. Using fraudulent, coercive or dishonest practices, or demonstrating incompetence or financial irresponsibility

in the conduct of insurance business;9. Having a producer’s license or its equivalent, denied, suspended, or revoked in any other State, province,

district or territory;10. Forging another’s name to an application for insurance;11. Knowingly accepting insurance business from an individual who is not licensed;12. Failing to comply with an administrative or court order imposing a child support obligation;13. Failing to pay State income tax or comply with any administrative or court order directing payment of State

income tax; or14. Cheating on a State pre-licensing examination.

If the Commissioner non-renews a license or denies an application for a license, he or she shall notify the applicantor licensee in writing the reason for such action. The licensee may appeal such action within thirty (30) days ofreceiving such notice. Once the Commissioner receives the appeal and a request for a hearing, he or she will hold ahearing within twenty (20) days.

MASSACHUSETTS INSURANCE COMMISSIONER

The various powers and duties of the Insurance Commissioner are described in Chapter 175 of Massachusetts law. The Commissioner is provided with the power to administer and enforce insurance laws and regulations throughout

NILA ©

L/H Licensing – Massachusetts Law

Chapter 20—7

Page 314: FORWARD - WordPress.com · Return of premium p 6-8 6.0 Annuities p 7-1 Annuity principles and concepts p 7-1 Accumulation period versus annuity period p 7-1 Owner, annuitant and beneficiary

this State. The Commissioner is appointed by the Governor and therefore this regulatory position is an appointedposition and not an elective one.

POWER AND DUTIES — The principal duty of the Commissioner of Insurance is to administer and enforce theinsurance laws that are in effect today. It is not the power or duty of this office to propose legislation, make lawsor pass laws. That is the responsibility of the state legislature. The Commissioner possesses several powers and dutiesincluding but not limited to the power or duty to:

! Issue insurance related licenses (i.e., producers, advisers, etc.) or a certificate of authority to an insurer.Chapter 175, Sections 4 and 32 state that the Commissioner must conduct an extensive examinationdetermining that an insurer is financially sound and qualified to transact insurance business. Prior to issuinga certificate of authority, the Commissioner requires the insurer to file an affidavit signed by the officers ofthe company certifying the amount of expenses incurred in the organization and the fact that the company hasno outstanding liabilities except organization expenses. In addition, no insurer may issue any contracts ofinsurance until it has obtained this certificate and stated it has complied with all the regulations of theCommonwealth. The Commissioner also receives license fees.

! If a licensee or insurer breaks any State laws the Commissioner may, after a hearing, suspend the applicablelicense or certificate of authority, or place the guilty party on probation. A hearing is to be held to providean opportunity for the offending producer to defend himself or herself. The Commissioner also refersinformation regarding licensees who break the law to the Attorney General (or the proper district attorney)for prosecution. The Attorney General will then prosecute licensees who break the insurance laws. TheCommissioner does not prosecute producers who engage in illegal activities. Therefore, the Commissioneracts with regard to insurance regulatory issues whereas the Attorney General prosecutes those who engagein criminal or other illegal activity.

Chapter 175, Sections 174 -A and 176 - D (7) state that notice of hearings concerning the suspension orrevocation of a license are sufficient when sent to the offender registered mail to the last residence or businessaddress on record with the Commissioner. Whenever a person is suspected of committing an unfair tradepractice, the Commissioner shall send a notice of hearing to the offending party that will also include astatement of the charges against the offender. The hearing to be held shall not be less than twenty-one (21)days following the notice.

! Approve insurance policy forms and premium rates. This State is a “file and use” State which means thatan insurer submits or files proposed rates to the Commissioner and may use them unless the Commissionerlater informs them that such rates are unfair. The Commissioner also sets the rates used by auto insurers inthis State. Chapter 175, Section 113 - B states that all rates must not be excessive, inadequate or unfairlydiscriminatory. Chapter 175:2-B and Section 192 also state that all policy forms to be used must be filed withthe Commissioner for review. If the Commissioner does not send to the insurer a rejection of the formswithin thirty (30) days after receiving them, the insurer may use them. Forms are also approved so that thepublic may compare them if desired.

! Examine the books and records of insurers who are authorized currently and who wish to do business in thisState to make sure they are in sound financial condition. In other words, the Commissioner regulates aninsurer for solvency. He or she may also examine the affairs of any person engaged in the insurance businessin order to determine if that person has or is engaging in any unfair or deceptive practice. Chapter 175,Section 4 states that when the Commissioner determines the nature, scope and frequency of insurerexaminations, he or she shall consider financial statement analysis and ratios, changes in management or

NILA ©

L/H Licensing – Massachusetts Law

Chapter 20—8

Page 315: FORWARD - WordPress.com · Return of premium p 6-8 6.0 Annuities p 7-1 Annuity principles and concepts p 7-1 Accumulation period versus annuity period p 7-1 Owner, annuitant and beneficiary

ownership, actuarial opinions or reports of independent CPAs in order to determine solvency. An insurer isrequired to have reserves in order to assure solvency.

! Hold public hearings to review illegal acts by licensees or insurers.

! Require an affidavit of domestic insurers which will verify its financial condition in at least in every fifthyear after it becomes authorized. This means that domestic insurers must be examined at least once every fiveyears to determine its financial condition, its ability to meet its obligations, if it has complied with State law,the equity of its dealings with policy holders and any other information relating to its business practices andmethods. Chapter 175, Section 4 and RL 63.26 state that every insurer must allow the Commissioner toexamine its books and records at any time.

! Examine and audit insurers at his or her discretion. The Commissioner may also delegate examining dutiesto a deputy or other authorized person. Examination expenses incurred by the Division are paid by theinsurer being examined and must be paid within thirty days of receiving notice of the charges. TheCommissioner may also conduct an examination upon the request of five or more stockholders, creditors,policy holders or persons with a financial (i.e., pecuniary) interest who believe that the insurer is financiallyunsound.

! Determine, set or compute the reserves of life insurers. These reserves or the “net value” of a life insurancecompany are computed by December 31 every year. The Commissioner does not activate these reserves. Anst

insurer is required to have reserves in order to assure solvency.

! Make rules and regulations regarding producer licensing examinations. The Commissioner makesarrangements, including contracting with an outside testing vendor (i.e., Thomson Prometric), to collectapplicable fees and administer the State examinations.

! Regulate insurance advertising. Producers may use advertisements in the sale of insurance products onlyif such ads are submitted by the insurer for approval by the Commissioner and are subsequently approved foruse by the insurer.

! Issue a cease and desist order to producers when applicable. Chapter 175, Sections 174 - K and 176- D (7and 10) states that when the Commissioner believes that a producer has or is engaging in any unfair salespractice, he or she will issue a cease and desist order until a hearing can be held. Following the hearing, ifthe Commissioner does find that the offender has engaged in such illegal activity, the cease and desist orderremains in force and the offender may be ordered to compensate the insurer or policy holder for any loss ordamage suffered. Anyone who violates this order may be fined up to $10,000 per violation and be subjectto license suspension or revocation. Anyone failing to appear when summoned by the Division of Insurancemay be subject to a $1,000 fine, up to one year in prison and the revocation of any insurance license held.

ILLEGAL AND UNFAIR SALESOR MARKETING PRACTICES

Chapter 175 and 176 of State law identifies several types of illegal or unfair sales practices. Any licensee or insurerengaging in these activities will be subject to a monetary fine and license suspension or revocation after a hearing. These illegal practices are also referred to as unfair or deceptive acts or unfair methods of competition. Any personengaging in such activities may be subject to a monetary penalty or fine of not more than $1,000. In addition, the

NILA ©

L/H Licensing – Massachusetts Law

Chapter 20—9

Page 316: FORWARD - WordPress.com · Return of premium p 6-8 6.0 Annuities p 7-1 Annuity principles and concepts p 7-1 Accumulation period versus annuity period p 7-1 Owner, annuitant and beneficiary

Commissioner may order the offender to make restitution to the insurer or policy holder who suffered a loss. Further,in any action to recover on an insurance policy, the court may also award punitive damages, in addition to the amountof the claim, not to exceed 25% of the claim, if it finds that the party seeking to recover has been damaged by aviolation of Massachusetts law. The following are illegal or deceptive practices which are stated in Massachusetts lawand regulations.

LARCENY — Chapter 175, Section 176 of Massachusetts law states that a producer engaged in negotiating, selling,or renewing a life insurance policy or annuity who commits larceny shall be subject to a fine or prison. Anindependent producer who is allowed to collect premiums from insureds may hold these premiums in a premium trustaccount for a limited period of time until due the insurer (i.e., 30 days). If premiums are not paid to the insurer withinthe allotted time, the insurer sends a written demand to the producer. If the producer ignores or fails to reply to thewritten demand, he or she may be charged with larceny since this is considered to be holding premiums in trust forthe insurer. Premiums collected and personal funds may not be commingled in the premium trust account. If aproducer misappropriates, illegally commingles or uses premiums for personal use, he or she may be charged withlarceny. The monetary penalty for larceny is not more than $1,000.

! Premiums may be deposited in a premium trust account by an independent producer.! If premiums are not paid to the insurer by the producer within 30 days after their receipt, a written demand

will be sent.! Ignoring the written demand for premiums due constitutes larcenous activity.! Failing to refund premiums due those who are entitled to a refund may be considered larceny as identified in

Section 187 of Massachusetts law.

MISREPRESENTATION — This unfair practice and other prohibited acts are identifies in Chapter 175. Accordingto Sections 181 through 186 and 176 - D:3(1) and (11), this type of unfair marketing practice involves a licensee orany insurance company officer or representative making false written or oral statements which misrepresent the terms,conditions, privileges, or benefits provided by a policy. Any licensee who includes or omits a statement tending tomislead or deceive a member of the public has engaged in this unfair act.

False or misleading statements regarding the benefits of a policy are also considered misrepresentations. For example,if a simple misrepresentation is made by a producer such as (innocently) miscalculating a premium payment, thereis generally no problem. However, if the misrepresentation is of a material nature, the policy may be void. A producerwho guarantees that a policy will pay a dividend is generally engaging in a material misrepresentation or confirming(i.e., guaranteeing) future dividends in a life insurance proposal is also an example of this unfair practice. When thelicensee "knows" the statements are false, the misrepresentation is generally material.

If a producer, for instance, sells a life insurance policy to a retiree after claiming the contract to be purchased is anannuity, an obvious material misrepresentation has occurred. An annuity may pay a fixed interest rate such as 9%. The retiree is enthralled with the idea of receiving such an income. The producer touts the advantages of the annuityand convinces the retiree to buy. The producer has no intention to sell anything but a life insurance policy. So theproducer completes a life insurance application and submits it to the insurer and a policy is issued. This is a classiccase of material misrepresentation since the producer sold a life policy while stating that it included advantages andbenefits (i.e., 9% guaranteed interest rate) that it did not.

! Another description of misrepresentation involves a producer making an illegal or unfair comparison ofpolicies in order to persuade a person to lapse or surrender his current life insurance plan in force andpurchase a new one from the producer. This form of misrepresentation may also be referred to as twisting. Twisting and other types of misrepresentations are illegal under Massachusetts law and regulations. Twistinglaws are designed to protect the consumer/public.

NILA ©

L/H Licensing – Massachusetts Law

Chapter 20—10

Page 317: FORWARD - WordPress.com · Return of premium p 6-8 6.0 Annuities p 7-1 Annuity principles and concepts p 7-1 Accumulation period versus annuity period p 7-1 Owner, annuitant and beneficiary

! Anyone engaging in misrepresentations of a material nature may be fined not more than $1,000 or receivea prison sentence of not more than six months.

! If twisting or other types of misrepresentation cause a person to lapse his current policy in force, he or shemay take action against the parties involved (i.e., insurer and producer). The aggrieved person has two yearsafter the new policy is secured to follow this course of action. All premiums paid will be returned and allbenefits of the lapsed contract will be reinstated at the expense of the "twisting" insurer.

REBATING — According to Chapter 175, Sections 182 and 184 of Massachusetts law, no insurer, licensee, policyowner or beneficiary shall offer anything of value to a prospective insured or policy owner that is not specified inthe contract in return for the purchase of a life insurance policy or annuity.

! Any valuable consideration or special favor not stated in a policy of insurance which is used to induce anapplicant to buy insurance is known as rebating.

! The licensee offering a rebate is called the offeror. The person knowingly accepting the rebate is the offeree.! An insurer or its representative may not provide a special offer or advantage to members of the public in order

to persuade them to purchase a policy.! The most common form of rebating involves a producer offering to share commissions (i.e., a kick-back)

with an applicant in return for purchasing the policy. The insurance applicant may not legally accept theoffer.

! Rebating is currently illegal in most States but is allowed in California and Florida.! Other examples of rebating involve offering stocks, bonds, securities, season tickets to sporting events, or

almost anything of value such as a leather jacket or other clothing, satchel or briefcase, that in the opinionof the Insurance Department is being used as an inducement to buy or renew.

! When committed knowingly, both parties are guilty of rebating.! A person engaging in rebating may be fined; receive a prison sentence for not less than a month nor more than

a year; or both.

UNFAIR DISCRIMINATION — Chapter 176 - D:3 (7) also states that no insurer may discriminate unfairly inthe assessment of premium rates. No insurer may discriminate between individuals of the same class in an equalexpectation of life nor discriminate with regard to premium rates for any life insurance policy or annuity or with regardto dividends paid on a policy. No discrimination may occur with regard to premium rates based on any informationsuch as color, creed, national origin, marital status or any other criteria not generally utilized during underwriting.

INSURANCE FRAUD REGULATIONS — Massachusetts law regarding fraudulent actions and their penaltiesis reviewed in Chapter 175, Section 181. An insurance producer or other person who knowingly and willingly makesfalse statements or representations in connection to any application for life insurance or endowments may be guiltyof fraud. Any person who makes false statements, conceals, or withholds material information for the purpose ofobtaining any fee, commission, money, or other benefit may be guilty of fraud.

! Individuals guilty of fraud may be fined not less than $100 nor more than $500, receive a prison sentenceof not less than one month nor more than one year, or be assessed a fine and prison sentence.

! For example, assume that an insurance producer conceals in a life insurance application the fact that theapplicant suffers from an incurable disease. The policy is issued and a year later the insured dies. Uponinvestigation the insurer discovers the truth. Not only will it deny the claim and return the premium, but theywill probably terminate the producer contract and report this to the Department of Insurance. The attorneygeneral may then prosecute the guilty producer with fraud.

NILA ©

L/H Licensing – Massachusetts Law

Chapter 20—11

Page 318: FORWARD - WordPress.com · Return of premium p 6-8 6.0 Annuities p 7-1 Annuity principles and concepts p 7-1 Accumulation period versus annuity period p 7-1 Owner, annuitant and beneficiary

Fraud can be contrasted with perjury. Perjury involves making a false statement as part of a sworn statement. Insurance claim forms are considered to be sworn statements when signed by any party. If a person falsifies infor-mation (i.e., claim form) in a sworn statement in order for an insured to collect benefits from a policy, he or she maybe guilty of perjury.

! The penalty assessed to a person guilty of perjury is a fine of not less than $100 nor more than $1,000, orby imprisonment for not more than one year.

UNFAIR CLAIMS PRACTICES — Chapter 176 - D 3 (9) of Massachusetts law identifies various illustrationsof unfair claim practices. If committed by an insurer, the Commissioner may suspend the insurer's certificate ofauthority. Any employees of the insurer who are responsible for unfair claim practices may be disciplined as well. Some of the more common types of unfair claim practices include:

! Misrepresenting insurance policy provisions or pertinent facts concerning coverages provided.! Refusing to pay claims without conducting an equitable and fair investigation based on all available

information.! Failing to effectuate a prompt, fair, and equitable settlement of claims in which liability has become

reasonably clear.! Failing to adopt and implement reasonable standards for the prompt investigation of covered claims or failing

to affirm or deny coverage for a claim within a reasonable time after a proof of loss has been submitted to theinsurer.

! Failing to acknowledge and act reasonably and promptly upon communications with respect to claims.! Seeking to pay life or health insurance claims for less than face amount.! Failing to provide claim or proof of loss forms in a timely fashion.! Forcing insureds to institute legal action to recover amounts due under a policy by offering less than the

amounts ultimately recovered in actions brought by insureds.! Unfairly refusing to pay claims or making known to an insured that the insurer has a policy of contesting

claims by going to arbitration, which may induce the claimant to accept a lower amount to which he or sheis entitled.

! Failing to settle claims promptly after relevant information has been collected by the insurer.! Failing to promptly provide a reasonable response to an insured or beneficiary with relation to the facts or

applicable law regarding denial of a claim.

IMPERSONATION — Chapter 175, Section 175 of State law defines impersonation. A licensee who advertises ormarkets himself or herself as something they are not has engaged in impersonation. The following are examples ofimpersonation: someone claiming to be a producer when they are not; a producer claiming to be a licensed insuranceadviser; or any licensee, including a producer, who advertises themselves by a name or trade name other than the oneappearing on the license on record with the Commissioner. A licensee may use a name other than the one whichappears on his or her license as long as the Commissioner or the Insurance Department provides consent. Forexample, if Jane Brown owns an insurance agency she must advertise herself as the "Jane Brown Insurance Agency." She cannot call herself the "Acme Insurance Agency" or the “Brown” insurance agency unless she receives approvalfrom the Division of Insurance. A specific fine applies to impersonation. Anyone engaging in this unfair practice maybe fined from $10 to $100. In addition, the guilty party will be subject to license suspension or revocation.

DEFAMATION — Chapter 176-D:3 (3) of Massachusetts law states that no person may make false or untruestatements regarding a competing insurer. Any such comments or remarks will be deemed to be defamation. Falsestatements regarding the financial condition or authority of a competing insurer are generally viewed as defamation. For example, if a producer or insurer informs a customer or potential customer that a competing insurer is unreliable

NILA ©

L/H Licensing – Massachusetts Law

Chapter 20—12

Page 319: FORWARD - WordPress.com · Return of premium p 6-8 6.0 Annuities p 7-1 Annuity principles and concepts p 7-1 Accumulation period versus annuity period p 7-1 Owner, annuitant and beneficiary

because it is not authorized in all fifty States, the former has engaged in defamation. Many insurers choose not to belicensed in various states due to stringent regulations or requirements in that particular State.

EMBEZZLEMENT — Chapter 266, Section 56 of State law specifies that any producer who embezzles funds froman insurer, including premiums or any other funds, will be subject to a prison sentence in State prison for a term ofup to five (5) years.

BOYCOTT, COERCION AND INTIMIDATION — Section 176 - D: 3 (4) and 3-A describes this unfairpractice and states that no person or persons may enter into any agreement to commit any act of boycott, coercion orintimidation which results or tends to result in the unreasonable restraint of or monopoly in the business of insurance. For example, a lender may not inform a borrower that in order to receive a loan he or she must purchase property orcasualty insurance from a specific insurer or agency or purchase credit life or disability through the bank. This iscoercion or intimidation and is illegal. Producers are not permitted to intimidate a consumer.

COMPLAINT HANDLING PROCEDURES / FAILURE TO MAINTAIN — Section 176 D:3 (7) also statesthat all insurers are required to maintain proper and adequate complaint handling procedures for the benefit of thepublic. Failure to do so may result in a monetary fine or insurer license suspension. The Investigation Division isthe agency of the Division of Insurance which will receive consumer complaints with regard to an insurer’s failureto maintain or handle consumer complaints. A complaint is any written communication primarily expressing agrievance. Producers and adjusters must maintain any complaints received for a period of two years with a recordof their disposition.

FALSE ADVERTISING REGULATIONS — According to Chapter 175 , the Commissioner shall regulate alladvertising in this State with regard to the sale or marketing of life insurance, annuities, and endowments. Advertisingincludes any making, publishing, disseminating, circulating, or placing at the disposal of the public any notices,circulars, direct mailers, or other tools which describe the benefits of a policy or the services provided by an insurerand its licensees. Advertisements also include the following:

! Prepared sales talks utilized by a licensee.

! Audio visual aids used in a sales presentation.

! Radio and television announcements or ads.

! Newspaper and magazine ads.

! Circulars or "flyers" distributed by mail or in person.

! Billboards, signs, and other like types of advertisements.

An insurer or its representatives may not disseminate nor distribute false information in any of its advertisements. Some examples of false or misleading information include:

! Making false statements concerning the possible dividends or share of surplus previously paid or to be paidon any life insurance policy.

! For example, a producer who informs an applicant that dividends to be paid on a life policy are guaranteedis providing false information. Even if a particular policy has paid dividends for several consecutive years,no dividends, especially the amount of a dividend, can be guaranteed or promised.

! It is also unlawful to make misleading or false statements concerning the financial condition of any personor insurer or to make misrepresentations regarding the legal reserves of any insurer. It is also illegal to stateor imply to prospective or current customers that an insurer may not be financially stable or reliable in otherareas since they are not authorized in all 50 States. Comments such as these are defamatory. Thoseresponsible for such action may be charged with defamation.

NILA ©

L/H Licensing – Massachusetts Law

Chapter 20—13

Page 320: FORWARD - WordPress.com · Return of premium p 6-8 6.0 Annuities p 7-1 Annuity principles and concepts p 7-1 Accumulation period versus annuity period p 7-1 Owner, annuitant and beneficiary

! No insurer or person is allowed to create false financial statements that may deceive the public accordingto Chapter 176-D.

ADDITIONAL STATE REGULATORY INFORMATION

ADVERTISING REGULATIONS— Chapter 176-D of State law specifies that a producer or insurer may notengage in false statements or untrue advertising in the marketing of insurance products. Insurers engaged in thisactivity will be subject to suspension or revocation of its certificate of authority. Furthermore, insurers must alwaysidentify their trade name in any of their advertisements. Insurers are required by the State to keep a copy of all theiradvertisements for a period of four years.

SHARING COMMISSIONS — An insurance licensed person is permitted to share commissions with any otherproducer, company or corporation as long as such entities are licensed in the same line of insurance as the businessbeing solicited. No commissions may legally be shared with an unlicensed person. Whoever violates this law may be fined not less than $50 nor more than $500.

CHANGE OF ADDRESS — Chapter 175, Section 162 - M (f) states that whenever a licensed individual orbusiness entity changes his, her or its business, residence or email address, the Division of Insurance must be notifiedof the change within 30 days. If a producer is moving to another State, he or she must inform the Division ofInsurance in writing within 30 days. If a non-resident producer is moving to another State other than Massachusetts,he or she must inform the Division within 30 days as well. If the non-resident producer is moving to Massachusetts,he or she has 90 days to notify the Division, cancel the current home State license and apply for a license in this State.This person will also need a letter of clearance or certification letter from the Insurance Department of the currenthome State to verify that the license is being relinquished.

ASSUMED NAMES — Chapter 175, Section 162 - P states that no individual or business entity it permitted toadvertise or conduct insurance transactions under a name that is not file with Division of Insurance. Any partyengaging in such activity will be deemed to be operating under an assumed name. Any party must notify theCommissioner of any name under which he, she or it is doing business (i.e., D.B.A.) prior to using such assumedname.

REPORTING OF ACTIONS — Chapter 175, Section 162-V of State law describes the reporting of actionsagainst any producer by another Commissioner or jurisdiction. A producer must report to the MassachusettsCommissioner any administrative action against him or her by another State or governmental agency within 30 daysof a final disposition of the matter (or if criminal, within 30 days of the pre-trial hearing). This report shall includea copy of the order, consent to order and any other relevant documents as required by the Commissioner.

LIFE AND HEALTH INSURANCE INSOLVENCY / GUARANTY FUND — The purpose of the GuarantyFund is to protect policy owners, insureds, beneficiaries and others against failure in the performance of contractualobligations due to the impairment or insolvency of the insurer issuing the policies. Therefore, the Fund protects theinsurance buying public against the insolvency of insurers. In other words, this Fund pays the claims of persons whoseinsurer has become insolvent. All insurers authorized in this State to sell life and health insurance must participatein the Fund. This means that the Guaranty Fund is funded by all authorized insurers transacting life and healthinsurance business in this State. In addition, it is also prohibited to unfairly advertise with regard to the InsuranceGuaranty Fund. In other words, it is illegal for a producer to inform or give the impression to a client or prospectiveclient that the Guaranty Fund always covers the full amount of a claim. There are three classes of assessments thatmay be levied on insurers when an insolvency arises including: (1) a Class A assessment to pay for administrative

NILA ©

L/H Licensing – Massachusetts Law

Chapter 20—14

Page 321: FORWARD - WordPress.com · Return of premium p 6-8 6.0 Annuities p 7-1 Annuity principles and concepts p 7-1 Accumulation period versus annuity period p 7-1 Owner, annuitant and beneficiary

costs and general expenses; (2) a Class B assessment for expenses incurred in connection with impaired or insolventdomestic insurers; and (3) a Class C assessment for expenses incurred by insolvent foreign and alien insurers. TheFund guarantees $300,000 in death coverage for a life insurance policy with $100,000 in guaranteed cash value ifa whole life policy. There is also a guarantee of $100,000 in cash for an annuity, and $100,000 for an applicablehealth insurance policy.

INSURANCE INFORMATION AND PRIVACY PROTECTION ACT — The Insurance Informationand Privacy Protection Act is designed to protect the public against the publishing or making public personalinformation of an individual. Whenever an application is completed, a policy is delivered or a policy is renewed andpersonal information is collected from an insured or applicant, a written notice should be provided to them. At thelatest, when new insurance is being sought, this notice must be provided at the time of policy delivery. This noticeshall state: (1) if personal information is to be collected from persons other than the individual proposed for coverage;(2) what type of information is going to be collected; (3) the circumstances under which disclosure of personalinformation may be made without authorization of the applicant/insured; and (4) the fact that information obtainedfrom a report prepared by an insurance-support organization may be retained by the organization and disclosed to otherpersons. Insurers are allowed to secure personal information in order to make underwriting decisions according tothe Fair Credit Reporting Act and State regulations. State law also provides an individual with the ability to accessthe personal information compiled by an insurance company or organization. Once a request is made, the insuranceorganization must provide the individual with the information in writing, by phone or by some other means (i.e., mail)within 30 days. The insurer must also notify the individual of any other person to whom such information wasprovided during the previous two years. If any of the information is untrue, false or incorrect, the individual mayrequest the insurance organization to amend it or make appropriate changes. Once requested, the insuranceorganization has 30 days to make corrections or to inform the individual that it is refusing to do so (and the reasonsfor the refusal). An "insurance organization" also includes an institutional source or insurance support organization.

EXAMINATION OF BOOKS AND RECORDS — Chapter 175 states that the Commissioner or appointeddeputies may audit an insurer at any time. Insurers must keep up-to-date records of all transactions and such recordsmay be reviewed or examined by the Commissioner at his or her discretion. Any examination or audit expenses willbe paid by the insurer being examined.

TYPES OF INSURERS

Massachusetts law defines various types of insurers which legally operate in this State, including but not limited to:

1. Authorized Insurer — This is a company that has been allowed to conduct insurance business in this State.It is also known as an admitted insurer or a licensed insurer. Authorized carriers receive a “certificate ofauthority” to do business in this State.

2. Unauthorized Insurer — This is an insurer that is not allowed nor permitted to transact insurance in thisState. Any licensee soliciting or negotiating insurance on behalf of an unauthorized insurer will be subjectto license suspension or revocation. This type of insurer is also called a non admitted insurer.

3. Domestic Insurer — This is an insurer chartered and formed under the laws of this State. A domesticcompany generally has its principal or home office in this State as well. The Commissioner of Insurance isrequired to examine domestic insurers at least once every five years to ensure that they are financially stable. Massachusetts Mutual of Springfield, Massachusetts is an example of a domestic insurer. A domestic insurersatisfying State requirements is also considered to be authorized.

NILA ©

L/H Licensing – Massachusetts Law

Chapter 20—15

Page 322: FORWARD - WordPress.com · Return of premium p 6-8 6.0 Annuities p 7-1 Annuity principles and concepts p 7-1 Accumulation period versus annuity period p 7-1 Owner, annuitant and beneficiary

4. Foreign Insurer — A foreign company is one which is authorized in this State but it's charter and principaloffice is located in another State. Massachusetts Mutual is also licensed in Connecticut to sell life insurance. Therefore, in Connecticut, Mass Mutual functions as a foreign company.

5. Alien Insurer — This is a company whose principal office and charter is located in another country. If analien insurer meets Massachusetts requirements it may also be authorized in this State. Nippon Life of Tokyo,Japan or Sun Financial Services (i.e., Sun Life) of Toronto, Canada would function in this State as an aliencompany.

6. Stock Company — This type of insurer is owned by its stockholders and issues nonparticipating policies. Stock insurers do not pay dividends to policyholders. Surplus at year's end is divided among shareholders.

7. Mutual Company — This form of company is owned by its policy owners. Mutual companies issueparticipating policies which may permit policyholders to receive dividends if they are payable. Mutualinsurers and stock insurers differ according to ownership.

MASSACHUSETTS REGULATIONS PERTINENT TO

LIFE INSURANCE ONLY

LIFE INSURANCE SOLICITATION AND SALES PRESENTATIONS — Chapter 175 and Regulation31describe State law with regard to the solicitation of life insurance policies. These regulations require insurersto provide a life insurance purchaser with information which will improve his or her ability to select the mostappropriate policy needed. This information will also improve and increase the understanding of the purchaser withregard to policy coverages, benefits and other features. Policy summaries may not be misleading or deceptive withregard to information describing the cost of insurance or the cost indices mentioned in the summary. Life insurancesolicitation regulations do not apply to annuities, group life credit life, variable life or any type of life insurance issuedin connection with a pension, profit or welfare plan.

According to Massachusetts law, the Commissioner shall regulate all advertising in this State with regard to the saleor marketing of life insurance, annuities, and endowments. Advertising includes any making, publishing,disseminating, circulating, or placing at the disposal of the public any notices, circulars, direct mailers, or other toolswhich describe the benefits of a policy or the services provided by an insurer and its licensees.

Policy Summary and Buyer’s Guide — The insurer is required to provide a Life Insurance Buyer's Guide toall buyers of life insurance. A policy summary and appropriate illustrations must also be provided to the buyer whichpresents an overview of the policy benefits and terms. Both of these items are to be provided prior to a licensee (i.e.,producer) accepting any initial premium or taking an application unless the policy being solicited includes a free lookperiod of at least 10 days. If the policy includes this free-look period, the buyer's guide and policy summary may beprovided at policy delivery. Any licensee, as well as an insurer, is required to comply with this regulation.

“Illustration” includes a presentation or depiction that includes non-guaranteed elements of a policy of lifeinsurance over a period of years and is one of the following three types: (1) a basic illustration which is a ledger orproposal used in the sale of a life insurance policy that demonstrates both guaranteed and non-guaranteed elements;(2) a supplemental illustration is an illustration furnished in addition to a basic illustration that may be presented in

NILA ©

L/H Licensing – Massachusetts Law

Chapter 20—16

Page 323: FORWARD - WordPress.com · Return of premium p 6-8 6.0 Annuities p 7-1 Annuity principles and concepts p 7-1 Accumulation period versus annuity period p 7-1 Owner, annuitant and beneficiary

a different format, but may only depict a scale of non-guaranteed elements; and (3) an in force illustration is anillustration furnished at any time after the policy depicted has been in force for one year or more. Another type ofillustration is a self-supporting illustration. This is an illustration of a policy for which it can be demonstrated thatfor all illustrated points in time on or after the fifteenth policy anniversary, the twentieth policy anniversary for last-survivor policies, or upon policy expiration if sooner, the accumulated value of all policy cash flows equals or exceedsthe total policy owner value available.

An illustration used in the sale of a life insurance policy must be clearly labeled “life insurance illustration” andcontain the following basic information: (1) name of insurer; (2) name and business address of the producer; (3) name,age and sex of the proposed insured; (4) the underwriting or rating classification upon which the illustration is based;(5) the generic name of the policy, the product name and the applicable form number; (6) initial death benefit; and(7) dividend option election or application of non-guaranteed elements, if applicable. When using an illustration inthe sale of a life insurance policy, neither an insurer nor its producer may: represent the policy as anything other thana life insurance policy; use or describe non-guaranteed elements in a manner that is misleading or has the capacity ortendency to mislead; state or imply that the payment or amount of non-guaranteed elements is guaranteed; providean applicant with an incomplete illustration; represent in any way that premium payments will not be required for eachyear of the policy in order to maintain the illustrated death benefits (unless that is the fact); use the term “vanish” or“vanishing premium” or a similar term that implies that the policy becomes paid-up, to describe a plan for using non-guaranteed elements to pay a portion of future premiums; or use an illustration that is not self-supporting. If an interestrate used to determine the illustrated non-guaranteed elements is shown, it must not be greater than the earned interestunderlying the disciplined current scale.

Cost Comparison — When applications are solicited by producers and when contracts are issued, insurers arerequired to provide policy owners with cost comparison information in order to compare the suitability of lifeinsurance products. There are many indexes used to compare prices including, but not limited to: (1) traditional netcost methods; (2) the interest adjusted net cost method; (3) the cash accumulation methods; (4) the comparativeinterest rate method; (5) the surrender cost index ; and (6) the net payment cost index. The surrender cost index isimportant to the consumer who places a high priority upon the growth of cash value in the policy. It aids in costcomparisons if the policyowner plans to surrender the policy for its cash value in ten or twenty years. The netpayment cost index is most useful for policyowners whose main concern is death protection and are not as concernedwith the build-up of cash value. It helps the consumer compare costs of death protection between policies that willbe held for ten or twenty years.

LIFE INSURANCE BENEFITS AND PROTECTION OF BENEFICIARIES — According to Chapter175 of Massachusetts law, if proceeds of a life insurance policy are retained by the insurer at the death of the insuredor upon maturity, no person or entity is entitled to encumber, assign or attach these proceeds. Further, no person orentity will be permitted to attach, encumber, commute, alienate or anticipate any portion of the proceeds for anyreason.

! Creditors of the beneficiary (or of the deceased) may not attach or encumber death proceeds of a life insurancepolicy left with the insurer at interest. Such proceeds will not be subject to the beneficiary's debts, contracts,engagements, judicial judgments, or other obligations. In other words, the beneficiary is protected from theclaims of his or her creditors. A spendthrift clause in a policy protects the beneficiary against creditors.

! The settlement option selected by the beneficiary will determine in what manner and for how long the policyproceeds will be protected. A beneficiary also possesses the right to sue an insurer on behalf of the deceasedinsured as well.

NILA ©

L/H Licensing – Massachusetts Law

Chapter 20—17

Page 324: FORWARD - WordPress.com · Return of premium p 6-8 6.0 Annuities p 7-1 Annuity principles and concepts p 7-1 Accumulation period versus annuity period p 7-1 Owner, annuitant and beneficiary

ASSIGNMENT AND INCIDENTS OF OWNERSHIP IN A GROUP POLICY – Chapter 175 of Massa-chusetts law reviews the concept of assignment and the sole incident of ownership which a covered employeepossesses under a group term life policy.

! Assignment involves the transfer of a right or rights under a life insurance policy from the owner of thecontract to another party.

! An absolute assignment involves transferring all rights under the policy to another party.! A collateral or conditional assignment involves the transferring of one or more of the rights under a policy

but not all. For example, if the policyholder engages in a conditional assignment using the cash value ascollateral in order to secure a loan from a bank (i.e., the lender or “assignor”), and the insured dies beforepaying back the loan, the death benefit will be divided between the assignor (i.e., the bank) and the primarybeneficiary according to their interests in the policy (i.e., the bank gets its money from the death benefit firstand the remainder goes to the beneficiary).

! The owner of a group life insurance policy is the employer. Therefore, the employer holds the ownershiprights in the contract with one exception. The covered employee possesses an “incident” of ownership in thepolicy which is the right to name and change his or her beneficiary.

! Therefore, a person (i.e., employee) whose life is insured under a group policy may assign any or all incidentsof ownership granted to him under the policy. The sole incident of ownership under the group plan is the rightto name (or change) the beneficiary.

! A person covered by a group policy also has the right to convert such coverage to an individual policy withinthe conversion period (i.e., 31 days) without proving insurability. If this right is exercised, the employee isresponsible for the payment of the premium.

REPLACEMENT OF LIFE INSURANCE POLICIES – Chapter 175, CMR 34.01 through .09 ofMassachusetts law define replacement of life insurance policies. Replacement is a legal activity as long as state lawis followed and the replacing insurer and producer satisfy strict replacement requirements.

! Replacement involves any transaction where new life insurance is to be purchased and it is known or shouldbe known to the licensee that any previously existing life insurance has been or is to be: (1) lapsed orsurrendered; (2) converted into paid-up insurance; (3) continued as extended term insurance; (4) or reissuedwith a reduction in cash values or other types of policy benefits.

! Licensees (i.e., producers) and insurers possess several responsibilities to an applicant and the other insurerwhose policy might be replaced. A signed disclosure notice or statement must be provided to the applicantdescribing possible benefits that could be lost if a current policy is replaced. This notice includes informationrecommending that the applicant request that the replacing insurer provide policy yield index information for5, 10 and 20 years. This notice must be signed by the producer and the applicant.

! The insurer whose policy is being replaced must be provided the opportunity to conserve the business. Thereplacing insurer must send to the existing insurer, within 7 working days, a written notice of the pendingreplacement and a copy of the policy summary. The conservation effort involves the attempt by the insurerwhose policy is being replaced to persuade their policyholder from surrendering the policy.

! Massachusetts law states that conservation efforts do not involve late payment reminders nor reinstatementoffers. All insurers issuing life insurance policies in this State are subject to replacement regulationsincluding direct response companies (i.e., direct mail). When a replacement occurs the policyowner must beprovided with a 20 day free-look period.

! Replacement regulations apply to all types of life insurance, annuities and endowments except for group lifeor credit life. It is not considered replacement when the replacing insurer and existing insurer are one in thesame.

NILA ©

L/H Licensing – Massachusetts Law

Chapter 20—18

Page 325: FORWARD - WordPress.com · Return of premium p 6-8 6.0 Annuities p 7-1 Annuity principles and concepts p 7-1 Accumulation period versus annuity period p 7-1 Owner, annuitant and beneficiary

REFUND OF PREPAID PREMIUMS – Massachusetts law Chapter 175 states that at the death of the insured,the proceeds payable (i.e., death benefit) by the insurer shall also include premiums paid (i.e., return premium) for anyperiod beyond the end of the policy month in which death occurred.

! For example, if Ted paid an annual premium of $600 on January 1st, and he died March 7th, his beneficiarywill receive the face amount of the policy plus the balance of the prepaid or unearned premium.

! To compute what this prepaid premium amount is, divide the annual premium ($600) by 12 months in a year. This results in a total of $50 per month. Since the insured died March 7th, go forward to the first day of thenext month and count the remaining months until the end of the year (April through December = nine months). Multiply nine months by the $50 per month figure for a total of $450. This will be returned to the beneficiarysince it was unearned. Ted paid premiums for April through December but since he died prior to this time,there is no need for the protection. Therefore, his beneficiary is refunded the unearned or prepaid premiumamount.

! If Ted pays his annual premium four months before he dies, his beneficiary will receive the face amount pluseight months of prepaid premium. This regulation applies to annual premium payments for all types of lifeinsurance EXCEPT single-premium life. The refund of prepaid premium will be paid to the beneficiaryunless it is owed or due some other person (i.e., an employer).

POLICY LOAN INTEREST RATE – Chapter 175 of Massachusetts law states that premiums must be paid bya policy owner for three full years before a policy loan may be secured. In other words, whole life policies mustprovide a cash value not more than three years after the policy is issued.

! Policy owners may borrow from their cash value of traditional whole life products, if one exists, at any timeafter this initial three year period.

! In Massachusetts, the interest rate charged by the insurer for policy loans may be a maximum fixed interestrate of 8%; or an adjustable (i.e., variable) rate may be included. The adjustable or variable rate will be basedupon some sort of current bond index such as Standard & Poor's or Moody's Corporate Bond Index, tripleAAA bonds, or Grade A bonds.

! Interest due is payable on the anniversary date of the policy. If loans (plus interest) are not repaid, the amountof the debt is deducted from the face amount when the insured dies. Outstanding loan amounts will not causea policy to be canceled.

! A delay clause permits an insurer to withhold a request policy loan for up to six months if it so desires. Generally, loan requests are granted within a few weeks. Insurers are generally not required to make policyloans less than $25.

! A variable or adjustable policy loan interest rate for an individual policyowner is determined once a year orevery 12 months.

! The basic adjustable or variable rate determinations are figured every three months based upon the bond indexat that time. This can be compared with a variable home mortgage rate based on the Treasury Bill rate. TheT-Bill rates are determined each week. The bond index rate is determined every three months. The policyloan interest rate for the individual policyowner and the mortgage rate for the individual homeowner arefigured (on an anniversary date) once a year.

GROUP LIFE INSURANCE (Chapter 175) – State law defines group life to be that type of insurance coveringnot less than 10 employees at the date of issue and the policy is issued to an employer which covers all eligibleemployees. No employee shall be excluded by the policy if he or she has been employed by the employer/policyownerfor one year or more. No less than 75% of the eligible employees shall be insured. If employees of the group passa medical exam, not less than 40% may be covered. If coverage is provided in connection with a loan or debt, no morethan two persons may be covered in connection with the credit transaction. Group plans issued in this state may be

NILA ©

L/H Licensing – Massachusetts Law

Chapter 20—19

Page 326: FORWARD - WordPress.com · Return of premium p 6-8 6.0 Annuities p 7-1 Annuity principles and concepts p 7-1 Accumulation period versus annuity period p 7-1 Owner, annuitant and beneficiary

contributory (i.e., employee and employer share the premium) or non-contributory (i.e., employer pays all thepremium).

Group life plans shall not be issued until they have been on file with the Commissioner’s office for 30 days. If thepolicy form is rejected by the Commissioner, the Division of Insurance must provide the reason for such action inwriting to the insurer. Policy forms will be approved if they, at least, include the following provisions: (1) two-yearincontestable clause; (2) an entire contract provision, (3) misstatement of age provision; (4) a certificate of coverage,(5) an enrollment period; (6) a 31-day grace period; and (7) a beneficiary provision.

Group life coverage issued in this State must also provide a conversion period in order to allow an employee to takehis or her life insurance coverage with them as long as the group coverage is converted, without requiring insurability, to an individual plan of life insurance. The employee / certificateholder shall be notified of such privilege and itsduration within fifteen days after the happening of the event (e.g., termination of employment), provided that if suchnotice be given more than 15 days but less than ninety days after the happening of such event, the time allowed forthe exercise of such privilege of conversion shall be extended for fifteen days after the giving of such notice. If suchnotice be not given within ninety days after the happening of such event, the time allowed for the exercise of suchconversion privilege shall expire at the end of such ninety days.

In the event that there is no beneficiary when an insured dies, the insurer has the right to pay the policy proceeds toan executor of the deceased’s estate or to other relatives of the deceased employee. In addition, the insurer also hasthe right to pay from the death benefit up to $250 to any person who has incurred any funeral or final medical expensesconnected with the insured/ employee’s death (facility of payment provision).

VARIABLE LIFE INSURANCE — Any insurer issuing variable life insurance shall establish one or moreseparate accounts. Assets in the separate account shall be valued at least as often as variable benefits are determinedbut in any event at least monthly. No insurer shall knowingly employ or contract with any person who within the lastten years: (1) has been convicted of a felony or misdemeanor arising out of conduct involving embezzlement,fraudulent conversion or misappropriation of funds or securities; (2) has been found by any State regulatory officialto have violated any State insurance law involving fraud, deceit or knowing misrepresentation, or (3) has been foundby federal authorities to have engaged in fraud, deceit or knowing misrepresentation involving a federal law. Allpersons with access to the cash, securities or other assets of a separate account shall be covered under a surety bondin an amount no less than $ 100,000.

MASSACHUSETTSREGULATIONS PERTINENT TO

ACCIDENT & HEALTH ONLY

REDUCTION IN BENEFITS BY SOCIAL SECURITY OFFSET PROHIBITED (CHAPTER 175) According to Massachusetts state law, once an individual begins to receive monthly income benefits from a groupdisability income policy (i.e., LTD), the amount of the benefit will not be decreased or reduced even if there aremodifications to the Social Security program. For example, if Bill begins to receive $800 per month from his groupdisability income policy and six months later Social Security alters its benefit structure, Bill's income benefit will notbe affected since he was already receiving benefits prior to the change in Social Security.

NILA ©

L/H Licensing – Massachusetts Law

Chapter 20—20

Page 327: FORWARD - WordPress.com · Return of premium p 6-8 6.0 Annuities p 7-1 Annuity principles and concepts p 7-1 Accumulation period versus annuity period p 7-1 Owner, annuitant and beneficiary

ADVERTISEMENTS OF ACCIDENT AND SICKNESS INSURANCE (REGULATION 71) — TheCommissioner of Insurance regulates all advertising in this state. However, this does not mean that the Commissioneror Division of Insurance approves all individual advertisements. They have neither the time nor resources to reviewevery individual advertisement. All advertisements are subject to state regulations in order to protect the public andmake sure that they are true representations and not misleading. Relevant advertising information is as follows:

! Advertisements may not be misleading! Testimonials must be genuine. They promote an insurer and its products.! Advertisements may include statistical information but only if it is accurate and relevant.! All insurers must include their trade name in all advertisements and appropriately identify themselves in such

advertisements. They must also keep a copy of all advertisements for at least four years.

MINIMUM STANDARDS OF ACCIDENT AND HEALTH POLICIES (CHAPTER 175):

! All A&H policies must contain an outline of coverage describing the policy’s benefits. This outline ofcoverage must also include any exclusions or limitations contained in the policy.

! Any exclusions, limitations, and reductions in coverage must also appear on the face (i.e., first) page of thepolicy.

! If a medical exam is required in order for coverage to be issued, this fact must be listed in the advertisement. ! If an impairment rider is added to an A&H policy, an insured must sign a form stating that the rider has been

explained to them by the agent or the insurer.

HEALTH MAINTENANCE ORGANIZATIONS (Chapter 176-G) — An HMO is a prepaid group practice planoriginally designed to reduce the high cost of medical care and services. Generally, a covered member (i.e., enrolleeor subscriber) must go to the HMO facility to receive treatment. In addition, the covered enrollee may seek treatmentfrom only the physicians whose names appear on the HMOs list of doctors.

! An HMO covers regular medical services for injuries or illnesses. Additional medical services provided byHMOs include but are not limited to: mental, nervous and emotional disorders; alcoholism and drug treatment; pregnancy and infant care; cardiac rehabilitation; cytological screening and mammogram; nursing care;outpatient treatment; lab fees; speech, physical and occupational therapy; and prenatal and post natal care. An HMO does not cover administrative expenses (i.e., this is not a medical expense). Many HMOs do notprovide chiropractic care (although some do).

! HMOs that operate in this State must provide coverage for dependents including occupational, physical andspeech therapy and psychological counseling. Dependent children are covered from birth (i.e., newborns). In addition, HMOs must provide for continuation of group coverage if a covered member is laid off due toplant closing or partial closing. Additional HMO regulations include but are not limited to: (1) HMOs maydisclose patient information to law enforcement authorities if there is reason to believe the insured incommitting fraud or engaging in other misconduct. An HMO may also disclose information if it has to dowith coordination of benefits with another health plan, subrogation, peer review or utilization review; and (2)HMOs must also cover bone marrow transplants, glucose treatment (i.e., diabetes) and hospice related carefor a person during his or her final six months (or less).

PREFERRED PROVIDER ORGANIZATIONS (Chapter 176) — This is an alternative to an HMO whichcovers medical services of members. One of the basic differences between an HMO and a PPO is that a coveredmember may choose his or her own physician under a PPO. In a PPO, the organization contracts with doctors whowill provide medical services. The contract between the doctor and the organization is known as a Preferred ProviderArrangement (PPA). These contracts (PPAs) must conform to state law and be approved by the Division of Insurance

NILA ©

L/H Licensing – Massachusetts Law

Chapter 20—21

Page 328: FORWARD - WordPress.com · Return of premium p 6-8 6.0 Annuities p 7-1 Annuity principles and concepts p 7-1 Accumulation period versus annuity period p 7-1 Owner, annuitant and beneficiary

and the Commissioner. According to State regulations, PPOs must meet standards for quality of health care,controlling health care costs, assuring reasonable levels of access to health care services and proper utilization ofhealth care services. PPOs, like other providers, must maintain adequate records of all treatment and transactions.

CONSOLIDATED OMNIBUS BUDGET RECONCILIATION ACT (COBRA) — Chapter 175, Section110 of State law describes COBRA. This federal act requires compliance by all employers with twenty (20) or moreemployees and ensures that a laid-off worker may continue health insurance coverage formerly provided by anemployer for up to 18 months after the lay-off. The former employee is responsible for the payment of the premium.The beneficiary of a deceased employee will be allowed to continue health coverage for up to 36 months after thedeath of the employee. The beneficiary is also responsible for paying the premium.

All group plans must include a provision providing that, in the event that the insured person leaves the group coveredby such insurance, said person shall remain insured under such policy for a period of 31 days thereafter unless,during such period, he or she shall otherwise be entitled to similar benefits. In addition, every policy which is issuedor subsequently renewed by agreement between the insurer and policyholder shall include coverage such that, in theevent that the insured person leaves the group covered by such insurance because his or her employment is terminateddue to a plant closing or covered partial closing, said person shall remain insured under such policy for a period onninety days thereafter unless during such period, he shall otherwise be entitled to similar benefits.

LONG-TERM CARE (REGULATION 65) ) The purpose of Massachusetts long-term care regulations is topromote the sale and benefits of long-term care policies to citizens of the Commonwealth. This type of policy is onewhich is designed to provide benefits for not less than 12 months. The following regulatory information applies tonon-group long term care insurance. Standard coverage provided by long term care policies include custodial care,intermediate care, home health care and skilled nursing care. Some policies also provide coverage for adult day care,which includes other nursing services, occupational, speech and physical therapy and social or recreational activities. Activities of daily living (ADL) are benefit triggers such as dressing, grooming, personal hygiene, eating, bathing,accident prevention, walking, continence, taking medication or use of special aids may also be covered. MassachusettsLTC law states that individual policies that are not intended to be federally qualified may not include benefit eligibilitystandards that are more stringent than a requirement that the insured be unable to perform at least two activities ofdaily living due to a loss of functional capacity or severe cognitive impairment. Other types of coverage that areavailable include: adult foster care covering room, board or personal care; care management which requires priorapproval of covered services; chore care such as washing, cleaning or maintenance which allows a senior citizen toremain in his or her home; or homemaker care.

! Respite care is also available from some insurers as well as special day care. Long term care does not providecoverage for hospital confinement.

! A managed long term care (LTC) plan arranges for delivery of LTC services through a LTC delivery system.

! Massachusetts law stipulates that all LTC policies issued in this State must include skilled nursing, homehealth, intermediate and custodial care. Skilled nursing, intermediate nursing and custodial care daily benefitsmay not be less than $50 per day. Home health care benefits provided may not be less than 50% of the dailybenefit provided for custodial or skilled nursing care. If a policy provides for lifetime benefits they may notbe for less than 730 days nor less than a dollar amount of $36,000.

! Exclusions allowed in a LTC policy include pre-existing conditions; illness arising from war, commissionof a felony, intentionally self-inflicted injuries, attempted suicide or aviation related causes unless a fare-paying passenger; illness contracted while in the armed services; treatment in a government facility; anybenefits if paid by Medicare (i.e., hospital confinement, etc.), Medicaid, Workers’ Compensation or no-fault

NILA ©

L/H Licensing – Massachusetts Law

Chapter 20—22

Page 329: FORWARD - WordPress.com · Return of premium p 6-8 6.0 Annuities p 7-1 Annuity principles and concepts p 7-1 Accumulation period versus annuity period p 7-1 Owner, annuitant and beneficiary

auto insurance; or services provided for alcohol or drug rehabilitation. Post claims underwriting is notpermitted. In other words, acceptability must be determined before the policy is issued.

! Limitations are allowed for mental and nervous disorders, Alzheimer's disease and alcoholism. Minimumbenefits shall not be for less than 365 days or one year. LTC policies must include a guaranteed renewableprovision. Direct response insurers (i.e., direct mail) and others selling LTC must include in the policy a free-look period of at least 30 days. The free-look period is also referred to as the right to return provision. Elimination periods may not be for more than 365 days. Individual policies may offer deductibles in lieu ofeliminations periods but not both. Lifetime maximum benefit periods may not cover fewer than 730 daysbeyond the policy’s elimination period. Preexisting conditions limitations may not be for more than sixmonths and must be identified on the front or face page of the policy. If a LTC policy is to be terminated bythe insurer for a legitimate reason it must give the policyholder 30 days written notice of termination. Thepolicyholder also has the right to designate additional persons to receive such notice to protect against anunintentional lapse of the policy. A reinstatement provision shall also be included in the policy. This allowsreinstatement if requested within five months after termination and shall allow for the collection of anyoverdue premium by the insurer.

! When LTC policies are delivered, they must be accompanied by a disclosure statement. Any riders orendorsements added to such policies must be accepted or rejected in writing by the policy owner. In addition,LTC policies must include a statement on its face page that "This is a Limited Policy." The policy's exclusionand limitations must be listed separately and prominently. If a replacement of LTC policies is to take place,a notice regarding replacement must be provided to the insured to inform him or her of any loss or reductionin benefits that will be experienced because of the replacement. Direct response insurers must provide a noticeregarding replacement to a prospective insured at the time it sends an application for coverage, even if it isnot proposing a replacement. Whenever a LTC policy is replaced, any preexisting condition limitation orprovision in the new plan will be waived.

! To prevent against an unintentional lapse, the insurer issuing this type of policy must receive from theapplicant a written designation of at least one person, in addition to the applicant, who is to receive notice oflapse or termination of the policy due to nonpayment of premium (or a written waiver stating that the applicantdoes not desire to designate another to receive notice). If a LTC policy is to be terminated an insurer mustprovide ate least thirty days written notice of such termination.

MEDICARE SUPPLEMENT / MEDIGAP (REGULATION 71 and Chapter 176) ) A Medicare Supplementpolicy, also referred to as Medigap insurance, pays benefits for medical services that are not covered by Medicare. Medicare includes deductibles and coinsurance amounts which must be satisfied by the insured. These "gaps" incoverage will be covered if the insured owns a Medigap policy. Massachusetts law stipulates the following withregard to these plans:

! No Medicare Supplement policy shall be issued which contains a waiting period or pre-existing conditionlimitation or exclusion. However, benefits provided under Medicare are excluded by Medigap policies. Anylimitations included must be prominently identified. Most Medigap policies generally exclude coverage foreyeglasses and hearing aids.

! Medicare Supplement policies must be guaranteed renewable. This must be captioned or listed on the firstpage of the policy. These policies shall not be sold to anyone not covered by Medicare Parts A and B. Again,no Medicare Supplement policy shall contain benefits which duplicate Medicare benefits. These policiesgenerally include a free-look or “right to return” period of 30 days.

NILA ©

L/H Licensing – Massachusetts Law

Chapter 20—23

Page 330: FORWARD - WordPress.com · Return of premium p 6-8 6.0 Annuities p 7-1 Annuity principles and concepts p 7-1 Accumulation period versus annuity period p 7-1 Owner, annuitant and beneficiary

! Individual case management or mail service prescription drug programs may be offered by insurers as longas they have been approved by the Commissioner of Insurance. All policies issued in this State must includean open enrollment period. This period is February 1 st through March 31 st each year.

! In cases where replacement of a Medicare Supplement policy is involved, a notice regarding replacementmust be provided to an insured advising him or her of any reduction of benefits that might arise if the policyis replaced. When replacement occurs, an insured must be wary of any loss of benefits, increase in cost andinsurability questions (i.e., can I qualify?). A Medigap policy may only be sold to a person when "combined"with the applicant's or insured's health coverage already in force, and it would insure no more than 100% ofthe applicant's or insured's actual medical expenses covered under the combined policies.

! All advertisements for Medicare Supplement insurance must be submitted to the Commissioner for reviewat least 15 days prior to its use by the insurer. First year commissions to a sales representative may not exceed200% of the commission or other compensation paid for selling or servicing the policy in the second year. In other words, excessive commissions are not permitted which would unduly influence an producer. Thecommission provided in subsequent years (i.e., renewal) must be the same as that provided in the second year.Insurers selling Medigap insurance must possess a loss ratio of 65%. This means that an insurer must pay outin claims, 65 cents of every dollar collected in premiums.

! According to Massachusetts law, Medigap insurance shall cover full semi-private benefits for hospitalizationfor the first 365 days per benefit period, to the extent not covered by Medicare. Services in a skilled nursingfacility certified by Medicare during the first 100 days are covered to the extent not covered by Medicare ($10per day is covered for the 101st through 365th day). Benefits for services in all skilled nursing facilities shallbe available for a combined maximum of 365 days per benefit period.

Medicare Part B eligible expenses are covered by Medigap insurance to the extent not covered by Medicare. Medical expenses covered include outpatient expenses such as: hospital diagnostic services; initial visit foremergency treatment within 72 hours of the causative accident; surgical services; x-ray therapy; maintenancedialysis; services necessary to continue active treatment which was begun during hospitalization of at least3 days and which is rendered within 100 days following discharge (excluding a mental disorder); and initialvisit for emergency treatment of sudden and serious illness requiring immediate treatment. Medical servicesprovided by psychologists, chiropractors, podiatrists, nursing midwives, dentists and physicians to the extentcharges are reasonable are covered. Regulation 71 states that duplicate coverage is expensive andunnecessary. Applicants must disclose if they have multiple policies.

! Medigap also covers approved charges for self-dialysis training by a renal dialysis facility and for equipmentand supplies provided by such a facility for use in the home. Physiotherapist services are also covered up to$100 less any Medicare payment. Coverage is also provided for those traveling outside the U.S. for the sameservices and the same level of payment as is provided within this country.

COORDINATION OF BENEFITS (Chapter 175 and 176, REGULATION 38) ) The purpose ofMassachusetts regulations in this area is to establish an order in which plans pay their claims when a person iscovered by more than one plan. A plan that does not contain a coordination of benefits (COB) provision may nottake benefits of another plan into account when determining its benefits. Any policy or plan containing a COBprovision must provide information to persons covered under the plan about the rules used to determine primary andsecondary coverage and to determinate calculable expenses.

! COB rules state that the primary plan must pay or provide its benefits as if the secondary plan or plans didnot exist. A plan or policy that does not include a COB provision is not permitted to take the benefits of

NILA ©

L/H Licensing – Massachusetts Law

Chapter 20—24

Page 331: FORWARD - WordPress.com · Return of premium p 6-8 6.0 Annuities p 7-1 Annuity principles and concepts p 7-1 Accumulation period versus annuity period p 7-1 Owner, annuitant and beneficiary

another plan into account when determining benefits to be paid, EXCEPT for coverage that is designed tosupplement a part of a basic package of benefits. This supplementary coverage shall be excess to any otherparts of the plan. A policy may take benefits of another plan into account only when it is secondary to thatother plan.

! The benefits of the policy which covers the person as an employee member or subscriber are determinedbefore those of the plan which covers the person as a dependent.

! COB and Dependents ) If two or more plans cover a dependent child whose parents are divorced orseparated, the plan of the parent who has custody is primary. If the dependent is covered by two or more plansand the parents are not divorced or separated, the plan of the parent whose birthday falls earlier in the yearis primary. If both parents have the same birthday, the plan that has covered one of them the longest isprimary.

! The secondary plan may reduce its benefits so that the total benefits paid or provided by all plans during aclaim determination period are not more than the total allowable expenses.

SMALL GROUP HEALTH PLANS (CHAPTER 176 AND REGULATION 66) ) Massachusetts lawregarding small group health plans applies to all health plans issued to any eligible small business whether issueddirectly by a carrier, through an association or intermediary.

! SMALL GROUP HEALTH PLAN DEFINITIONS ) The following are relevant definitions pertainingto State regulations covering small group health plans.

1. Eligible Employee ) Any employee who works on a full-time basis with a normal work week of 30or more hours and includes an owner, sole proprietor or a partnership provided they are included asan employee under the plans (does not include temporary or substitute employees). The employeemust have been hired to work for a period of not less than 5 months.

2. Eligible Dependent ) This includes the spouse or children of an eligible employee.

3. Eligible Business ) A business that on at least 50% of its working days during the preceding yearemployed from one to not more than 50 eligible employees (the majority of whom worked inMassachusetts), including owners and those who are self-employed.

4. Intermediary ) This is a trade association, chamber of commerce or other organization formed forpurposes other than obtaining insurance and which offers as a service to its members the option ofpurchasing a health insurance plan.

! Limited Open Enrollment — Each carrier electing to schedule enrollment periods under Massachusettslaw shall file with its initial actuarial opinion a statement describing the beginning and ending dates for thecarrier's open enrollment periods (i.e., when insurability does not have to be proven). For any subsequentopen enrollment period, the carrier shall file with the Commissioner 60 days prior to the beginning if any openenrollment period notice of the dates of the open enrollment period. If a carrier does not schedule openenrollment periods pursuant to Massachusetts law, then the carrier must maintain continuous open enrollmentfor eligible small businesses.

! Insurance carriers shall not be required to issue a health benefit plan to an eligible small business if the carriercan demonstrate to the satisfaction of the Commissioner that within the prior twelve months the eligible small

NILA ©

L/H Licensing – Massachusetts Law

Chapter 20—25

Page 332: FORWARD - WordPress.com · Return of premium p 6-8 6.0 Annuities p 7-1 Annuity principles and concepts p 7-1 Accumulation period versus annuity period p 7-1 Owner, annuitant and beneficiary

business has: (1) made at least three or more late payments in a 12 month period; (2) committed fraud,misrepresented the eligibility of an employee, or misrepresented information necessary to determine groupsize, group participation rate, or the group premium rate; (3) failed to comply in a material manner with ahealth benefit plan provision, including carrier requirements for employer group premium contributions; or(4) been covered by three or more health benefit plans within the same class of business during the four yearsimmediately preceding the date of application for coverage.

! Any insurance carrier who denies coverage to an eligible small business under the provisions of Massachusettslaw shall: (1) provide to the small business, in writing, the specific reason(s) for the denial of coverage; and(2) make available to the Commissioner, upon request, the documentation for such denial.

! Renewability — Every health benefit plan shall be renewable with respect to all eligible persons and eligibledependents at the option of the eligible small business except as provided by Massachusetts law. A carriershall not be required to renew the health benefit plan of an eligible small business if the small business: (1)has not paid the required premiums; (2) has committed fraud, misrepresented whether or not a person is aneligible employee, or misrepresented information necessary to determine the size of a group, the participationrate of a group, or the premium rate for a group; (3) failed to comply in a material manner with health benefitplan provisions including carrier requirements regarding employer contributions to group premiums; (4) fails,at the time of renewal, to satisfy the definition of an eligible small business; or (6) is not actively engaged inbusiness. The carrier must provide 60 days notice of non-renewal, if applicable.

! Premiums — The insurance carrier may determine a particular eligible small business premium to be chargedby basing the premium on one or more of the following case characteristics: (1) age; (2) sex; (3) industry;(4) group size. A carrier may establish participation-rate rate adjustments for any health benefit plan or plansfor any ranges of participation rates below the following minimum participation requirements: (1) for groupsof five or fewer - not to exceed 100%; or (2) for groups of six or more - not to exceed 75%. The participation-rate rate adjustments must be based upon sound analysis of the differences in the experience of eligible smallbusinesses with different participation rates.

! Group health insurance for larger or small groups may be purchased by employers for their employees; bya labor union for its members; or by a state, city or municipality for its employees. Group health insurancemay generally not be purchased by civic or cultural groups.

OTHER COVERAGES AND STATE MANDATED PROVISIONS ) The following information describescoverages that are mandated by the State of Massachusetts.

! AIDS (Chapter 175 and 176) ) According to Massachusetts law, AIDS related information must be treatedas highly confidential. No AIDS-related information must be communicated by an insurer, producer oremployee without the written authorization of the individual to whom such information relates. No insurermay request or require that a person submit to an AIDS-related test unless prior written informed consentof the individual has been obtained. An insurer may request or require an individual to submit to an AIDS-related test only if it is not unfairly discriminatory. Insurers and producers who learn of any violations ofthese requirements must take immediate and corrective action. Insurers and producers must also maintainrecords of such violations and the corrective actions taken which shall be available to the Division ofInsurance upon request. An insurer cannot decline coverage based on the presence of symptoms. The insurermay only decline an application based on the presence of infection as demonstrated by positive blood testresults. In addition, no questions on an application may inquire about the applicant's marital status, livingarrangements or sexual orientation.

NILA ©

L/H Licensing – Massachusetts Law

Chapter 20—26

Page 333: FORWARD - WordPress.com · Return of premium p 6-8 6.0 Annuities p 7-1 Annuity principles and concepts p 7-1 Accumulation period versus annuity period p 7-1 Owner, annuitant and beneficiary

INSURER NOTIFICATION ) The insurer shall provide written notification to the individual including: (1) the purpose of the AIDS test and if it is a condition of obtaining insurance; (2) information concerning theeffect of the test results on the approval of the application; and (3) the nature of the test, the meaning of theresults and a description of the confidentiality standards identifying who may have access to test results. There must also be included a statement concerning the availability of free and anonymous AIDS testing inthis State as well as the fact that counseling is also available. The insurer must always implementreasonable safeguards to protect the privacy of AIDS-related information.

TEST RESULTS ) If test results are positive, the insurer must notify the individual within 45 days of thetest. Notification may be made through a physician or directly to the individual by certified letter.

APPLICATIONS ) Insurers or producers are prohibited from requesting any information related to thesexual orientation of any applicant for insurance, proposed insured, policyowner or beneficiary. An insurermay ask if an applicant has ever tested positive for AIDS or HIV-related conditions.

! DEPENDENT CHILD AGE LIMIT — Chapter 176 states that newborn children will be covered byindividual or group health plans from the moment of birth for up to thirty-one days for any condition. Ifcoverage is to be continued beyond this period, the proper premium must be paid. Proof of birth must beprovided to the insurer such as a letter from a hospital or an attending physician. Providing a birth certificateis not required since such forms are not produced until weeks or months following birth.

State law mandates that preventive care services must be provided up until the age of six. Coverage must alsobe provided for a newly adopted child from the moment the adoption is finalized. Such policies are alsorequired to provide coverage for dependent children up to age 19 and for full-time students (under a parentsplan) until age 25. Disabled adult children, physically handicapped or mentally challenged children may becovered by a parents plan as long as the insurer is notified of the specific condition, verification is providedand the proper premium is paid. Coverage for newborns must also include hearing screening as well.

! MASSACHUSETTS COMPREHENSIVE HEALTH CARE PLAN — MassHealth pays for healthcare for certain low and medium income people residing in Massachusetts. Those eligible who are under age65 and who are not living in a nursing home or other long-term care facility include: (1) families withchildren under age 19; (2) children under age 19; (3) pregnant women; (4) disabled persons; (5) adults whowork for a qualified employer; (6) people out of work for a long time; or (7) people who are HIV positive. The plan also provides benefits for certain persons age 65 or older if they are parents or caretaker relativesof children under age 19, or are disabled and working forty hours or more per month. Proof of one’s monthlyincome before taxes and deductions for every person in your family must be provided in order to be eligible. The Massachusetts child health care plan is part of MassHealth.

1. COVERAGES AND LIMITS — Covered health care services include: (1) in-patient hospitalservices; (2) out-patient services such as hospitals, clinics, doctors, dentists, family planning andhome health care; (3) medical services such as lab tests, x-rays, therapies, pharmacy services, dentalservices, eyeglasses, hearing aids, medical equipment and supplies, adult day care and adult fostercare; (4) mental health and substance abuse services (in-patient and out-patient); (5) well-childscreening (for children under age 21) including medical, vision, dental and hearing tests as well asshots, prescription and non-prescription drugs; (6) for disabled adults who also receive benefits fromMedicare Part B (premiums, coinsurance and deductibles); and (7) transportation services. Ifeligible, coverage begins 10 calendar days before the date the Division of Insurance receives themedical benefits request, if all necessary information is received within 60 days. Exclusions ofcoverage are similar to those found in most health care plans.

NILA ©

L/H Licensing – Massachusetts Law

Chapter 20—27

Page 334: FORWARD - WordPress.com · Return of premium p 6-8 6.0 Annuities p 7-1 Annuity principles and concepts p 7-1 Accumulation period versus annuity period p 7-1 Owner, annuitant and beneficiary

MASSACHUSETTS INDIVIDUAL MANDATE FOR MINIMUM CREDITABLE COVERAGE — Thepurpose of this law was to establish criteria for the lowest threshold health benefit plan that an individual mustpurchase in order to satisfy the legal requirement that a Massachusetts resident have health insurance (that constitutesminimum creditable coverage) as to avoid paying a penalty to the Department of Revenue. Benefit standards havebeen determined by the Commonwealth Health Insurance Connector Authority. Effective 1 / 1 / 09, a health benefitplan must include core benefits and a broad range of medical benefits including: (1) preventive and primary; (2)emergency services; (3) hospitalization; (4) ambulatory patient services; (5) prescription drugs; and (6) mental healthand substance abuse services. In addition, no health benefit plan may include a deductible exceeding $2,000 for anindividual or $4,000 for a family. No policy may include a prescription drug deductible exceeding $250 for anindividual or $500 for a family. Plans may include in network and out of network benefit levels and may not imposean overall annual maximum benefit limitation for services or dollar amounts (i.e., no cap on total benefits for aparticular illness or for a single year). However, benefit limitations may be included for substance abuse treatment,physical therapy, inpatient rehabilitation care services and durable medical equipment.

NILA ©

L/H Licensing – Massachusetts Law

Chapter 20—28

Page 335: FORWARD - WordPress.com · Return of premium p 6-8 6.0 Annuities p 7-1 Annuity principles and concepts p 7-1 Accumulation period versus annuity period p 7-1 Owner, annuitant and beneficiary

MASSACHUSETTSLIFE, ACCIDENT & HEALTHSAMPLE LAW EXAMINATION

1. A licensed producer in Massachusetts represents which of the following?

A. The insured C. The Commonwealth of Massachusetts

B. The customer D. The insurer or the insured

2. An adviser is also known as:

A. A broker C. An agent

B. A consultant D. An attorney

3. An insurer having its charter and principal office in Massachusetts is known as a:

A. Foreign company C. Domestic company

B. Alien company D. Non admitted company

4. An individual who acts as a producer during a license suspension will be subject to:

A. A fine of not less than $10 nor more than $500

B. A fine of not less than $20 nor more than $500

C. A fine of not less than $50 nor more than $500

D. A fine of not less than $20 nor more than $1,000

5. If a producer has engaged in any criminal practices and the Commissioner becomes aware of such action, such illegalities

must be reported to:

A. The Governor C. The Director of Consumer Affairs

B. The Labor Secretary D. The Attorney General

6. A producer collecting premiums from clients for his own personal use best describes:

A. Misrepresentation C. Rebating

B. Twisting D. Larceny

7. A producer who claims to be an adviser has engaged in:

A. Twisting C. Impersonation

B. Rebating D. Larceny

8. Which of the following is an unfair or deceptive practice?

A. Sharing commissions C. False advertising

B. Receiving compensation D. False start

9. Which of the following insurers may pay dividends to policyholders?

A. Domestic company C. Mutual company

B. Stock company D. Alien company

NILA ©

L/H Licensing – Massachusetts Law

Chapter 20—29

Page 336: FORWARD - WordPress.com · Return of premium p 6-8 6.0 Annuities p 7-1 Annuity principles and concepts p 7-1 Accumulation period versus annuity period p 7-1 Owner, annuitant and beneficiary

10. A stock and mutual insurer differ according to:

A. Profits C. Ownership

B. Members D. Articles of incorporation

11. A producer who unfairly compares policies to induce an acquaintance to lapse his current policy and purchase another

has engaged in:

A. Twisting C. Acquisitions

B. Rebating D. Churning

12. An adviser license, once issued, is good for:

A. 1 year C. 3 years

B. 2 years D. 4 years

13. A producer who supplies a client with a kick-back of commissions in return for the purchase of the policy best describes:

A. Twisting C. Defamation

B. Rebating D. Larceny

14. Failing to pay an insurer the premiums owed to it describes:

A. Larceny C. Twisting

B. Rebating D. Defamation

15. A producer who combines premiums collected with personal funds has engaged in:

A. Misappropriation C. Commingling

B. Rebating D. Misapplication

16. An insurer licensed in this State but incorporated in Vermont best describes a(n):

A. Domestic company C. Alien company

B. Foreign company D. Stock company

17. If an insurer cancels a producer appointment it must notify which of the following?

A. Governor C. Division of Insurance

B. Attorney General D. Registry of Licenses

18. A public adjuster represents:

A. An insurer C. A policyholder

B. A broker D. A beneficiary

19. Which of the following attempts to determine the amount of an auto loss to assist in the settling of a claim?

A. Adjuster C. Agent

B. Appraiser D. Claims manager

NILA ©

L/H Licensing – Massachusetts Law

Chapter 20—30

Page 337: FORWARD - WordPress.com · Return of premium p 6-8 6.0 Annuities p 7-1 Annuity principles and concepts p 7-1 Accumulation period versus annuity period p 7-1 Owner, annuitant and beneficiary

20. A producer must complete a Continuing Education requirement within the first three years after being issued a license.

This initial requirement is:

A. 30 credit hours C. 60 credit hours

B. 45 credit hours D. 105 credit hours

21. Dan is a producer licensed in this State. To make sure that his largest commercial insurance account renews their policy,

he makes a gift of a pair of diamond earrings to the owner's wife. Therefore, Dan has engaged in:

A. Twisting C. Defamation

B. Misrepresentation D. Rebating

22. Two agencies are competing for a new client. Agency A mentions to the prospective client that all of the companies with

which they do business are licensed in all U.S. States. It also mentions that Agency B may not be dependable since they

are only licensed in forty-two States. Therefore, Agency A may have engaged in:

A. Perjury C. Defamation

B. Fraud D. Larceny

23. Which of the following would not be considered rebating?

A. A producer offers to pay a portion of the insured's premium

B. A producer provides a television set to a current customer

C. A producer offers season tickets to a prospective client

D. A producer offers an advantageous premium payment plan to an insured

24. An insurer which does not hold a certificate of authority and is not permitted to conduct insurance business in this State,

best describes?

A. An authorized insurer C. An admitted insurer

B. An unauthorized insurer D. A revocable insurer

25. An individual who illegally compares an existing policy with a new one he or she is trying to solicit in order to induce the

policyholder to lapse the current one has engaged in:

A. Rebating C. Defamation

B. Twisting D. Fraud

26. John is licensed as a resident producer in New Hampshire. If he desires to be legally licensed to solicit insurance in this

State, which of the following will issue him a nonresident license?

A. New Hampshire Department of Insurance

B. The Department of Labor

C. The Massachusetts Division of Insurance

D. The National Association of Insurance Commissioners (NAIC)

27. The maximum monetary penalty that may be assessed an individual engaging in perjury is:

A. $100 C. $1,000

B. $500 D. $10,000

28. A producer license once issued is good for three years and expires on the holder’s:

A. Birth date C. Date of incorporation

B. Anniversary date D. Interogatory date

NILA ©

L/H Licensing – Massachusetts Law

Chapter 20—31

Page 338: FORWARD - WordPress.com · Return of premium p 6-8 6.0 Annuities p 7-1 Annuity principles and concepts p 7-1 Accumulation period versus annuity period p 7-1 Owner, annuitant and beneficiary

29. After the initial three years of licensure, a producer must complete how many hours of Continuing Education:

A. 10 hours every three years C. 45 hours every three years

B. 30 hours every three years D. 60 hours every three years

30. Mr. Fox paid his $1,800 annual life insurance premium on March 1, 1999. He then died in an auto accident on June 2,

1999. Which of the following amounts will the beneficiary receive?

A. The face amount of the policy

B. The policy's cash value

C. The death benefit plus $800 of prepaid premium

D. The death benefit plus $1,200 of unearned premium

31. Which of the following are powers or duties of the Commissioner of Insurance?

A. Examines domestic companies once every two years

B. Prosecutes producers who break insurance laws

C. Sets life insurance premium rates.

D. Computes the reserves of life insurance

32. A licensee who does not comply with State Continuing Education requirements will be subject to which of the following?

A. Permanent license revocation

B. License suspension until compliance

C. A fine of not less than $20 nor more than $500

D. Imprisonment for not more than six months

33. A physician who falsifies an insurance claim form may be subject to which of the following penalties?

A. Revocation of license to practice medicine

B. A fine for misrepresentation

C. A prison sentence for fraud

D. A fine or prison sentence for perjury

34. Which of the following is considered a form of twisting?

A. Offering anything of value not listed in the policy

B. An illegal inducement to surrender a policy

C. Failing to provide claim forms

D. Advertising oneself as a producer when in fact one is not

35. The individual accepting a rebate in a life insurance transaction is known as:

A. An offeror C. An offeree

B. An assignor D. A lessee

36. An individual who continues to act as a producer without a license may be fined which of the following amounts?

A. Not less than $10 nor more than $100 C. $750

B. $60 D. $1,000

NILA ©

L/H Licensing – Massachusetts Law

Chapter 20—32

Page 339: FORWARD - WordPress.com · Return of premium p 6-8 6.0 Annuities p 7-1 Annuity principles and concepts p 7-1 Accumulation period versus annuity period p 7-1 Owner, annuitant and beneficiary

37. Which of the following best defines a domestic insurer?

A. An insurer which is incorporated under the laws of another country

B. An insurer which places insurance with an unauthorized company

C. An insurer permitted to conduct insurance business in this State

D. A licensed insurer whose principal office is located in another State

38. All of the following statements are true concerning Massachusetts law regarding life insurance policy loan interest rates,

except:

A. A policy owner may borrow from the cash value of a contract after it has been in effect for three years

B. Policies may include a maximum fixed interest rate of 8%

C. Adjustable policy loan interest rates are based upon annual treasury bill yields

D. Variable policy loan interest rates on an individual policy are figured every twelve months

39. Walter owns a $50,000 term life policy and pays his $300 annual premium on May 1, 1992. He dies May 2, 1992. What

will his beneficiary receive?

A. $50,000

B. $50,000 plus interest

C. $50,000 plus eleven months of prepaid premiums

D. $50,000 plus $300 of prepaid premium

40. A collateral assignment generally occurs in connection with a debt agreement. Which of the following best describes thistype of assignment?

A. A transfer of all rights under a policy to another partyB. A transfer of one or more rights to another party but not allC. An offer of a consideration that is not specified in the policyD. A transfer of some but not all risks provided by the policy

41. Which of the following insurer classifications issues participating life insurance policies?

A. Stock company C. Surplus lines insurerB. Mutual insurer D. Non-admitted insurer

42. Dick is an producer for a stock life insurance company. His brother Tom has a serious heart condition and has not beenable to purchase life insurance because of it. Dick submits an application with premium to his insurer but does not dis-close his brother's ailment and a policy is issued. Tom dies from a heart attack three months later. The insurer uncovershis health history during its claim investigation. What action will the insurer take?

A. Pay the claimB. Deny the claimC. Pay the claim but charge back Dick's commissionD. Deny the claim and return premiums

43. Which of the following are powers or duties of the Commissioner of Insurance?

A. Activates the reserves of an insurer C. Sets life insurance premium ratesB. Prosecutes producers who break laws D. Approves premium rates

44. Acting as an producer without a license after a license suspension subjects a person to a fine of:

A. $5 C. $65B. $10 D. $750

NILA ©

L/H Licensing – Massachusetts Law

Chapter 20—33

Page 340: FORWARD - WordPress.com · Return of premium p 6-8 6.0 Annuities p 7-1 Annuity principles and concepts p 7-1 Accumulation period versus annuity period p 7-1 Owner, annuitant and beneficiary

45. An accident and health producer who does not comply with State Continuing Education requirements will be subject towhich of the following?

A. Banishment to ChileB. License suspension C. A fine of not less than $50 nor more than $500D. Imprisonment for not more than twelve months

46. Which of the following is considered a form of misrepresentation?

A. Commingling premiums C. Misappropriation of premiumB. Twisting D. Concealment in an application

47. The individual offering a rebate in a life insurance transaction is known as:

A. An offeror C. An offeree

B. An assignor D. A lessee

48. An individual who acts as a producer without a valid insurance license in force may be fined which of the following

amounts?

A. Not less than $10 nor more than $100 C. $1,000

B. $750 D. None of the above

49. Which of the following best defines a foreign insurer?

A. An insurer which is incorporated under the laws of another country

B. An insurer which places insurance with an unauthorized company

C. An insurer permitted to conduct insurance business in this State

D. A licensed insurer whose principal office is located in another State

50. The Commissioner of Insurance approves which of the following?

A. Premium rates C. Commission schedules for producers

B. Insurance regulations D. Underwriting procedures

51. Which of the following insurer classifications issues non-participating insurance policies?

A. Stock company C. Surplus lines insurer

B. Mutual insurer D. Non-admitted insurer

52. Each of the following is a covered medical service provided by an HMO, EXCEPT:

A. Cardiac rehabilitation expenses C. Pre-natal care expenses

B. Administrative expenses D. Dependent care expenses

53. All of the following may purchase group health insurance, EXCEPT:

A. An employer for the employees C. A civic group

B. A labor union D. A municipality for its workers

54. Statistical information may be used in accident and health advertising as long as it is.

A. Written in ten point print C. Less than one page in length

B. Approved by formula D. Accurate and relevant

NILA ©

L/H Licensing – Massachusetts Law

Chapter 20—34

Page 341: FORWARD - WordPress.com · Return of premium p 6-8 6.0 Annuities p 7-1 Annuity principles and concepts p 7-1 Accumulation period versus annuity period p 7-1 Owner, annuitant and beneficiary

55. Testimonials are generally utilized to:

A. Verify coverage C. Prove solvency

B. Promote a company's products D. None of the above

56. Which of the following is true regarding advertising by insurers in this State?

A. The insurer must include excerpts of the policy in its advertising

B. The insurer may only advertise in a major media market

C. The insurer must not spend more than 2% of its revenue on advertising

D. The insurer must include its trade name in all advertisements

57. When an insured begins to receive disability income benefits from a policy, which of the following is true?

A. Once eligible for Social Security benefits, income received from disability policies cease

B. Benefits from disability income plans cannot be reduced by any changes in Social Security once benefits have

begun

C. Once Social Security benefits increase, disability income plan benefits will decrease by the proportional amount

D. All of the above are true

58. A producer who uses health insurance premiums collected for his own personal use has engaged in:

A. Fraud C. Rebating

B. Perjury D. Larceny

59. Health Maintenance Organization provide health care to member subscribers. Each of the following is mandated by

Massachusetts law to be covered by an HMO, EXCEPT:

A. Bone marrow transplants

B. Glucose treatment

C. Hospice care during a member's final twelve months

D. Treatment for diabetes

60. A long term care policy provides nursing home coverage. Standard coverages include custodial, skilled nursing and home

health care. Which of the following is also covered by this type of policy?

A. Occupational therapy C. Chore care

B. Activities of daily living D. All of the above

61. Which of the following is not covered by a long term care policy?

A. Personal hygiene

B. Health care for a senior so he or she may remain in their home

C. Hospital confinement

D. Respite care

62. All of the following are excluded in a long term care policy, EXCEPT:

A. Pre-existing conditions

B. Injury to passenger or a commercial airliner

C. Treatment in a government facility

D. Intentionally self-inflicted injuries

NILA ©

L/H Licensing – Massachusetts Law

Chapter 20—35

Page 342: FORWARD - WordPress.com · Return of premium p 6-8 6.0 Annuities p 7-1 Annuity principles and concepts p 7-1 Accumulation period versus annuity period p 7-1 Owner, annuitant and beneficiary

63. Medicare Supplement insurance is available to seniors who wish to be covered for health care expenses not paid by

Medicare Part A or B. Which of the following would be covered by this type of policy?

A. Self dialysis training by a renal dialysis facility

B. Physiotherapist services

C. Illness occurring while a senior is traveling outside the country

D. All of the above

64. A business that on at least 50% of its working days during the preceding year employed from one to not more than 50

eligible employees may qualify for which of the following?

A. Non-contributory group insurance C. A small group health insurance plan

B. Contributory group insurance D. An association health insurance plan

65. An insurance carrier providing health insurance to small groups will not be required to renew the plan if the small business

has:

A. Failed to hire an equal number of male and female employees

B. Modified its business plan without notice

C. Reduced its payroll due to layoffs

D. Failed to provide the carrier with required information

66. According to Massachusetts law, AIDS-related information must be treated as highly confidential. No AIDS-related

information must be communicated by an insurer, producer or employee without:

A. Medical confirmation from an attending physician

B. Written authorization of the individual to whom such information relates

C. Authorization from the insurance carrier's underwriters

D. Division of Insurance approval

67. Hospice services must be included in all group health insurance policies issued in the State of Massachusetts. Benefits

will be provided for insureds who have a life expectancy of:

A. Three months or less C. Nine months or less

B. Six months or less D. Twelve months or less

68. Which of the following is a power or duty of the Commissioner of Insurance?

A. To pass legislation C. Enforce insurance regulations

B. Approve insurance law D. Pass new insurance regulations

69. In order to secure a producer license in this State, an applicant must:

A. Be a resident of a New England State

B. Be of full age

C. Possess a college degree

D. Have been a resident of this State for at least three years prior to applying for a license

70. When can a license candidate actually begin to engage in sales activities in this State?

A. As soon as he or she passes the State licensing examination

B. When the actual license has been issued by the Division of Insurance

C. When the candidate completes the required pre-licensing education

D. As soon as appointment with potential clients are secured

NILA ©

L/H Licensing – Massachusetts Law

Chapter 20—36

Page 343: FORWARD - WordPress.com · Return of premium p 6-8 6.0 Annuities p 7-1 Annuity principles and concepts p 7-1 Accumulation period versus annuity period p 7-1 Owner, annuitant and beneficiary

71. Jack T. Quinn has been issued an insurance license. The license has been issued in the name of the “Jack T. Quinn

Insurance Agency Inc.” When Jack places an advertisement in the local yellow pages promoting his agency, the

advertisement must read:

A. Jack T. Quinn Insurance C. Jack T. Quinn Agency

B. Jack T. Quinn Insurance Agency D. Jack T. Quinn Insurance Agency Inc.

72. What is the maximum fine that may be assessed for an individual acting as a producer without a license?

A. $ 20 C. $ 500

B. $ 100 D. $ 1,000

73. A producer who is found guilty of embezzlement will be subject to which of the following prison sentences?

A. Five years C. Ten years

B. Seven years D. Fifteen years

74. An individual who has a previous felony or a history of engaging in illegal activities may secure an insurance license if

all requirements are satisfied and he or she receives:

A. Oral permission from an employee of the Division of Insurance

B. Written permission from the insurer who will sponsor the producer

C. Written permission from the Commissioner of Insurance

D. Oral permission from the insurer who plans to sponsor the producer

75. Insurers who are authorized to sell life and health insurance in this State are required to maintain a copy of all

advertisements for a period of up to:

A. Two years C. Four years

B. Three years D. Five years

76. When a producer has possibly engaged in an unfair sales practice, the Commissioner may hold a hearing to discuss the

situation. The Commissioner must provide to the producer what amount of notice of the hearing?

A. 20 days C. 30 days

B. 21 days D. 31 days.

77. Which of the following is responsible for remitting a producer’ appointment fee ?

A. The producer C. An insurance agency

B. The insurance company D. The Division of Insurance

NILA ©

L/H Licensing – Massachusetts Law

Chapter 20—37

Page 344: FORWARD - WordPress.com · Return of premium p 6-8 6.0 Annuities p 7-1 Annuity principles and concepts p 7-1 Accumulation period versus annuity period p 7-1 Owner, annuitant and beneficiary

MASSACHUSETTSLIFE, ACCIDENT & HEALTH

SAMPLE LAW EXAM ANSWERS

1. D 2. B 3. C 4. B 5. D

6. D 7. C 8. C 9. C 10. C

11. A 12. C 13. B 14. A15. C

16. B 17. C 18. C 19. B 20. C

21. D 22. C 23. D 24. B 25. B26. C

27. C28. A29. C30. D31. D

32. B33. D34. B35. C36. B

37. C 38. C 39. C 40. B41. B

42. D 43. D 44. C 45. B 46. B

47. A 48. D 49. D 50. A51. A52. B

53. C54. D55. B56. D57. B

58. D59. C60. D61. C62. B

63. D64. C65. D66. B67. B

68. C69. B70. B71. D72. C

73. A74. C75. C76. B77. B

NILA ©

L/H Licensing – Massachusetts Law

Chapter 20—38